Strategic Cost Management 2021 Edition.

Download as docx, pdf, or txt
Download as docx, pdf, or txt
You are on page 1of 567

Philippine Copyright, 2021

by

MA. ELENITA BALATBAT CABRERA


GILBERT ANTHONY B. CABRERA
BERNADETTE ANN B. CABRERA

Any copy of this book not bearing the signature of the author(s) shall be considered as
proceeding from an illegal source.

The Internet addresses listed in the text were accurate at the time of publication. The inclusion of
a website does not indicate endorsement by the authors or GIC Enterprises & Co., Inc. and they
do not guarantee the accuracy of information presented at these sites.

ALL RIGHTS REVERVED


ISBN 978-621-416-104-1
Published & Printed by:

GIC ENTERPRISES & CO., INC.


*National Book Development Board Registered
2017 C.M. Recto Avenue, Sampaloc,
Manila, Philippines
About the Authors

MA. ELENITA B. CABRERA


BBA MBA CPA CMA

Dean Cabrera graduated Magna Cum Laude from the University of the East with a degree of Bachelor of Business Administration, major in Accounting
and was one of the topnotchers when she passed the CPA Licensure Board Examination. She earned her Master Business Administration major in
Financial Management from the University of the Philippines and is a candidate for Doctor of Education at the University of the East. She is a holder of
a Certificate in Management Accounting from the Institute of Certified Management Accountants of Victoria, Australia

Dean Cabrera worked with SGV & Co. as Staff Auditor. She taught Financial Accounting, Financial Management, Management Advisory Services,
Auditing Theory and Practice in various colleges and universities and authored books in these subjects. She previously held the position of Dean of the
College of Business Administration at the Lyceum of the Philippines University.

A former Vice Chairman of the Professional Regulatory Board of Accountancy, she was the BOA representative to the Financial Reporting Standards
Council (FRSC), Philippine Interpretations Committee (PIC) and Auditing and Assurance Standards Council (AASC). She served as the Chairman of
the PRC CPE Council of Accountancy and Chairman of the CHED Technical Committee for Accountancy Education. She was a World Bank Project
Philippine Institute of Certified Public Accountants (PICPA) awards as Outstanding CPA in Education, Honorary Life Membership, Distinguished
Accountancy Author and 2018 Accountancy Hall of Fame.

GILBERT ANTHONY B. CABRERA


BBA MBA CPA

Gilbert received his bachelor’s degree in Accountancy from the University of East, Cum Laude. He obtained a Master in Business Administration
degree with concentrations on International Finance and Accounting from the University of Maryland, College Park, Robert H. Smith School of
Business.

A certified public accountant, he has public accounting experience with SGV & Co. (Ernst and Young Member Firm) and teaching experience with the
University of the East, Manila and University of Maryland, Robert H. Smith School of Business. Presently, he is a Senior Vice-President, Risk and
Finance of Global Insurance Brokerage in California, USA, An active member of the Association of Filipino Finance Managers in California, he is also a
former Board Member of Bay Are Red Cross

BERNADETTE ANN B. CABRERA


BBA MBA CPA

Bernadette received her bachelor’s degree in Accountancy from De Lasalle University. She obtained a Master in Business Administration degree from
Vanderbilt University, Owen Graduate School of Management.

A certified public accountant, she has public accounting experiebce with SGV & Co. (Ernst and Young Member Firm). She spent majority of her career
in the USA as a management consultant for Ernst & Young, New York, specializing in Strategy Risk anf Operational Management, and People and
Organizational Change Presently, she runs her own Executive Coaching business where she coaches CEOs and the other executives on Personal
Leadership and High Perfoming Teams.
I

Preface
The central focus of this 2021 Edition of Stategic Cost Management is how management accounting helps managers better
plan, control and decide for their companies. This book can be immediately after the student has had an introductory course in
Cost Accounting and Control.

The pace of change in organizations continues to be rapid. This book reflects changes occurring, the challenges managers of
business concerns face and how-to-best deal with these challenges using financial and non financial information.

Examples of key additons and changes in topic areas are

 Growing significance of the global economy.


 Increased coverage if strategy and strategic users of cost information,
 Increased discussion of decision users of cost management information occurring such as areas as quality management,
cost estimation and performance measurements.
 Systematic incorporation of new evolving management thinking including supply chain analysis, enterprise resource
planning systems, balanced scorecard, managing productivity and marketing effectiveness.
 Increased attention to behavioral issues such as motivation managers and employees, effect of management control and
transfer pricing on manager’s behavior and earnings management.

A primary concern in the organization and content of this book is the flexibility in meeting the needs of Strategic Cost
Management courses varying in length, content, and student profile.

To aid the student, each chapter begins with measurable learning outcomes that focus on important areas of coverage. Sufficient
text material is available to permit the reader of this textbook to choose topics and depth of coverage as desired. A modular,
flexible organization adopted in this book would facilitate diverse approaches to teaching and learning.

In recognition of the widespread application of strategic cost management concepts, many examples and problems in the book
deal with sevice, not for profit, trading organizations as well as with manufacturing operations. Students gain an understanding of
technical information and learn how to apply that information in appropriate situations. To enhance understanding, relevant
examples on how the concepts apply to the global business environment hace also been included.

The mix of assignment material include questions that review concepts and procedures, exercises that review the direct
application of the basic concepts, problems of varying difficulty that enhance the learning process and cover the concept in
greater depth.

Some problems do not have a single correct answer. Rather they contain multiplt dimensions demanding a broad managerial
perspective. Other problems do not just focus exclusively on computation but may require drawing from one’s knowledge in
marketing, finance economies, economies, quantitative techniques and human resources aspects. Students are therefore
challenged to critically analyze multidimensional issues while still numerical problem-solving skills.

A special note of gratitude goes to

 Our family fir the love and patience,


 Our colleagues in the academe for their ideas, encouragement and support
Contents in Brief
Preface

Unit 1 Introduction to Strategic Cost Management


1
Chapter 1 Overview of Cost Management and Strategy
2
2 The Professional Environment of Cost
20
Management
3 Contemporary Business Environment and Strategic Focus
54
of Cost Management
4 Developing a Competitive Strategy and Contemporary
Contemporary Cost Management
Techniques 69

Unit II Strategic Cost Management Concepts and


Techniques for Planning Control 97
98
Chapter 5 Strategy and the Master Budget
6 Organizational Innovations:
Totally Quality Management; Just-in-Time 144

Production System
7 The Balanced Scoreboard:
179
A Tool to Implement Strategy
8 Cost Planning for Product Life Cycle:
204
Life-Cycle Costing and Long-Term Pricing;
Target Costing and Theory of Constraints
9 Decentralized Operations and Segment 238

Reporting
10 Variable Costing: A Tool for Evaluating
Management Performance 253
dd

Unit III Cost Management Concepts and Tools for


Decision Making 278

Chapter 11 Relevant Costs for Non- Routine 279

Decision Making
335
12 Quantitative Techniques for Decision Making
13 Capital Investment Decision 413

Unit IV Cost Management Concepts and


Techniques for Control and Strategic 470

Performance Measurement
Chapter 14 Management Control and Strategic
Performance Measurement; Strategic 471

Investment Units and Transfer Pricing


15 Financial and Non- Financial Performance 529
Measures
16 Managing Productivity and Marketing
Effectiveness 557

17 Executive Performance Measures and


Compensation 583

Glossary 599

Appendix Time Value Money Concepts 615

References 629
Contents in Brief
Preface
Unit I Introduction To Strategic Cost
Management 1
Chapter 1 Overview of Cost Management and Strategy 2

2 The Professional Environment of Cost


Management 20

3 Contemporary Business Environment and


Strategic Focus of Cost Management 54

4 Developing a Competitive Strategy and


Contemporary Cost Management
Techniques 69
Unit II Strategic Cost Management Concepts and
Techniques for Planning and Control 97
Chapter 5 Strategy and the Master Budget 98

6 Organizational Innovations:
Total Quality Management; Just-in-Time
Production System 144

7 The Balanced Scorecard:


A Tool to Implement Strategy 179

8 Cost Planning for Product Life Cycle:


Life-Cycle Costing and Long-Term Pricing;
Target Costing and Theory of Constraints 204

9 Decentralized Operations and Segment


Reporting 238

10 Variable Costing: A Tool for Evaluating


Management Performance 253
Unit III Cost Management Concepts and Tools for
Decision Making 278
Chapter 11 Relevant Costs for Non-Routine Decision Making 279

12 Quantitative Techniques for Decision Making 335

13 Capital Investment Decision 413


Unit IV Cost Management Concepts and
Techniques for Control and Strategic
Performance Measurement 470
Chapter 14 Management Control and Strategic
Performance Measurement; Strategic
Investment Units and Transfer Pricing 471

15 Financial and Non-Financial Performance


Measures 529

16 Managing Productivity and Marketing


Effectiveness 557

17 Executive Performance Measures and


Compensation 583
Glossary 599

Appendix Time Value of Money Concepts 615

References 629
Contents
Preface
Unit I Introduction To Strategic Cost Management 1
Chapter 1 OVERVIEW OF COST MANAGEMENT AND STRATEGY 2
Expected Learning Outcomes 2
Introduction 3
Users of Cost Management Information 4
Uses of Cost Management Information 5
Management Accountants Role in Strategic Cost
Management 7
Relationship Between Cost Accounting and Cost
Management 12
Strategic Decision Making and the Cost Management
Accountant 12
Basic Cost Management Perspectives 12

Review Questions and Problems 13

2 THE PROFESSIONAL ENVIRONMENT OF COST


MANAGEMENT 20
Expected Learning Outcomes 20
Organization Structure and The Management
Accountant 21
The Chief Financial Officer and the Controller 22
The Controller as the Top Management
Accountant 23
Basic Functions of Controllership 25
Qualifications of the Controller 26
The Chief Financial Officer and the Treasurer
Treasurership 27
Ethical Standards for Management Accountants 28
Code of Conduct for Management Accountants 28
Company Code of Conduct 32
Typical Ethical Challenges 33
Code of Conduct on the International Level International 34
Certification 34
Institute of Management Accountants (IMA) 35
Philippine Association of Management Accountants
(PAMA) 36

Review Questions and Problems 37


3 CONTEMPORARY BUSINESS ENVIRONMENT AND
STRATEGIC FOCUS OF COST MANAGEMENT 54
Expected Learning Outcomes 54
Contemporary Business Environment 55
The Global Business Environment 56
Advances in Manufacturing Technologies 56
Advances in Information technologies,
The Internet and E-Commerce 56
A Greater Focus on Customers 57
New Forms of management Organization 58
Changes in the Social, Political and Cultural
Environment of Business 59
Strategic Focus of Cost Management 60
Cost Management and Accounting Systems 61
When Should the Internal Accounting System be
Changed? 61
Integrative Framework 63

Review Questions and Problems 66

4 THE PROFESSIONAL ENVIRONMENT OF COST


MANAGEMENT 69
Expected Learning Outcomes 69
Developing a Competitive Strategy 70
Strategic Measures of Success 71
Competitive Strategies 74
Distinctive Aspects of the Two Competitive
Strategies 74
Contemporary Cost Management Techniques 76

Review Questions and Problems 143


Unit II Strategic Cost Management Concepts and 97
Techniques for Planning and Control
5 STRATEGY AND THE MASTER BUDGET 98
Expected Learning Outcomes 99
Role of a Budget 99
Importance of Strategy in Budgeting 100
Formulation of Strategy 100
Strategic Goals and Long-term Objectives 102
Long Range Planning 102
Short-Term Objectives and the Master Budget 102
The Management Process of Preparing the Master
Budget 104
Organization for Budget Preparation 104
Budget Guidelines 105
The Budget Period 105
The Initial Budget Proposal 106
Budget Negotiation, Review and Approval,
Revisions 106
The Master Budget 107
Steps in Developing as Master Budget 108
Preparation of Comprehensive Master Budget
Illustrated 108
Budgeting in Service Industries 118
Budgeting in Not-for-Profit Organization 118
Budgeting in International Setting 119
Alternative Approaches in Budgeting 119
Ethical Issues in Budgeting 120

Review Questions and Problems 123

6 ORGANIZATIONAL INNOVATIONS: TOTAL QUALITY


MANAGEMENT; JUST-IN-TIME PRODUCTION SYSTEM 144
Expected Learning Outcomes 144
Introduction 145
Total Quality Management 145
Core Principles of TQM 145
TQM Implementation Guidelines 148
Types of Conformance 149
Costs of Quality 150
Just-in-Time Production System 157
Key Features 157
Financial Benefits of JIT 159
Performance Measures and Control in JIT
Production 160
JIT Effects on Costing System 161

Review Questions and Problems 162

7 THE BALANCED SCORECARD: A TOOL TO IMPLEMENT


STRATEGY 179
Expected Learning Outcomes 179
The Balanced Scorecard 180
Four Perspectives of the Balanced Scorecard 182
Features of a Good Balanced Scorecard 185
Pitfalls in Implementing a Balanced Scorecard 186
Evaluating the Success of a Strategy 187
The Price-Recovery Component 193
The Productivity Component 194
Internal Business Process Performance 196

Review Questions and Problems 199

8 COST PLANNING FOR PRODUCT LIFE CYCLE:


LIFE CYCLE COSTING AND LONG-TERM PRICING;
TARGET COSTING AND THEORY OF CONSTRAINTS 204
Expected Learning Outcomes 204
Cost Management for Product Life Cycle 206
Why Design is Important 207
Common Design Models 208
Cost Management over the Sales Life Cycle 211
Phases of the Sales Life Cycle 211
Target Costing 216
How to Reduce Costs to a Target Cost Level 216
Steps in Implementing a Target Cost Approach 217
Theory of Constraints 221
Steps in Theory of Constraints Analysis 222

Review Questions and Problems 227


9 DECENTRALIZED OPERATIONS AND SEGMENT
REPORTING 238
Expected Learning Outcomes 238
Decentralized Operations 239
Underlying Concepts of Decentralized Operations 239
Advantage of Decentralization 240
Limitations of Decentralization 241
Segment Reporting 242
Levels of Segmented Statement 243
Sales and Contribution Margin 246
Traceable and Common Fixed Costs 245
Problems Related to Proper Cost Assignment 246
Omission of Costs 246
Inappropriate Methods for Allocating Costs
Among Segments 246
Arbitrarily Dividing Common Costs
Among Segments 247

Review Questions and Problems 248

10 VARIABLE COSTING: A TOOL FOR EVALUATING


MANAGEMENT PERFORMANCE 253
Expected Learning Outcomes 253
Inventory Costing and Capacity Analysis 254
Absorption Costing 254
Variable Costing 254
Underlying Concept of Variable Costing 255
Advantages of Using Variable Costing Disadvantages of 255
Using Variable Costing 255
Comparison between Variable Costing and
Absorption Costing 256
Reconciliation of Net Income under Variable Costing
with Net Income under Absorption Costing 257
Why Managers Prefer Direct Costing to Absorption
Costing 262
Variable Costing and Performance Evaluation of
Managers 263
Variable Costing and Segmented Reporting 263
Segmented Reporting: Variable-Costing Basis 264
Variable Costing for Planning and Control 266

Review Questions and Problems 263


Unit III Cost Management Concepts and Tools for 278
Decision Making
11 RELEVANT COSTS FOR NON-ROUTINE
DECISION MAKING 278
Expected Learning Outcomes 279
The Decision Making Process 280
Identifying Relevant Costs 281
Approaches in Analyzing Alternatives in Nonroutine
Decision Making 284
Short Run Vs. Long Run: Other Factors to Consider 286
Types of Decisions 286
Make or Buy Decision 286
Adding or Dropping Products/Segments 288
Sell Now or Process Further 290
Special Sales Pricing 292
Utilization of Scarce Resources 293
Shutdown or Continue Operations 295
Pricing Products and Services 298

Review Questions and Problems 302

12 QUANTITATIVE TECHNIQUES FOR DECISION


MAKING 335
Expected Learning Outcomes 335
Rationale in Using Quantitative Techniques 336
Probability 337
Decision Making under Certainty 337
Decision Making under Uncertainty 338
Assigning Probabilities 338
Types of Probabilities 339
Basic Terms Used with Probability 339
Rules in Combining Probabilities 339
Probability Distributions 341
Payoff (Decision) Tables 344
Expected Value of Perfect Information 345
Decision Tree 346
Underlying Concept 346
Advantages of Decision Tree Analysis 346
Limitations of Decision Tree Analysis 347
Steps in Making a Decision Tree 247
Illustrative Problem: Preparation of a
Decision Tree 348
Learning Curve 351
Simulation Techniques 352
Advantages and Limitations of Simulation 354
Monte Carlo Technique 355
Sensitivity Analysis 355
Queuing 356
Linear Programming 357
Nature and Applications 357
Steps in the Formulation of a Linear Program 357
Computational Methods of Linear
Programming 358
Graphic Method 358
Algebraic Method 362
Simplex Method 362
Shadow Prices 365
Program Evaluation and Review Techniques
(PERT) 368
Basic Underlying Concept 368
Expected Activity Time 369
Concept of Critical Path 370
Cost Estimating 370
Crashing the Network 372
PERT-Cost Network 372
Variation in Activity Time 375
Variation Along a Path 378
Accountant's Role in PERT 379
Benefits and Limitations of PERT 379
Gantt Chart 380
Steps in Preparing a Gantt Chart 380
Illustration of Gantt Chart 381
Advantages of Gantt Chart 381
Inventory Modeling 382
Inventory cost 382
EOQ model 383
Reorder Point 385
Safety Stock 386

Review Questions and Problems


13 CAPITAL INVESTMENT DECISION
Expected Learning Outcomes 413
Capital Budgeting Defined 414
Characteristics of a Capital Investment Decision 414
Categories of Capital Investments Elements of Capital 415
Budgeting 416
Net Initial Investment or Project Cost 416
Net Cash Returns 417
Minimum or Lowest Acceptable Rate of Return 419
Process of Capital Budgeting 422
Categories of Project Cash Flows 423
Screening Capital Investment Proposals 425
Payback period 425
Bail-out period 427
Accounting Rate of Return 428
Discounted Cash Flow Techniques 430
Net Present Value 431
Discounted Rate of Return 434
Payback Reciprocal 437
Profitability Index 437
Discounted Payback period 439
Preference Decisions - The Ranking of Investment
Projects 440
Comparing Preference Rates 441
Comparing Projects with Unequal Lives 441
Replacement Chain (Common Life) Approach 441
Equivalent Annual Annuity (EAA) Approach 443
Inflation and Capital Budgeting 445

Review Questions and Problems 447

Unit IV Cost Management Concepts and techniques for


Control and Strategic Performance Measurement 470
14 MANAGEMENT CONTROL AND STRATEGIC
PERFORMANCE MEASUREMENT; STRATEGIC
INVESTMENT UNITS AND TRANSFER PRICING 471
Expected Learning Outcomes 471
Performance Evaluation and Control 472
Strategic Performance Measurement 473
Basic Concepts 473
Decentralization and Segment Reporting 473
Prerequisites to the Initiation and Maintenance of
an Effective Strategic Performance
Measurement or Responsibility Accounting 474
Strategic Business Units (SBUs) and their Evaluation 475
Cost SBU 476
Profit SBU 479
Investment SBU 482
Revenue SBU 488
Transfer Pricing 491
The Need for Transfer Price 491
Alternative Transfer Pricing Schemes 492
Minimum Transfer Price 492
Market-based Transfer Price 493
Cost-based Transfer Price 494
Negotiated Transfer Price 496
Distress Prices 497
Transfer Price for Services 497
Multinational Transfer Pricing 498
Review Questions and Problems 505
15 FINANCIAL AND NON-FINANCIAL PERFORMANCE
MEASURES 529
Expected Learning Outcomes 529
Financial and Nonfinancial Performance Measures 530
Steps in Designing Accounting-based Performance
Measures 531
Step 1: Choosing among Different
Performance Measures 631
Step 2: Choosing the Time Horizon of the
Performance Means 532
Step 3: Choosing Alternative Definitions for
Performance Measures 532
Step 4: Choosing Measurement Alternatives for
Performance Measures 533
Long-term Assets: Gross or Net Book Values 534
Step 5: Choosing Target Levels of Performance 534
Step 6: Choosing the Timing of Feedback 535
Performance Measurement in Multinational 539
Companies
Computation of Foreign Division's ROI in the Foreign 540
Currency
Distinguishing Performance of Managers from 541
Performance of Organization Units
Intensity of Incentives, Financial and Nonfinancial 542
Incentives 543
Review Questions and Problems
16 MANAGING PRODUCTIVITY AND MARKETING
EFFECTIVENESS 557
Expected Learning Outcomes 557
Managing Productivity 558
Measuring Productivity 558
Partial Productivity 560
Partial Operational Productivity 562
Partial Financial Productivity 563
Advantages of Partial Productivity Measures 563
Limitations of Partial Productivity Analysis 563
Total Productivity 564
Benefits and Limitations of Total Productivity
Measures 565
Managing Marketing Effectiveness 565
Summary of Variance Analysis to Assess Marketing
Effectiveness 567

Review Questions and Problems 562

17 EXECUTIVE PERFORMANCE MEASURES AND


COMPENSATION 583
Expected Learning Outcomes 583
Objectives of Management Compensation 584
Executive Performance Measures and
Compensation 584
Cash Compensation 585
Noncash Compensation 585
Bonus Plans 586
Bases for Bonus Compensation 586
Bonus Compensation Pools 587
Bonus Payment Options 588
Performance Measures at the Individual Activity
Level 590
Performing Tasks 590
Team-based Compensation Arrangements 590
Environment and Ethical Responsibilities 591

Review Questions and Problems 592

Glossary 599

Appendix Time Value of Money Concepts 615

References 629

CHAPTER
1
OVERVIEW OF COST
MANAGEMENT AND
STRATEGY

EXPECTED LEARNING OUTCOMES


After studying the chapter, you should be able to...
1. Explain what strategy is.
2. Relate strategic management. cost management to strategic
3. Describe the nature of cost management information and how they are developed.
4. Explain the objective, scope and benefits derived from proper cost management
5. Enumerate and describe the various users of cost management information
6. Explain how cost management information is used for the following management functions
 Strategic Management
 Planning and Decision-Making
 Management and Operational Control
 Reportorial and compliance to legal and various regulatory requirements

7. Understand the cost management accountant's role in strategic cost management


8. Describe the role of the cost management accountant in the development and implementation
of strategic decision for the business firm
9. Enumerate and explain the basic cost management perspectives 1863

CHAPTER 1
OVERVIEW OF COST MANAGEMENT AND STRATEGY

INTRODUCTION
The growing pressures of global competition, trade wars among countries, technological
innovation and changes in business processes have made cost management much more
important, critical and dynamic than ever before. Business managers must think and act
competitively and doing so requires a strategy.
Strategy is a set of policies, procedures and approaches to business that produce long-term
success while strategic management involves the development of a sustainable competitive
position. Strategic cost management involves the development of cost management information
to facilitate the principal management function which is strategic management.
In today's business environment, the development and use of information especially cost
management information is a critical factor in the effective management of a firm or
organization.
Cost management information is the information that the manager needs to effectively manage
the firm, profit-oriented as well as not-for-profit organization. This includes both financial
information about cost and revenues as well as relevant nonfinancial information about
productivity, quality and other key success factors for the firm.
Cost management is the practice of accounting in which the accountant develops and uses cost
management information. For competitive success, it is not enough to emphasize only on
financial information. This could lead manager to stress cost reduction (a financial measure)
while ignoring or even lowering quality standards (a nonfinancial measure). This decision could
be a critical mistake which could lead to the loss of customers and market share in the long run.
If a firm is to compete successfully, importance should be given to nonfinancial and long-term
measures of operating performance such as product and manufacturing advances, product quality
and customer loyalty. Cost management information, is thus a value-added concept. It adds value
by helping a firm be more competitive.

Effective strategic management is very important to the success of organization and is thus the
pervasive theme of this book.
every firm or
Strategic thinking involves anticipating changes. Products and production Flexibility is
important. The ability to make fast changes is critical as a result of processes are designed to
accommodate expected changes in customer-demands. the demand of the new management
concepts of e-commerce, speed to market, and flexible manufacturing. Product life cycle - the
time from the introduction of a shorter. Success in the recent past days or months is no longer a
measure of new product to its removal from the market is expected to become shorter and
ultimate success; the manager must be "driving" the firm by using the windshield, not the rear-
view mirror.

The strategic emphasis also requires creative and integrative thinking, that is, the ability to
identify and solve problems from a cross-functional view. The business functions are often
identified as marketing, production, finance, and accounting/ controllership. Instead of viewing a
problem as a production problem, a marketing problem, or a finance and accounting problem,
cross-functional teams view it from an integrative approach that combines skills from all
functions simultaneously The integrative approach is necessary in a dynamic and competitive
environment. The firm's attentions is focused on satisfying the customers' needs; all of the firm's
resources, from all functions, are directed to that goal.

USERS OF COST MANAGEMENT INFORMATION


Cost management information is useful in all organizations: business firms, governmental units,
and not-for-profit organizations. Business firms are usually categorized by industry, the main
categories being merchandising, manufacturing, and service. Merchandising firms purchase
goods for resale. Merchandisers that sell to other merchandisers are called wholesalers; those
selling directly to
consumers are retailers.
Governmental and not-for-profit organization provide services, much like the firms in service
industries. However, these organizations provide the services for which no direct relationship
exists between the amount paid and the services provided. Instead, both the nature of these
services and the customers to receive them are determined by government or philanthropic
organizations. The resources are provided by governmental units and/or charities. The services
provided by these organizations are often called public goods to indicate that no typical market
exists for them. Public goods have a number of unique characteristics, such as the impractically
of limiting consumption to a single customer (clean water and police and fire protection are
provided for all residents).

USES OF COST MANAGEMENT INFORMATION


Cost management information is needed for each of the following management functions,
namely:
1. Strategic Management
Strategic management involves the development of a sustainable competitive position in which
the firm's competitive advantage spells continued success. A strategy is a set of goals and specific
action plans that if achieved, provide the desired competitive advantage. Strategic management
involves identifying and implementing these goals and action. plans. Management must make
sound strategic decisions regarding the choice of products, manufacturing methods, marketing
techniques and channels and other long-term issues.
The strategic emphasis requires an integrative approach which combines skills from all business
function, namely, marketing, production, finance and accounting/controllership, is necessary in a
dynamic and competitive environment.
Due to increasing strategic issues, cost management has moved from a traditional role of product
costing and operational control to a broader strategic focus: strategic cost management.
Strategic cost management is the development of cost management information to facilitate the
principal management function, strategic management.

2. Planning and Decision-making


Cost management information is needed to support recurring decision such as replacing and
maintaining equipment, managing cash flows, budgeting raw materials purchases, scheduling
production, pricing and
managing distribution of products to customers, and so forth
Planning and decision-making involves budgeting and profit planning, cash flow management
and other decision related to the firm's operation such as deciding whether to lease or buy a
facility, whether to replace or just repair as equipment, when to change a marketing plan or when
to begin new product development.
3. Management and Operational Control
Cost management information is needed to provide a fair and effective basis for identifying
inefficient operations and to reward and motivate the most effective manages.
Operational Control takes place when mid-level manages (e.g., product managers, regional
managers) monitors the activities of operating-level managers and employers (e.g., production
supervisions, department heads). Management control on the other hand, is the evaluation of
mid- level manager by upper-level manager (e.g., Controller or the Chief Financial Officer
(CFO)).
4. Reportorial and Compliance to Legal Requirements
Reportorial and compliance responsibilities require management to comply with the financial
reporting requirements to regulatory agencies such as the Securities and Exchange Commission
(SEC) Bureau of Internal revenue (BIR), and other relevant government authorities and
agencies.
The financial statement preparation role has recently received a renewed new focus and interest
as accounting scandals have shown how crucial and
important accurate financial information is for investors.
The financial statement information also serves the other three management functions as this
information is often an important part planning and decision making, control and strategic
management.
MANAGEMENT ACCOUNTANT'S ROLE IN STRATEGIC COST
MANAGEMENT
Cost Management is the practice of accounting in which the accountant develops and uses cost
management information. This area of accountancy practice is performed by management
accountants. Management accountants are the accounting professionals who develop and analyze
cost management information and other accounting information.
Management Accounting involves the application of appropriate techniques and concepts to
economic data so as to assist management in establishing plans for reasonable economic
objectives and in the making of rational decisions with a view toward achieving these objectives.
It is the process of identification, measurement, accumulation, analysis, preparation,
interpretation, and communication of financial information, which is used by management to
plan, evaluate and control activities within an organization. It also comprises the preparation of
financial reports for non-management groups such as shareholders, creditors, regulatory agencies
and tax authorities.
Management accountants (including cost accountants) are concerned with providing information
to managers, that is, people inside an organization who direct and control the operations. They
provide a variety of reports. Some reports focus on how well managers and business units have
performed while other reports provide timely and frequent updates on key indicators, analysis of
business situation or opportunity and analytical reports that are needed to investigate specific
problems.
Management accountants at appropriate levels are involved actively in the process of managing
the entity. The process includes making strategic, tactical and operating decisions and helping to
coordinate the efforts of the entire organization. The management accountant participates, as part
of management, in assuring that the organization operates as a unified whole in its long-run
intermediate and short-run best interests.
Management accounting is concerned primarily with providing information to internal managers
who are charged with planning and controlling the operations of the firm and making a variety of
management decisions. Generally, management accountants do the following tasks:
(a) Scorekeeping or data accumulation which enables both internal and external parties to
evaluate organizational performance and position.
(b) Interpreting and reporting of information that helps manager to focus on operating
problems, opportunities as well as inefficiencies. This is commonly associated with current
planning and control and the analysis and investigations of recurring routine internal accounting
reports to signal situations in which management action may be required.
(c) Problem-solving or quantification of the relative merits of possible courses of action as
well as recommendations as to the best procedure. This is
commonly associated with non-recurring decisions.

Three important guidelines help management accountants provide the most value when
scorekeeping provide the most value when scorekeeping, problem-solving and attention directing
(interpreting and reporting). These are
1. Employ a cost-benefit approach
2. Recognize behavioral as well as technical considerations and
3. Use appropriate cost concepts for different purpose
Management accountants continually face resource-allocation decisions, such as whether to
purchase a new software package or hire a new employee. The cost- benefit approach should be
used in making these decisions: Resources should be spent if they are expected to better attain
company goals in relation to the expected costs of those resources. The expected benefits from
spending should exceed the expected costs. The expected benefits and costs may not be easy to
quantify. Nevertheless, the cost-benefit approach is useful for making resource-allocation
decisions.
Specifically, the management accountant provides a system which allows management to receive
the necessary information used in performing its administrative functions of:
(a) planning which involves setting of goals for the firm, evaluating the various ways to meet the
goals and picking out what appears to be the best way to meet the goals;
(b) controlling which involves the evaluation of whether actual performance conforms with
planned goals; and
(c) decision making which involves determination of predictive information
(e.g. relevant costs) for making important business decisions.
Planning
A key activity for all companies is planning. Planning involves identifying alternatives and
selecting a course of action and specifying how the action will be implemented to further the
organization's objectives.
The plan communicates a company's goals to employees and specifies the resources needed to
achieve them. The plans of management are often expressed formally in budgets. Cash budgets,
capital budgets, and projected statements of financial position are examples of contributions
which accounting can make in resource planning while break-even analysis, projected income
statements are examples of useful tools in profit planning.
Control
Control of organizations is achieved by evaluating the performance of managers and the
operations for which they are responsible. The distinction between evaluating managers and
evaluating the operations they control is important. Managers are evaluated to determine how
their performance should be rewarded or punished, which in turn motivates them to perform at a
high level. Based on an evaluation indicating good performance, a manager might receive
substantial bonus compensation. An evaluation indicating a manager performed poorly might
lead to the manager being fired. In part because evaluations of managers are typically tied to
compensation and promotion opportunities, managers work hard to ensure that they will receive
favorable evaluations.
Cost variance analysis, financial statements analysis, gross profit variance analysis are some of
the accounting control reports used to inform managers when activities which are part of their
responsibility are deviating from the plan. The reports used evaluate the performance of
managers and the operations they control are referred to as performance reports.
Although there is no generally accepted method of preparing a performance report, such reports
frequently involve a comparison of current period performance with performance in a prior
period or with planned (budgeted) performance.
Performance reports may not provide definitive answers, but they are still extremely useful.
Managers can use them to "flag" areas that need closer attention and to avoid areas that are under
control. It would not seem necessary, for example, to investigate labor, rent, depreciation, or
other costs, because these costs are either equal to or relatively close to the planned level of cost.
Typically, managers follow the principle of management by exception when using performance
reports. This means that managers investigate departures from the plan that appear to be
exceptional; they do not investigate minor departures from the plan.
Operations are evaluated to provide information as to whether or not they should be changed
(i.e., expanded, contracted, or modified in some way). An evaluation of an operation can be
negative even when the evaluation of the manager responsible for the operation is basically
positive.
Company plans often play an important role in the control process. Managers can compare actual
results with planned results and decide if corrective action is necessary. If actual results differ
from the plan, the plan may not have been followed properly, the plan may have not been well
thought out, or changing circumstances may have made the plan out of date.
Figure 1-1 presents the major steps in the planning and control process. Once a plan has been
made, actions are taken to implement it. These actions lead to results, which are compared with
the original plan. Based on this evaluation, managers are rewarded (e.g., given substantial
bonuses or promoted if performance is judged to be good) or punished (e.g., given only a small
bonus, given no bonus, or even fired if performance is judged to be poor). Also, based on the
evaluation process, operations may be changed. Changes may consist of expanding (e.g adding a
second shift), contracting (e.g., closing a production plant), or improving operations (e.g.,
training employees to do a better job answering customer product inquiries). Changes may also
consist of revising an unrealistic plan.

Figure 1-1: Planning and Control Process

Plan

Decisions to change operations Action taken to


Or reverse implement plan

Results

Decisions to reward or punish Comparison of planned

managers and actual results

Evaluation

Thus, accounting serves management at all stages of the management process, from the formulation of
objectives and so on up to the feedback of performance information which in turn helps in the
reformulation of objectives.

Decision Making
As indicated in Figure 1-1, decision making is an integral part of the planning and control
process - decisions are made to reward or punish managers, and decisions are made to change
operations or revise plans. Should a firm add a new product? Should it drop an existing product?
Should it manufacture a component used in assembling its major product or contract with
another company to produce the component? What price should a firm charge for a new product?
These questions indicate just a few of the key decisions that confront companies. And how well
they make these decisions will determine future profitability and, possibly, the survival of the
company. Recognizing the importance of making good decisions, we will devote all of Chapter
11 to the topic.
In summary, the management accountant develops cost management information for the Chief
Financial Officer, other managers and employee teams to use to management the firm and make
the firm more competitive and successful.
RELATIONSHIP BETWEEN COST ACCOUNTING AND COST MANAGEMENT
Cost accounting is a systematic set of procedures for recording and reporting measurements of
the cost of manufacturing goods and performing services in the aggregate and in detail. It
includes methods for recognizing, classifying, allocating, aggregating and reporting such costs
and comparing them with standard costs.
Cost management needs the output of cost accounting. Its purpose is to provide managers with
information which aids decision. There are no generally accepted principles which specify how
management accounting information is to be reported. While systems such as direct costing and
standard costing exist in management accounting, each accounting report should be tailored to
the needs of the decision and the decision maker. The most effective systems result when the
manager-decision maker and the accountant work together until the accountant understands the
decision to be made and the manager understands the source of information that the accountant
will report.
Managers use cost management information to choose strategy, to communicate it and to
determine how best to implement it. They use this information to coordinate their decisions about
designing, producing and marketing a product or service.
STRATEGIC DECISION MAKING AND THE COST MANAGEMENT ACCOUNTANT
Basic Cost Management Perspectives
For cost management process to succeed, it is necessary that managers must complement their
measurement skills with basic management perspectives that "go beyond the numbers". This will
enable them to make intelligent planning, control, and decision making for the enterprise. These
are:
a.A Strategic Management Perspective
b. An Enterprise Risk Management Perspective
c. A Corporate Social Responsibility Perspective
d. A Process Management Perspective
e.A Leadership Perspective
f. An Ethical Perspective
A. A Strategic Management Perspective
An enterprise generates profit by attracting customers willing to pay for the goods and services it
offers. Customers usually compare the goods and services offered by a company to the same
goods and services offered by other companies. The key to a company's success is creating value
for customers while differentiating itself from its competitors. Identifying how a company will
do this is what strategy is all about. However, a chosen strategy is only as good as how
effectively it is implemented. The management accountant provides input that aids in developing
strategy, building resources and capabilities, and implementing strategy.
A strategy is a "game plan" that enables the company to attract customers by distinguishing itself
from competitors. The focal point of a company's strategy should be its target customers. A
company can only succeed if it creates a reason for its target customers to choose it over a
competitor. These reasons or what are more formally called customer value propositions are the
essence of strategy.
Customer value premises tend to fall into three broad categories, namely, customer intimacy,
operational excellence, and product leadership.
Customer Intimacy Strategy
Companies that adopt a customer intimacy strategy are in essence saying...
"You should choose us because we can customize our products and services to meet your
individual needs better than our competitors."
Operational Excellence
Companies that pursue the second customer value premise called operational excellence are
saying to their target customers...
"You should choose use because we deliver products and service faster, more conveniently, and
at a lower price than our competitors".

Product Leadership
Companies pursuing the third customer value premise are saying to their
target customers...
"You should choose us because we offer higher quality products than our competitors".

B. An Enterprise Risk Management Perspective


A simplified framework for an Enterprise-wide Risk Management Process follows:
Risk Management System Top Management’s Involvement
Oversight Activites:

Establish goals and objectives, Set management policy, establish


roles and responsibilities, context, set limits and tolerance,
common language, and oversight etc.
structure
Risk Management Process:

Step 1: Assess Risks Identify source, Ensure that process captures all
measure business risks

Step 2: Develop/Design Action Ensure that all available tools


Plans: Reduce, avoid, retain, and methodologies are used
transfer, exploit

Step 3: Implement Action Plans Review effectiveness of plans,


Check capabilities

Step 4: Monitor and report risk Review and evaluate regular


management performance reports on performance

Step 5: Continuously improve risk Evaluate recommendations for


management capabilities
improvement

Every business manager should recognize the fact that every strategy, plan and decision involves
risks.
Enterprise risk management is a process used by an entity to identify those risks and develop
responses to them that enable it to be assured of meeting its goals.
Examples of Business Risks Examples of Controls to Reduce Business Risk
1. Losing market share due to the Develop an approach for legally gathering
unforeseen actions of competitors information about competitors' plans and practices
2. Products harming customers Develop a formal and rigorous new product
testing program
3. Intellectual assets being stolen from Create Create firewalls that prohibit computer hackers
firewalls that prohibit computer from corrupting or stealing intellectual property
computer files
4. A website malfunctioning Thoroughly test the website before going “live”
on the internet
5. A supplier strike halting the flow of Establish Establish a relationship with two companies
raw materials capable of providing needed raw materials
6. Poor weather conditions shutting down Develop contingency plans for overcoming
operations weather-related disruptions
7. A poorly designed incentive system causing Create a balanced set of performance measures
compensation employees to make bad decisions that motivates the desired behaviour
8. Inaccurate budget estimates causing excessive Implement a rigorous budget review process
or insufficient production
9. Poor environmental stewardship causing Create a reporting system that tracks key
reputational and financial damage environmental performance indicators
10. Failing to comply with equal employment Create a report that tracks key metrics related to
opportunity laws compliance with the laws
The following schedule shows examples of business risks that companies face and the
corresponding example of a control that could be implemented to help reduce each the risks
mentioned:

Although these types of controls cannot completely eliminate risks, they enable companies to
proactively manage their risks rather than passively reacting to unfortunate events that have
already occurred.

C. A Corporate Social Responsibility Perspective


Corporate social responsibility (CSR) is a concept where business organizations consider the
needs of all stakeholders when making decisions. They are responsible not only for creating
strategies that produce financial results that satisfy shareholders but also to serve other
customers, suppliers, communities and stakeholders such as environmental and human rights
advocate whose interests are tied to the company's performance.

Examples of corporate social responsibilities that are of interest to the stockholders groups
follow. Companies should
1. Provide its customers with:
Safe, high-quality products that are fairly priced.
b. Easy-to-use information systems for shopping and tracking orders.
c. Competent, courteous, and rapid delivery of products and services.
d. Easy-to-use information systems for shopping and tracking orders.

II. Provide suppliers with:


a. Hassle-free acceptance of timely and complete deliveries. b. Fair contract terms and prompt
payments.
c. Reasonable time to prepare orders.
d. Cooperative rather than unilateral actions.

III. Provide stockholders with:


a. Competent management.
b. Full disclosure of enterprise risks.
c. Easy access to complete and accurate financial information.
d. Honest answers to knowledgeable questions.

IV. Provide its employees with:


a. Fair compensation.
b. Safe and humane working conditions.
c. Non-discriminatory treatment and the right to organize and file
grievances.
d. Opportunities for training, promotion, development. and personal

V. Provide the communities with:


a. Payment of fair taxes.
b. Resources that support charities, schools, and civic activities.
c. Honest information about plans such as plant closings.
d. Reasonable access to media: sources.

VI. Provide environmental and human rights advocate with:


a. Recycling and resource conversation data.
b. Full disclosure of suppliers located in developing countries.
c. Greenhouse gas emission data.
d. Child labor transparency.

D. A Process Management Perspective


Most companies organize themselves by functional departments, such as Marketing Department,
the Research and Development Department, and the Accounting Department. These departments
tend to have a clearly defined "chain of command" that specifies superior and subordinate
relationships. However, effective managers understand that business processes, more so than
functional departments, serve the needs of company's most important stakeholders it customers.
A business process is a series of steps that are followed in order to carry out some tasks in a
business. These steps often span departmental boundaries, thereby requiring managers to
cooperate across functional departments. The term value chain is often used to describe how an
organization's functional departments interact with one another to form business processes. A
value chain, as shown below consists of the major business functions that add value to a
company's products and services.
Business Functions Making Up the Value Chain
Research and Product Design Manufacturing Distribution Customer

Development Service

Managers need to understand the value chain to be effective in terms of planning, control, and
decision making. For example, if a company's engineers plan to design a new product, they must
communicate with the Manufacturing Department to ensure that the product can actually be
produced, the Marketing Department to ensure that customers will buy the product, the
Distribution Department to ensure that large volumes of the product can be cost-effectively
transported to customers, and the Accounting Department to ensure that the product will increase
profits
From a contro.l and decision-making standpoint, managers also need to focus on process
excellence instead of functional performance. For example, if the Purchasing Department
focuses solely on
minimizing the cost of purchased materials, this narrowly focused attempt at cost Department,
more complaints in the Customer Service Department, and reduction may lead to greater scrap
and rework in the Manufacturing greater challenges in the Marketing Department because
dissatisfied customers are turning their attention to competitors.
Managers frequently use a process management method known as lean thinking, or what is
called Lean Production in the manufacturing sector. Lean Production is a management approach
that organizes resources such as people and machines around the flow of business processes and
that only produces units in response to customer orders.
E. Leadership Perspective
To achieve success, organizational leaders must be able to unite the behaviours of the fellow
employee who have diverse needs, beliefs and goals to the workplace. Leaders need to
understand how (a) internal motivation (b) external incentives and (c) cognitive bias influence
human behaviour.
(a) Internal motivation refers to motivation that comes from within one's self. A leader who is
perceived by employees as credible and respectful of their value to the company can increase the
extent to which those employees are intrinsically motivated to pursue strategic goals.
To be perceived as a credible and respectful leader, he (she) must possess the following attributes
1. Technical competence (spanning the value chain)
2. Personal integrity (in terms of work ethic and honesty)
3. Strong communication skills (including oral presentation skills and writing skills)
4. Strong mentoring skills (to help others realize their potential)
5. Strong listening skills (to learn from his (her) co-workers and be responsive to their needs,
and
6. Personal humanity (in terms of giving recognition to all employees who contribute to the
organization's success.
(b) External Incentives such as bonus compensation, are given by many organizations to
highlight important goals and to motivate employees
to achieve them.
(c) Cognitive Bias. Leaders should be aware that are all people (including themselves) should
possess cognitive biases or distorted thought processes such as promoting false assertion that can
adversely affect planning, controlling and decision making.
While cognitive biases cannot be eliminated effective leaders should take steps and reduce their
negative impacts. These steps include
1. They should acknowledge their own susceptibility to cognitive bias (e.g., being overly
optimistic in assessing future outcome or overestimating ones strengths and underestimating
ones weaknesses relative to others)
2. They should acknowledge the presence of cognitive bias in others and introduce techniques to
minimize their adverse consequences.
To reduce if not totally eliminate cognitive biases, a leader may W routinely appoint independent
team of employees to assess the credibility of recommendations set forth by other individuals
and groups.
F. An Ethics Perspective
Without ethics, the economy would operate much less efficiently less product would be available
to consumers, quality would be lower and most likely, process would be higher. Ethical
behaviour is the lubricant that keep the economy running smoothly.
In other words, without fundamental trust in the integrity of the business, the economy would
operate much less efficiently. Therefore for the benefit of everyone - including profit-making
companies - it is vitally important that that business be conducted within an ethical framework
that builds and sustains trust.
Professional management accountants have developed and implemented a set of Ethical
Standards for practitioners. Refer to pages 28 to 34 for Ethics Standards for Management
Accountants.

REVIEW QUESTIONS AND EXERCISES


Questions
1. Give four examples for firms you think would be significant users of
cost management information and explain why.
2. Give three examples of firms you think would not be significant users of cost management
information and explain why.
3. What is meant by the term cost management? Who in the typical firm or organization is
responsible for cost management?
4. List the four functions of management. Explain what type of cost management information is
appropriate for each.
5. Which is the most important function of management, and why?
6. Identify the different types of business firms and other organizations that use cost management
information and explain how the information is used.
7. Name a firm or organization you know of that you are reasonably sure uses strategic cost
management and explain why it does so.
8. The opening paragraph of an accounting textbook says. "Managers need accounting
information and need to know how to use it." Critically evaluate this statement.
9. The owner of a small software company felt his accounting system was useless. He stated,
"Accounting systems only generate historical costs. Historical costs are useless in my business
because everything changes so rapidly."
Are historical costs useless in rapidly changing environments? b. Should accounting systems be
limited to historical costs?
10. A finance professor and a marketing professor were recently comparing notes on their
perceptions of corporations. The finance professor claimed the goal of a corporation should be to
maximize the value to the shareholders. The marketing professor claimed that the goal of a
corporation should be to satisfy customers. What are the similarities and differences in these two
goals?
11. How do management accountant support strategic decisions?
12. Define the term strategic cost management.
13. What is meant by a business strategy?
14. What information does cost accounting provide?
15. How do cost accountants support strategic decisions?
16. Pick any large company and describe three risks that it faces and how it responds to those
risks.
17. Pick three industries and describe how the risks faced by companies within those industries
can influence their planning.
18. Locate the website of any company that publishes a corporate social responsibility report
(also referred to as a sustainability report). Describe three nonfinancial performance measures
included in the report. Why do you think the company publishes this report?

Multiple Choice
1.Which of the following statements is false?
a. Cost accounting measures and reports short-term, long-term financial, and nonfinancial
information.
b. Cost management provides information that helps increase value for customers.
c. All strategies should be evaluated regarding the resources and capabilities of the
company.
d. A good cost accounting system is narrowly focused on a continuous reduction of costs.

2. Which of the following statements is correct?


a. a.The best-designed strategies are valuable whether or not they are effectively
implemented.
b. b. To take advantage of changing market opportunities, the annual budget should be
strictly enforced.
c. c. Linking rewards to performance is a major deterrent to good. management
performance.
d. d. An important strategic decision is making the correct investments in productive assets.

3. All of the following statements are true except


a. A budget is a tool used to plan and express strategy.
b. Financial accounting reports financial and nonfinancial information that helps managers
implement company strategies.
c. Feedback links planning and control.
d. Control includes deciding what feedback to provide that will help with future decision
making.

4. All of the following statements are false except


a. Attention-directing activities should focus on cost-reduction opportunities, and not on
value-adding opportunities.
b. For strategic decisions, scorekeeping is the most prominent role played by management
accounting.
c. A budget may be used as a planning tool, but not as a control tool.
d. Management accountants often are simultaneously doing problem- solving, scorekeeping,
and attention-directing activities.

5. Management accounting
a. focuses on estimating future revenues, costs, and other measures to forecast
activities and their results.
b. provides information about the company as a whole.
c. reports information that has occurred in the past that is verifiable and reliable.
d. provides information that is generally available only on a quarterly or annual
basis.
6. The person MOST a(n) likely to use management accounting information is application.
a. a.banker evaluating a credit application
b. shareholder evaluating a stock investment.
c. governmental taxing authority.
d. assembly departinent supervisor.

7. Which of the following description refers to management accounting information?


a. It is verifiable and reliable.
b. It is driven by rules.
c. It is prepared for shareholders.
d. It provides reasonable and timely estimates.

8. Which of the following groups would be LEAST likely to receive detailed management
accounting reports?
a. Stockholders
b. Sales representatives
c. Production supervisors
d. Managers

9. Management accounting information includes


a. tabulated results of customer satisfaction surveys.
b. the cost of producing a product.
c. the percentage of units produced that are defective.
d.all of the above
10. Which of the following types of information are used in management accounting?
a. a.Financial information
b. b.Nonfinancial information
c. Information focused on the long term
d. All of the above

11. Management accounting includes


a. implementing strategies.
b. developing budgets.
c. preparing special studies and forecasts.
d. all of the above

12. Financial accounting is concerned PRIMARILY with


a. external reporting to investors, creditors, and government authorities.
b. cost planning and cost controls.
c. profitability analysis.
d. providing information for strategic and tactical decisions.
e.

13. Financial accounting provides a historical perspective, whereas management accounting


emphasizes
a. the future.
b. past transactions.
c. a current perspective.
d. reports to shareholders.

14. Strategy specifies


a. how an organization matches its own capabilities with the opportunities in the
marketplace.
b. standard procedures to ensure quality products.
c. incremental changes for improved performance.
d. the demand created for products and services.

15. Control includes


a. implementing planning decisions.
b. evaluating performance,
c. providing feedback to help with future decision making:
d. all of the above.

16. Linking rewards to performance


a. helps to motivate managers.
b. allows companies to charge premium prices.
c. should only be based on financial information.
d. d.does all of the above

17. Control measures should


a. be set and not changed until the next budget cycle.
b. be be flexible to allow for employees who are slackers.
c. be kept confidential from employees aployees so that competitors don't have an
opportunity to gain a competitive advantage.
d. be linked by feedback to planning

18. For control decisions, emphasis is placed on the _____________ role(s) of management
accounting.
a. problem-solving
b. scorekeeping
c. attention-directing
d. both (b) and (c)

19. Which of the following terms does not represent a main focus of cost management
information?
a. Usefulness
b. Timeliness
c. Relative accuracy
d. Compliance with external reporting requirements

20. Strategic management can be defined as the development of sustainable:


a. a.chain of command
b. competitive position
c. cash flow
d. business entity

21. The control area of management is primarily concerned with:


a. a.standards and variances
b. monitoring and evaluation

c. structure and discipline


d. organization and implementation
22. Cost management has moved from a traditional role of product costing and operational
control to a broader strategic focus, which places an emphasis on:
a. a.non-competitive pricing
b. domestic marketing
c. short-term thinking
d. integrative thinking

23. Dramatic improvements in communication have resulted in increasing global competition,


which has required firms to:
a. a.completely replace existing cost information systems.
b. expand existing cost information systems.
c. modify existing cost information systems to handle more data.
d. develop cost management systems to handle more data.

24. All the information the manager needs to effectively manage the firm or not-for-profit
organization is termed:
a. a.planning information.
b. b.cost management information.
c. c.financial information.
d. life cycle information.

25. Those who develop cost management information are most often referred to as:
a. a.cost accountants.
b. operational accountants.
c. c.management accountants.
d. d.industrial accountants.

26. The main focus of cost management information must be:


a. a.reliability and usefulness.
b. timeliness and reliability.
c. c.objectivity and reliability.
d. usefulness and timeliness.

27. The development of a sustainable competitive position - understanding what specific


activities are needed for the firm to succeed, and making the appropriate strategic choices - is
termed:
a. strategic cost management.
b. strategic management.
c. total quality management
d. activity-based management.

28. The development of cost information to facilitate the principal management function is
termed:
a. life cycle costing.
b. activity-based costing
c. total quality management
d. strategic cost management

29. The ability to deliver a product or service faster than the competition is termed:
a. just-in-time.
b. statistical quality control.
c. flexible manufacturing.
d. speed-to-market.

30. A set of policies, procedures and approaches to business to produce long-term success is
termed a:
a. critical success factor.
b. competitive position.
c. mission.
d. strategy.

Exercises
Exercise 1: Ethics and the Manager
Raymond Diaz was recently hired as assistant controller of RD Chem Inc., which processes
chemicals for use in fertilizers. Diaz was selected for this position because of his past experience
in chemical processing. During his first month on the job, Diaz made a point of getting to know
the people responsible for the plant operations and learning how things are done at RD Chem.
During a conversation with the plant supervisor, Diaz asked about the company procedures for
handling toxic waste materials. The plant supervisor replied that be was not involved with the
disposal of wastes and suggested that Diaz might be wise to ignore this issue. This response
strengthened Diaz' determination to probe this area further to be sure that the company
was not vulnerable to litigation.
Upon further investigation, Diaz discovered evidence that RD Chem was using a nearby
residential landfill to dump toxic wastes. It appeared that some members of RD Chem’s
management team were aware of this situation and may have been involved in arranging for this
dumping; however, Diaz was unable to determine whether his superior, the controller, was
involved.

Uncertain how he should proceed, Diaz began to consider his options by outlining the following
three alternative courses of action:
 Seek the advice of his superior, the controller.
 Anonymously release the information to the local newspaper.
 Discuss the situation with an outside member of the board of directors with whom he is
acquainted.

Required: For each of the three alternative courses of action that Raymond Diaz has outlined,
explain whether or not the action is appropriate according to the Standards of Ethical Conduct
for Management Accountants.

Exercise 2: Enterprise Risk Management


The table below refers to seven industries.
Required: For each industry, provide an example of a business risk faced by companies that
compete within that industry. Then, describe an example of a control that could be used to reduce
the business risk that you have identified.
Industry Example of Business Risk Example of Control to Reduce the
Business Risk
1.Airlines (e.g., Philippine Airlines)
2.Pharmaceutical drugs (e.g., Pfizer)
3.Package delivery (e.g., LBC)
4. Banking (e.g., BPI)
5. Oil and gas (e.g., Shell)
6. E-commerce (e.g., Lazada)
7. Automotive (e.g., Toyota)

Exercise 3: Ethics in Business

Consumers and lawyers in more than 10 regions accused a prominent nationwide chain of auto
repair shops of misleading customers and selling them unnecessary parts and services, from
brake jobs to front-end alignments.

"In the face of declining revenues, shrinking market share, and an increasingly competitive
market management attempted to spur performance of its auto centers The automotive service
advisers were given product-specific sales quotas-sell so many springs, shock absorbers,
alignments, or brake jobs per shift and paid a commission based on sales [Flailure to meet
quotas could lead to a transfer or a reduction in work hours. Some employees spoke of the
"pressure, pressure, pressure" to bring in sales. This pressure-cooker atmosphere created
conditions under which employees felt that the only way to satisfy top management was by
selling products and services to customers that they didn't really need.

Suppose all automotive repair businesses routinely followed the practice of attempting to sell
customers unnecessary parts and services.

Required:
1. How would this behaviour affect customers? How might customers attempt to protect
themselves against this behaviour?
2. How would this behaviour probably affect profits and employment in the automotive service
industry?
Exercise 4: Ethics in Decision Making
Assume that you are the chairman of the Department of Accountancy at Mountain State
University. One of the accounting professors in your department, Dr. Cruz, has been consistently
and uniformly regarded by students as an awful teacher for more than 10 years. Other accounting
professors within your department have observed Dr. Cruz’s classroom teacher and they concur
that his teaching skills are very poor. However, Dr. Cruz was granted tenure 12 years ago,
thereby ensuring him life-long job security at Mountain State University.

Much to your surprise, today you received a phone call from an accounting professor at
University of Eastern Philippines. During this phone call you are informed that the University is
on the verge of making a job offer to Dr. Cruz. However, before extending the job offer, the
faculty at the University wants your input regarding Dr. Cruz’s teaching effectiveness while at
Mountain State University.

Required: How would you respond to the professor from University of Eastern Philippines?
What would you say about Mr. Cruz’s teaching ability? Would you describe your answer to this
inquiry as being ethical? Why?

Exercise 5: Ethics and Decision Making


Assume that you just completed a December weekend vacation to a resort within the
Entertainment City in Pasay City. During your trip you won P100,000 gambling. When the
casino exchanged your chips for cash they did not record any personal information, such as your
driver's license number or social security number. Four months later while preparing your tax
returns for the prior year, you stop to contemplate the fact that the Bureau of Internal Revenue
requires taxpayers to report all gambling winnings on Form XXX.

Required: Would you report your gambling winnings to the Bureau of Internal Revenue so that
you could pay income taxes on those winnings? Do you believe that your actions are ethical?
Why?

CHAPTER
2
THE PROFESSIONAL
ENVIRONMENT OF
COST MANAGEMENT

EXPECTED LEARNING OUTCOMES

After studying this chapter, you should be able to…

1. Describe the position of the management accountant in the organization structure


of the business firm

2. Explain the role and the relationship between the Chief Financial and the
Controller

3. Describe the functions and responsibilities of the Controller as the top


management accountant

4. Explain the role and the relationship between the Chief Financial Officer and the
Treasurer

5. Describe the functions and responsibilities of the Treasurer

6. Understand the ethical standards for management accountants

7. Realize the need for a company code of conduct

8. Be familiar with typical ethical challenges that management accountants


encounter

9. Describe the international certifications that are available to management


accountants
CHAPTER 2

THE PROFESSIONAL ENVIRONMENT


OF COST MANAGEMENT

ORGANIZATION STRUCTURE AND THE MANAGEMENT ACCOUNTANT

Many of the activities constituting the field of management accounting are interrelated and thus
must be coordinated, ranked and implemented by the management accountant in such a fashion
as to meet the objectives of the organization as perceived by him or her. A major function of the
management accountant is that of tailoring the application of the process to the organization so
that the organization’s objectives, short-term and long-term, are achieved effectively.

Management accounting is intended to include persons involved in such functions as


controllership, treasury, financial analysis, planning, and budgeting, cost accountants thus may
have titles as controller, treasurer, budget analyst, cost analyst, and accountant, among others.

The accounting function is usually “staff”, with responsibility for providing line managers and
also other staff managers, with specialized services. This includes advice and help in the areas of
budgeting, controlling, pricing and special decisions.

Line authority is the authority to command action or give orders to subordinates. Line managers
are directly responsible for attaining the objectives of the business firm as efficiently as possible.
Sales and production managers typically have line authority. Staff authority is the authority to
advise but not command others; it is exercised laterally or upward. Staff managers give support,
advice and service to line departments. Examples of staff authority are found in personnel,
purchasing, engineering and accounting.

Except for exercising line authority over his department, the chief accounting officer usually the
controller generally fills the staff role in his company as contrasted with the line roles of sales
and production executives. Theoretically, the controller transmits the best accounting procedures
to be followed by the line people to the President who will communicate such through a manual
of instructions. In practice however, the controller holds delegated authority from top line
management to direct the line people on how to apply these procedures. This is known as
functional authority which Is the right to command action, laterally or downward, with regard to
a specific function or specialty.

THE CHEF FINANCIAL OFFICER AND THE CONTROLLER.

The chief financial officer (CFO) – also called the financed director in many countries – is the
executive responsible for overseeing the financial operations of an organization. The
responsibilities of the CFO vary among organizations, but they usually include the following
areas:
 Controllership – includes providing financial information for reports to managers and
reports to shareholders and overseeing the overall operations of the accounting system.

 Treasury – includes banking and short-term and long-term financing, investments, and
management of cash.

 Risk management – includes managing the financial risk of interest-rate and exchange-
rate changes and derivatives management.
 Taxation – includes income taxes, sales taxes, and international tax planning.

 Internal Audit – includes reviewing and analyzing financial and other records to attest to
the integrity of the organization’s financial reports and to adherence to its policies and
procedures.

In some organizations, the CFO is also responsible for information systems. In other
organizations, an officer of equivalent rank to the CFO – called the chief information officer – is
responsible for information systems.

The controller (also called the chief accounting officer) is the financial executive primarily
responsible for management accounting and financial accounting. This book focuses on the
controller as the chief management accounting executive. Modern controllers do not do any
controlling in terms of line authority except over their own departments. Yet, the modern concept
of controllership maintains that the controller does control in a special sense. That is, by
reporting and interpreting relevant data (problem-solving and attention-directing roles), the
controller exerts a force or influence the impels management toward making better-informed
decisions.
Figure 2-1 is an illustrative organization chart of the CFO and the corporate controller of an
apparel company

Figure 2-1: Reporting Relationships for the CFO and the Corporate Controller

Chairman Chief Executive Officer Board of Directors


(CEO)

President Chief Operating Officer


(COO)

Chief Financial Officer (CFO)

Controller Treasurer

The controller as the Top Management Accountant


Controllership is the practice of the established science of control which is the process by which
management assures itself that the resources are produced and utilized according to plans in
order to achieve the company’s objectives.
In most organizations, the top managerial position is held by the controller. The controller
provides reports for planning and evaluating company activities (e.g., budgets and performance
reports) and provides the information needed to make management decisions (e.g., decisions
related to construction of a new factory or decisions related to adding or dropping a product).
The controller also has responsibility for all financial accounting reports and tax filing with the
Bureau of Internal Revenue and other taxing agencies, as well as coordinating the activities of
the firm’s external auditors.

A simplified illustration of the organization chart for the controller’s office is shown in Figure 2-
2. Note that one of the areas reporting to the controller is cost accounting. Most medium-sized
and large manufacturing companies have such a department. Cost accountants estimate costs to
facilitate management decisions and develop cost information for purposes of valuing inventory.

The controlle4r is an internal part of the top management team. If one wants a high-level career
in management accounting, he/she will need not only strong accounting skills but also skills
required of all high-level executives. These skills include excellent written and oral
communication skills, solid interpersonal skills and a deep knowledge of the industry in which
the firm competes.

The controller’s authority is basically staff authority in that the controller’s office gives advice
and service to other departments. However, in his own department, he has line authority. In the
modern concept of controllership, it is maintained that the controller does control in a special
sense. That is, by reporting and interpreting relevant data, the controller exerts a force or
influence that impels management toward logical decisions consistent with objectives.

Figure 2-2: A Typical Organization Chart Showing the Functions of the Controller

Controller

Budgeting and Financial Cost


Financial
Performance Analysis and Managemen
Reporting Special Studies
Reporting

Systems Taxation
Developments Reporting
Basic Functions of Controllership
The basic principal functional responsibilities and activities of controllership may be organized
as follows:
1. Planning. Establish and maintain an integrated plan of operation consistent with the
company’s goals and objectives, both short and long term, analyzed and revised, as
required, communicated to all levels of management, with appropriate systems and
procedures installed.
2. Control. Develop and revise standards against which to measure performance and
provide guidance and assistance to other members of management in insuring
conformance of actual results to standards.
3. Reporting. Prepare, analyze, and interpret financial results for utilization by
management in the decision-making process, evaluate the data with reference to
company and unit objectives; prepare and file external reports as required to satisfy
government regulatory bodies, shareholders, financial institution, customers, and
general public
4. Accounting. Design, establish, and maintain general and cost accounting systems at all
company levels, including corporate, divisional, plant, and unit to properly record all
financial transactions in the books of accounts and records in accordance with sound
accounting principles with adequate internal control.
5. Other Primary Responsibilities. Manage and supervise such functions as taxes,
including interface with the respective taxing authorities and agents; maintain
appropriate relationships with internal and external auditors; develop and maintain
systems and procedures; develop record retention programs; supervise assigned
treasury functions; institute investor and financial public relations programs; office
management; and direct other assigned functions.

As circumstances warrant, there may be many deviations from the basic functions just described.
It should be pointed out that the controller’s efforts should not be diluted and render less
effective by assigning to him unrelated functions of an operational nature. The financial planning
and control functions are too important to the success of the business to burden the controlle4r
with activities that others can perform.

Qualification of the Controller

The qualifications of an effective controller would include:


1. An excellent technical foundation in accounting and finance with an understanding and
through knowledge of accounting principles.
2. An understanding of the principles of planning, organizing, and control.
3. A general understanding of the industry in which the company competes and the social,
economic, and political forces involved.
4. A thorough understanding of the company, including its technologies, products, policies,
objectives, history, organization, and environment.
5. The ability to communicate with all levels of management and a basic understanding of
other functional problems related to engineering, production, procurement, industrial
relations, and marketing.
6. The ability to express ideas clearly in writing or in making informative presentations.
7. The ability to motivate others to achieve positive action and results.

The controller may have the technical capability and be able to lay out the assigned tasks as well
as supervise and direct his personnel, but he must also have integrity and the ability to
communicate if he is to succeed, he must be fair, reasonable, and sincere with all concerned if he
is to be recognized for the importance of the controllership function.

As in any executive position, the controller must be able to work with people at all levels, have
respect for the ideas and opinions of others, and have the resourcefulness, to meet all challenges.

THE CHIEF FINANCIAL OFFICER AND THE TREASURER

Although organizational structures vary from firm of firm, the role of finance is assigned to the
Chief of Financial Officer (CFO) or the Vice President-Finance who reports to the President.

The financial vice-president’s key subordinates are the Treasurer and the Controller. This book
has extensively dealt with the role of the Controller in the previous section.

Treasureship

Treasureship is concerned with the acquisition, financing and management of assets of a


business concern to maximize the wealth of the firms for its owners.

In addition to the position of the controller, many companies have a position called treasurer.
The treasurer has custody of cash and funds invested in various marketable securities. In
addition to money management duties, the treasurer is generally responsible for maintaining
relationships with investors, banks, and othe3rcreditors.thus, the treasurer plays a major role in
managing cash and marketable securities, preparing cash forecasts and obtaining financing from
banks and other lenders. Both the controller and the treasurer report to the chief financial officer
(CFO) who is the senior executive responsible for both accounting and financial operations.

In most firms the treasurer has the following responsibilities:


1. Funds Procurement
This involves raising of funds in accordance with the firms planned capital structure. This
responsibility many require negotiating for loans, short-term or long-term, issuing equity of debt
instruments at the best term and conditions possible.

2. Banking and Custody of Funds


This involves direct management of cash and cash equivalents and maintenance of good relations
with banks and other non-bank institution.

3. Investment of Funds
This involves management of the company’s placements and securities or purchase of debt or
equity instruments such as ordinary or preference shares in other corporate entities. This
responsibility also includes analysis of decisions related to investment in property, plant and
equipment.

4. Operating Responsibilities related to


(a) Credit and Collection
(b) Inventory Management
(c) Corporate Pension and Retirement Fund
(d) Investor Relations
(e) Insurance
(f) Compliance with legal and regulatory provisions relating to funds procurement, use
and distribution as well as coordination of the finance function with accounting
function.

ETHICAL STANDARDS FOR MANAGEMENT ACCOUNTANTS

In recent years, many concerns have been raised regarding ethical behavior in business and in
public life. Allegations and scandals of unethical conduct have been directed toward managers in
virtually all segments of society, including government, business, charitable organizations, and
even religion. Although these allegations and scandals have received a lot of attention, it is
doubtful that they represent a wholesale breakdown of the moral fiber of the nation. After all,
hundreds of millions of transactions are conducted every day that remain untainted.
Nevertheless, it is important to have an appreciation of what is and is not acceptable behavior in
business and why. Fortunately, the institute of Management Accountants (IMA) of the United
States has developed a very useful ethical code called the Standards of Ethical Conduct for
Practitioners of Management Accounting and Financial Management. Even though the
standards were specifically developed for management accountants, they have much broader
application.
Code of Conduct for Management Accountants

The institute of Management Accountants (IMA) issued the Standards of Ethical Conduct for
Practitioners of Management Accounting and Financial Management. These standards are
presented in Figure 2-3. There are two parts to the standards. The first part provides general
guidelines for ethical behavior. In a nutshell, the management accountant has ethical
responsibilities in four broad areas namely

1. To maintain a high level of professional competence,

2. To treat sensitive matters with confidentiality,

3. To maintain personal integrity, and

4. To be objective in all disclosing.

The second part of the standards gives specific guidance what should be done if an individual
finds evidence of ethical misconduct within an organization.

The ethical standards provide sound, practical advice for management accountants and
managers. They require professional behavior, especially in avoiding conflicts of interest. They
require management accountants to bring bad news to the attention of their supervisors, and to
work competently.

Most of the rules in the ethical standards are motivated by a very practical consideration – if
these rules were not generally followed in business, then the economy could come to a halt. The
following are examples of the consequences of not abiding by the standards:

1. Suppose employees could not be trusted with confidential information. Top managers
would therefore be reluctant to distribute confidential information within the company.
This could result to decisions being made based on incomplete information and could
lead to deterioration of operations.

2. Suppose employees accept bribes from suppliers. Then contracts would tent to go to
suppliers who pay the highest bribe rathe3r than to the most competent suppliers. Would
you like to fly in an airplane whose wings were made by the subcontractor who was
willing to pay the highest bribe to a purchasing agent?

3. Suppose the CEOs or president of companies routinely lied in their annual reports to
shareholders and grossly distorted financial statements. If the basic integrity of the
company’s financial statement could not be relied on, investors and creditors would have
little basis for making informed decisions. Rational investors would suspect the worst and
would pay less for securities issued by companies. As a result, less funds would be
available for productive investments and many firms might be unable to raise any funds
at all. This ultimately, would lead to slower economic growth, fewer goods and services,
and higher prices.

As these examples suggest, it ethical standards were not generally adhered to, there would be
understandable consequences for everyone. Following ethical rules such as those in the
Standards of Ethical Conduct for Practitioners of Management Accounting and Financial
Management is not just a matter of being “nice”, it is absolutely essential for the smooth
functioning of an advanced market economy.

Figure 2-3: Standards of Ethical Conduct for Practitioners of Management Accounting and
Financial Managements

Practitioners of management accounting and financial management have an obligation to the


public, their profession, the organization they serve, and themselves, to maintain the highest
standards of ethical conduct. In recognition of this obligation, the institute of Management
Accountants has promulgated the following standards of ethical conduct for practitioners of
management accounting and financial management. Adherence to these standards, both
domestically and internationally, is integral to achieving the Objectives of Management
Accounting. Practitioners of management accounting and financial management shall not
commit acts contrary to these standards nor shall they condone the commission of such acts by
others within their organizations.

Competence. Practitioners of management accounting and financial management have a


responsibility to:

 Maintain an appropriate level of professional competence by ongoing development of


their knowledge and skills.
 Perform their professional duties in accordance with relevant laws, regulations, and
technical standards.
 Prepare complete and clear reports and recommendations after appropriate analysis of
relevant and reliable information.

Confidentiality. Practitioners of management accounting and financial management have a


responsibility to:

 Refrain from disclosing confidential information acquired in the course of their work
except when authorized, unless legally obligated to do so.
 Inform subordinates as appropriate regarding the confidentiality of information acquired
in the course of their work and monitor their activities to assure the maintenance of that
confidentiality.
 Refrain from using or appearing to use confidential information acquired In the course of
their work for unethical or illegal advantage personality or through third parties.

Integrity. Practitioners of management accounting and financial management have a


responsibility to:
 Avoid actual or apparent conflicts of interest and advise all appropriate parties of any
potential conflict.
 Refrain from engaging In any activity that would prejudiced their ability to carry out their
duties ethically.
 Refuse any gift, favor, or hospitality that would influence or would appear to influence
their actions.
 Refrain from either actively or passively subverting the attainment of the organization’s
legitimate and ethical objectives.
 Recognize and communicate professional limitations or other constraints that would
preclude responsibility judgement or successful performance of an activity.
 Communicate unfavorable as well as favorable information and professional judgements
or opinions.
 Refrain from engaging in or supporting any activity that would discredit the profession.

Objectivity. Practitioners of management accounting and financial management have a


responsibility to:

 Communicate information fairly and objectively.


 Disclose fully all relevant information that could reasonably be expected to influence an
intended user’s understanding of the reports, comments, and recommendations presented.

Resolution of Ethical Conflict. In applying the statement of ethical conduct, practitioners of


management accounting and financial management may encounter problems in identifying
unethical behavior or in resolving an ethical conflict. When faced with significant ethical issues,
practitioners of management accounting and financial management should follow the established
policies of the organization bearing on the resolution of such conflict. If these policies do not
resolve the ethical conflict, such practitioner should consider the following courses of action:

 Discuss such problems with the immediate superior except when u tappers that the
superior is involved, in which case the problem should be presented initially to the next
higher managerial level. If a satisfactory resolution cannot be achieved when the problem
is initially presented, submit the issues to the next higher managerial level.
 If the immediate superior is the chief executive, or equivalent, the acceptable reviewing
authority may be a group such as the audit committee, executive committee, board of
directors, board of trustees, or owners. Contact with levels above the immediate supe4rior
should be initiated only with the superior’s knowledge, assuming the superior is not
involved. Except where legally prescribed, engaged by the organization is not considered
appropriate.
 Clarify relevant ethical issues by confidential discussion with an objective advisor (e.g.,
IMA Ethics Counseling Service) to obtain a better understanding of possible courses of
action.
 Consult on your own attorney as to legal obligations and rights concerning the ethical
conflict.
 If the ethical conflict still exists after exhausting all levels of internal review, there may
be no other recourse on significant matters than to resign from the organization and to
submit an informative memorandum to an appropriate representative of the organization.
After resignation, depending on the nature of the ethical conflict, it may also be
appropriate to notify other parties.

*institute of Management Accountants, formerly National Association of Accountants,


Statements on Management Accounting: Objectives of Management Accounting, Statement No.
1B, New York, NY, June 17, 1982 as revised in 1997.

COMPANY CODE OF CONDUCT

Ethical standards serve a very important practical function in advanced market economy.
Without widespread adherence to ethical standards, material living standards would fall. A
former president of CMA emphasizes the importance of ethics in business:

“Employees like to work for a company that they can trust. Customers like to deal with an
ethically reliable business. Suppliers like to sell to firms with which they can have a real
partnership. Communities are more likely to cooperate with organizations that deal honestly and
fairly with them. If the business community is to functions effectively, all of the players need to
act ethically.”

It is unfortunate though, that some companies place so much emphasize on short-term profits
that may make it seem like the only way to get ahead is to act unethically.
Those who engage in unethical behavior often justify their actions with one or more of the
following reasons:

(1) The organization expects unethical behavior,

(2) Everyone else is unethical, and/or

(3) Behaving unethically is the only way to get ahead.

To counter the first justification for unethical behavior, many companies have adopted formal
ethical codes of conduct. These codes are generally broad-based statements of a company’s
responsibilities to its employees, its customers, its suppliers and the community in which the
company operates. Codes give broad guidelines rather than that spell out specific do’s and don’ts
or suggest proper behavior in a specific situation. Companies with a strong code of ethics can
create strong customer and employee loyalty. While liars and cheats may win on occasion, their
victories are often short-term. Companies in business for the long-term find that it pays to treat
all of their constituents and loyalty.

TYPICAL ETHICAL CHALLENGES


Ethical issues can confront management accountants in many ways. Here are two example:

 Case A. Roger Cruz, a management accountant, knows that reporting a loss for a
software division will result in yet another series of layoffs, and has concerns about the
commercial potential of software for which R&D costs are currently being capitalized as
an asset rather than being shown as an expense for internal reporting purposes. The
division manager argues that the new product will be successful and profitable but
presents little evidence to support her argument. The last two products from this division
have been unsuccessful. The management accountant has many friends in the division
and wants to avoid a personal confrontation with the division manager.

 Case B: A packaging supplier, bidding for a new contract, offers the management
accountant of the purchasing company an all-expense paid weekend to the Boracay
Resort. The supplier does not mention the new contract when giving the invitation. The
accountant is not a personal friend of the supplie4r. he knows cost issues are critical in
approving the new contract and is concerned that the supplier will ask for details about
bids by competing packaging companies.

In both cases, the management accountant is faced with an ethical dilemma. Case A involves
competence, objectivity, and integrity. The management accountant should request that the
division manager provide credible evidence that the new product is commercially viable. If the
manager does not provide such evidence, expensing R&D costs in the current period is
appropriate. Case B involves confidentiality and integrity. Ethical issues are not always clear-cut.
The supplier in Case B may have no intention of raising issues associated with the bid. However,
the appearance of a conflict of interest in Case B is sufficient for many companies to prohibit
employees from accepting “favors” from suppliers. Figure 2-3 includes the IMA’s guidance on
“Resolution of Ethical Conflict”, the accountant in Case B should discuss the invitation with his
immediate supervisor. If the visit is approved, the supplier should be informed that the invitation
has been officially approved subject to his following corporate policy (which includes the
confidentiality of information).

CODES OF CONDUCT ON THE INTERNATIONAL LEVEL

In July 1990, the International Federation of Accountants (IFAC) in which the Philippines
through the PICPA is a member, issued the “Guidelines on Ethics for Professional Accountants”
which governs the activities of all professional accountants throughout the world; regardless of
whether they are practicing as independent CPAs. Employed in government service or employed
as internal accountants. In addition to outlining ethical requirements in matters dealing with
competence, objectivity, independence, and confidentiality, the IFAC’s code also outlines the
accountant’s ethical responsibilities in matters relating to taxes, fees and commissions,
advertising and solicitation, the handling of monies and cross-border activities. Where cross-
border activities are involved, the IFAC ethical requirements must be followed if these
requirements are stricter than the ethical requirements of the country in which the work is being
performed.
The Board of Accountancy of the Professional Regulation Commission approved the
implementation of the Revised Code of Ethics for Professional Accountants in the Philippines
effective January 1, 2016.

INTERNATIOLNAL CERTIFICATIONS

The three certifications available to management accountants are so follows:

 Certificate of Management Accounting (CMA)


 Certificate in Public Accounting (CPA)
 Certificate in Internal Auditing (CIA)

CMA. A Certified Management Accountant is one who has passed the rigorous qualifying
examination, has met an experience requirement, and participates in continuing educations. The
CMA Certificate is graduated by the Institute Management Accountants (IMA)

CPA. A Certified Public Accountant is one who has met the pre-qualification educational
requirements, passed the CPA licensure examinations given by the Professional Regulatory
Board of Accountancy and has satisfied all other legal and regulatory requirements of a public
accountant. The CPAs main responsibility is to provide assurance concerning the reliability of
the information contain in the firm’s financial statements.

CIA. Since one of the management control responsibilities of the management accountant is to
develop effective systems to detect and prevent errors and fraud in the accounting records, it is
common for the management accountant to have strong ties to the control-oriented organization
such as the Institute of Internal Auditors (JIA) granting Certification in Internal Auditing (CIA).
To attain the status of Certified Internal Auditor an individual must pass a comprehensive
examination designed to ensure technical competence and have the required number of years of
work experience.

INSTITUTE OF MANAGEMENT ACCOUNTANTS (IMA)

Management accountants have gained status in recent years as they now spend more time
analyzing a company’s operations and less with the problems pf recording and computing costs
of products. The Institute of Management Accountants (IMA), the principal organization of
management accountants in the United States, has instituted a program to provide certifications
for management accountants and financial managers. The Certified Management Accountant
(CMA) examination was first given in 1972. A listing of the required subject areas in the CMA
examination indicates the breadth of knowledge expected of the professional management
accountant. The examination consists of the following four parts: Economics, Finance, and
Management; Financial Accounting and Reporting; Management Reporting, Analysis and
Behavioral Issues; and Decision Analysis and Information Systems. The Certified in Financial
Management (CFM) examination was first given in 1996. The CFM examination is similar to the
CMA examination with one major difference: The Financial Accounting and Reporting section is
replaced with Corporate Financial Management. The IMA also promulgated a code of ethics for
management accountants, which is discussed in the previous section.

The Institute of Management Accounting (IMA) is a professional organization that publishes the
monthly magazine Strategic Finance. Since 1973, the IMA has conducted a comprehensive
examination to test the knowledge a management accountant must have to be successful in a
complex and fast-changing business world. More than 3,000 individuals take the exam each year.
Those who pass the exam are issued a Certificate in Management Accounting and are proud to
indicate the designation CMA on resumes and business cards. For details in student and
professional membership in the IMA and for information on the CMA examination, visit the
IMA Web site.

One of the contributions of the IMA is the development of standards of ethical conduct and
maintenance of an ethics hotline that members can call to discuss ethical conflicts. One mat also
Visit the IMA website to review these ethical standards.

PHILIPPINE ASSOCIATION OF MANAGEMENT ACCOUNTANTS (PAMA)

PAMA was established in 1972 as the National Association of Accountants (NAA) Philippine
Chapter, Inc. it is affiliated with NAA in New York. It was founded primarily to provide its
members with educational and professional activities that supplement in the knowledge of
management accounting practices and methods. Monthly technical meetings, seminars and
workshops are held to present relevant and current topics by leading speakers from the
government, private and educational sectors. The open forum provides the nerve for the
exchange of ideas and experiences among the participants and the speakers. Publication of
technical materials is also part of the Association’s efforts to service its members.

To propagate and professionalize Management Accounting in the Philippines, PAMA conducts


the Certificate in Management Accounting (CMA) Program through its confusing education arm,
the Philippine Institute of Management Accounting (PIMA). Basic objectives of the program are:

1. To establish management accounting as a recognized profession by identifying the role of


the management accountant and the underlying body of knowledge, and by outlining a
course of study by which such knowledge can be acquired.

2. To foster higher educational standards in the field of management accounting

3. To assist employees, educators and students by establishing an objective measure of an


individuals’ knowledge and competence in the field of management accounting.
REVIEW QUESTIONS AND EXERCISES

QUESTIONS

1. Name the three international professional certifications available to management


accountant.

2. What type of professional certification is most relevant for the management accountant
and why?

3. Which is the most important function of management, and why?

4. What roles do management accountants perform?

5. Where does the management accounting function fit into an organization’s structure?

6. What guidelines do management accountants use?

7. What are the ethical responsibilities of accountants?

8. “Planning is really more vital than control”. Do you agree? Why?

9. “The controller is both a line and a staff executive”. Do you agree? Why?

10. Prepare an organization chart (highlighting the accounting functions) of Bettina


Company, which has the following positions:

a. VP, Sales
b. Internal audit manager
c. Treasurer
d. Payroll clerk
e. General accounting manager
f. Accountants receivable clerk
g. Budget and standard cost analyst
h. Controller
i. Cost systems analyst
j. Special studies manager
k. Assistant controller
l. Accounts payable clerk
m. General ledger bookkeeper
n. Performance analyst
o. Tax manager
p. Cost accounting manager
q. Cost clerk
r. Billing clerk
s. VP, finance
t. Systems and EDP Manager
u. VP, production
v. Assistant manager
w. President

11. How does a controller help “control” a company?

12. Discuss the potential behavior implications of management accounting.

Exercises

Exercises 1 (Problem Solving, Scorekeeping, and Attention Directing)

For each of the following activities, identify the main role the accountant is performing –
problem solving, scorekeeping, or attention directing.

1. Preparing a monthly statement of Australian sales for the IBM marketing vice
president.
2. Interpreting differences between actual results and budgeted amounts on a
performance report for the Customer Warranty Department of General Electric.
3. Preparing a schedule of depreciation for forklift trucks in the Receiving Department
of a Hewlett-Packard plant in Scotland.
4. Analyzing, for a Mitsubishi international manufacturing manager, the desirability of
having some auto parts made in Korea.
5. Interpreting why a Birmingham distribution center did not adhere to its delivery costs
budget.
6. Explaining a Xerox Shipping Department’s performance report.
7. Preparing, for the manager of production control of a U.S steel plant, a cost
comparison of two computerized manufacturing control systems.
8. Preparing a scrap report for the Finishing Department of a Toyota parts plant.
9. Preparing the budget for the Maintenance Department of Mount Sinai Hospital.
10. Analyzing, for a General Motors product designer, the impact on product costs of
some new headlight lamps.

Exercise 2 (Management Accounting Information System)

The items that follow are associated with a management accounting information system.

a. Repairing a defective part


b. Providing information for planning and control
c. Designing a product
d. Measuring the cost of design
e. A budget that shows how much should be spent on design activity
f. Using output information to make a decision.
g. Usage of materials
h. A report comparing the actual costs of quality with the expected costs of quality
i. Surveying customers to assess post purchase costs
j. Insurance of post purchase costs
k. Costing out products
l. Assigning the cost of labor to a product
m. Report showing the cost of a product
n. Measuring the cost of quality

Required:
Classify the items into one of the following categories:

1. Inputs
2. Processes
3. Outputs
4. System objectives

Exercises 3 (Role of Management Accountants)

Management accountants are actively involved in the process of managing the entity. This
process includes making strategic, tactical, and operating decisions while helping to coordinate
the efforts of the entire organization. To fulfill these objectives, the management accountant
accepts certain responsibilities that can be identified as (1) planning, (2) controlling, (3)
evaluating performance, (4) ensuring accountability of resources, and (5) external reporting.

Required:
Described each of these responsibilities of the management accountant and identify examples of
practices and techniques.

Exercise 4 (Line Versus Staff)

The job responsibilities of two employees of Boots Manufacturing follow.

Jamie Reyes, Cost Accounting Manager. Jamie is responsible for measuring and collecting costs
associated with the manufacture of the garden hose product line. She is also responsible for
preparing periodic reports comparing the actual costs with planned costs. These reports are
provided to the production line managers and the plant manager. Jamie helps explain and
interpret reports.

Stephen Santos, Production Manager. Stephen is responsible for the manufacture of the high-
quality garden hose. He supervises the line workers, helps develop the production schedule, and
is responsible for seeing that production quotas are met. He is also held accountable for
controlling manufacturing costs.

Required:
Identify Jamie and Stephen as line or staff and explain your reasons.

Exercise 5 (Professional Ethics and End-of-Year Games)

Jane Tan is the new division controller of the snack foods division of Yummy Foods. Yummy
Foods has reported a minimum 15% growth in annual earnings for each of the past 5 years. The
snack foods division has reported annual earnings growth of over 20% each year in this same
period. During the current year, the economy went into a recession. The corporate controller
estimates a 10% annual earnings growth rate for Yummy Foods this year. One month before the
December 31 fiscal year-end of the current year, Tan estimates the snack foods division will
report an annual earnings growth of only 8 percent. Louie Ryan. The snack foods division
president, is less than happy, but he says with a wry smile, “Let the end-of-year games begin”.

Tan makes some inquiries and is able to compile the following list of end-of-year games that
were more-or-less accepted by the previous division controller.

a. Deferring routine monthly maintenance in December on packaging equipment by


an independent contractor until January of next year.
b. Extending the close of the current fiscal year beyond December 31 so that some
sales of next year are included in the current year.
c. Altering dates of shipping documents of next January’s sales to record them as
sales in December of the current year.
d. Giving salespeople a double bonus to exceed Decembe4r sales targets.
e. Deferring the current periods advertising by reducing the number of television
sports run in December and running more than planned in January of next year.
f. Deferring the current period’s reported advertising costs by having Yummy Foods
outside advertising agency delay billing December advertisements until January
of next year or having the agency alter invoices to conceal the December date.
g. Persuading carriers to accept merchandise for shipment in December of the
current year although they normally would not have done so.

Required:

1. Why might the snack foods division president want to play the end-of-year games
described above?

2. The division controller is deeply troubled and reads the Standards of Ethical
Conduct for Management Accountants. Classify each of the end-of-year games
(a-g) as (i) acceptable, or (ii) unacceptable according to that document.

3. What should Tan do if Ryan suggests that end-of-year games are played in every
division of
Yummy Foods and that she would greatly harm the snack foods division if she
does not play along and paint the rosiest picture possible of the division’s results?
Exercise 6 (You get what you measure!)

Each year, the president of Quark Electronics selects a single performance measure, and offers
significant financial bonuses to all key employees if the company achieves a 10 percent
improvement on the measure in comparison to the prior year. He recently expressed the opinion
that "this focuses my managers on a single, specific target and gets them all working together to
achieve a major objective that will increase shareholder value."

Pilar Hernandez is a new member of the company's board of directors, and she has begun to
question the president's approach to rewarding performance. In particular, she is concerned that
placing too much emphasis on a single performance measure may lead managers to take actions
that increase performance in terms of the measure but decrease the value of the firm. Is this
possible?

Required:

a. What negative consequence might occur if the performance measure is sales to


new customers + total sales in the current year versus the prior year? (Note: To
receive a bonus, managers would need to increase this ratio compared with the
prior year.)

b. What negative consequence might occur if the performance measure is cost of


goods sold (divide) sales in the current year versus the prior year? Note: To
receive a bonus, managers would need to decrease this ratio compared with the
prior year.)

c. What negative consequence might occur if the performance measure is selling


and administrative expense (divide) sales in the current year versus the prior year?
(Note: To receive a bonus, managers would need to decrease this ratio compared
with the prior year.)

Exercise 7 (The Roles of Managers Accountants)

Listed below are a number of terms that relate to organizations, the work of management, and the
role of managerial accounting:

Budgets Line
Chief Financial Officer Managerial accounting
Controller Nonmonetary data
Decentralization Performance report
Directing and motivating Planning
Feedback Precision
Financial accounting Staff

Choose the term or terms above that most appropriately complete the following statements.

1. ________________ is concerned with providing information for the use of those who
are inside the organization, whereas _____________is concerned with providing
information for the use of those who are outside the organization.

2. ________________ consists of identifying alternatives, selecting from among the


alternatives the one that is best for the organization, and specifying what actions will
be taken to implement the chosen alternative.

3. When _______________, managers oversee day-to-day activities and keep the


organization functioning smoothly.

4. The accounting and other reports coming to management that are used in controlling
the organization are called ______________.

5. The delegation of decision-making authority throughout an organization by allowing


managers at various operating levels to make key decisions relating to their area of
responsibility is called ______________

6. A position on the organization chart that is directly related to achieving the basic
objectives of an organization is called a ________________ position.

7. A _____________ position provides service or assistance to other parts of the


organization and does not directly achieve the basic objectives of the organization.

8. The manager in charge of the accounting department is generally known as the


______________

9. The plans of management are expressed formally in _____________

10. A detailed report to management comparing budgeted data to actual data for a
specific time period is called a _____________

11. The ____________ is the member of the top management team who is responsible for
providing timely and relevant data to support planning and control activities and for
preparing financial statements for external users.

12. Managerial accounting places less emphasis on and more emphasis on


_____________ than financial accounting.
Exercise 8 (Ethics in Business)

Maria Reyes was hired by a popular fast-food restaurant as an order-taker and cashier. Shortly
after taking the job, she was shocked to overhear an employee bragging to a friend about
shortchanging customers. She confronted the employee who then snapped back: "Mind your own
business. everyone does it and the customers never miss the money." Maria didn't know how to
respond to this aggressive stance.

Required:

What would be the practical consequences on the fast-food industry and on consumers if cashiers
generally shortchanged customers at every opportunity?

Exercise 9 (Ethics and the Manager)

Happyville, Inc., operates a chain of department stores located in the northwest. The first store
began operations in 1965, and the company has steadily grown to its present size of 44 stores.
Two years ago, the board of directors of Happyville approved a large-scale remodeling of its
stores to attract a more upscale clientele.

Before finalizing these plans, two stores were remodeled as a test. Lisa Perez, assistant
controller, was asked to oversee the financial reporting for these test stores, and she and other
management personnel were offered bonuses based on the sales growth and profitability of these
stores. While completing the financial reports, Perez discovered a sizable inventory of outdated
goods that should have been discounted for sale or returned to the manufacturer. She discussed
the situation with her management colleagues; the consensus was to ignore reporting this
inventory as obsolete, since reporting it would diminish the financial results and their bonuses.

Required:

1. According to the Standards of Ethical Conduct for Practitioners of Management


Accounting and Financial Management, would it be ethical for Perez not to report the
inventory as obsolete?

2. Would it be easy for Perez to take the ethical action in this situation?
Exercise 10 (Preparing an Organization Chart)

Verona University is a large private school located in Mountain Province. The " university is
headed by a president who has five vice presidents reporting to him. These vice presidents are
responsible for auxiliary services, admissions and records, academics, financial services
(controller), and physical plant.

In addition, the university has managers who report to these vice presidents.
These include managers for central purchasing, the university press, and the university
bookstore, all of whom report to the vice president for auxiliary services; managers for computer
services and for accounting and finance, who report to the vice president for financial services;
and managers for grounds and custodial services and for plant and maintenance, who report to
the vice president for physical plant.

The university has four colleges - business, humanities, fine arts and engineering and
quantitative methods - and a law school. Each of these units has a dean who is responsible to the
academic vice president. Each college has several departments.

Required:

1. Prepare an organization chart for Verona University.

2. Which of the positions on your chart would be line positions? Why would they be
line positions? Which would be staff positions? Why?

3. Which of the positions on your chart would have need for accounting information?
Explain.

Exercise 11 (Ethics in Business)

Pedro Santos is the controller of a corporation whose stock is not listed on a national stock
exchange. The company has just received a patent on a product that is expected to yield
substantial profits in a year or two. At the moment, however, the company is experiencing
financial difficulties; and because of inadequate working capital, it is on the verge of defaulting
on a note held by its bank.

At the end of the most recent fiscal year, the company's president instructed Santos not to record
several invoices as accounts payable. Santos objected since the invoices represented bona fide
liabilities. However, the president insisted that the invoices not be recorded until after year-end,
at which time it was expected that additional financing could be obtained. After several very
strenuous objections - expressed to both the president and other members of senior management
- Santos finally complied with the president's instructions.

Required:
1. Did Santos act in an ethical manner? Explain fully.

2. If the new product fails to yield substantial profits and the company becomes
insolvent, can Santos' actions be justified by the fact that he was following orders
from a superior? Explain.

Multiple Choices

1. ____________ means reporting and interpreting information that helps managers to focus
on operating problems, imperfections, inefficiencies, and opportunities.

a. Scorekeeping
b. Attention directing
c. Problem solving
d. None of the above

2. Management accounting is considered successful when it

a. helps creditors evaluate the company's performance.


b. helps managers improve their decisions.
c. is accurate.
d. is relevant and reported annually.

3. The Institute of Management Accountants (IMA)

a. is a professional organization of management accountants.


b. is a professional organization of financial accountants
c. issues standards for management accounting
d. issues standards for financial accounting.

4. Line management includes.

a. manufacturing managers
b. human-resource managers
c. information-technology managers
d. management-accounting managers.

5. Staff management includes

a. manufacturing managers
b. human-resource managers
c. purchasing managers.
d. distribution managers.
6. Responsibility of a CFO include all EXCEPT

a. providing financial reports to shareholders


b. managing short-term and long-term financing
c. investing in new equipment
d. preparing tax returns.

7. The Standards of Ethical Conduct for management accountants include concepts related
to

a. competence, performance, integrity, and reporting


b. competence, confidentiality, integrity and objectivity
c. experience, integrity, reporting, and objectivity.
d. none of the above as ethical issues do not affect management accountants.

8. Ethical challenges for management accountants include

a. whether to accept gifts from suppliers, knowing it is an effort to indirectly influence


decisions.
b. whether to report unfavorable department information that may result in unfavorable
consequences for a friend.
c. whether to file a tax return this year.
d. both (a) and (b).

9. If a financial manager/management accountant has a problem in identifying unethical


behavior or resolving an ethical conflict, the first action (s)he should normally take is to

a. consult the board of directors.


b. discuss the problem with his/her immediate superior
c. notify the appropriate law enforcement
d. resign from the company.

10. Katrina is a financial manager who has discovered that her company is violating
environmental regulations. If her immediate superior is involved, her appropriate action
is to

a. do nothing since she has a duty of loyalty to the organization.


b. consult the audit committee.
c. present the matter to the next higher managerial level.
d. confront her immediate superior.

11. If financial manager/management accountant discovers unethical conduct in his/her


organization and fails to act, (s)he will be in violation of which ethical standard(s)?

a. "Actively or passively subvert the attainment of the organization's legitimate and


ethical objectives."
b. "Communicate unfavorable as well as favorable information."
c. "Condone the commission of such acts by others within their organizations."
d. All of the answers are correct.

12. Corporate social responsibility is

a. effectively enforced through the controls envisioned by classical economics.


b. the obligation to shareholders to earn a profit.
c. the duty to embrace service to the public interest.
d. the obligation to serve long-term, organizational interests.

13. A common argument against corporate involvement in socially responsible behavior is


that

a. it encourages government intrusion in decision making


b. as a legal person, a corporation, is accountable for its conduct
c. it creates goodwill.
d. In a competitive market, such behavior incurs costs that place the company at a
disadvantage.

14. Integrity is an ethical requirement for all financial managers/ management accountants.
One aspect of integrity requires

a. performance of professional duties in accordance with applicable laws


b. avoidance of conflict of interest.
c. refraining from improper use of inside information.
d. maintenance of an appropriate level of professional competence.
15. A financial manager/management accountant discovers a problem that could mislead
users of the firm's financial data and has informed his/her immediate superior. (S)he
should report the circumstances to the audit committee and/or the board of directors only
if

a. the immediate superior, who reports to the chief executive officer, knows about the
situation but refuses to correct it
b. the immediate superior assures the financial manager/management accountant that the
problem will be resolved.
c. the immediate superior reports the situation to his/her superior.
d. the immediate superior, the firm's chief executive officer, knows about the situation
but refuses to correct

16. In which situation is a financial manager/management accountant permitted to


communicate confidential information to individuals or authorities outside the firm?

a. There is an ethical conflict and the board has refused to take action.
b. Such communication is legally prescribed.
c. The financial manager/management accountant knowingly communicates the
information indirectly through a subordinate.
d. An officer at the financial manager/management accountant's bank has requested,
information on a transaction that could influence the firm's stock price.

17. Which ethical standard is most clearly violated if a financial manager/management


accountant knows of a problem that could mislead users but does nothing about it?

a. Competence
b. Legality
c. Objectivity
d. Confidentiality

18. produces information that helps workers, managers, and executives in organizations make
better decisions.

a. Governmental accounting
b. Management accounting
c. Auditing
d. Financial accounting

19. is the recognition and evaluation of business transactions and other economic events for
appropriate accounting action.

a. Identification
b. Analysis
c. Communication
d. Evaluating

20. ____________ is the quantification of business transactions or other economic events


that have occurred or forecasts of those that may occur

a. Accumulation
b. External reporting
c. Measurement
d. Internal reporting

21. ____________ is a determination of the reasons for the reported activity and its
relationship with other economic events and circumstances.

a. Analysis
b. Measurement
c. Evaluation
d. Accumulation

22. includes strategic, tactical and operating aspects


a. Controlling
b. Communication
c. Planning
d. Evaluating

23. _____________ judges implications of historical and expected events nd helps to choose
the optimum course of action.

a. Controlling
b. Communication
c. Planning
d. Evaluating

24. Which of the following is a basic feature of a financial accounting system?

a. Internal audience
b. Historical data
c. Subjective information
d. Disaggregate information

25. Which of the following is NOT a basic feature of a financial accounting system?

a. objective information
b. reports on past performance
c. future oriented reports
d. highly aggregated data

26. Which of the following is a basic feature of a managerial accounting system?

a. external audience
b. reports are current and future oriented
c. objective data only
d. reports on the entire organization

27. Which of the following is NOT a basic feature of a managerial accounting system?

a. financial measures only


b. subjective information
c. internal audience
d. informs local decision and actions

28. Which of the following is a basic feature of a managerial accounting system?


a. The scope tends to be highly aggregate.
b. There are no regulations governing the reports.
c. The reports are generally delayed and historical.
d. The audience tends to be stockholders, creditors and tax authorities.

29. Which of the following groups would be LEAST likely to receive detailed management
accounting reports?

a. management accountants
b. scientists and engineers
c. stockholders
d. managers

30. _____________ indicate whether the organization is creating long-term value and
profitability.

a. Strategic information
b. ROI
c. Net income
d. Critical success factors

31. ____________ is when a firm compares itself with the best practice of competitors or
other comparable organizations.

a. Process improvement
b. Benchmarking
c. Employee empowerment
d. Total quality philosophy

32. Which of the following is NOT a function of a management accounting system?

a. operating control
b. product and customer costing
c. management control
d. financial reporting

33. Which of the following functions provides feedback information about the efficiency of
tasks performed?

a. operating control
b. product and customer costing
c. management control
d. financial reporting

34. Which of the following functions provides information on the performance of managers
and operating units?
a. Operational control
b. Product and customer costing
c. Management control
d. Financial reporting

35. Which of the following is NOT a role of management accounting information in


operating control?

a. to provide feedback information about quality


b. to provide feedback information about timeliness
c. to provide feedback information about the efficiency of tasks performed
d. to provide performance measures for decentralized organizational units

36. Which of the following is NOT a role of management accounting information in product
and customer costing?

a. to measure the cost of resources used to produce a service


b. to assess the profitability of the organization's services by linking resources generated
c. to provide feedback information about the quality, timeliness, and efficiency of tasks
performed
d. to assess customer profitability for a particular segment

37. An organization develops a code of ethics because

a. it is required by law.
b. the Chief Executive Officer demands it.
c. it wishes to reduce ethical conflicts by avoiding ambiguity or misunderstandings.
d. it wishes to punish those whose ethical standards are different from its own.

38. If an individual faces a conflict between the organization's stated and practiced values
experts recommend that

a. the individual resign immediately and call the media


b. the individual call the media.
c. delay action and work with respected leaders in the organization.
d. delay action and hope the problem goes away.

39. The elements of an ethical control system include the following ЕХСЕРТ

a. reward system for turning in those who violate the ethical code
b. a statement of the organization's values and code of ethics.
c. an ongoing internal audit of the ethical control system.
d. a statement of the employee's ethical responsibilities.
40. Certified Management Accountants are required to adhere to the following ethical
standards, EXCEPT

a. Competence
b. Ingenuity
c. Integrity
d. Objectivity

41. A study of organization that are among the best in the world at performing a particular
task

a. Business process
b. Benchmarking
c. Control
d. Feedback

42. An activity that consumes resources or takes time out that does not add value for which
customers are willing to pay

a. Non-Value added activity


b. Value-added activity
c. Process reengineering
d. Total quality management

43. Accounting and other reports that help managers monitor performance and focus on
problems and/or opportunities that might otherwise go unnoticed

a. Feedback
b. Performance report
c. Financial accounting
d. Managerial accounting
CHAPTER

3
CONTEMPORARY BUSINESS
ENVIRONMENT AND STRATEGIC
FOCUS OF COST MANAGEMENT
______________________________________________
EXPECTED LEARNING OUTCOMES

After studying this chapter, you should be able to…

1. Describe the more recent changes in contemporary business environment such as


● The Global Business Environment

● Advances in Manufacturing Techniques

● Advances in Information ...

● A greater focus on customers

● New forms of organization

● Changes in the Social, Political & Cultural Environment

2. Explain the strategic focus of cost management

3. Describe the relationship between cost management and accounting systems

4. Explain the concept of integrative framework on how the accounting system is used
in the firm’s organizational structure

◆◆◆
CHAPTER 3

CONTEMPORARY BUSINESS ENVIRONMENT AND


STRATEGIC FOCUS OF COST MANAGEMENT

CONTEMPORARY BUSINESS ENVIRONMENT

The business environment in recent years has been characterized by increasing competition and
relentless drive for continuous improvement. These changes include (1) an increase in global
competition; (2) advances in manufacturing technologies; (3) advances in information
technologies, the Internet, and e- commerce; (4) a greater focus on the customer; (5) new
forms of management organization; and (6) changes in the social, political, and cultural
environment of business. As businesses turned global and product lines expanded, operations
have become more complex, forward-looking companies saw a tremendous need for
management oriented data that was separate from financial-oriented data.

Corporate executives are now using cost data to chart successful futures for their companies.
Adapting management accounting system to better meet management's needs for information is
crucial to an organization's survival when competing in global markets. Global competitors now
have relatively free access to markets around the world. As a result, domestic markets on
virtually every country face greater challenges from foreign competition. With increased reliance
on global markets, companies need not only respond quickly to changing market conditions but
also tailor products to different consumer tastes and demands and this has to be done at a level
that assures profit and gives satisfactory returns to shareholders.

In today's automated environment management accountants use their management control


systems to support and reinforce manufacturing and other operating strategies. It is in this light
that one learns to appreciate the role of a management accountant which is more of an
influencing role rather than just an informing role.

The change in the business environment in at least the last two decades where organization have
to transform themselves to become more competitive, have profound effect in the practice of
management accounting. Of particular importance are the changes in business, especially the
increase in global competition and the changes in management techniques, that have created the
need for a new, strategic approach to management and to cost management.

The Global Business Environment

The growth of international markets and trade are the key development that drive the extensive
changes in the contemporary business changes in the contemporary business environment. Profit-
oriented business and not-for-profit organization, consumers and regulators are all affected
.

significantly by the rapid growth of economic independence and increased competition from
other continues. The growing number of alliance among large multinational, the increasing trade
agreements among countries indicate clearly that the opportunities for growth and profitability
lie in global markets. As low-cost, high quality goods are traded worldwide, most consumers are
benefited. Manager, business owners and investors benefit likewise when sales and production
activities are pursued in foreign countries.

Global business environment is very competitive and firms need cost management information to
sustain competitiveness. They also need financial and nonfinancial information about doing
business and competing effectively.

Advances in Manufacturing Technologies

Firms around the world adopt new manufacturing technologies to remain competitive in the face
of the increased global competition. Many firms adopt methods applied in some Japanese
manufacturing firms that produced significant cost and quality improvements using quality
teams, and statistical quality control. Some firms include just-in-time inventory method in order
to reduce the cost and waste of maintaining large levels of raw materials and unfinished product.
A key competitive edge that forms have is the ability to deliver the product or service faster than
the competition. This is known as speed-to-market.

Advances in Information Technologies, The Internet and E-Commerce

The increasing use of information technology, the internet and e-commerce is perhaps the most
fundamental of all business changes in recent years. This new economy is manifested in the rapid
growth of Internet-based firms (the dot-com's such as Amazon, eBay, and E-trade) and the
increased use of the Internet for business data processing, communication, and sales. These
technologies have resulted in the growing focus in cost management by reducing the time
required to process transactions, thereby expanding the individual's access to information within
the firm, the industry and the business environment around the world.

A Greater Focus on Customers

To succeed in this era, customer value is the key focus that businesses of all types must be
concerned with. A key change in increased customer demand for product functionality and
quality. As business firms seek to add new features and new products as quickly as possible,
shorter product life cycle thereby increasing the overall intensity of competition. The new
business process focuses on customer satisfaction.
Contemporary Business Environment and Strategic Focus of Cost Management 57

Producing value for the customer has changed the orientation of managers from low-cost
production of large quantities to quality, service, faster delivery and the ability to respond to the
customer’s desire for specific feature. Today, many of the critical success factors are customer
oriented. Cost management practices are also changing, cost management reports now include
specific measures of customer preference and customer satisfaction.

The value of a product or service to the customer is affected by such diverse attributes as product
price, quality, functionality, user-friendliness, customer service, warranty and maintenance costs.
By managing activities that will increase customer value, the firms can establish a competitive
advantage by creating better customer value for the same or lower cost than that of competitors.
Cost information plays an important part in the process called strategic cost management.
Generally, firms chose a strategic position corresponding to one of two general strategies:

(a) cost leadership, and


(b) superior product through differentiation.

A focus on customer value means that the management accounting system should produce
information about both realization and sacrifice. The system should be able to measure various
attributes of customer value.

Successful pursuit of cost leadership and/or differentiation strategies requires an understanding


of a firm's value chain (internal) and supply chain (external).

New Forms of Management Organization

Management organization has changed in response to the changes in marketing and


manufacturing. Because of the focus on customer satisfaction and value, the emphasis has shifted
from financial and profit-based measures of performance to customer-related, nonfinancial
performance measures such as quality, time to delivery and service. Similarly, the hierarchical
command-and-control type of organization is being replaced by a more flexible organizational
from that encourages teamwork and coordination among business functions. Ins response to
these changes, cost management practices are also changing to include reports that are useful to
cross-functional teams of managers; the reports reflect the multinational roles of these teams and
include a variety of operating and financial information: product quality, unit cost, customer
satisfaction, and production bottlenecks. The changes in management organization and
marketing in the environment of business are summarized in Figure 3-1.
Figure 3-1: Comparison of Prior and Contemporary Business Environments

Prior Business Contemporary Business Environment


Environment

Management Organization
Type of information recorded and Almost exclusively financial data Financial and operating data, the
reported firm’s strategic success factors
Management organizational Hierarchical, command and Network-based organization forms,
structure control teamwork focus - employee has more
responsibility and control, coaching
rather than command and control
Management focus Emphasis on the short term, Emphasis on the long term, focus on
short-term performance critical success factors, commitment
measures and compensation, to the long-term success of the firm,
concern for sustaining the including adding shareholder value
current stock price, short
tenure and high mobility of top
managers
Manufacturing
Basis of compensation Standardization, economies of Quality, functionality, customer
scale satisfaction
Manufacturing process High volume, long production Low volume, shot production runs,
runs, significant levels of in- focus on reducing inventory levels
process and finished inventory and other non-value-added activities
and costs
Manufacturing Technology Assembly-line automation isolated Robotics, flexible manufacturing
technology applications systems, integrated technology
applications connected by networks
Required labor skills Machine-paced, low-level skills Individually and team-paced, high-
level skills
Emphasis on quality Acceptance of a normal or usual Goal of zero defects
amount of waste
Marketing
Products Relatively few variations, long Large number of variations, short
product life cycles product life cycles
Markets Largely domestic Global

Changes in the Social, Political, And Cultural Environment of Business


Significant changes have taken place in the social, political, and cultural environments that affect
business. Although the nature and extent of these changes vary a great deal from country to
country, they include a more ethically and racially diverse workforce, a renewed sense of ethical
responsibility among managers and employees, and an increased deregulation of business by the
national government.

The new business environment requires firms to be flexible and adaptable and to place greater
responsibility in the hands of a more highly skilled workforce. Additionally, the changes tend to
focus the firm factors outside the production of its product or provision of its service to the
ultimate consumer and the global society in which the costumer lives.

STRATGIC FOCUS OF COST MANAGEMENT

A competitive firm incorporates the emerging and expected change in the contemporary
environment of business into its business planning and practices. This firm is customer-driven,
uses advanced manufacturing technologies when appropriate, anticipates the effect of changes in
regulatory policies and customer tastes, and recognizes its complex social, political and cultural
environment.

Guided by strategic or long-term thinking, the management accountant focuses that make the
company successful rather than just focusing on cost control and other financial measure.
Cost management should focus not on the measurement per -- but on the identification of those
measures that are critical to the firm's success.

Phases of the development of cost management systems should consider the following:

Stage 1: Cost management systems are basic transaction reporting systems.


Stage 2: As they develop into the second stage, cost management systems focus on external
financial reporting. The objective is reliable financial reports; accordingly, the
usefulness for cost management is limited.
Stage 3: Cost management systems track key operating data and develop more accurate and
relevant cost information for decision making; cost management information is
developed.
Stage 4: Strategically relevant cost management information is an integral part of the system.

Stages 1 and 2 of cost system development focus on the management accountant's measurement
and reporting role. Stage 3 shifts to operational control. Stage 4, the management accountant
Developing a Competitive Strategy; Contemporary Cost Management Techniques 95

becomes an integral part of management, not just a reporter but a full business partner, with the
skills of identifying, summarizing and reporting critical factors necessary for the firm's success.

Critical Success Factors (CSFs) are measures of those aspects of the firm's performance essential
to its competitive advantage and, therefore, to its success. Many of these critical success factors
are financial, but many are nonfinancial. The CSFs for any given firm depend on the nature of
the competition it faces.

COST MANAGEMENT AND ACCOOUNTING SYSTEMS

The term cost management is widely used in businesses today. Unfortunately, there is no
uniform definition. We use cost management to describe the approaches and activities of
managers in short-run and long-run planning and control decisions that increase value for
customers and lower costs of products and services. For example, managers make decisions
regarding the amount and kind of material being used, changes of plant processes, and changes
in product designs. Information from accounting systems helps managers make such decisions,
but the information and the accounting systems themselves are not cost management.

Cost management has a broad focus. For example, it includes — but it not confined to the
continuous reduction of costs. The planning and control of costs is usually inextricably linked
with revenue and profit planning. For instance, to enhance revenues and profits, managers often
deliberately incur additional costs for advertising and product modifications.

Cost management is not practiced in isolation. It's an integral part of general management
strategies and their implementation. Examples include programs that enhance customer
satisfaction and quality, as well as programs that promote "blockbuster" new product
development.

WHEN SHOULD THE INTERNAL ACCOUNTING SYSTEM BE CHANGED?

The succeeding sections will analyze organizational innovations. These innovations illustrate
that internal accounting systems are an integral part of the organization's architecture. When
managers change the architecture of their organization by decentralizing decision rights and
empowering employees via TQM programs because the firm's business strategy changes,
accounting systems are likewise modified. Similarly, when JIT production systems are installed,
accounting system changes follow. However, there were no organization changes associated with
productivity measurement systems and these accounting systems were not widely implemented.

There is no such thing as the ideal management accounting system. Each organization has
different circumstances that lead to different management accounting decisions. Also, accounting
must continually deal with trade-offs among external users wanting information describing firm
performance and internal users wanting information for decision making and control. Surviving
organizations must meet the demands of changing technologies and markets by revising their
business structures and organizational architectures. Because organizational architectures are in a
constant state of change, the accounting system must regularly adapt.

There are certain signs that indicate that the internal accounting system is not working well. One
sign is dysfunctional behavior on the part of managers because of poorly chosen performance
measures. Managers will make decisions to positively influence performance measure. If those
performance measures are not consistent with the goals of the organization, management will
make decisions that do not coincide with the organization’s goals. Another sign of problems with
the accounting system is poor operating decisions. If product mix and pricing decisions based on
management accounting are not adding to the organizational value, then the accounting system is
either providing inaccurate estimates of opportunity costs and/or creating dysfunctional
incentives.

Often changes in customers' organizational architectures cause suppliers to change their


architecture (and modify their organizational accounting systems). If your major customers are
architects, they are likely responding to technological and market conditions. The way in which
knowledge is generated and disseminated has probably changed. These changes are likely
affecting your firm's organizational architecture.

Organizations should not necessarily look to the latest management accounting fads to give them
direction in changing their management accounting systems. Activity-based costing (ABC), for
example, is only appropriate for certain types of organizations. Each organization must
continually evaluate and improve its management accounting system to meet the challenges of a
changing environment and a changing organization.

This text will emphasize the dual role of internal accounting systems for decision making and
control. Because the internal accounting system is performing two separate roles (it is also being
used for taxes and financial reporting). trade-offs between these roles must be made. In its
decision-making role, the accounting system is the first-place managers tum to help them
estimate opportunity costs.
However, accounting numbers are not forward-looking opportunity costs.
Accounting systems record historical costs, which are backward looking.
Therefore, accounting numbers are useful for decision making only under very strong
assumptions, primarily that the future will look like the past.

INTEGRATIVE FRAMEWORK
Succeeding chapters of this book will describe and analyze cost management accounting
systems. Besides being used for both decision making and control. These accounting systems
support external reporting for shareholders, taxes, and government regulations. Thus, one of the
central themes of this text is trade. offs arise when the accounting system is designed for multiple
purposes. In addition to providing a better understanding of the internal uses of the accounting
system, this book reinforces the importance of viewing the accounting system as part of the
firm's organizational architecture. This analytic structure will help readers better understand, use,
and design future accounting systems as well as other systems that evaluate and reward
performance and partition decision rights.

Figure 3-2 shows the integrative framework for understanding how the accounting system is
used in the firm's organizational architecture. Starting at the top, two external factors
(technological innovation and market conditions) affect the firm's business strategy. The
business strategy then interacts with the firm's organizational architecture to provide incentives
for managers and employees. These incentives affect the actions taken, which in turn affect the
value of the firm. Thus, Figure 3-2 emphasizes that external factors like technology and market
conditions affect investments, organizational architecture, incentives, actions, and ultimately the
value of the firm.

Figure 3-2 provides two important observations:

1. Changes in the accounting system rarely occur in a vacuum. Accounting system


changes generally occur at the same time as changes in the firm's business strategy and
other organizational changes, particularly with regard to the partitioning of decision
rights and the performance evaluation and reward systems.
2. Alterations in the firm's organizational architecture, including changes in the
accounting system, are likely to occur in response to changes in the firm's business
strategy caused by external shocks from technology and shifting market conditions.
Figure 3-2: The determinants of business architecture, and firm value

Three significant managerial implications are derived from these two observations. First, before
implementing an accounting or other organizational change, it is important to understand what is
driving the change. Second, an accounting system should not be adopted merely because other
firms are doing so; they may be reacting to a different set of external shocks. Third, an
accounting system should not be changed without concurrent, consistent changes in the way
decision rights are partitioned as well as in the performance reward systems. All three parts of
the organization's architecture must be internally consistent and coordinated.
REVIEW QUESTIONS AND EXERCISES

Question

1. How does a company determine and implement its mission?


2. How have firms responded to changes in business?
3. Contrast the use of strategic financial measures with the use of strategic nonfinancial
measures.
4. What are some of the factors in the contemporary business environment that are
causing changes in business firms and other organizations, and how are the changes
affecting the way those firms and organizations use cost management information?
5. What are the implications of strategic analysis for cost management?
6. The competitive environment for both manufacturing and service companies have
become far more challenging and demanding in the last 25 years. Discuss the changes in
competition and in the nature and type of the new requirements for management
accounting information.
7. Explain how a customer focus can result in increased profits for a company.
8. In most organizations, customer satisfaction is one of the top priorities. As such,
attention to customers is necessary for success. Briefly describe the four types of
demands customers are currently placing on organizational performance.

Exercises

Exercise 1

Joel De Castro, CEO of De Castro Enterprises, Inc. (DCEl), is considering a special offer
to manufacture a new line of women's clothing for a large department store chain. DCEl
has specialized in designer women's clothing sold in small, upscale retail clothing stores
throughout the country, in order to protect the very elite brand image, De Castro has not
sold clothing to the large department stores. The current offer, however, may be too good
to turn down.
The department store is willing to commit to a large order, which would be very
profitable to De Castro, and the order would be automatically renewed for two more
years, presumably to continue after that point.
Required: Analyze the choice Joel faces, based on competitive analysis
Exercise 2

Jim Castelo's lifelong hobby has been racing small sailboats. Jim has been successful
both at the sport and in the design of new pieces of equipment to be used on small
sailboats to make them easier to sail and more effective in racing.
Jim is now thinking about starting a mail-order business in his garage to sell products he
favors, as well as some he has designed himself. He plans to contract out most of the
manufacturing for the parts and equipment to machine shops and other small
manufacturers in his area.
Required: Develop a strategic analysis for Jim's new business plan. What should be his
competitive position; that is, how should be choose to compete in the existing market for
sailboat supplies and equipment? How is he likely to use cost management information in
building his business?

Exercise 3

Consider the following cases, each of which is a consulting client:


1. Performance Bicycles, a mail-order company that supplies bicycles, parts, and
bicycling equipment and clothing.
2. The Nathalie Hotel, a downtown hotel that serves primarily convention and business
travelers.
3. The Barangay Asenso Public Health Clinic, which is supported by tax revenues of
Barangay Asenso and public donations.
4. The American motorcycle company.
5. The Metro pharmaceutical company.
6. St. Martha's College, a small, private liberal arts college..

Required: For each client, determine the competitive strategy and the related critical
success factors for the organization or firm.
Multiple Choice

1. The strategic approach to management requires integrative thinking, i.e., the ability to
identify and solve problems:
0. from a cross-functional view c. quickly and decisively
0. without using accounting data d. under considerable stress.
a.
2. Firms have responded to the recent changes in business in all but which one of the
following ways?
a. down-sizing the workforce
b. developing smaller structures
c. outsourcing service functions
d. promoting more effective policies

3. The competitive strategy in which the firm succeeds by producing at the lowest cost in
the industry is termed:
a. differentiation c. price strategy
b. cost advantage d. cost leadership

4. The competitive strategy in which the firm succeeds by developing and maintaining a
unique value for the product, as perceived by the customers is termed:
a. differentiation c. price strategy
b. cost advantage d. cost leadership

5. Skills or competencies that the firm employs especially well are called:
a. critical skills. c. essential strategies.
b. core competencies. d. competitive factors.

6. Which one of the following describes the type of information that cost management
must provide that is not provided by traditional cost accounting systems?
a. information of a record keeping nature
b. reported financial information
c. reported nonfinancial information
d. information that address the strategic objectives of the firm

7. Activities that firms in the industry must perform in the process of taking raw material
and converting it into final product are known as:
a. value activities c. production activities
b. conversion activities d. production activities
CHAPTER

4
DEVELOPING A COMPETITIVE
STRATEGY AND CONTEMPORARY
COST MANAGEMENT TECHNIQUES
______________________________________________
EXPECTED LEARNING OUTCOMES

After studying this chapter, you should be able to…

1. Understand the basic approach of how a firm’s competitive strategy is developed

2. Explain the strategic measures of success, financial and nonfinancial factors

3. Describe the critical success factors in a business firm and how they can be measured

4. Explain the consequences of lack of strategic information

5. Describe the two basic competitive strategies, namely,


a. Cost Leadership
b. Product Differentiation

6. Understand and describe the contemporary cost management techniques such as


a. Total Quality Management
b. Just-in-Time Production System
c. Process Reengineering
d. Benchmarking
e. Mass Customization
f. Balanced Scorecard
g. Activity-based Costing and Management
h. Theory of Constraints
i. Life Cycle Costing
j. Target Costing
k. Computer-Aided Design and Manufacturing
l. Automation
m. E-Commerce
n. The Value Chain

DEVELOPING A COMPETITIVE STRATEGY

A strategy is a set of policies, procedures and approaches to business that produce long-term
success.

◆◆◆
CHAPTER 4

DEVELOPING A COMPETITIVE STRATEGY;


CONTEMPORARY COST MANAGEMENT
TECHNIQUES

Finding a strategy begins with determining the purpose and long-range direction or on in other
words, the mission of the company. The mission is developed into specific performance
objectives which are then implemented by specific corporate or company’s strategies, that is,
specific actions to achieve the objectives that will fulfill the mission. A form succeeds by
implementing a strategy.

Strategy specifies how an organization matches its own capabilities with the opportunities in the
market place to accomplish its objectives. In other words, strategy describes how a compete will
compete and the opportunities its employee should seek and pursue. Companies follow one of
two broad strategies. Some companies such as Jollibee, Pure Gold and Cebu Pacific Airline
compete on the basis of providing a quality product or service at low prices. This is also known
as “Cost Leadership” strategy. Other Companies such as Rustan’s Department Store and BGC
Shangri-La Hotel compete on their ability to offer unique products or services that are often
process higher than the products or services of competitors. This is known as “Product
Differentiation” strategy.

Deciding between these strategies is a big part of what managers do. Management accountants
work closely with managers in formulating strategy by providing information about the sources
of competitive advantage – for example, the cost, productivity, or efficiency advantage of their
company relative to competitors or the premium prices a company can charge relative to the cost
of adding features that make its products or services distinctive. The management accountant
also helps formulate a strategy by answering questions such as:

 Who are our most important customers?


 How sensitive are their purchases to price, quality, and service?
 Who are our most important suppliers?
 What substitute products exists in the marketplace, and how do they differ from our
product in terms of price and quality?
 Is the industry demand growing or shrinking?
 Is there overcapacity?

Strategic Cost management is often used to describe Cost Management that specifically focuses
on strategic issues such as these.
CHAPTER 4

DEVELOPING A COMPETITIVE STRATEGY;


CONTEMPORARY COST MANAGEMENT
TECHNIQUES
STRATEGIC MEASURES OF SUCCESS

Firms use cost management to support their strategic goals. The strategic cost management
system develops strategic information, including both financial and non-financial information.

Financial performance measures include among others


a. growth in sales and earnings
b. cash flows
c. stock price

They show the impact of the firm’s policies and procedures in the firm’s current financial
position and therefore, its current return to the shareholders.

Non-financial measures of operation include among others


a. market share
b. product quality
c. customer satisfaction
d. growth opportunities

The nonfinancial actors show the firm’s current and potential competitive position as measured
from three additional perspective, namely:
1. the customer
2. internal business process and
Figure 4-1 shows the Financial and Nonfinancial Measures of Success (Critical Success Factors)

Figure 4-1: Financial and Nonfinancial Measures of Success or Critical Success Factors and
How to Measures CSF

Critical Success Factors How to measure CSF

Financial Measures of Success


Sales Level of sales in critical product groups, sales
trend, percent of sales from new products, sales
forecast accuracy
Profitability Earnings from operations, earnings trend,
dividend growth
Liquidity Cash flow, trend in cash flow, interest coverage,
asset turnover, inventory turnover, receivables
turnover, credit ratings
Market Value Share price
Non-Financial Measures of Success Customer Factors
Customer satisfaction Customer returns and complaints, customer
Dealer and distributor survey
Coverage and strength of dealer and distributor
channel relationships; e.g., number of dealers per
Marketing and selling state or region
Trends in sales performance, training, market
Timeliness of delivery research activities; measured in hours or peso
On-time delivery performance, time from order to
Quality customer receipt
Customer complaints, warranty expense
Internal Business Process
Quality Number of defects, number of returns, customer
survey, amount of scrap, amount of rework, field
service reports, warranty claims, vendor quality
defects
Productivity Cycle time (from raw materials to finished
product); labor efficiency; machine efficiency;
amount of waste, rework, and scrap
Flexibility Setup time, cycle time
Equipment readiness Downtime, operator experience, machine
capacity, maintenance activities
Safety Number or accidents, effects and accidents
Learning and Innovation
Product innovation Number of design changes number of new
patents or copyrights, skills or research and
development staff
Timeliness of new product Number of days over or under the announced
ship date
Skill development Number of training hours, amount of skill
performance improvement
Employee turnover, number of complaints,
Employee morale employee survey
Rate of turnover, training, experience,
Competence adaptability, financial and operating performance
measures
Other factors
Government relations Number of violations, community service activities
Without strategic information, the firm is likely to stray from its competitive course, to make
strategically wrong manufacturing and marketing decisions, to choose the wrong products or the
wrong customers. Some of the consequences of a lack of strategic information are shown in
Figure 4-2.

Figure 4-2: Consequences of Lack of Strategic Information

 Decision making based on intuition


 Lack of clarity about direction and goals
 Lack of clear and favorable perception of the firm by customers and suppliers
 Incorrect investment decisions; choosing products, markets or manufacturing processes
inconsistent with strategic goals
 Inability to effectively benchmark competitors, resulting in lack of knowledge about more effective
competitive strategies
 Failure to identify most profitable products, customer and markets
.

COMPETITIVE STRATEGIES

For a firm to sustain a competitive position, it must purposefully or as result of market forces
arrive at one of the two competitive strategies, namely

Cost Leadership and Product Differentiation

Cost Leadership

This is a competitive strategy in which a firm succeeds in producing products or services at the
lowest cost in the industry. A firm that is a cost leader makes sustainable profits at lower prices,
thereby limiting the growth of competitions in the industry through its success in price wars and
undermining the profitability of competitors which must meet the firm’s low price.

Product Differentiation

The differentiation strategy is implemented by creating a perception among consumers that the
product or service is unique in some important way, usually by being of higher quality, features
or innovation. This perception allows the firm to charge higher prices and outperform the
competition in profits without reducing cost significantly. Most industries, including automobile,
consumer electronics, and industrial equipment, have differentiated firms. The appeal of
differentiation is especially strong for product lines which the perception of quality of
differentiation is important, as in cosmetics, jewelry and automobiles. Tiffany, Roles, Ferrari and
BMW are good examples of forms that emphasize differentiation.

Distinctive Aspects of the Two Competitive Strategies


Aspect Cost Leadership Differentiation
Strategic Target Broad cross section of the Focused section of the market
market
Basis of competitive Lowest cost in the industry Unique product or service
advantage
Product line Limited selection Wide variety, differentiating
features
Production emphasis Lowest possible cost with high Innovation in differentiating
quality essential product products
features
Markets emphasis Low price Premium price and innovative
differentiating features
Looking more closely at differentiated firms, the keys CSFs and execution issues are in marketing and
product development — developing customer loyalty and brand recognition, emphasizing superior and
unique products, and developing and using detailed and timely information about customer needs and
behavior. This is where the marketing and product development within the firm provide leadership and
the management accountants support these efforts by gathering, analyzing, and reporting the relevant
information.

Other Strategic Issues

A firm succeeds by adopting and effectively implementing one of the strategies explained earlier.
Recognize that although one strategy is generally dominant, a firm is most likely to work hard at
process improvement throughout the firm, whether cost leader or differentiator, and on occasion
to employ both of the strategies at the same time. However, a firm following both strategies is
likely to succeed only if it achieves one of them significantly. A firm that does not achieve at
least one strategy is not likely to be successful. This situation is what Michael calls "getting stuck
in the middle". A firm that is stuck in the middle is not able to sustain a competitive advantage.
For example, giant retailer Makati Supermarket been stuck in the middle between trying to
compete with Pure Gold on cost and price, and with style conscious target on differentiation..
CONTEMPORARY COST MANAGEMENT TECHNIQUES
Managers commonly use the following tools to implement the firm's broad and to facilitate the
achievement of success on critical success factors: just-in-time (JIT), total quality management,
process reengineering, benchmarking, mass customization, balanced scorecard, activity-based
costing and management, theory of constraints (TOC), life cycle costing, target costing,
computer-aided design and manufacturing, automation, e-commerce and the value chain and
supply-chain analysis.

The basic concepts of these cost management techniques are discussed in the succeeding section:

a. Total Quality Management


To survive in an increasingly competitive environment, firms realizes that they must
produce high-quality products. As a result, an increasing number of companies have
instituted total quality management programs to ensure their products are of the highest
quality and that production processes efficient.
Total quality management (TQM) is a technique in which management develops policies
and practices to ensure that the firm's products and services exceed customers'
expectations.
Currently, there is no generally agreed upon "perfect" way to institute a TQM program.
But most companies with TQM develop a company that stresses listening to the needs of
customers, making products right the first time, reducing defective products that must be
reworked, and encouraging workers to continuously improve their production process.
That is why some TQM programs are referred to as continuous quality improvement
programs.
TQM affects product costing by reducing the need to track the cost of scrap and rework
related to each job. If TQM is able to reduce these costs to a very low level, the benefit of
tracking the costs is unlikely to exceed the cost to the accounting system.
Total Quality Management (TQM) is a formal effort to improve quality throughout an
organization's value chain. The two major characteristic TQM are
(1) a focus on serving customers, and
(2) systematic problem-solving using teams made up of front-line workers.
Chapter 6 includes more detailed discussion of the TOM.
b. Just-in-Time (JIT)
Just-in-Time (JIT) is the philosophy that activities are undertaken only as needed or
demanded. JIT is a production system also known as pull-it-through approach, in which
materials are purchased and units are produced only as needed to meet actual customer
demand. In a JIT system, inventories are reduced to the minimum and in some cases,
zero.

Just-in-Time (JIT) production is a system in which each component on a production line


is produced immediately as needed by the next step in the production line. In a JIT
production line, manufacturing activity at any particular workstation is prompted by the
need for that station's output at the following station. Demand triggers each step of the
production process, starting with customer demand for a finished product at one end of
the process and working all the way back to the demand for direct materials at the other
end of the process. In this way, demand pulls a product through the production line. The
demand-pull feature of JIT production systems achieves close coordination among work
centers. It smoothes the flow of goods, despite low quantities of inventory.

Financial Benefits of JIT

JIT tends to focus broadly on the control of total manufacturing costs instead of
individual costs such as direct manufacturing labor. For example, idle time may rise
because production lines are starved for materials more frequently than before.
Nevertheless, many manufacturing costs will decline. JIT can provide many financial
benefits, including
1. Lower investment in inventories.
2. Reductions in carrying and handling costs of inventories.
3. Reductions in risk of obsolescence of inventories.
4. Lower investment in plant space for inventories and production.
5. Reductions in setup costs and total manufacturing costs.
6. Reduction in costs of waste and spoilage as a result improves quality.
7. Higher revenues as a result of responding faster to customers.
8. Reductions in paperwork.

Major Features of JIT Production System


There are five main features in a JIT production system:
1. Production is organized in manufacturing cells, a grouping of all different types of
equipment used to manufacture a given product.
2. Workers are trained to be multiskilled so that they are capable of performing a variety
of operations and tasks.
3. Total quality management is aggressively pursued to defects.
4. Emphasis is placed on reducing setup time, which is the time require to get equipment,
tools and materials ready to start the production of a component or product, and
manufacturing lead time, which is time from when an order is ready to start on the
production line to when it becomes a finished good.
5. Suppliers are carefully selected to obtain delivery of quality-tested parts in a timely
manner.
A more detailed discussion of JIT Product System is covered in Chapter 6.
c. Process Reengineering

Reengineering is a process for creating competitive advantage in which a firm


reorganizes its operating and management functions. often with the result that jobs are
modified, combined, or eliminated. It has been defined as the "fundamental rethinking
and radical redesign of business processes to achieve dramatic improvements in critical,
contemporary measures of performance, such as cost, quality, service, and speed.

Process reengineering, a more radical approach to improvement than TQM, is an


approach where a business process is diagrammed in detail, questioned and then
completely redesigned in order to eliminate unnecessary steps, to reduce opportunities for
errors and to reduce costs. A business process is any series of steps that are followed in
order to carry out some task in a business.

The main objective of this approach IS the simplification and elimination of wasted effort
and the central idea is that all activities that do not add value to product or service should
be eliminated. In its most simplified version. the steps used in process reengineering are
1. A business process is diagrammed in detail.
2. Every step in the business process must be analyzed and justified.
3. The process is redesigned to include only those steps that make the product or
service more valuable.

This process can yield the following anticipated results:


1. Process is simplified
2. Process is completed in less time
3. Costs are reduced, and
4. Opportunities for errors are reduced

Process reengineering has one basic recurrent problem, that is – employee resistance. As
with other improvement projects, employees fear loss of jobs which may lead to lost
morale and failure to improve the bottom line (i.e., profits). For the process to prosper
and succeeds employees must be convinced that the end result of the improvement will be
more secure, rather than less secure jobs. They can be made to understand that improving
the processes, the company can generate more business, produce a better product at lower
cost and will have the competitive strength to prosper.

d. Benchmarking
Benchmarking is a process by which a firm
 determines its critical success factors
 studies the best practices of other firms (or other units within a firm) for achieving
these critical success factors, and
 then implements improvements in the firm's processes to match or beat the
performance of those competitors.

Today benchmarking efforts are facilitated by cooperative networks of noncompeting


firms that exchange benchmarking information.
e. Mass Customization

Many manufacturing and service firms increasingly find that customers expect products
and services to be developed for each customer's unique needs. And many firms have
been successful with a strategy that targets customer's unique needs.

Mass customization is a management technique in which marketing and production


processes are designed to handle the increased variety that results from delivering
customized products and services to customers.

The growth of mass customization is in effect another indication of the increased


attention given to satisfying the customer.
f. Balanced Scorecard

The balanced scorecard is an accounting report that includes the firm’s critical success
factors in four areas

(a) financial performance,


(b) customer satisfaction.
(c) internal business process, and
(d) innovation and learning.

The concept of balance captions the intent of broad coverage, financial and nonfinancial
of all the factors that contribute to the success of the firm in achieving its strategic goals.
The use of the balanced scorecard is thus a critical ingredient of the overall approach that
firms take to become and remain competitive.

This is discussed in more details in Chapter 7.


g. Activity-based Costing and Management
Activity analysis is used to develop a detailed description of the specific activities
performed in the operation of the firm. Many firms have found that they can improve
planning, product costing, operational control, and management control by using activity
analysis to develop a detailed description of the specific activities performed in the firm's
operations. The activity analysis provides the basis for activity-based costing and activity-based
management. Activity-based costing (ABC) is used to improve the accuracy of cost analysis by
improving the tracing of costs to products or to individual customers. Activity-based
management (ABM) uses activity analysis to improve operational control and management
control. ABC and ABM are key strategic tools for many firms, especially those with complex
operations, or great diversity of products.

h. Theory of Constraints (TOC)

The Theory of Constraints is a sequential process of identifying and removing constraints


in a system.

The Theory of Constraints emphasizes the importance of managing the organization's


constraints or barriers that hinder or impede progress toward an objective. Since the
constraint is whatever is holding back the organization, improvement efforts usually must
be focused on the constraint to be really effective.

The basic sequential steps followed in applying TOC are


1. Analyze all the factors of production (materials, labor, facilities, methods, etc.)
required in the production chain.
2. Identify the weakest link, which is the constraint.
3. Focus improvement efforts on strengthening the weakest link.
4. If improvement efforts are successful, eventually the weakest link will improve
to the point where it is no longer the weakest link.
5. At this point, a new weakest link (new constraint) must be identified and
improvement efforts must be shifted over that link.

The Theory of Constraints approach is a perfect complement to Total Quality


Management and Process Reengineering - it focuses improvement efforts where they are
likely to be most effective.

TOC is discussed more extensively in Chapter 8.


i. Life Cycle Costing

Life-cycle costing is a management technique to identify and monitor the costs of a


product throughout its lifecycle. It consists of all steps from product design and purchase
of raw material to delivery of and service of the finished product. The steps include
(l) research and development
(2) product design, including prototyping, target costing and testing
(3) manufacturing, inspecting, packaging and warehousing
(4) marketing, promotion and distribution
(5) sales and service.

Cost management traditionally has focused only on costs incurred up to the third step
manufacturing. Management accountants now strategically manage the product's full life
cycle of costs, including upstream and downstream costs as well as manufacturing costs.
j. Target Costing

Target costing involves the determination of the desired cost for a product or the basis of
a given competitive price so that the product will earn a desired profit. The basic
relationship that is observed in this approach is

Target cost = Market demand price – Desired profit

The entity using target costing must often adopt strict cost-reduction measures to meet the
market price and remain profitable. This is a common strategic approach used by
intensely competitive industries where even small price differences attract consumers to
the lower-priced product.

k. Computer-Aided Design and Manufacturing

More companies are using computer-aided design (CAD) and computer-aided


manufacturing (CAM) to respond to changing consumer tastes more quickly. These
innovations allow companies to significantly reduce the time necessary to bring their
products from the design process to distribution stage.

Computer-aided design is the use of computers in product development, analysis, and


design modification to improve the quality and performance of the product. Computer-
aided manufacturing (CAM) is the use of the computers to plan, implement, and control
production.

l. Automation

Automation involves and requires a relatively large investment in computers, computer


programming, machines, and equipment. Many firms add automation gradually, one
process at a time. To improve efficiency and effectiveness continuously, firms must
integrate people and equipment into the smoothly operating teams that have become a
vital part of manufacturing strategy. Flexible manufacturing systems (FMS) and
computer-integrated manufacturing (CIM) are two integration approaches. A flexible
manufacturing system (FMS) is a computerized network of automated equipment that
produces one or more groups of parts or variations of a product in a flexible manner. It
uses robots and
.

computer-controlled materials-handling systems to link several stand-alone, computer-


controlled machines in switching from one production run to another.

Computer-integrated manufacturing (CIM) is a manufacturing system that totally


integrates all office and factory functions within a company via a computer-based
information network to allow bour-by-hour manufacturing management.

The major characteristics of modern manufacturing companies that are adopting FMS and
CIM are production of high-quality products and services, low inventories, high degrees
of automation, quick cycle time, increased flexibility, and advanced information
technology. These innovations shift the focus from large production volumes necessary to
absorb fixed overhead to a new emphasis on marketing efforts, engineering, and product
design.

m. E-Commerce

A number of. internet-based companies have emerged and been proven successful in last
decade. This E-Commerce business model adopted by Amazon.com and eBay has also
attracted many investors to pursue the use of Internet in conducting business. Established
companies will undoubtedly continue to expand into cyberspace — both for business-to-
business transactions and for retailing. The Internet has important advantages over more
conventional marketplaces for some kinds of transaction such as mortgage banking. It is
also very likely that a blockbuster business may be built around the concept of selling
low-value, low-margin and bulky items like groceries over the Internet.

n. The Value Chain

Value chain refers to the sequence of business functions in which usefulness is added to
the products or services of a company. The term value refers to the increase in the
usefulness of the product or service and a result its value to the customer.
The value chain is an analysis tool that firms use to identify the specific steps required to
provide a product or service to the customer. The key idea of this concept is that the firm
studies each step in its operation to determine how each activity contributes to the firm's
competitiveness and profits.

Analyzing the firm's value chain helps management discover


 which steps or activities are not competitive
 where costs can be reduced, or
 which activity should be outsourced, and
 how to increase value for the customer at one or more of the steps of the value
chain.
When properly implemented, these approaches can (a) enhance quality, (b) reduce cost, (c)
increase output, and (d) eliminate delays in responding to customers. These techniques are
introduced here and most are covered more fully in later Chapters.

Internal value chain is the set of activities required to design, develop, produce, market and
deliver products or services to customers, If customer values are emphasized managers are
forced to determine which activities in the value chain are important to customers, A
management accounting system should track information about a wide variety of activities than
span the internal value chain.

Illustrative Case 4-1: Value Chain Analysis

Jack Reyes, a consultant for the Red Archer basketball team, has been asked to complete a value-
chain analysis of the franchise with a particular focus on comparison with a nearby competing
team, the Roaring Lions. Jack has been able to collect selected cost data, as shown below, for
each of the six steps in the value chain. Single-ticket prices range from P45.00 to P80 and
average paying attendance is approximately 2,200 for Red Archers and 5,000 Roaring Lions.

Average Cost per Person and Scheduled Games

Red Archer Steps in the Value Chain Roaring Lions


P. 45 Advertising and general promotion expenses P.50
.28 Ticket sales: local sporting goods stores and at the .25
ball park
.65 Ballpark operations .80
.23 Management compensation .18
.95 Players’ salaries 1.05
.20 Game-day operations, special entertainment, and .65
game-day promotions
P2.76 Total cost P3.43

Required:

Develop an analysis of the value chain to help Jack better understand the nature of the
competition between the Archers and the Lions, and to identify opportunities for adding value
and/or cost reduction at each step.
Illustrative Case 4-1 – Analysis of the Value Chain

The cost figures Jack has assembled suggest that the two team's operations are generally quite
similar, as one would expect in basketball. However, an important difference is the amount that
the Lions team spends on game-day operations — more than three times than that of the Red
Archers. That difference has, in part, built a loyal set of fans in Lions, where gate receipts
average more than twice that of Archers (P285,000 versus P 123,500). It happens that the Lions
have found an effective way to compete — by drawing attendance to special game-day events
and promotions.

To begin to compete more effectively and profitability, Archers might consider additional value-
added services, such as game-day activities similar to those offered in Lions. While Archers costs
per person are somewhat lower than Lion's, the cost savings are not enough to offset the loss in
revenues.

On the cost side, the comparison with Lions shows little immediate promise for cost reduction;
Archer spends on the average less than Lions in every category except management
compensation. Perhaps this is a further indication that instead of reducing costs, Archer should
spend more on fan development. The next step in Jack's analysis might be survey of Archer fans
to determine the level of satisfaction and to identify desired services that are not currently
provided.
REVIEW QUESTIONS

Questions

1. Identify and explain the two types of competitive strategy.

2. Identify three or four well-known firms that succeed through cost leadership.

3. Identify three or four well-known firms that succeed through product differentiation.

4. Explain the process of identifying a sustainable competitive advantage for a firm.

5. What is the meaning of "getting stuck in the middle" in the context of competitive
strategy, and how does the situation arise?

6. What is the role of the cost manager regarding nonfinancial performance measures such
as delivery speed and customer satisfaction?

7. Explain the difference between short-term and long-term performance measures and give
two or three examples of each.

8. What is a critical success factor, and what is its role in strategic management and in cost
management?

9. Identify four or five potential critical success factors for a small auto-repair shop.

10. What is a balanced scorecard? What is the primary objective when using a balanced
scorecard?

11. Name the ten contemporary management techniques and describe each briefly.

12. How do managers implement strategy?

13. What should managers do to compete effectively?

14. How do companies add value?

15. What does the phrase "what gets measured gets done" mean? Provide an example of a
negative consequence for an organization if this phrase is taken literally
16. What are some new measures of performance that management accountants are
beginning to consider as part of their domain?

17. Is customer satisfaction a qualitative or a quantitative measure of performance? Or, is it a


combination of both measures?

18. Define competitive benchmarking and continuous improvement.

19. Define value-added and nonvalue-added activities. Which of the following would be
value-added for an automotive manufacturer? Nonvalue-added?

a. painting automobiles
b. moving auto parts from the warehouse to the plant
c. inspection of final product
d. assembling the engines
e. costs to store finished goods
f. costs to store raw materials
g. production of bumpers
h. production of headlights
i. inspection of intermediate product
j. production of spark plugs for automobile
Exercises

Exercise 1 (Strategy, Competitive Advantage)

In the mid-1970s a large retailer of auto parts, Best Parts, Inc. (BPI), was looking for
ways to invest an accumulation of excess cash. BPI’s success was built on a carefully
developed inventory control system that guaranteed a customer would be able to purchase
a desired part 99m percent of the time on demand, and the remaining 1 percent of the
time within one business day. The speed and quality of service set BPI apart from other
parts dealers, and the business continued to grow.

On the advice of close friends and consultants, the owner and CEO of BPI decided to
invest a significant portion of the excess cash in small chain of gift and craft stores. The
stores would be placed in shopping malls.

Required:

Determine the competitive advantage (cost leadership, or differentiation) of BPI in the


auto-parts business. Assess how this competitive advantage would or would not facilitate
success in the new venture.

Exercise 2 (Strategy, Contemporary Management Techniques)

One of the large auto manufacturers in the 1970s developed a sport version of its family
sedan. The new version was equipped with a small V-8 engine and other performance
improvements. The car was called the Pirouette, because of its graceful appearance and
performance. Unfortunately, there was a difficulty in servicing the vehicle. The engine
was too large for the space available, and it had to be moved slightly on the engine
mounts in order for one of the spark plugs to be changed.

Required:

Comment on the strategic competitive advantage of the Pirouette. What type of


management technique was likely used in its design? What type of design approach
should have been used?

Exercise 3 (Value Chain Analysis)

Ram Radio manufactures yacht radios, navigational equipment and depth sounding and
related equipment from a small plant near MNR-North, Tuguegarao City. One of Ram’s
most popular products, making up 40 percent of its revenues and 35 percent of its profits,
is a marine radio, model VF4500, which is installed on many of the new large boats
produced in the Northern Luzon. Average production and sales are 500 units per month.
Ram has achieved its success in the market though excellent customer service and
product reliability. The manufacturing process consist primarily of assembly of
components purchased from various electronics forms, plus a small amount of
metalworking and finishing. The manufacturing operations cost P110 per unit. The
purchased parts cost Ram P250, of which P130 is for parts that Ram could manufacture
in its existing facility for P80 in materials for each unit, plus an investment in labor and
equipment that would cost P35,000 per month.

Ram is considering outsourcing to another MNR firm, Basher Enterprises, the marketing,
distribution and servicing for its units. This would save Ram P125,000 in monthly
materials and labor costs. Yje cost of the contract would be P105 per radio.

Required:

1. Prepare a value-chain analysis for Ram to assist in the decision whether to purchase or
manufacture the parts, and whether to contract out the marketing, distribution, and
servicing of the units.

2. Should Ram (a) continue to purchase the parts or manufacture them and (b) continue
to provide the marketing, distribution and service or outsource this activity to Basher?
Explain your answer.

Multiple Choice

1. Well-implemented just-in-time production and purchasing techniques


a. result in large stockpiles of inventory to keep production running.
b. strengthen a company' ability to compete in the marketplace.
c. increase a reliance on long-term consumer forecasts.
d. reduce a company's competitive edge.

2. Computer-integrated manufacturing (CIM) plants allow management to do all EXCEPT


a. create brand recognition.
b. diagnose the reason for a defect.
c. access timely and accurate information reading production costs.
d. respond faster to changes in customer preferences.
Developing a Competitive Strategy; Contemporary Cost Management Techniques 89

3. ___________ is/are when a firm compares itself with the best practice of competitors or
other comparable organizations.
a. Value chain c. Key success factors
b. Supply chain d. Benchmarking

4. R&D, production and customer service are business functions that are all included as of
the value chain.
a. the value chain. c. marketing.
b. benchmarking. d. the supply chain.

5. The value chain is the sequence of business functions in which


a. value is deducted from the products or services of an organization.
b. value is proportionately added to the products or services of an organization.
c. products and services are evaluated with respect to their value to the supply chain.
d. usefulness is added to the products or services of an organization.

6. _____________ is the generation of, and experimentation with, ideas related to new
products, services, or processes.
a. Research and development
b. Design of products, services, or processes
c. Production
d. Marketing

7. _____________ is the acquisition, coordination, and assembly of resources to produce a


product or deliver a service.
a. Research and development c. Production
b. Customer services d. Marketing

8. ______________ is an operational factor that directly affects the economic ability of the
organization.
a. Customer focus c. Continuous
improvement
b. A key success factor d. Supply chain

9. Customers are demanding improved performance related to


a. reduced costs
b. both reduced costs and increased quality
c. lower cost, improved quality, and improved customer service
d. lower cost, improved quality, improved customer service, and innovative products
and services.
90 Chapter 4
.

10. The sequence of activities that creates a good or service is:


a. an organization. c. a customer chain.
b. a value chain. d. an information system.

11. There are four broad classes of activities in the value chain. Research and development
would be in which class?
a. activities relating to getting ready to make the product
b. activities related to making the product
c. activities to dealing with the customer
d. other activities that support the first three activities

12. There are four broad classes of activities in the value chain. Storing work in process
would be in which class?
a. activities relating to getting ready to make the product
b. activities related to making the product
c. activities related to dealing with the customer
d. other activities that support the first three activities

13. There are four broad classes of activities in the value chain. Billing activities would be in
which class?
a. activities relating to getting ready to make the product
b. activities related to making the product
c. activities related to dealing with the customer
d. other activities that support the first three activities

14. There are our four broad classes of activities in the value chain. Accounting activities
would be in which class?
a. activities relating to getting ready to make the product
b. activities related to making the product
c. activities related to dealing with the customer
d. other activities that support the first three activities

15. Which of the following activities is value-added?


a. Processing c. storing
b. Moving d. inspecting

16. Which of the following four general steps to improve the effectiveness and efficiency of
an organization's activities would be performed first?
Developing a Competitive Strategy; Contemporary Cost Management Techniques 91

a. identify what is now being done


b. measure current performance
c. Analysis
d. Improve

17. Employees improve effectiveness and efficiency by performing general steps regarding
the organization's activities. The following describes which step?

The employee measures the performance of each activity in the (value chain) from the
perspective of customer while assuring that the overall performance of activities meets
the requirements of the organization’s other stakeholders.
a. Identify what is now being done.
b. Measure current performance.
c. Analyze.
d. Improve.

18. Which of the following would NOT happen when quality is bad?
a. Rework
b. Scrap
c. zero-defects
d. an increase in the cost of good units increases

19. Which of the following statements is true?


a. The customer will choose the product with the lower price.
b. If two products provide the same services and quality, the customer will choose
the product with the lower price.
c. If two products provide the same services and quality, the customer will choose
the product with the higher price.
d. The customer will choose the product with the higher price.

20. A key advantage of cross-functional teams in today's manufacturing and service


environments is
a. information is shared much more quickly by people in different functions and this
helps speed products to market.
b. the physical size of the organization gets smaller.
c. there are fewer computing system requirements.
d. the number of senior managers decreases.

21. Rewarding team performance based on team output can cause problems for team
members because
92 Chapter 4
.

a. some team members work different shifts than others.


b. not all team members pull their weight.
c. the team leader has to get a much higher share of the reward and team members
resent it.
d. day shift workers are entitled to a higher share of the reward than evening shift
workers

22. Continuous education has the following advantages, EXCEPT


a. Employees learn about organizational changes and improvements.
b. employees skills are kept up-to-date.
c. employees learn too much too quickly and get frustrated.
d. employees become more committed to their jobs as they believe the organization
is investing in them.

23. Goal congruence means


a. an employee has set high goals for him/herself.
b. an employee has set low goals for him/herself.
c. an employee's goals are aligned with those of the organization.
d. an employee will never attain his or her goals.

24. A management approach that emphasizes the importance of managing constraints


a. Decentralization c. Control
b. Theory of Constraints d. Business Process

25. A production system in which units are produced and materials purchased only as needed
to meet actual customer demand is called
a. Total quality management c. Process reengineering
b. Just-in-time d. Benchmarking

26. In Process Reengineering, two objectives are to simplify and to eliminate


a. Constraint c. Nonconstraint
b. Non-valued-added activities d. Losses

27. A detailed report to management comparing budgeted data with actual data for a specific
time period is called a
a. Performance report c. Financial accounting report
b. Feedback d. Budget

28. The critical success factors for a business today are all:
Developing a Competitive Strategy; Contemporary Cost Management Techniques 93

a. planning-oriented c. sales-oriented
b. production-oriented d. customer-oriented
29. A process by which a firm identifies its critical success factors, studies the best practices
of other firms for these critical success factors, and then implements improvements in the
firm's processes to match or beat the performance of these competitors is termed:
a. continuous improvement c. strategic management
b. reengineering d. benchmarking

30. A technique in which management develops policies and practices to ensure that the
firm's products and services exceed the customer's expectations is:
a. continuous improvement c. critical success factoring
b. benchmarking d. total quality management

31. A process for creating competitive advantage in which a firm reorganizes its operating
and management functions, often with the result that jobs are modified, combined, or
eliminated is termed:
a. benchmarking c. target costing
b. life cycle costing d. reengineering

32. A strategic technique to help firms effectively improve the most common and important
critical success factor - cycle time, is:
a. activity-based costing c. the theory of constraints
b. benchmarking d. continuous improvement

33. The competitive strategy of "cost leadership" allows a firm to out-perform competitors by
producing products or services:
a. with lowered quality standards.
b. in smaller operational units.
c. at lower costs achieved by increased productivity.
d. with attractive added features.

34. The competitive strategy of "differentiation" requires that a product or service must be:
a. always readily available.
b. price competitive.
c. produced at the lowest possible cost.
d. unique in some important way, usually of being of higher quality.

35. Many firms find that a consideration of critical success factors yields a renewed focus on
the three key factors of:
a. product design, manufacture and distribution
b.
c. cost, price and volume
d. innovation, regulation and utilization
e. cost, quality and speed of product development and delivery

36. After critical success factors (CSFs) have been identified, the next step in developing a
competitive is to develop relevant and reliable measure for these CFSs. If these measures
are not developed, a firm cannot hope to:
a. make profit for any extended period.
b. increase sales above previous year(s).
c. develop policies to enhance profitability.
d. monitor its progress toward achieving its strategic goals.

37. In order to achieve a firm's objectives, the strategic cost accounting system must collect,
record and report:
a. the right king of information.
b. information on a very regular basis.
c. only incremental information.
d. detailed information.

38. The "balanced scorecard" accounting report can be made more effective by developing it
at a detail level so that employees:
a. can see how it is put together.
b. appreciate all the effort that goes into its preparation.
c. respect management for including them in its formulation.
d. can see how their actions contribute to the success of the firm.

39. Both leading and lagging indicators should be used in the development of the "balanced
scorecard" accounting report because:
a. leading indicators are future oriented and lagging indicators are primarily
historical output measures.
b. one type of indicator will always correct the other type.
c. leading indicators are express non-quantitatively while lagging indicators are
expressed only in quantitative terms.
d. both answer a and answer b are correct.

40. The objective of the value chain analysis is to identify stages of the value chain where the
firm can:
a. justify increases in the price of the product or service.
b. increase value to the customer or reduce cost.
c. sublet production to other producers.
d. Answer b is most correct, but answer a and c are possibly true.

41. The second step in value chain analysis is to identify the cost driver(s) at each value
activity. The objective of the second step is to identify activities where the firm has a
current or future
a. revenue potential. c. cost advantage.
b. legal responsibility. d. cost overrun.

42. In value chain analysis, the third step choice made by a firm to emphasize its strong
research and development reputation is an example of:
a. low cost manufacturing. c. cost leadership.
b. price leadership. d. innovative design.

43. In regard to critical success factors, which one of the following would not be considered a
financial measure of success?
a. cash flow c. sales growth
b. brand growth d. earnings growth

44. In regard to critical success factors, which one of the following would not be considered a
non-financial customer measure of success?
a. education c. customer satisfaction
b. on-time delivery d. customer service

45. In regard to critical success factors, which one of the following would not be considered a
non-financial internal business process measure of success?
a. cycle time c. high product quality
b. Yield d. market share

46. Which of the following financial critical success factors is measured by earnings from
operations?
a. profitability c. liquidity
b. sales d. flexibility

47. Many firms are finding it is difficult to compete successfully on cost leadership or
differentiation alone, and they must, in fact, compete on both:
a. cost and design c. cost and price
b. price and functionality d. design and
functionality
CHAPTER
STRATEGY AND THE
5 MASTER BUDGET

EXPECTED LEARNING OUTCOMES

After studying this chapter, you should be able to...

1. Explain what a budget is and its role in the management process


2. Describe the importance of strategy in budgeting and its relationship to the strategic long-
term and short-term goals of the firm
3. Explain the management process of preparing the master budget
4. Prepare a sales budget including a computation of expected cash receipts
5. Prepare a production budget, direct materials budget, direct labor budget including the
computation of expected cash disbursements for purchases of material and payment of direct
labor
6. Prepare a manufacturing overhead budget and a selling and administrative expense budget
and expected cash disbursements relative thereto
7. Prepare a cash budget
8. Prepare a budgeted income statement and a budgeted statement of financial position
9. Describe the alternative approach in budgeting such as zero-base budgeting, activity-based
budgeting and Kaizen (Continuous improvement) budgeting
10. Explain the ethical issues in budgeting
CHAPTER 5

STRATEGY AND THE MASTER BUDGET

Repeated references to budget allowances have been made throughout previous


chapters and we have seen how closely accounting and budgeting are related and how
one depends on the other. Accounting draws some of its data from planned
performances established in the budget; in turn, recorded historical data provide a basis
for determining budget estimates.

ROLE OF A BUDGET

A budget is a financial plan of the resources needed to carry out tasks and meet
financial goals. It is also a quantitative expression of the goals the organization wishes
to achieve and the cost of attaining these goals.

The act of preparing a budget is called budgeting. The use of budgets to control a firm's
activities is known as budgetary control.

Budgets and the budgeting process are intertwined with all aspects of management. In
addition to being a plan of operations, a budget plays an important role in allocating
resources, coordinating operations, identifying constraints and limitations, and
communicating expected actions and results, authorizing activities, motivating and
guiding implementation, providing guidelines for control of operations, managing cash
flows, and serving as criteria in performance evaluations.

In the process of preparing a firm's budget, managers need to be forward-looking in


evaluating upcoming events and situations as they relate to the firm's strategic goals.
Budget preparation allows management the opportunity and time to work out any
potential problems that the company might meet in the coming periods. The firm is able
to minimize if not totally avoid the adverse effects that anticipated problem could have
on operation. Budgeting can also help managers identify current and potential
bottleneck in operation. Critical resources can then be mustered to ease any bottleneck
in operations and prevent them from becoming obstacles to attaining budgeting goals.
Completion of a budget for all units of an organization also mandates coordinating
operations among all budgeted units and synchronizing the operating activities of
various department. Budgets help firms to run smoother operations and achieve better
results

Properly conceived, budgeting can mean the difference between a general drift that
may or may not lead to a desired goal and a carefully plotted course toward a
predetermined objective that holds drift to a minimum. Budgets make the decision-
making process more effective by helping managers meet uncertainties, The objective
of budgeting is to substitute deliberate, well-conceived business judgment for accidental
success in enterprise management. Budgets should not be expressions of wishful
thinking but rather descriptions of attainable objectives.

IMPORTANCE OF STRATEGY IN BUDGETING

As discussed in the earlier chapters, a firm's strategy is the path it chooses for attaining
its long-term goals and mission. It is the starting point in preparing its plans and
budgets. For example, a very large and successful Retail Department Store in the
Philippines considers itself the leader of top-of-the-line consumer goods (wardrobe,
designer bags and shoes, accessories, housewares, and so forth. One of its
departments is the Supermarket and Grocery Department, selling food items, fresh and
processed and other household needs. Even though this department is a little bit
profitable, for competitive reasons, the firm made a strategic decision to leave this line
of business. Subsequent budgets of this firm reflect this strategic decision.

Formulation of Strategy

The process of determining a company's strategy starts with the assessment of external
factors that affect operation and evaluating internal factors than can be its strength and
weakness.

External factors typically include


 Competition
 Technical, economic political, regulatory, social and environmental factors

A careful examination of such factors can help the firm identify opportunities, limitations
and threats.
Internal factors on the other hand, include operating characteristics such as

 Financial strength
 Managerial talent and expertise
 Functional structure, and
 Organizational culture

Matching the firm's strengths with its identified opportunities, resources and threats
enables it to form its strategy.

Having analyzed external factors surrounding the organization and assessed the
internal situations it is in, management can match opportunities with strength and
competitive advantages of the firm and determine its strategies and long-term
opportunities.

Figure 5-1 summarizes the activities involved in the development of a firm's product
strategy.

Figure 5-1: Development of a Firm's Product Strategy


Strategic Goals and Long-Term Objectives

An organization presents its strategic goals and long-term objectives through capital
budget and master budgets.

Strategy provides the framework or parameters within which a long-range plan is


developed. A firm's long-range plan identifies required actions over a 5-year to 10-year
period to attain the goals set forth in their strategies.

Long Range Planning

Long range planning often entails capital budgeting, which is a process of evaluating
proposed major projects such as purchases of new equipment, construction of a new
factory, and addition of new products and planning for resource requirements. Capital
budgets are prepared to bring an organization's capabilities into line with the needs of
its long-rang plan and long-term forecast. An organization's capacity is a result of capital
investments made in prior budgeting periods.

Capital budgeting is discussed in more details in Chapter 13.

Short-Term Objectives and The Master Budget

Short-term objectives are goals for the coming period, which can be a month, a quarter,
a year, or any length of time desired by the organization for planning purposes. A firm
determines short-term objectives for the budget period based on strategic goals, long-
term objectives and plans, operating results of past periods, and expected future
operating and environmental factors including economic, industry and marketing
conditions.

Figure 5-1 presents the relationship between strategic goals, long-term objectives and
plan, short-term goals, budget, operations and control.
Figure 5-2: The Relationship between Strategic Goals, Long-Term Objectives and
Plans, Short-Term Goals, Budget, Operations and Control
THE MANAGEMENT PROCESS OF PREPARING THE MASTER BUDGET

Top Management Involvement

For a budget to be effective, top management needs to be involved and show strong
interest in budget results Too much involvement, however, may make the budget and
alienate lower managers. The right answer is a good balance of top management
involvement with lower-level managers.

Top management ensures that budget guidelines are being followed through the budget
review and approval process. Active involvement by top management in reviewing and
approving the proposed budget is an effective way to discourage lower-level managers
from playing budget games (e.g., budgets with easy, target and adding stock to a
budget).

Budgeting processes usually include formation of a budget committee; determination of


the budget guidelines: preparation of the initial budget proposal; budget negotiation,
review and approval: and budget revision.

Organization for Budget Preparation

It is essential that the manager of an entity assigns the most qualified personnel to the
preparation of the budget. A budget committee with representation from the different
functional areas (marketing, production, finance, and administration) is generally
considered an effective body to oversee preparation and administration of the budget.
The controller may be selected to serve as head of the committee for two major
reasons:

1) Controller's position is independent from the operating parts of the organization.

2) He has the skills and experiences in coping with the intricacies of setting up a
budget.

The controller acts as a coordinator in the budgeting operation. He recommends how


budgets should be prepared, assembles the budgets, prepares periodic reports showing
variances of the actual results from the budgeted results, interprets variances and offers
suggestions for improvement whenever possible.

The budget committee decides how budgets shall be prepared, passes on the final
budget, and settles disputes in one segment of the business and another when
differences of opinion arise. The committee also receives budget reports and makes
policy decisions with respect to budget revisions and other problems of budget
administration.

Budget Guidelines

One of the responsibilities of the budget committee is to provide initial budget guidelines
that set the tone for the budget and govern budget preparation. All responsibility centers
(or budget units follow the initial budget guidelines in preparing their budgets.

The starting point in developing budget guidelines is the firm's strategy. In developing
the initial budget guidelines, the budget committee also needs to consider development
that have occurred since the adoption of the strategic plan; the general outlook of the
economy and the market; the goal of the organization for the budgeting period; specific
corporate policies such as mandates for downsizing, reengineering, and the operating
results of the year to date.

The Budget Period

As a general rule, the period covered by a budget should be long enough to show the
effect of managerial policies but short enough so that estimates can be made with
reasonable accuracy. This suggests that different types of budgets should be made for
different time spans.

A master budget is an overall financial and operating plan for a coming fiscal period and
the coordinated program for achieving the plan. It is usually prepared on a quarterly or
an annual basis. Long range budgets called capital budgets. which incorporate plans for
major expenditures for plant and equipment or the addition of product lines, might be
prepared to cover plans for as long as 5 to 10 years. Responsibility budgets which are
segments of the master budget relating to the aspect of the business that is the
responsibility of a particular manager are often prepared monthly. Cash budgets may be
prepared on a day-to-day or monthly basis. Some companies follow a continuous
budgeting plan whereby budgets are constantly reviewed and updated. The updating is
accomplished, for example, by extending the annual budget one additional month at the
end of each month. A review of the budget may also suggest that the budget be
changed as a result of changing business and operating conditions.
The Initial Budget Proposal

Based on the initial budget guidelines, each responsibility center prepares its initial
budget proposal. In preparing an initial budget proposal, the following factors should be
considered by a budget unit.

Internal factors:
 Introduction of new products.
 Adoption of new manufacturing processes
 Changes in availability of equipment or facilities.
 Changes in product design or product mix.
 Changes in expectations or operating processes of other budget units that the
budget unit relies on for its input materials or other operating factors.
 Changes in other operating factors or in the expectations or operating processes
in those other budget units that rely on the budget unit to supply them
components.

External factors:
 Competitor's actions.
 Changes in the labor market.
 Availability of raw materials or components and their prices.
 Industry's outlook for the near term.

Budget Negotiation, Review and Approval, Revision


The head of the budget units examines the initial budget proposal to determine whether
the proposal is within the budget guidelines. The head also checks to see if the budget
goals can be reasonably attained and in line with the goals of the budget units at the
next level up, and the budgeted operations are consistent with the budgeted activities of
another budget unit. Negotiation occur at all levels of the organization. As budget units
approve their budgets, the budgets go through the successive levels of the organization
until they reach the final level, when the combined unit budgets become the budget of
the organization.

The budget committee reviews and gives final approval to the budget. The chief
executive officer then approves the entire budget and submits the budget to the board
of directors.
Systematic, periodic revision of the approved budget or the use of a continuous budget
can be an advantage in dynamic operations because the updated budget provides
better operating guidelines. Regular budget revision, however, may encourage
responsibility centers not to prepare their budgets with due diligence. Organizations with
systematic budget revisions need to ensure that revisions are allowed only if
circumstances have changed significantly.

The Master Budget

A master budget is a comprehensive budget for a specific period, it consists of many


interrelated operating and financial budgets. Some firms refer to the process of
preparing a master budget as profit planning or targeting.

Figure 5-3 delineates the relationships among components of a master budget.

Figure 5-3: The Master Budget


Steps in Developing a Master Budget

The major steps in developing a Master Budget may be outlined as follows:

1. Establish basic goals and long-range plans for the company. These will serve as
guidelines in the preparation of budget estimates.
2. Prepare a sales forecast for the budget period.
3. Estimate the cost of sales and operating expenses.
4. Determine the effect of budgeted operating results on assets, liabilities and
ownership equity accounts. The cash budget is the largest part of this step, since
changes in many asset and liability accounts will depend upon the cash flow
forecast.
5. Summarize the estimated data in the form of a projected income statement for
the budget period, the projected statement of financial position as of the end of
the budget period and the projected cash flow statement.

Preparation of Comprehensive Master Budget Illustrated

Gilbert Manufacturing Company manufactures a special line of tools. As of December


31, 20X1, the Statement of Financial Position of the firm is as follows:

Gilbert Company
Statement of Financial Position
December 21, 20X1
Assets Equities
Current Assets Current Liabilities
Cash P 150,000 Accounts Payable P 140,000
Accounts Receivable 220,000 Taxes Payable 156,000
Inventories 592,000 Current portion of long-
Other Current Assets 23,000 term debt 83,000
Total Current Assets P 958,000 Total Current Liabilities P 379,000
Long-Term Liabilities 576,000
Total Liabilities P955,000

Long-Term Assets Equity


Property, plant and Share capital P 350,000
equipment P 2,475,000 Retained earnings 1,035,000
Less: Accumulated Total P 1,655,000
Depreciation 850,000
Net P 1,625,000 Total Equities P 2,610,000
Total Assets P 2,610,000
The following information is available for the development of its Master Budget for 20X2:

Estimated Sales:
Units 6,400
Price per unit P 800
Finished goods inventory:
Beginning 900 units @ P500
Ending 1,000
Work in process inventory:
NONE

Raw Materials:
Material
R S
Materials required per unit of finished
product 3 units 5 units
Beginning inventory 2,200 4,000
Ending inventory 1,300 4,600
Unit cost P 10 P 30
Direct labor P146 Per unit produced

Overhead is estimated as follows


Variable
Indirect Materials and supplies P 5.85 Per unit produced
Materials handling 9.07 Per unit
Other indirect labor 5.07 Per unit
Fixed
Supervisor labor P 175,000
Maintenance & repair 85,000
Plant administration 173,000
Utilities 87,000
Depreciation 280,000
Insurance 43,000
Property taxes 117,000
Other 41,000
Marketing and Administrative expenses are budgeted as follows:
Variable Marketing Costs:
Sales commissions P 40.625 Per unit sold
Other marketing costs P 16.250 Per unit sold
Fixed Marketing Costs:
Sales salaries P 100,000
Advertising 193,000
Other 78,000
Administrative costs (all fixed):
Administrative salaries P 254,000
Data processing services 103,000
Legal and other professional fees 180,000
Depreciation - building, furniture
and equipment 94,000
Taxes - other than income 160,000
Other 26,000

Additional information:

The treasurer's office also provided the following information and estimates:

1) All sales are on account and collections from customers are expected to amount
to P5,185,000.
2) Equipment costing P300,000 with accumulated depreciation of P275,000 will be
sold at its net book value. New equipment costing P320,000 will be purchased
during the year.
3) Accounts payable will increase by P15,000 and assumed to be for materials
purchases only.
4) Income taxes will be provided at an average rate of 35% of income before taxes
while P252,000 will be paid during the year.
5) Dividends amounting to P140,000 will be paid during the year and the current
portion of the long-term debt shall also be settled at the end of the year. Interest
rate is 8% per annum.

REQUIRED: Prepare the Master Budget for Gilbert Company for the year ending
December 31, 20X2.

Based on the above preliminary data, each of Gilbert Company's budgets will now be
discussed and illustrated.
Sales Budget

The sales budget showing what products will be sold in what quantities at what prices,
is the foundation on which all other short-term budgets are built. The sales budget
triggers a chain reaction that leads to the development of many other budget figures in
an organization. The sales budget provides the revenue predictions from which cash
receipts from customers can be estimated and supplies the basic data for constructing
budgets for production cost and selling and administrative expenses. In short, the sales
forecast is the keystone of the budget structure. The accuracy and reasonableness of
the sales data will affect the whole budget. The sales forecast is made after
consideration of the following factors.

1. Past sales volume


2. General economic and industry conditions
3. Relationship of sales to economic indicators
4. Relative product profitability
5. Market research studies and competition
6. Pricing, advertising and other promotion policies
7. Production capacity
8. Quality of sales force
9. Seasonal variations
10. Long-term sales trends for various products

For Gilbert Company, the Sales Budget is presented as follows:

Schedule 1
Sales Budget
for 20X2
Price per Total sales
Units unit revenue
Estimated Sales 6,400 800 P 5,120,000

Production Budget

After the sales budget has been set, a decision can be made on the level of production
that will be needed for the period to support sales and the production budget can be set
as well. The production budget becomes a key factor in the determination of other
budgets, including the direct materials budget, the direct labor budget and the
manufacturing overhead budget. These budgets in turn are needed to assist in
formulating a cash budget.
Using the data from the previously prepared sales budget as well as the inventory
summary information, the following production budget is developed.

Schedule 2
Production Budget
For 20X2
Units to be sold 6400
Add: Desired ending inventory 1000
Total 7400
Less: Beginning inventory 900
Units to be produced 6500

Raw Materials Budget

After determining the number of units to be produced, the Raw Materials Purchases can
now be prepared, as follows:

Schedule 3
Raw Materials Purchases
For 20X2
Materials
R S
Units required for production
R (6500 x 3) 19,500
S (6500 x 5) 32,500
Add. Desired ending inventory 1,300 4,600
Total units required 20,800 37,100
Less: Beginning inventory 2,200 4,000
Units to be purchased 18,600 33,100
Unit price x P10 x P30
Total purchases P 186,000 P 993,000

Direct Labor Budget


The preliminary data show that the budgeted direct labor cost per unit produced is
P146. This must have been arrived at after considering such factors as skills level of the
workers, labor rate per hour, time requirement, conditions of union contracts, etc.
The direct labor is therefore budgeted as follows:
Schedule 4
Number of units to be produced 6,500
Multiply by: Direct labor cost per unit P146
Total budgeted direct labor costs P949,000
Overhead Costs Budget
Study of past records will show how the cost reacts to changes in volume or in relation
to other factors. Some overhead items may be projected on the basis of direct labor
hours or on materials costs or on machine hours.

The overhead costs budget for 20X2 is illustrated below using the basic information
from the preliminary data previously established.

Schedule 5
Budgeted Manufacturing Overhead
For 20X2
Variable overhead: units needed to produce 6500 units
Indirect materials and supplies (@P5.85) P 38,000
Materials handling (@P9.07) 59,000
Other indirect labor (@P5.07) 33,000
130,000
Total
Fixed manufacturing overhead
Supervisor labor 175,000
Maintenance & repairs 85,000
Plant administration 173,000
Utilities 87.000
Depreciation 280,000
Insurance 43,000
117,000
Property taxes
41,000
Others
1,001,000
Total
Total manufacturing overhead P 1,131,000

Budgeted Cost of Sales

The Budgeted Cost of Sales Statement can now be developed using the data from the
following:

Production Budget Schedule 2


Raw Materials Budget Schedule 3
Direct Labor Budget Schedule 4
Overhead Cost Budget Schedule 5
Budgeted Statement of
Cost of Sales Schedule 6
Schedule 6
Budgeted Statement of Cost of Sales
For 20X2
Beginning work in process inventory P___-___
Manufacturing costs
Direct materials
Beginning inventory
[(2200 R @P10) + (4000 S @P30)] P 142,000
Purchases (Schedule 3) 1,179,000
Total P1,321,000
Less: Ending inventory
[(1300 R @ P10) + (1600 S @ P30) 151,000
Total direct materials cost P1,170,000
Direct labor (6500 @ P146) 949,000
Manufacturing overhead (Schedule 5) 1,131,000
Total manufacturing cost P3,250,000
Less: Ending work in process inventory ____-____
Cost of goods manufactured P3,250,000
Add: Beginning finished goods
inventory (900 @ P500) 450,000
Total available for sale P3,700,000
Less: Ending finished goods
inventory (1000 @ P500) 500,000
Cost of sales P3,200,000
Marketing and Administrative Expense Budget

As with overhead costs, marketing and administrative expenses are also made up of
fixed budget forting variable components. The marketing and administrative expense
budget for 20X2 is shown on the next page. Previously provided data are used.

Schedule 7
Budgeted Marketing and Administrative Costs
For 20X2
Variable marketing costs
Sales commission (6400 @ P40.625) P 260,000
Others (6400 @ P16.25) 104,000
Total P 364,000
Fixed marketing costs
Sales salaries P 100,000
Advertising 193,000
Others 78,000
Total P 371,000
Total marketing costs P 735,000
Administrative costs (all fixed)
Administrative salaries P 254,000
Data processing services 103,000
Legal and other professional fees 180,000
Depreciation - building, furniture and
equipment 94,000
Taxes other than income 108,000
Others 26,000
Total 765,000
Total marketing and administrative costs P1,500,000
Cash Budget

Cash Receipts

Normally, the bulk of a firm's cash receipts comes from customers. The possibility of
cash from other sources (such as additional investments, sales of assets, borrowings)
should likewise be considered when cash receipts are being budgeted.

Cash Disbursements

Data converted from individual budgets previously illustrated supply the basic
information for the cash disbursements budget. However, various adjustments and
additions will have to be made when preparing the budget for prepayments, accruals as
well extraneous items (such as the purchase of new equipment, dividend payment) that
do not show up in any of the individual budgets already prepared. If the financial policy
of the company requires that a minimum cash balance be maintained at all times, the
cash budget must be altered to accommodate bank loans and their repayment.

Using the data collected in the various budgets and the information that has been
previously provided, the following Cash Budget Statement is developed.

Schedule 8
Gilbert Manufacturing Company
Cash Budget
For the Budget Year Ending December 31, 20X2
Cash balance, January 1, 20X1 P 150,000
Add: Estimated receipts
Collections from customers P5,185,000
Sale of assets 25,000
Total P5,210,000
Total cash available P5,360,000
Less: Estimated disbursements
Payments for material purchases P1,164,000
Direct labor 949,000
Manufacturing overhead 851,000
Marketing & Administrative expenses, Interest 1,458,000
Payments for income tax 252,000
Dividends 140,000
Reduction in long-term debt 83,000
Acquisition of new assets 320,000
Total disbursements P5,217,000
Cash balance, December 31 P 143,000
Budgeted Income Statement

After the cash budget has been completed, Gilbert Company prepares the budgeted
income statement showing the net income that is to any pred during the budget period.
The information needed to prepare the budget expected during the budget from the
previously provided preliminary data as well as from me stampment some budgets.

Schedule 9
Gilbert Manufacturing Company
Budgeted Income Statement
For the Budget Year Ending December 31, 20X2
Sales (Schedule 1) P 5,120,000
Less: Cost of sales (Schedule 6) 3,200,000
Gross profit P1,920,000
Less: Marketing and administrative costs (Schedule 7) 1,500,000
Net operating profit P 420.000
Less: Interest expense 52,000
Net income before taxes P 368.000
Less: Provision for income taxes (35%) 128,000
Net income after taxes P 240,000

* Rounded off.

Budgeted Statement of Financial Position

The budgeted statement of financial position is developed by beginning with the current
statement of financial position and adjusting it for the data contained in the other
budgets. Gilbert Company's budgeted statement of financial position is presented
below:

Schedule 10
Gilbert Manufacturing Company
Budgeted Statement of Financial Position
December 31, 20X2
Assets
Current assets
Cash (Schedule 8) P 143,000
Accounts receivable 155,000
Inventories 651,000
Other current assets 23,000
Total current assets P 972,000
Long-term assets
Property, plant and equipment P2,495,000
Less Accumulated depreciation 949,000
Net P1,546,000
P2,518,000
Total assets
Equities
Current liabilities
Accounts payable P 155,000
Taxes payable 32,000
Current portion of long-term debt ___-___
Total current liabilities P 187,000
Long-term liabilities 576,000
Total liabilities P 763,000
Equity
Share capital P 350,000
Retained earnings 1,405,000
Total Equities P2,518,000

Budgeting in Service Industries

A service organization achieves its budgeted goals and fulfills its mission through
providing services.

Budgeting for service firms, similar to budgeting for manufacturing or merchandising


firms, plans for the resources available from operation and the required resources in
operations to fulfill budgeted goals. The difference is in the absence of products or
merchandise purchase budget and their ancillary budget.

An important focal point in its budgeting is personal planning. A service frim must
ensure that it has personnel with the appropriate skills and competence to perform the
services required for the budgeted sales revenue.

Budgeting in Not-For-Profit Organization

A not-for-profit organization's objective is to provide services efficiently and effectively


as mandated in its charter, while not spending more than the allowed expenditure level.

The objectives of not-for-profit organizations such as governments, state universities or


colleges, secondary and primary schools, charity organizations, museums, and
foundations are different from those of for-profit organizations.
With no clear standard by which to measure performance in delivering services, and
with a clear mandate not to exceed budgeted expenditures, the master budget of a not-
for-profit organization often becomes an authorization document for allowable
expenditures and activities. In effect, the budget for a not-for-profit organization often
becomes the source of both the power and limitations of the budgeted unit.

Budgeting in International Setting

Subsidiaries or subdivisions of a multinational firm often have their own budgets. They
must follow the firm's budget procedure and coordinate their budgets with other
divisions of the firm. A Philippine subsidiary of an international firm in Belgium, for
example, must negotiate its budget with the European headquarters or the business unit
to which the subsidiary belongs. The superior divisions then approve the budget in
sequence until the final approval by the corporate budget committee.

Alternative Approaches in Budgeting

Zero-Base Budgeting

Zero-base budgeting is a budgeting process that requires managers to prepare budgets


from a zero base. A typical and traditional budgeting process is an incremental process
that starts with the current budget. The process assumes that most, if not all, current
activities and functions will continue into the budget period. The primary focus in a
typical budgeting process is on changes to the current operating budget.
A zero-base budgeting process on the other hand allows no activities or functions to be
included in the budget unless managers can justify their needs. Zero-base budgeting
requires managers or budgeting teams to perform in-depth reviews and analyses of all
budget items. Such a budgeting process encourages managers to be aware of activities
or functions that have outlived their usefulness or have been a waste of resources. A
tight, efficient budget often results from zero-base budgeting.

Activity-Based Budgeting

Activity-based budgeting (ABB) is a budgeting process based on activities and cost


drivers of operations. ABB starts with the budgeted output and segregates costs
required for the budgeted output into homogenous activity cost pools such as unit,
batch, product-sustaining activity cost pools based on similarity of their resource and
activity consumption cost drivers.
Activity-based budgeting can be a simple extension of a firm's activity-based costing
systems that has grouped its costs into activity cost pools. The firms need to review the
appropriates of its activity cost pools and accuracy of its activity costs for the budget
period, however, before employing them in budgeting. Either internal or external
relevant operating factors may have changed and rendered the data from the current
activity-based costing inaccurate or irrelevant, especially when a firm has experienced
inexplicable variances.

Kaizen (Continuous Improvement) Budgeting

Kaizen budgeting is a budgeting approach that explicitly demands continuous


improvement in operation processes and incorporates the improvements in the budget.
A firm using kaizen budgeting prepares budgets based on the desired future operating
processes for the budget period. This is an improvement over the current operating
processes, rather than the continuation of the current practices as is often the case in
traditional budgeting. It will be noted that continuous improvement (kaizen) has become
a common practice for firms operating in today's globally competitive environment.

Kaizen budgeting begins by analyzing practices to identify areas for improvement and
determine expected changes needed to attain the desired improvements. Budgets are
prepared based on improved practices or produces. As a result, budgeted cost often is
lower than those in the preceding period, and the firm expects to be able to manufacture
products or render services at a lower cost.

ETHICAL ISSUES IN BUDGETING

Ethical issues in budgeting include preventing concealment of information, avoidance of


having a higher budget goal, inclusion of budget slack, and spending the budget to
avoid having it cut back. Behavioral issues in budgeting encompass the difficulty level of
budget targets, the drawbacks and advantages of authoritative and participate
budgeting processes, the extent of involvement of top management in budgeting, and
the role of the budget department or controller on budgeting.

Ethical issues permeate all aspects of budgeting. A significant portion of information


used in budgeting is provided by people whose performance is evaluated against the
budget. Employees breach the code of ethics if they deliberately furnish data for
budgeting purposes that would lead to lower performance expectations.
Including a budget slack, or padding the budget, is the practice of knowingly including a
higher amount of expenditure in the budget than managers truly feel is needed.
Managers often justify such practices as insurance against uncertain future events.

Spending the budget is another serious ethical issue in budgeting. Managers may
believe that if they do not use up all the budgeted amounts, future budgets will be
reduced. To avoid cuts in their budgets, managers may resort to wasteful spending to
exhaust the remaining budgeted amount before the end of the period. As a result.
precious resources are wasted on activities that yield little or no benefit to the firm. Or
necessary assets are acquired to use up remaining funds. Furthermore, time is wasted
on unproductive efforts in trying to use up the budget.

Goal Congruence

Goal congruence is consistency between the goals of the firm and the goals of its
employees. A perfect goal congruence is the ideal for which many firms strive.
Realistically, perfect goal congruence almost never exists because resources for
satisfying short-term goals of individuals are often in conflict with those of the firm. For
example, employees desire to earn a high salary with minimum effort, whereas a firm
seeks to pay employees the lower possible compensation while receiving maximum
efforts from them.

A budget that aligns the goals of the firm with those of its employees has much better
chance of leading to successful operations. One approach that encourages goal
congruence is avoiding authoritative budgeting and using participative budgeting as
much as possible. An employee identifies a budget as their own in participative
budgeting, the goals of the firm and those of its employees become the same.

Authoritative or Participative Budgeting

Budgeting processes are either top down or bottom up. Authoritative budgeting in a top-
down budgeting process top management prepares budgets for the entire organization,
including those for lower-level operations. A participative budgeting process, is a
bottom-up approach that involves the people affected by the budget, including lower-
level employees, in preparing the budget.
Authoritative budgeting provides better decision-making control than participative
budgeting. Top management sets the overall goals for the budget period and prepare a
budget for operations to attain the goals. An authoritative budget, however, often lacks
commitment on the part of the lower-level managers and workers responsible for the
implementation of the budget.

A participative budget is a good communication device. The process of preparing a


participative budget often gives top management a better grasp of the problems their
people face and enables employees to gain a better understanding of the quandaries
top management deals with. A participative budget is more likely to gain the employees'
commitment to fulfill the budgetary goals.

Unless properly controlled, however, a participative budget may lead to easy budget
targets or targets not in compliance with the organization's strategy or budget targets.
An effective budgeting process therefore usually combines both top-down and bottom-
up budgeting approaches. Divisions prepare their initial budgets based on the budget
guidelines issued by the firm's budget committee. Senior managers review and make
suggestions to the proposed budget before sending it back to the divisions for revisions.
The final budget usually from more participation, not enforced negotiations.
REVIEW QUESTIONS, EXERCISES AND PROBLEMS

Questions

1. What is a budget? What is budgetary control?


2. Discuss some of the major benefits to be gained from budgeting.
3. What is a master budget? Briefly describe its contents. 4. Describe the flow of
budget data in an organization. Who are the participants in the budgeting
process, and how do they participate?
4. How can budgeting assist a company in planning its workforce staffing levels?
5. "As a practical matter, planning and control mean exactly the same thing." Do
you agree? Explain.
6. Budgets are half-used if they serve only as a planning device? Explain.
7. What are the two major features of a budgetary program? Which feature is more
important? Why?
8. Explain briefly how a budget can be used in costing products.
9. Why must sales and production be coordinated?
10. How can a labor hour budget be translated into a labor cost budget?
11. How are long-range plans for the acquisition of plant assets included in current
budgets?
12. What is the budget period? Is a budget prepared for a month, for a year, or for
some other interval of time? Explain.
13. What is a rolling, continuous, or progressive budget?
14. Explain how a comparison of actual results with a budget can be applied in the
control of operations.
15. Can a comparison of actual results with a budget lead to better future budgets?
Explain.
16. What is a self-imposed budget? What are the major advantages of self- imposed
budgets? What caution must be exercised in their use?
17. "The principal purpose of the cash budget is to see how much cash the company
will have in the bank at the end of the year." Do you agree? Explain.
18. How does zero-based budgeting differ from traditional budgeting?
Matching Type. Match the definitions enumerated on the right column with the terms
on the left column.

1. Sales forecast A. A quantitative benchmark for measuring


company achievement.
2. Management by exception
B. A budget reflecting long-range decisions of
the company.
3. Responsibility accounting
C. The most important input for budget
preparation. All estimates of activity depend
upon this information.
4. Statement of financial position
D. An integrated plan of action for the firm as
a whole, expressed in financial term.
5. Performance budget
E. A system that relates costs to
organizational structure.
6. Objective
F. An integrated statement of resource levels
and their sources.
7. Capital expenditures budget
G. A set of statements providing broad
direction for the firm.
8. Profit plan
H. The practice of focusing attention on those
activities where the actual performance
differs significantly from planned
9. Master budget performance.

I. A budget prepared after the fact, showing


10. Goals what costs should have been at the actual
level of activity.

J. An operating budget for a specific future


period of time.
Exercises

Exercise 1 (Schedule of Expected Cash Collections)

Peak sales for Mideast Products, Inc., occur in August. The company's sales budget for
the third quarter showing these peak sales is given below:

July August September Total


Budgeted sales P600,000 P900,000 P2,000,000 P2,000,000

From past experience, the company has learned that 20% of a month's sales are
collected in the month of sale, that another 70% is collected in the month following sale,
and that the remaining 10% is collected in the second month following sale. Bad debts
are negligible and can be ignored. May sales totaled P430,000 and June sales totaled
P540,000.

Required:
1. Prepare a schedule of expected cash collections from sales, by month and in
total, for the third quarter.
2. Assume that the company will prepare a budgeted statement of financial position
as of September 30. Compute the accounts receivable as of that date.

Exercise 2 (Production Budget)

Rock Telecom has budgeted the sales of its innovative mobile phone over the next four
months as follows:

Sales in Units
July 30,000
August 45,000
September 60,000
October 50,000

The company is now in the process of preparing a production budget for the third
quarter. Past experience has shown that end-of-month inventories of finished goods
must equal 10% of the next month's sales. The inventory at the end of June was 3,000
units.

Required:
Prepare a production budget for the third quarter showing the number of units to be
produced each month and for the quarter in total.
Exercise 3 (Materials Purchase Budget)

Mini Products, Inc., has developed a very powerful electronic calculator. Each calculator
requires three small "chips" that cost P200 each and are purchased from an overseas
supplier Mini Products has prepared a production budget for the calculator by quarters
for Year 2 and for the first quarter of Year 3, as shown below.

Year 2 Year 3
Budgeted First Second Third Fourth First
production in
calculators
60,000 90,000 150,000 100,000 80,000

The chip used in production of the calculator is sometimes hard to get, so it is


necessary to carry large inventories as a precaution against stockouts. For this reason,
the inventory of chips at the end of the quarter must be equal to 20% of the following
quarter's production needs. Some 36,000 chips will be on hand to start the first quarter
of Year 2.

Required:
Prepare a material purchases budget for chips, by quarter and in total, for Year 2. At the
bottom of your budget, show the peso amount of purchases for each quarter and for the
year in total.

Exercise 4 (Direct Labor Budget)

The Production Department of the Laguna Plant of JC Corporation has submitted the
following forecast of units to be produced at the plant for each quarter of the upcoming
fiscal year. The plant produces high-end outdoor barbeque grills.

1st quarter 2nd quarter 3rd quarter 4th quarter


Units to be produced 5,000 4,400 4,500 4,900

Each unit requires 0.40 direct labor-hours and direct labor-hour workers paid P11 per
hour.
Required:
1. Construct the company's direct labor budget for the upcoming fiscal year,
assuming that the direct labor work force is adjusted each quarter to match the
number of hours required to produce the forecasted number of units produced.
2. Construct the company's direct labor budget for the upcoming fiscal year,
assuming that the direct labor work force is not adjusted each quarter.

Instead, assume that the company's direct labor work force consists of permanent
employees who are guaranteed to be paid for at least 1,800hours of work each quarter.
If the number of required direct labor-hours is less than this number, the workers are
paid for 1,800 hours anyway. Any hours worked in excess of 1,800 hours in a quarter
are paid at the rate of 1.5 times the normal hourly rate for direct labor.

Exercise 5 (Manufacturing Overhead Budget)

The direct labor budget of Kiko Corporation for the upcoming fiscal year contains the
following details concerning budgeted direct labor-hours.
1st quarter 2nd quarter 3rd quarter 4th quarter
Budgeted direct 5,000 4,800 5,200 5,400
labor-hours

The company's variable manufacturing overhead rate is P1.75 per direct labor-hour and
the company's fixed manufacturing overhead is P35,000 per quarter. The only noncash
item included in the fixed manufacturing overhead is depreciation, which is P15,000 per
quarter.

Required:
1. Construct the company's manufacturing overhead budget for the upcoming fiscal
year.
2. Compute the company's manufacturing overhead rate (including both variable
and fixed manufacturing overhead) for the upcoming fiscal year.
Round off to the nearest whole centavos.

Exercise 6 (Selling and Administrative Budget)

The budgeted unit sales of Helene Company for the upcoming fiscal year are provided
below:

1st quarter 2nd quarter 3rd quarter 4th quarter


Budgeted unit sales 12,000 14,000 11,000 10,000

The company's variable selling and administrative expenses per unit are P2.75. Fixed
selling and administrative expenses include advertising expenses of P12,000 per
quarter, executive salaries of P40,000 per quarter, and depreciation of P16,000 per
quarter. In addition, the company will make insurance payments of P 6,00 in the 2 nd
Quarter and P6,000 in the 4 th Quarter. Finally, property taxes of P 6,000 will be paid in
the 3rd Quarter.

Required:
Prepare the company's selling and administrative expense budget for the upcoming
fiscal year.

Exercise 7 (Cash Budget Analysis)

A cash budget, by quarters, is given below for a retail company (000 omitted). The
company requires a minimum cash balance of P5,000 to start each quarter.

Quarter
1 2 3 4 Year
Cash balance, beginning P9 P? P? P? P?
Add collections from customers ? ? 125 ? 391
Total cash available 85 ? ? ? ?
Less disbursements:
Purchase of inventory 40 58 ? 32 ?
Operating expenses ? 42 54 ? 180
Equipment purchases 10 8 8 ? 36
Dividends 2 2 2 2 ?
Total disbursements ? 110 ? ? ?
Excess (deficiency) of cash available over
disbursements (3) ? 30 ? ?
Financing:
Borrowings ? 20 - - ?
Repayments (including interest)* - - (?) (7) (?)
Total financing ? ? ? ? ?
Cash balance, ending P? P? P? P? P?

* Interest will total P4,000 for the year.

Required:
Fill in the missing amounts in the table above.
Problems

Problem 1 (Schedule of Expected Cash Collections and Disbursements)

Cookie Products, a distributor of organic beverages, needs a cash budget for


September. The following information is available:

a. The cash balance at the beginning of September is P9,000.


b. Actual sales for July and August and expected for September as are follows:
July August September
Cash sales P 6,500 P 5,250 P 7,400
Sales on account 20,000 30,000 40,000
Total sales P26,500 P35,250 P47,400

Sales on account are collected over a three-month period in the following ratio: 10%
collected in the month of sale, 70% collected in the month following sale, and 18%
collected in the second month following sale. The remaining 2% is uncollectible.

c. Purchases of inventory will total P25,000 for September. Twenty percent of a month's
inventory purchases are paid for during the month of purchase. The accounts payable
remaining from August's inventory purchases total P16,000, all of which will be paid in
September.
d. Selling and administrative expenses are budgeted at P13,000 for September. Of this
amount, P4,000 is for depreciation.
e. Equipment costing P18,000 will be purchased for cash during September, and
dividends totaling P3,000 will be paid during the month.
f. The company must maintain a minimum cash balance of P5.000. An open line of
credit is available from the company's bank to bolster the cash position as needed.

Required:
1. Prepare a schedule of expected cash collections for September.
2. Prepare a schedule of expected cash disbursements during September for
inventory purchases.
3. Prepare a cash budget for September. Indicate in the financing section any
borrowing that will be needed during September.
Problem 2 (Production and Purchases Budgets)

Tiny Toys manufactures and distributes a number of products to retailers. One of these
products, Toyclay, requires three pounds of material P214 in the manufacture of each
unit. The company is now planning raw materials needs for the third quarter - July,
August, and September. Peak sales of Toyclay occur in the third quarter of each year.
To keep production and shipments moving smoothly, the company has the following
inventory requirements:

a. The finished goods inventory on hand at the end of each month must be equal to
5,000 units plus 30% of the next month's sales. The finished goods inventory on
June 30 is budgeted to be 17,000 units.
b. The raw materials inventory on hand at the end of each month must be equal to
one-half of the following month's production needs for raw materials. The raw
materials inventory on June 30 for material P214 is budgeted to be 64,500 pounds.
c. The company maintains no work in process inventories.

A sales budget for Toyclay for the last six months of the year follows:

Budgeted Sales
in Units
July…………………… 40,000
August……………….. 50,000
September…………… 70,000
October……………… 35,000
November…………… 20,000
December…………… 10,000

Required:

1. Prepare a production budget for Toyclay for the months July, August, September,
and October.

2. Examine the production budget that you prepared. Why will the company produce
more units than it sells in July and August and less units than it sells in September and
October?
4. Prepare a budget showing the quantity of material P214 to be purchased for July,
August and September and for the quarter in total.

Problem 3 (Cash Budget; Income Statement; Statement of Financial Position)

The statement of financial position of Picture This, Inc., a distributor of photographic


supplies, as of May 31 is given below:

PICTURE THIS, INC.


Statement of Financial Position
May 31

Assets
Cash………………………………………………………………. P 8,000
Accounts receivable…………………………………………….. 72,000
Inventory………………………………………………………… 30,000
Buildings and equipment, net of depreciation……………….. 500,000
Total assets………………………………………………………. P 610,000
Liabilities and Equity
Accounts payable , suppliers………………………………... P 90,000
Notes Payable……………………………………………….. 15,000
Share capital, no par………………………………………… 420,000
Retained earnings…………………………………………….. 85,000
Total liabilities and equity…………………………………… P 610,000

Picture This, Inc., has not budgeted previously, and for this reason it is limiting its
master budget planning horizon to just one month ahead - namely, June. The company
has assembled the following budgeted data relating to June:

a. Sales are budgeted at P250,000. Of these sales, P60,000 will be for cash; the
remainder will be credit sales. One-half of a month's credit sales are collected
in the month the sales are made, and the remainder is collected in the month
following. All of the May 31 accounts receivable will be collected in June.

b. Purchases of inventory are expected to total P200,000 during June. These


purchases will all be on account. Forty percent of all inventory purchases are
paid for in the month of purchase; the remainder is paid in the following month.
All of the May 31 accounts payable to suppliers will be paid during June.
c. The June 30 inventory balance is budgeted at P40,000.

d. Operating expenses for June are budgeted at PS1,000, exclusive of depreciation.


These expenses will be paid in cash. Depreciation is budgeted at P2,000 for the month.

c. The note payable on the May 31 statement of financial position will be paid during
June. The company's interest expense for June (on all borrowing) will be P500, which
will be paid in cash.

f. New warehouse equipment costing P9,000 will be purchased for cash during June.

g. During June, the company will borrow P18,000 from its bank by giving a new note
payable to the bank for that amount. The new note will be due in one year.

Required:
1. Prepare a cash budget for June. Support your budget with schedules showing
budgeted cash receipts from sales and budgeted cash payments for inventory
purchases.
2. Prepare a budgeted income statement for June.
3. Prepare a budgeted statement of financial position as June 30.

Problem 4 (Sales, Production and Materials Purchases Budget)

The following information is made available to you by the management of Nikko


Manufacturing Company for the year 20X3:

Sales forecast: 80,000 units of its only product at P30 per unit

The seasonal variations index based on prior years operation is as follows:


1st quarter 80 3rd quarter 110
nd th
2 quarter 100 4 quarter 110

A unit of product requires 3 units of raw materials which are purchased at P5.00 per
unit. Inventories at the beginning and end of the year are:

Beginning End
Finished goods 3,000 units 5,000 units
Raw materials 12,500 units 15,000 units
which are to be maintained at 20% of the succeeding quarter's estimated sales and
production requirements. Purchases of raw materials are to be made to correspond with
budgeted monthly production.

Required:
1) Budget of anticipated sales
2) Statement of production required
3) Statement of raw materials purchase requirements

Problem 5 (Schedule of Expected Cash Collections; Cash Budget)

Jo Tan, president of JTC Products, has just approached the company's bank with a
request for a P30,000, 90-day loan. The purpose of the loan is to assist the company in
acquiring inventories in support of peak April sales. Since the company has had some
difficulty in paying off its loans in the past, the loan officer has asked for a cash budget
to help determine whether the loan should be made. The following data are available for
the months April - June, during which the loan will be used:

a. On April 1, the start of the loan period, the cash balance will be P26,000. Accounts
receivable on April I will total P151,500, of which P141,000 will be collected during
April and P7,200 will be collected during May. The remainder will be uncollectible.

b. Past experience shows that 20% of a month's sales are collected in the month of
sale, 75% in the month following sale, and 4% in the second month following sale.
The other 1% represents bad debts that are never collected. Budgeted sales and
expenses for the period follow:

April May June


Sales………………………. P200,000 P300,000 P250,000
Merchandise purchases….. 120,000 180,000 150,000
Payroll…………………… 9,000 9,000 8,000
Lease payments………….. 15,000 15,000 15,000
Advertising……………… 70.000 80,000 60,000
Equipment purchases…….. 8,000 - -
Depreciation……………. 10,000 10,000 10,000

c. Merchandise purchases are paid in full during the month following purchase.
Accounts payable for merchandise purchases on March 31, which will be paid during
April, total P108,000.
d. In preparing the cash budget, assume that the P30,000 loan will be made in April and
repaid in June. Interest on the loan will total P1,200.

Required:
1. Prepare a schedule of expected cash collections for April, May, and June and
for the three months in total.
2. Prepare a cash budget, by month and in total, for the three-month period.
3. If the company needs a minimum cash balance of P20,000 to start each month,
can the loan be repaid as planned? Explain.

Problem 6 (Flexible Budget)

Factory Overhead
Flexible Budget Preparation

Summary flexible overhead budgets are to be prepared for two departments of Summer
Machine Company. Budget data follows:

Department 1
Manufacturing overhead varies at the rate of P6.50 for each machine hour, and the
fixed manufacturing overhead is budgeted for the year at P300,000. At normal capacity,
the department should operate at 200,000 machine hours.
Department 2
Manufacturing overhead varies at the rate of P7.00 for each direct labor hour, and the
fixed manufacturing overhead is budgeted for the year at P500,000. At normal capacity,
the department should operate at 400,000 machine hours. One direct labor is equal to
two machine hours.

Required: Prepare in summary form a flexible budget for each department showing
costs at normal capacity, at 90, 80, 70, and 60 percent of normal capacity.
Problem 7 (Cash Budget with Supporting Schedules)
Ju Products, Inc., is a merchandising company that sells binders, paper, and other
school supplies. The company is planning its cash needs for the third quarter. In the
past, Ju Products has had to borrow money during the third quarter to support peak
sales of back-to-school materials, which occur during August. The following information
has been assembled to assist in preparing a cash budget for the quarter:
a. Budgeted monthly absorption costing income statements for July-October are as
follows:
July August September October
Sales P 40,000 P 70,000 P 50,000 P 45,000
Cost of goods sold 24,000 42,000 30,000 27,000
Gross margin 16,000 28,000 20,000 18,000
Selling and administrative
expense:
Selling expense 7,200 11,700 8,500 7,300
Administrative expense* 5,600 7,200 6,100 5,900
Total expenses 12,800 18,900 14,600 13,200
Net operating income P 3,200 P 9,100 P 5,400 P 4,800

*Includes P2,000 depreciation each month


b. Sales are 20% for cash and 80% on credit.
c. Credit sales are collected over a three-month period with 10% collected in the month
of sale, 70% in the month following sale, and 20% in the second month following sale.
May sales totaled P30,000, and June sales totaled P36,000.
d. Inventory purchases are paid for within 15 days. Therefore, 50% of a month's
inventory purchases are paid for in the month of purchase. The remaining 50% is paid
in the following month. Accounts payable for inventory purchases at June 30 total
P11,700.
e. The company maintains its ending inventory levels at 75% of the cost of the
merchandise to be sold in the following month. The merchandise inventory at June 30 is
P18,000.
f. Land costing P4,500 will be purchased in July.
g. Dividends of P1,000 will b declared and paid in September.
h. The cash balance on June 30 is P8,000; the company must maintain a cash balance
of at least this amount at the end of each month.

i. The company has an agreement with a local bank that allows the company to borrow
in increments of P 1,000 at the beginning of each month, up to a total loan balance of
P40,000. The interest rate on these loans is 1% per month, and for simplicity, we will
assume that interest is not compounded. The company would, as far as it is able, repay
the loan plus accumulated interest at the end of the quarter.
Required:
1. Prepare a schedule of expected cash collections for July, August and September
and for the quarter in total.
2. Prepare the following for merchandise inventory:
a. A merchandise purchases budget for July, August and September.
b. A schedule of expected cash disbursements for merchandise purchases
for July, August, and September and for the quarter in total.
3. Prepare a cash budget for July, August, and September and for the quarter in
total.

Multiple Choice

1. The first step involved in preparing a master budget is


a. Preparing a forecasted income statement.
b. Preparing a general operating budget.
c. Preparing a forecasted statement of financial position.
d. Preparing detailed period budgets.
2. The second logical step in preparing a master budget would be to:
a. Estimate the cost of goods sold.
b. Forecast sales during the budget period.
c. Establish the basic goals and long-range plans for the company.
d. Forecast general and administrative expenses for the budget period.

3. Which of the following components of the master budget must be prepared before the
others?
a. Direct labor peso budget
b. Cost of goods sold forecast
c. Production budget
d. Raw materials purchase budget

4. Which of the following factors are not important to consider in making a sales
forecast?
a. Past sales volume
b. Distribution costs involved
c. Conditions within the industry
d. Plant capacity
e. None of the above.
5. The period budget should begin with a forecast of
a. overhead.
b. production.
c. sales
d. direct labor

6. The basic difference between a master budget and a flexible budget is:
a. Flexible budget considers only variable costs but a master budget considers all
costs.
b. Flexible budget allows management latitude in meeting goals whereas a master
budget is based on a fixed standard.
c. Master budget is based on one specific level of production and a flexible budget can
be prepared for any production level within a relevant range.
d. Master budget is for an entire production facility but a flexible budget is applicable to
a single department only.

7. A flexible budget is
a. Appropriate for control of factory overhead but not for control of direct materials and
direct labor.
b. Appropriate for control of direct materials and direct labor but not.for control of
factory overhead.
c. Not appropriate when costs and expenses are affected by the fluctuations in volume
limits.
d. Appropriate for any level of activity.

8. The purpose of a flexible budget is to


a. Allow management some latitude in meeting goals.
b. Eliminate cyclical fluctuations in production reports by ignoring variable costs.
c. Compare actual and budgeted results at virtually any level of production.
d. Reduce the total time in preparing the annual budget.

9 When using a flexible budget, what will occur to fixed costs (on a per unit basis) as
production increased within the relevant range?
a. Fixed costs per unit will decrease.
b. Fixed costs per unit will remain unchanged.
c. Fixed costs per unit will increase.
d. Fixed costs are not considered in flexible budgeting.

10. A budget that identifies revenues and costs with an individual controlling their
incurrence is:
a. Budgetary control.
b. Master budget.
c. Product budget.
d. Responsibility budget.
e. None of the above

11. In preparing quarterly budget estimates, who should be responsible for the cash
budget?
a. Sales manager
b. Production manager
c. Finance manager
d. General manager

12. The cash budget is prepared


a. Before all period budgets are prepared.
b. After all forecasted income statement but before the forecasted statement of
financial position.
c. As the last step in the master budget.
d. Only if the company has doubts about the debt-paying ability.

13. The calculation of reasonable probabilities about the future, based on the analysis of
all the latest relevant information by tested and logically sound statistical and
econometric techniques, and applied in terms of an executive's personal judgment and
knowledge of his business is:
a. Budgeting.
b. Planning and control.
c. Business forecasting.
d. Project feasibility studies
e. None of the above.
14. Zero-based budgeting (ZBB) is a
a. Budget system which measures actual or estimated results, in terms of benefit
accruing to the public and their unit cost.
b. Process which requires systematic consideration of all programs, projects and
activities with the use of defined ranking procedures; the activities analyzed and
presented in decision packages.
c. Undertaking that comprises all the functions and activities devoted to the
accomplishment of a major purpose for which government entity is established.
d. Line-item budgeting justified from a base of P1,000.
e. Budget system based on a no appropriation or zero limit of funds.

15. Management by exception uses which accounting tool to a great extent?


a. CVP analysis
b. Financial statement analysis
c. A work sheet
d. Variance analysis

The following information pertains to questions 16 through 20:

The Dilly Company marks up all merchandise at 25% of gross purchase. All purchases
are made on account with terms of 1/10, net/60. Purchase discounts which are recorded
as miscellaneous income, are always taken. Normally, 60% of each month's purchases
are paid for in the month of purchase while the other 40% are paid during the first 10
days of the first month after purchase. Inventories of merchandise at the end of each
month are kept at 30% of the next month's projected cost of goods sold.

Terms for sales on account are 2/10, net/30. Cash sales are not subject to discount.
Fifty percent of each month's sales on account are collected during the month of sale,
45% are collected in the succeeding month and the remainder are usually uncollectible.
Seventy percent of the collections in the month of sale are subject to discount while
10% of the collections in the succeeding month are subject to discount.

Projected sales data for selected months follow:


Sales on Account Gross Cash Sales
December P 1,900,000 P 400,000
January 1,500,000 250,000
February 1,700,000 350,000
March 1,600,000 300,000

16. Projected gross purchases for January are


a. P1,400,000.
b. P1,470,000.
c. P1,472,000.
d. P1,248,000.
e. None of the above.

17. Projected inventory at the end of December is


a. P420,000. d. P552,000.
b. P441,600 e. None of the above.
c. P393,750.
18. Projected payments to suppliers during February are
a P1,551,200. d. P1,509,552.
b. P1,535,688. e. None of the above.
c. P1,528,560.

19. Projected sales discounts to be taken by customers making remittances during


February are
a. P 5,250. d. P11,900.
b. P15,925. e. None of the above.
c. P30,000.

20. Projected total collections from customers during February are


a. P1,875,000. d. P1,188,100.
b. P1,861,750. e. None of the above.
c. P1,511,750,

Sales (90% collectible in 20X3)……………………………………….…. P1,500,000


Bad debt write-offs……………………………………………………….. 60,000
Disbursements of costs and expenses……………………………….. 1,200,000
Disbursements for income taxes………………………………..…… 90,000
Purchases of fixed assets.............................................................. 400,000
Proceeds from issuance of ordinary shares………………………….. 580,000
Proceeds from short-term borrowings……………………………….. 100,000
Payments on short-term borrowings……………………………….…. 50,000
Depreciation on fixed assets……………………………………….... 80,000

What is the cash balance at December 31, 20X3?


a. P150,000 c. P210,000
b. P170,000 d. P280,000
22. David Company has budgeted its activity for April 20X3. Selected data from
estimated amounts are as follows:

Net income P120,000


Increase in gross amount of trade accounts
receivable during month 35,000
Decrease in accounts payable during month 25,000
Depreciation expense 65,000
Provision for income taxes 80,000
Provision for doubtful accounts receivable 45,000

Items 23 through 25 are based on the following data:

Cash P 8,000
Accounts receivable (net of allowance for
uncollectible accounts of P2,000) 38,000
Inventory 16,000
Property, plant and equipment (net of allowance for
accumulated depreciation of P60,000) 40,000
P102,000
Accounts payable P 82,500
Ordinary shares 50,000
Retained earnings (deficit) (30,500)
P102,000

Additional information:
* Sales are budgeted as follows:
February P 110,000
March 120,000

* Collections are expected to be 60% in the month of sale, 38% the next month, and 2%
uncollectible.

* The gross margin is 25% of sales. Purchases each month are 75% of the next month's
projected sales. The purchases are paid in full the following month.

 Other expenses for each month, paid in cash, are expected Depreciation each
month is P5,000.
23. What are the budgeted cash collections for February 20X3?
a. P 63,800 c. P101,800
b. P 66,000 d. P104,000

24. What is the pro-forma income (loss) before income taxes for February 20X37
a. P(3,700) c. P3,800
b. P(1,500) d. P6,000

25. What is the projected balance in accounts payable on February 20X3?


a. P 82,500 c. P 90,000
b. P 86,250 d. P106,500

26. Reliance Company budgets sales at P2,000,000 and expects a net income before
tax of 10% of the sales.

Expenses are estimated as follows:


Selling 15% of sales
Administrative 9% of sales
Finance 1% of sales

Labor is expected to be 40% of the total manufacturing cost. Factory overhead is to be


applied at 75% of direct labor cost.
January 1 December 31
Materials 250,000 300,000
Work in process 200,000 320,000
Finished goods 350,000 400,000

Cost of goods sold will be:


a. P 500,000. d. P2,000,000.
b. P 700,000. e. None of the above.
c. P1,300,000.

27. Refer to Question No. 26. Total manufacturing cost will be:
a. P 741,000. d. P1,470,000.
b. P 882,000. e. None of the above.
c. P1,029,000.

28. Refer to Question No. 26. Factory overhead will amount to:
a. P 441,000. d. P1,029,000.
b. P 491,000. e. None of the above.
c. P 588,000.

29. Refer to Question No. 26. Materials purchases will be:


a. P250,000. d. P741,000.
b. P300,000. e. None of the above.
c. P491,000.

30. Dean Company is preparing a flexible budget for 20X3 and the following maximum
capacity estimates for department M are available:
At Maximum Capacity
Direct labor hours 60,000
Variable factory overhead P150,000
Fixed factory overhead P240,000

Assume that Dean's normal capacity is 80% of maximum capacity. What would be the
total factory overhead rate, based on direct-labor hours, in flexible budget at normal
capacity?
a. P6.00 c. P7.50
b. P6.50 d. P8.13
CHAPTER

6
ORGANIZATIONAL INNOVATIONS:
TOTAL QUALITY MANAGEMENT;
JUST-IN-TIME PRODUCTION SYSTEM

EXPECTED LEARNING OUTCOMES

After studying this chapter, you should be able to...

1. Describe the ultimate test of a quality product or service


2. Explain the core principles of Total Quality Management (TQM)
3. Understand the TQM Implementation Guidelines
4. Explain the broad groups of costs of quality, namely,
 Prevention cost
 Appraisal cost
 Internal cost
 External cost
5. Explain the users, limitations and reporting of quality cost information
6. Describe the nonfinancial measures of quality and customer satisfaction and how
they are reported.
7. Explain the concept of Just-in-Time production system
8. Enumerate and describe the key features and benefits of the JIT production system
9. Describe the performance measures and control in JIT production system
CHAPTER 6

ORGANIZATIONAL INNOVATIONS:
TOTAL QUALITY MANAGEMENT;
JUST -IN-TIME PRODUCTION SYSTEM

INTRODUCTION

A number of organizational innovations have been introduced in Chapter 4.Total Quality


Management(TQM), Just-in-Time (JIT) Production System, Reengineering, Target
Costing, Theory of Constraints, Productivity Measures and Activity-Based Costing and
so forth. This section describes TQM and JIT and their effect on management
accounting and then illustrates how to apply them to better understand the innovations
in terms of what is causing firms to adopt them and whether they will be successful. The
other innovations are discussed in succeeding chapters.

TOTAL QUALITY MANAGEMENT

What is Quality?

Although people often view quality differently because of differences in their roles in the
production-marketing-consumption chain and their expectations for the products or
services. The ultimate test of a quality product or service is whether the product or
service meets or exceeds customers' expectations. The requirement to meet or exceed
customers' expectations then serve as specifications for operations throughout the
organization. Each individual, department or subdivision throughout an organization
needs to strive for conformity to specifications that meet and improve upon customer
satisfaction.

Core Principles of TQM

As aptly described by Procter and Gamble, “Total quality (management)is the unyielding
and continually improving effort by everyone in an organization to understand, meet and
exceed the expectation of customers." This description of TQM points out that the core
principles of TQM are processes that
 Focus on satisfying the customer
 Strive for continuous improvement
 Involve fully the entire work force
 Support and involve top management actively
 Use clear and measurable objective
 Recognize quality achievements in a timely manner
 Provide training on TQM continuously

Focus on the Customer

TQM begins by identifying the firm's customers , external and internal ; determining their
needs , requirements , and expectations ; and then doing whatever it takes to satisfy
them . External customers are the ultimate recipients of the firm's products or services .
Internal customers are individuals or subunits within the firm involved in manufacturing
the product or providing the services . At some stage , everyone in a process or
organization is a customer or supplier to someone else , either inside or outside the
organization .

These requirements and expectations become the bases for specifications for internal
customer / suppliers and external suppliers to the firm . They are then translated into
specifications for each successive internal customer / supplier , including design
requirements , part characteristics , and manufacturing operations , and requirements
for external vendors . A firm can serve its ultimate , external customer better if the firm
fully meets all requirements of each internal customer .

Strive for Continuous Improvement (Kaizen)

The Coca - Cola Company believes that quality is not a destination ; it is a way of life .
Coca - Cola Company states : " We know we will never arrive ; there is no finish . "
Quality is a moving target . Without continuous improvement , quality disappears .

Taguchi and Wu believe that continuous quality improvement and cost reduction
( kaizen ) are necessary to remain competitive in today's global marketplace . " With
competitors forever trying to outperform us and customers exhibiting ever - changing
expectations , a firm can never reach the ideal quality standard . " Firms need to
continuously update specifications for both internal customers / suppliers and external
suppliers to better serve external customers .
Full Involvement of the Entire Workforce

The requirement of the firm's external customers can only be met if each of the internal
customers/suppliers in the process satisfies the requirements of the downstream
process or customer. Any breakdown in the process no matter how insignificant can
lead to a defective product or service and unsatisfied customers. Top management
must encourage everyone in the firm, from the lowest level employees, office clerks,
factory workers, accountants, and engineers to upper echelon professionals and
managers, to be actively involved and to participate in the firm's efforts to continuously
improve quality.
Employee involvement can range from simple information sharing, dialogue or group
problem solving, all the way to total self-direction. One proven effective approach for
employee involvement is quality (control) circles or quality circles (QCs for short). A
‘quality circle is a small group of employees from the same work area that meets
regularly to identify and solve work-related problems and to implement and monitor
solutions to the problems.

Active Support and nvolvement of Top Management

Most companies have found that successful implementation of TQM requires


unwavering and active leadership from the CEO and senior managers.
However, the CEO or top management alone cannot bring forth all the desired
benefits ofTQM. Oniy with support from all managers in the top echelon can
TQM attain the most desirable results. Also, they need to demonstrate their
dedication to total quality to employees at every level, all vendors and
suppliers to the firm, all customers and the community at every opportunity so
that everyone is aware of the primary importance of total quality in every
aspect of the firm's operations.

Use Clear and Measurable Objectives

Progress can easily be seen if objectives are clear. Measurable objectives forge
efforts toward the common goal. To ensure success of total quality
management, a firm must set unambiguous and measurable objectives.
Effective measurement can help to ensure and facilitate quality improvements
and supporting systems.

Timely Recognition of Quality Achievement

Quality achievement of people and subunits when recognized timely is the best
way to emphasize the firm's continuous struggie for better quality and to
ensure efforts toward total quality at every level.
Efforts and progress will most likely be short-lived if the firm makes no change to its
compensation / appraisal / recognition system.

Continuing Education and Training

Employee training programs serve as a communication link to convey


management commitment to total quality and provide employees with
necessary skills to achieve total quality, Mandatory continuing education and
training of employees at all levels is necessary to achieve the culture change
and continuous focus required in a TQM environment.

TQM IMPLEMENTATION GUIDELINES

A firm cannot implement a successful TOM program overnight. It usually takes


any organization serious about achieving TỘM several years of concerted and
dedicated efforts by all its members to become a world-class quality firm.

The implementation of TỌM is not an easy task and is indeed time consuming.
The Institute of Management Accountants believes that a typical organization
takes three to five years to make from traditional management to TQM, Although
Some specific projects can quickly yield high returns, a firm will most likely not
see many tangible benefits in the early years of implementation.

The Institute of Management Accountants (IMA) has devised an 11-phase process


spanning three years to establish TQM. It is recommended that full and genuine
involvement of all employees must be present to implement successfully TQM.

Year One- Preparation and Planning


 Create quality council and staff
 Conduct executive-quality training programs
 Conduct quality audits
 Prepare gap analysis (determine the gap between the best in class and
the firm's current practice)
 Develop strategy on quality improvement

Year-Two- Training and Implementation


 Conduct employee communication and training program
 Establish quality teams
 Create a measurement system and set goals

Year Three - Assessment, Review & Revise


 Revise compensation / appraisal / recognition systems
 Launch external initiative with suppliers
 Review and revise

TYPES OF CONFORMANCE

Quality involves conformance with specifications for product or services that meet
or exceed customer requirements and expectations. Conformance, however, Cân
differ among individuals or firms. The general types however, are

1. Goalpost Conformance (zero-defects conformance)


This is conformance to a quality specification expressed as a specified range around
the target. The target is the ideal or desired outcome of the operations. This is also
called zero-defects conformance with the specified range allowed for variations.
Management expects all outputs to be within the specified range of variations. For
example, a firm specifies a tolerance of + 0.05 inch thickness of its products with a
target of 0.5 thickness. This means that the firm meets the quality of standard when the
result shows that the thickness of its products is between 0.55 inch and 0.45 inch.

2. Absolute quality conformance (robust quality approach)


This is conformance which requires that all products or services to meet target value
exactly with no variation. For example, as absolute conformance requires that all sheet
metal to have a thickness of 0.50 inch, not 0.50 . 0.05 inch or even 0.0005 inch. Any
variation from the target values is less than ideal and can have economic
consequences. Robustness in quality comes with meeting the exact target consistently.
Any deviation from the target is a quality failure and weakness in the overall quality, of
the product or service. Generally, for firms desiring to atain long-term profitability and
customer satisfaction, absolue conformance is considered a better approach that zero-
defects conformance.

Each business function examines its own activities and works to meet improvement
goals. Accountants help operating managers understand how decisions in product
design affect quality (and costs) in manufacturing distribution and customer service.
One of the tools used is benchmarking which involves studying organizations that are
among the bestin the world at performing a particular task. Identification of quality-
related costs is also necessary if TQM is to be applied.
Cost of Quality

Three factors underlie the quality of a product or service. These factors are grade,
quay of design, and quality performance. The bulk of all quality costs are
associated with quality of conformance and these costs can be broken down into
four broad groups.

1. Prevention Costs
These are costs incurred to avoid poor-quality goods or services or reduce the
number of defects in products or services. These include

2. Appraisal Costs
These costs, also called inspection costs, are incurred to identify products before
the products are shipped to customers. These include

3. Internal Failure Costs


These are costs that result from identification of defects during the appraisal
process. Examples are
4. External Failure Costs
These are incurred when poor-quality goods or services are detected after delivery
to customers. They include

Prevention and appraisal costs are costs of conformance because they are incurred
to ensure that products and services meet customers' expectations.

Internal failure and external failure casts are costs of nonconformance because
they are costs incurred and opportunity costs because of rejection of products or
services.

The cost of quality is the sum of conformance and nonconformance costs.

Prevention costs and appraisal costs are incurred in an effort to keep poor quality
of conformance from occurring. The other two groups, known as internal failure
costs and external failure costs, are incurred because poor quality of conformance
has taken place. A company can reduce its total quality costs by focusing its efforts
on prevention and appraisal. By this means, failures are minimized and any defects
are detected before delivery of products to customers.
It will be noted that quality costs do not just relate to manufacturing, rather they
relate to all activities in a company from initial research and development through
customer servicing. Therefore, as part of the quality cost system, companies must
gather the quality costs from each activity and accumulate these costs on a quality
cost report This report then becomes the foundation of the quality cost system in
that it will show management the type of quality costs being incurred, as well as
the amount and trend of these costs.

Uses of Quality Cost Information


The quality cost information is used by managers in several ways. These are
1. Quality cost information provides a basis for establishing budgets for quality
costs as management looks for ways to reduce the total cost involved.
2. Quality cost information helps managers see the financial significance of quality.

3. Quality cost information helps managers identify the relative importance of the
quality problems faced by the firm.

4. Quality cost information helps managers see whether their quality costs are
poorly distributed and when needed, it helps them distribute the costs better.

Limitations of Quality Cost Information


1. Some important quality costs are typically omitted from the quality costs report.
Examples are opportunity cost of lost sales, cost of top management time in
designing and administering the quality program.
2 Simply measuring and reporting quality costs does not solve quality programs. Only
management action can solve them.
3. A log may exist between when quality improvement programs are put into effect and
when the results are seen.

Reporting Quality Costs

The purpose of reporting quality costs is to make management aware of the


magnitude of quality costs and to provide a baseline against which the impact of
quality improvement activities could be measured. Tasks for reporting quality
costs include data definitions, identification of data sources, data collection, and
preparation and distribution of quality cost reports.

Illustration of a Cost of Quality Report

Figure 6-1 illustrates a cost of quality report. Bean Company is a small Metro Manila
manufacturing company with annual sales of around P9 million. The firm operates in a
highly competitive environment and has been experiencing increasing pressures to
raise quality and lower cost from new and existing competitors. The report shows that
the external failure costs for such items as warranty claims, customer dissatisfaction,
and market share loss accounted for 75 percent of the total cost of quality in year 0
(P1,770,000 + P2,360,000 or 22.13% + 29.59%).
To be more competitive and to increase market shares, Bean began a corporate wide
three-year TQM process. The firm started with substantial increases in prevention and
appraisal expenditures. The investment started to pay off in year 2. The internal failure,
external failure, and total quality costs have all decreased.

Nonfinancial Measures of Quality and Customer Satisfaction


To evaluate how well their actual performance satisfies customer needs and wants,
companies supplement financial measures with nonfinancial measures of quality
of design and conformance quality. Nonfinancial measures indicate the future
needs and preferences of customers, as well as specific areas that need
improvement. In this sense, nonfinancial measures of quality are leading
indicators of future long-run performance, unlike financial measures of quality that
focus on the short run. Management accountants usually maintain and present
these nonfinancial measures.

We focus first on nonfinancial measures of customer satisfaction, including


nonfinancial measures of quality of design and external failure, and then we look
at internal performance measures, including nonfinancial measures of prevetion,
appraisal, and internal failure.

Nonfinancial Measures of Customer Satisfaction

To evaluate how well they are doing, companies measure customer satisfaction
Over time. Some measures are:
l. On-time delivery rate (the percentage of shipments made on or before the
scheduled delivery date)
2. Delivery delays (the difference between the scheduled delivery date and
the date requested by the customer)
3. Percentage of products that fail soon or often
4. Number of customer complaints (Companies estimate that for every customer
who actually complains, there are 10 to 20 others who have had bad experiences with
the product or service but did not complain.
5. Number of defective units shipped to customers as a percentage of total
units shipped
6. Market research information on customer preferences and customer satisfaction
with specific product features.

Nonfinancial Measures of Internal Performance

To satisfy their customers, managers must constantly improve the quality of work
done inside their company. Most companies use nonfinancial measures of internal
quality to supplement financial measures, such as prevention, appraisal, and
internal failure costs. Examples of trends to gauge quality are:

1. Number of defects for each product line


2. Employee turnover (ratio of number of employees who leave the company
to the average total number of employees)
3. Process yield (ratio of good output to total output)

Time as a Competitive Tool


Many companies consider "time" as a driver of strategy. They need to measure time to
manage it properly. Two common operational measures of time are customer-response
time and on-time performance.

Managing customer-response time and on-time performance requires understanding


the causes and costs of delays. Delays can occur, for example, at a machine in
manufacturing or at a check-out counter in a store.

Customer-Response Time

Customer- response time is the duration from the time a customer places an order
for a product or service to the time the product or service is delivered to the customer.
Fast responses to customers are of strategic importance in industries such as
construction, banking, car rental, and fast food.

Figure 6-2 describes the components of customer- -response time. Manufacturing


lead tine (also called manufacturing cycle time) is the duration between the time
an order 1s received by Manufacturing to the time it becomes a finished good.
Manufacturing lead time is the sum of waiting time and manufacturing time for a
order. Delivery time is how long it takes to deliver a completed order to the
customer.

Several companies have adopted manufacturing lead time as the base for allocating
indirect manufacturing costs to products. They believe that using manufacturing
lead time as a cost allocation base motivates managers to reduce the time it takes
to manufacture products. Over time, total overhead costs decrease and operating
income rises.

On-Time Performance

On-time performance refers to situations in which the product or service is actually


delivered by the time it was scheduled to be delivered. On-time performance
increases customer satisfaction. Commercial airlines gain loyal passengers as a
result of consistent on-time service. But there is a trade-off between customer-
response time and on-time performance. Deliberately scheduling longer customer-
response times, such as airlines lengthening scheduled arrival times, makes
achieving on-time performance easier-but it could displease customers!

Illus trati
ve Problem 6-1. Measurement of Quality Service

The Sterling Moving Corporation transports household goods from one city to another
within the Luzon Area. It measures quality of service in terms of (a) time required to
transport goods, (b) on-time delivery (within two days of agreed-upon delivery date).
and (c) number of lost or damaged shipments. Sterling is considering investing in a new
scheduling and tracking system costing Pi60,000 per year, which should help it improve
performance with respect to items (b) and (c) The following information describes
Sterling's current performance and the expected performance if the new system is
implemented:

Sterling expects each percentage point increase in on-time performance will result
in revenue increases of P20,000 per year. Sterling's contribution margin
percentage is 45%.

REQUIRED:
1. Should Sterling acquire the new system? Show your calculations.
2. Calculate the minimum amount of revenue increase needed for the benefits
from the new system to equal the costs.

Solution:
1. Additional costs of the new scheduling and tracking system are Pl60,000 per
year. Additional annual benefits of the new scheduling and tracking system
are

Because the
expected benefits of 210,000 pesos exceed the costs of 160,000 pesos.
Sterling should invest in the new system.

2. As long as Sterling earns a contribution margin of P40.000 (to cover


incremental costs of P160,000 minus relevant variable-cost savings of
P120.000) from additional annual sales. investing in the new $yste
beneficial.
This contribution margin corresponds to additional sales of
P40,000 +0.45 = P88,889.

JUST-IN-TIME PRODUCTION SYSTEM

The initial discussion on Just-in-Time Production system is covered in Chapter4.

Just-in-time (JIT) production, also called lean production, is a demand-pull


manufacturing system because each component in a production line is produced as
soon as and only when needed by the next step in the production line. In a J1
production line, manufacturing activity at any particular workstation is prompted
by the need for that workstation's output at the following workstation. Demand
triggers each step of the production process, starting with customer demand for a
finished product at the end of the process and working all the way back to the
demand for direct materials at the beginning of the process. In this way, demand
pulls an order through the production line.
production systems achieves close coordination among workstations. It smoothes
The demand-pull feature of JIT
the flow of goods, despite low quantities of inventory. JIT production systems aim
to simultaneously (1meet customer demand in a timely way, (2} with high-quality
products and (3) at the lowest possible total cost.

Key Features

The key features of a JIT production system are


1. Maintaining a limited number of suppliers
A company must learn to rely on a few ultra reliable suppliers who are willing to make
frequent deliveries in small lots, Dependability is essential since companies are highly
vulnerable to any interruption in. supply when JIT is in use. If a single part is
unavailable, the entire assembly operation may have to be shut down.

2. Improving plant layout


Manufacturing flow lines in the company's plant must be improved. In a JIT system, all
machines needed to make a particular product are often brought together in one
location (focused factory approach) so that partially completed units are not shifted from
place to place all over the factory to mínimize handling and moving. By doing so, the
company can dramatically reduce throughput time (also known as cycle time) which is
the time required to make a product.

3. Reducing Setup Time


Setups involve activities such as moving materials, changing machine settings, setting
up of equipment, and running tests that must be performed whenever production is
switched over from making one type of item to another. If unproductive setup time can
be made negligible and batch sizes can be made very small, then the company can
respond quickly to the market, reduce cycle times and make it much easier to spot
manufacturing problems before they result in a large number of defective units.

4. Improving Production Scheduling


JIT businesses schedule production in small batches just in time to satisfy needs. There
should be good coordination of efforts throughout the value chain - from the time the
orders are received from the customers to the time the goods are manufactured and
readied for delivery to the same customers.

5. Targeting Zero Defects


Defective units create big problems in a JIT environment because they could delay the
shipment of the order and may generate a ripple effect that delays other orders.
Although it may be next to impossible to obtain a zero defect goal, companies have
found that they can come very close through continuous monitoring". This procedure
makes the company's production workers directly responsible for spotting defective
units (which should be rare) and the supervisors and other workers determining
immediately the cause of the defect and correcting it before any further defective units
are produced. Early detection saves rework and scrap
Costs.

6. Maintaining Flexible Workforce


Companies employing JIT must have workers who are flexible and
Multi-skilled. Workers assigned to a particular JIT product flow line are
often expected to know how to operate all the equipment on that line.
Cross training increases flexibility, boosts employee morale, and
decreases costs.

Financial Benefits of JIT

Early advocates saw the benefit of JIT production as lower carrying costs Ot
inventory. But there are other benefits of lower inventories:

(1) greater transparency of the production process,


(2) heightened emphasis on eliminating the specific causes of rework, scrap,
and waste, and
(3) lower manufacturing lead times.

In computing the relevant benefits and costs of reducing inventories in JIT


production systems, the cost analyst should take into account all benefits.

Illustrative Problem 6-2. Implementing a JIT Production System

Nelson Corporation, a manufacturer of copper fittings is considering implementing


a JIT production system. The following data are gathered. To implement JIT
production:

(a) P100,000 annual tooling costs to reduce setup times must be incurred.
(b) Average inventory will be reduced by P500,000.
(c) Relevant costs of insurance, storage, materials handling, and setup will
decline by P30,000 per year.
(d) Required rate of return on inventory investments is 10% per year.
(e) Quality will be improved and rework will be reduced on 500 units per year,
resulting in savings of P50 per unit.
() Better quality and faster delivery will enable the company to charge P2
more per unit on the 20,000 units it sells each year.

REQUIRED:
Should Nelson implement the JIT production system?

Answer:
On the basis of the information provided, the ff. computations may be made:

Since the total annual benefits and cost savings exceed the annual JIT implementation
costs by P45,000, Nelson should implement a JIT production
system. Furthermore, better quality of the product and faster delivery would surely
result to more customer satisfaction which will provide long-term benefits to the
company.

Performance Measures and Control in JIT Production


The following list describes measures that managers use to evaluate and control
JIT production and how these measures are expected to be affected:
1. Financial performance measures such as inventory turnover ratio which is
expected to increase
2. Nonfinancial performance measures of time, inventory and quality, such
as

Personal observation and nonfinancial performance measures provide the most


161
timely, intuitive and easy to understand measures of plant performance. Timely,
meaningful feedback is critical because the lack of inventories in a demand pull
stem makes it urgent to detect and solve problems
quickly.

JIT Effects on Costing System

JIT Systems reduce overhead costs through the reduction of materials handling
warehousing and inspection costs. It also facilitates direct tracing of some costs
usually classified as indirect. For instance, the use of manufacturing cells makes
it cost effective to trace materials handling and machine operating costs to spec1C
products or product families made in these cells. These costs then become direct
costs of those products. Likewise, the use of multi-skilled workers in these cells
allows the costs of setup, maintenance and quality inspection to be traced as direct
costs.
162 Chapter 6

REVIEW QUESTIONS. EXERCISES AND PROBLEMS


Questions
1. Define quality.
2. What is TỌM? At what point can a firm consider its effort to achieve total
quality management complete?
3. What are the core principles of total quality management?
4. Why is continuous quality improvement essential to achieve TQM and
critical to an organization's success and competitive position?
5. Describe the processes for an effective implementation of TQM.
6. Why is it often necessary to revise a firm's compensation and appraisal
systems when implementing TQM?
7. What are the purposes of conducting a quality audit?
8. What is gap analysis?
9. Name three types of costs associated with each of the following cost
categories:
a. Prevention
b. Appraisal
c. Internal failure
d. External failure
10. Which of the following cost categories tend to increase during the early
years of TỌM? Which of them tend to decrease over the years due to
successful total quality management? Why?
a. Prevention
b. Appraisal
c. Internal failure
d. External failure
11. What is cost of conformance? Nonconformance?
12. Many organizations found that investments in prevention and appraisal
usually resulted in major cost savings in other areas. Explain this
phenomenon.
13. What roles do management accountants play in TỌM?

14. How can management accountants meet the challenges ofTQM?


15. Describe JIT purchasing and its benefits.
16. Describe how the Internet can be used to reduce the costs of placing
purchase orders.
17. What are the main features in a JIT production system'?
18. Indicate whether each of the following costs should be higher or lower for
a world-class (or JIT) manufacturer than for a conventional manufacturer.
Briefly state why.
(a) Product warranty costs.
(b) Salaries of quality control inspectors.
(c) Amounts paid to vendors for parts and components.
(d) Wage rates for direct laborers.
(e) Total supervisory salaries.
(f) Warehousing costs, including rent or depreciation on space,
salaries and wages of employees, utilities, etc.
19. In a Just-in-Time (JIT) System, what is meant by the pull approach to the
flow of goods, as compared to the push approach used in conventional
systems?
20. Identify the benefits that can result from reducing the setup time for a
product.
Exercises
Exercise 1 (Quality Cost Classification)
Classify these following items into types of cost of quality:
a. Warranty repairs
b. Scrap
c. Allowance granted due to blemish
d. Contribution margins of lost sales
e. Tuition for quality courses
f. Raw materials inspections
g. Work-in-process inspection
h. Shipping cost for replacements
i. Recalls
j. Attorney's fee for unsuccessful defense of complaints about quality
k. Inspection of reworks
I. Overtime caused by reworking
m. Machine maintenance
n. Tuning of testing equipment

Exercise 2 (Cost of Quality Report)

The Bali Company manufactures custom-designed milling machines and


incurred the following cost of quality in 20X3 and 20X4:
The total sales in each of the two years were P6,000,000. The firm's cost of
goods sold is typically one-third of the net sales.
Required:
1. Prepare a cost-of-quality report that classifies the firm's costs under the
proper cost-of-quality category.
Calculate the ratio of each cost-of-quality category y to sales in each of the
two years. Comment on the trends in cost of quality between 20X3 and
20X4.
3.. Give three examples of nonfinancial measures that Bali might want to
monitor as part of a total quality management effort.

Exercise 3 (Cost of Quality Category)

The management of Boogie Company thinks that its total costs of quality can
be reduced by increasing expenditures in certain key costs of quality
categories. The following costs of quality have been identified by
management:
Required:
1. Classify these costs into the four costs of quality categories.
2. Determine the total pesos being spent on each of the categories.
3. Based on the company's expenditures by cost of quality categories, on
which cost category should the company concentrate its efforts to decrease
its overall costs of quality?

166 Chapter 6
Exercise 4 (Cost of Quality Analysis, Nonfinancial Quality Measures)

The Gabriel Corporation manufactures and sells industrial grinders. The


owing table presents financial information pertaining to quality in 20X3
20X4 (in thousands)
Required:
1. Classify the cost items in the table into prevention, appraisal, internal
failure, or external failure categories.
2. Calculate the ratio of each COQ category to revenues in 20X3 and 20X4.
Comment on the trends in costs of quality between 20X3 and 20X4.
3. Give two examples of nonfinancial quality measures that Gabriel
Corporation could monitor as part of a total quality-control effort.

EXERCISE 5 (Costs of Quality Analysis, Nonfinancial Quality Measure)

Canada Industries manufactures two types of refrigerators, Victoria and Vancouver.


Information on each refrigerator is as follows:
Required:
1. Calculate the costs of quality for Victoria and Vancouver, classified into
prevention, appraisal, internal failure, and external failure categories.
2. For each type of refrigerator, calculate the ratio of each CQQ category as a
percentage of revenues. Compare and comment on the costs of quality for
Victoria and Vancouver.
3. Give two examples of nonfinancial quality measures that Canada Industries
could monitor as part of a total quality-control effort.

168 Chapter 6

Exercise 6 (Quality Improvement, Relevant Cost Analysis)


The Jimmy Corporation manufactures and sells 3,000 premium quality multimedia
projectors at P12.000 per unit each year. At the current production level, the firm's
manufacturing costs include variable costs of P2,500 per unit and annual fixed cost of
P6.000.000. Additional selling, administrative, and other expenses, not including 15
percent sales commissions, are PI0,000,000 per year.

The new model, introduced a year ago, has experienced a flickering problem.On
average the firm has to rework 40 percent of the completed units. The firm still has to
repair under. warranty 15 percent of the units shipped. The additional work required for
rework and repair makes it necessary for the firm to add additional capacity with annual
fixed costs of P1,800,000: The variable costs per unit are P2,000 for rework and
P2.500, including transportation cost, for repair.

The chief engineer, Ayen Anicete, has proposed a modified manufacturing


process that will almost entirely eliminate the flickering problem. The new
process will require PI2,000,000 for new equipment and installation, and
P3,000,000 for training. Anicete believes that current appraisal costs of
P600,000 per year and P50 per unit can be eliminated within one year after the
installation of the new process. The firm currently inspects all the units before
shipment. Furthermore, warranty repair cost will be only P1,000 for no more
than 5 percent of the units shipped.

Jimmy believes that none of the fixed costs of rework or repair can be saved
and that a new model will be introduced in three years. The new technology
will most likely render the current equipment obsolete.

The accountant estimates that repairs cost the firm 20 percent of its business.

Required:
1. What are the additional costs of choosing the new process?
2. What are the benefits of choosing the new process?
3. Should Jimmy use the new process?
4. What factor should be considered before making the final decision?
5. A member of the board is very concerned about the substantial amount of
additional funds needed for the new process. Because the current model
will be replaced in the about three years, the board member suggests that
the firm should take no action and the problem will go away in three years.
Do you agree?
Problems

Problem 1
Anthony Foods manufactures food seasonings and packaged dry sauce
mixes for sales in grocery stores. Anthony started a quality improvement
program in 20X3. lt expanded its training and quality assurance programs
and began monitoring emplovee satisfaction and estimating lost sales due
to quality problems. The data in the table below summarize the quarterly
results of operating its TQM program over the last two years.

Required:
a. Prepare a cost-of-quality report that classifies each expense as being in one
of four categories; appraisal, prevention, internal failure, or external failure.
b. What conclusions can you draw from the data presented about Anthony
Foods' TQM program?

Problem 2 (Applying TOM in Manufacturing versus Administration)

One large company that has been successful in applying Total Quality
Management (TỌM) principles in manufacturing reports that it has had less
Success in applying the same techniques in improving administrative functions
such as order taking, distribution, and human resources. This company (which
has won several quality awards and has significantly improved its product
quality) used state-of-the-art TỌM methods to train all of its employees in how
to apply TQM. However, the company has not been able to achieve the same
cost reductions and service quality enhancements in administrative areas as it
has in the manufacturing area. Assuming that this phenomenon extends to
other companies, why do you think that TỌM works better in manufacturing
than in nonmanufacturing/services areas?

Multiple Choice
1. Implementation of total quality management (TQM) in a firm:
a. must follow a rigid, predetermined process to be successful.
b. involves some lower-level managers and all senior executives.
c. takes from 3-5 years.
d. is a bottoms up process, with senior management involved only in the final
phase.
2. Goalpost quality conformance differs from absolute quality conformance like:
a. "generally" differs from "always."
b. "range" differs from point.
c. probable" differs from "certain."
d. many" differs from "one."
3. The quality cost of prevention is:
a. exampled by the cost of servicing warranties.
b. refers only to zero-defect programs.
. c. an upstream cost.
D. downstream cost
4. Typically, as prevention costs increase, other costs of quality:
a. not affected.
b. change, but the direction cannot be predicted.
c. increase, but at a slower pace.
d. Decrease

5. Examples of the quality cost of prevention include all of the following,


except:
a. tuition for external training.
b. additional tolerance controls for machinery.
. c. depreciation of a training room.
d. an annual award for lowest rework rate.

6. Appraisal costs are incurred to measure and analyze data to test product
or service in conformity to specifications, but not to:
a. reduce error or prevent recurrence of error.
b. change procedures.
c. change policy.
D. check on quality standards.

7. The key difference(s) between internal failure cost and external failure cost is
(are):
a. when the cost happens.
b. where the cost happens.
c. both when and where the cost happens.
D. whether the cost happens.

8. Which one of the following is not listed as a practice that successful TQM firms
use to ensure having quality suppliers?
a. forming long-term relationships with suppliers as working partners.
b. setting measures that truly reflect the needs and expectations of suppliers.
c. reducing the supplier base.
d. selecting suppliers based on price and their capability and willingness to
improve quality, cost, delivery, flexibility, and for their dedication to continuous
improvement.

9. Conformance to a quality specification expressed as a specified range around a


target is
a. endzone conformance.
b. target conformance.
C goalpost conformance.
d absolute quality conformance.

10. Conformance that requires all products or services to meet exactly the target
value with no variation allowed is
a. Endzone conformance
b. Target conformance
c. Goalpost conformance
d. Absolute quality conformance

11. Just in time purchasing requires


a. Larger and less frequent purchase orders
b. Smaller and less frequent purchase orders
c. Smaller and more frequent purchase orders
d. Larger and more frequent purchase orders

12. A demand-pull system in which each component in a production line is produced


immediately as needed by the next step in the production line is referred to as
a. Just in time purchasing
b. Materials requirements planning
c. Relevant total costs
d. Economic order quantity

13. All of the following are potential financial benefits of just in time except
a. Lower investments in inventories
b. Lower investments in plant space for inventories
c. Reducing the risk of obsolescence
d. Reducing manufacturing lead time

14. Cost quality reports usually do not consider


a. External failure costs
b. Opportunity costs
c. Internal failure costs
d. Appraisal costs

15. Changing to an activity-based costing/management system will not


a. Change the way that resources are allocated.
b. Change all the way that costs are allocated
c. Change all the people’s jobs
d. Change the way that performance is evaluated

16. Resistance to changing a management accounting and control system (MACS)


can occur for the reasons listed below, except
a. Employees are set in their ways and will act defensively
b. An employee’s compensation and rewards may be altered
c. The balance of power may shift unfavorably for the employee
d. Employees have to wait for a vote of shareholders before a MACS can be
changed.

17. One common mistake that managers make when changing to a new cost
management system is
a. They involve too many in making the change
b. They take too long to implement the change
c. They over-budget for the cost of the change
d. They try to change too many things simultaneously

18. The just in time manufacturing (JIT) system is also called the
a. Job in training system
b. Job in transit system
c. Zero cost system
d. Zero inventories system

19. The traditional focus in management accounting has been to develop


a. Only quantitative performance measures
b. Only qualitative performance measures
c. Both quantitative and qualitative performance measures
d. Neither quantitative nor qualitative measures

20. A well-designated MACS develops and uses


a. Both quantitative and qualitative information for control, motivation and
performance evaluation
b. Only quantitative information for control, motivation and performance
evaluation
c. Only qualitative information for control, motivation and performance
evaluation
d. Neither quantitative nor qualitative information for control motivation, and
performance evaluation.

174 Chapter 6

21. Product quality and profitability are:


a. closely related
b. inversely related
c. loosely related
d. indirectly linked

22. A product that meets or exceeds customer expectation is:


a. the norm in today's marketplace.
b. A quality product.
c. A result of planning for quality
d. All of the above answers are correct.

23. Core principles of total quality management include:


a. focusing on customer satisfaction.
b. striving on continuous improvement.
c. involvement of the entire work force.
d Answer a. b and care all core principles.

24. Continuous improvement (Kaizen, n today's global economy:


a. is desirable if a firm promotes quality.
b. is necessary if a firm is to remain competitive.
c. speaks only to the control of defects.
d Is practically impossible.

25. Implementation of total quality management (TQM) in a firm:


a. must follow a rigid, predetermined process to be successful.
takes from 5-0 years
b. involves all lower-level managers and as well as senior executives
c. is a bottoms up process, with senior management involved only in
d. the final phase

26. Goalpost quality conformance differs from absolute quality conformance like:
a. "generally' differs from "always".
b. "range" differs from "point".
c. "probable" differs from "certain".
d. "many'" differs from "one".

Organizational Innovations ... 175

27. The Taguchi Quality Loss Function demonstrate s that as the quality measure of a
product declines, the loss due to quality defects:
a. increases as a quadratic function.
b. increases in direct proportion.
c. increases in an inverse proportion.
d. decrease as a quadratic function.

28. The quality cost of prevention is:


a. exampled by the cost of servicing warranties.
b. refers only to zero-defect programs.
c. an upstream cost.
d. a downstream cost.

29. Typically, as prevention costs increase, other costs of quality


a. are not effected.
. b. change, but the direction cannot be predicted.
c. increase, but at a slower pace.
d. decrease.

30. Examples of the quality cost of prevention include all of the following,
except:
a. tuition for external training.
b. additional tolerance controls for machinery.
c. depreciation of a training room.
d. an annual award for lowest rework rate.

31. Appraisal costs are incurred to measure and analyze data to test product
or service conformity to specifications, but not to:
a. reduce error or prevent recurrence of error.
b. change procedures.
c. change policy.
d. check on quality standards.

32 . The key difference(s) between internal failure cost and external failure
cost is (are):
a. when the cost happens.
b. where the cost happens.
c. both when and where the cost happens.
d. whether the cost happens.
176 Chapter 6

33. quality costs are viewed as conformance versus non-conformance


quality expert Philip Crosby would argue that there is no such thing as
a quality problem, but only a problem of:
a. Design
b. Material
c. Labor
d. All of the above
e.
34. If one were to classify quality costs as prevention, appraisal, internal failure and
external failure, the lowest and highest costs would tend to be respectively:
a. appraisal and external failure.
b. Appraisal and internal failure
c. Prevention and internal failure
d. Prevention and external failure

35. Regardless of the differences in form and control, a common feature that should be
present in any Cost of Quality is that the report:
a. promotes total quality management (TQM).
b. stratifies costs by product line.
c. stratifies costs by department.
d. stratifies costs by plant.

36. Whichever of the many helpful tools a firm chooses for identifying significant quality
problems, the tool(s) will be most effective if:
a. management accountants are not involved in their selection
b. management accountants take a pro-active role throughout the process.
c. the firm hires technical experts to choose the tool(S).
d. the firm leaves selection of the tool(s) to the supervisors.

37. Which one of the following is not a category of costs of quality?


a. Promotion
b. External failure
c. internal failure
d. Appraisal
38. Costs incurred to keep quality defects from occurring are
a. external failure costs.
b. Appraisal costs
. c. Internal failure costa
d. prevention costs.
39. Costs incurred in measurement and analysis of data to ascertain conformity of
products and services to the specifications are
a. external failure costs
b. appraisal costs
c. Internal failure costs
d. prevention costs

40. Costs incurred as a result of a poor quality found through appraisal prior to delivery
to customers are
a. external failure costs
b. appraisal costs
c. Internal failure costs
d. Prevention costs

41. Warranty costs would be classified as


a. prevention costs
b. retention costs
c. appraisal costs
d. external failure costs

42. Rework costs would be classified as


a. Prevention costs
b. Retention costs
c. Appraisal costs
d. Internal failure costs

43. Test acquisition costs would be classified as


a. prevention costs
b. retention costs
c. appraisal costs
d. Internal failure costs

44. Re inspection costs would be classified as


a. prevention costs
b. retention costs
c. appraisal costs
d. Internal failure costs

45. Costs of meetings would be classified as


a. prevention costs
b. retention costs
c. appraisal costs
d. internal failure costs

46. Finished goods inpection costs would be classified as


a. prevention costs
b. retention costs
c. appraisal costs
d. internal failure costs

47. Cost conformance includes


a. prevention costs and appraisal costs
b. internal failure costs and external failure costs
c. prevention costs and internal failure costs
d. appraisal costs and external failure costs.
CHAPTER

7
THE BALANCED SCORECARD:
A TOOL TO IMPLEMENT
STRATEGY
__________________________________________

EXPECTED LEARNING OUTCOMES

After studying this chapter, you should be able to…

1. Define the balanced scorecard

2. Understand the four perspectives of the balanced scorecard

3. Know the features of a good balanced scorecard

4. Analyse changes in operating income to evaluate success of a strategy

5. Evaluate internal process performance and compute the delivery cycle time, the
throughput time and the manufacturing cycle efficiency (MCE)

◆◆◆
CHAPTER 7

THE BALANCED SCORECARD:


A TOOL TO IMPLEMENT STRATEGY

THE BALANCED SCORECARD

The balanced scorecard translates an organization's mission and strategy into a set of
performance measures that provides the framework for implementing the strategy. The balanced
scorecard does not focus solely on achieving financial objectives. It also highlights the
nonfinancial objectives that an organization must achieve to meet its financial objectives. The
scorecard measures an organization’s performance from four perspectives: (1) financial, (2)
customer, (3) internal processes, and (4) learning and growth. A company's strategy influences
the measures it uses to track performance in each of these perspectives.

Strategic information using critical success factors such as growth in sales and earnings, cash
flow, stock price, market share, product quality, customer satisfaction, and growth opportunities
provides a road map for a firm to chart its competitive course and serves as a benchmark for
competitive success. To emphasize the importance of using strategic information, both financial
and nonfinancial, accounting reports of a firm's performance are now often based on critical
success factors in different dimensions.

Financial performance measures summarize the results of past actions and are important to a
firm's owners, creditors, employees and so forth. Nonfinancial performance measures
concentrate on current activities which will be the drivers of future financial performance. Thus,
effective management requires a balanced prospective on performance measurement, a viewpoint
that some call the "balanced scorecard" perspective. The balance scorecard integrates
performance measures in four key areas:

(1) financial perspective,


(2) customer satisfaction,
(3) internal business processes, and
(4) innovation and learning.
The Balanced Scorecard: A Tool to Implement Strategy 181

A balanced scorecard consists of an integrated system of performance measures that are derived
from and support the company's strategy. Different companies will have different balanced
scorecards because they have different strategies. A well-constructed balanced scorecard
provides a means for guiding the company and also provides feedback concerning the
effectiveness of the company's strategy.

It is called the balances scorecard because it balances the use of financial and nonfinancial
performance measures to evaluate short-run and long-run performance in a single report. The
balanced scorecard reduces managers; emphasis on short-run financial performance, such as
quarterly earnings. That's because the nonfinancial and operational indicators, such as product
quality and customer satisfaction, measure changes that a company is making for the long run.
The financial benefits of these long-run changes may not appear immediately in short-run
earnings, but strong improvement in nonfinancial measures is an indicator of economic value
creation in the future.

The balanced scorecard is depicted in Figure 7-1.

Figure 7-1: The Balanced Scorecard

How do we look Financial Perspective


to the firm’s
owners? Goals Measures
In which
How do our
activities must
customers see
we excel?
us?
Internal Operations
Customer Perspective Perspective

Goals Measures Goals Measures

Innovation and Learning


Perspective

Goals Measures

How can we continually


Improve and create value?
CHAPTER 7

THE BALANCED SCORECARD:


A TOOL TO IMPLEMENT STRATEGY
182 Chapter 7
.

FOUR PERSPECTIVES OF THE BALANCED SCORECARD


(1) Financial Perspective. Measures of profitability and market value among others, as indicators
of how well the firm satisfies its owners and shareholders. These financial measures show the
impact of the firm's policies and procedures on the firm's current financial position and therefore
its current return to the shareholders.
Objectives Measures
Revenue Growth: Percentage of revenue from new products
Increase the number of new products Percentage of revenue from new
Create new applications applications

Develop new customers and markets Percentage of revenue from new sources

Adopt a new pricing strategy Product and customer profitability

Cost Reduction:
Reduce unit product cost Unit product cost

Reduce unit customer cost Unit customer cost

Reduce distribution channel cost Cost per distribution channel

Asset Utilization: Return on investment

Improve asset utilization Economic value added

(2) Customer Satisfaction. Measures of quality service and low cost, among others, as indicators
of how well the firm satisfies its customers.
Objectives Measures

Core: Market share (percentage of market)


Percentage growth of business from existing
Increase market share customers
Increase customer retention Percentage of repeating customers
Increase customer acquisition Number of new customers
Increase customer satisfaction Ratings from customer surveys
Increase customer profitability Customer profitability
Performance Value: Price
Decrease price Postpurchase costs
Decrease postpurchase costs Ratings from customer surveys
Improve product functionality Percentage of returns
Improve product quality On-time delivery percentage
Increase delivery reliability Aging schedule
Improve product image and reputation Ratings from customer survey
200 Chapter 4
.

(3) Internal Business Processes. Measures of the efficiency and effectiveness with which the
firm produces the product or service.

Objectives Measures
Number of new products vs. planned Percentage
revenue from proprietary products
Innovation:
Time to market (from start to finish)
Increase the number of new produces
Quality costs
Increase proprietary products
Output yields
Decrease new product development time
Percentage of defective units
Operations:
Unit cost trends
Increase process quality Output/input(s)
Increase process efficiency Cycle time and velocity
Decrease process time MCE
Postsales Service: First-pass yields
Increase service quality Cost trends

Increase service efficiency Output/input

Decrease service time Cycle time

(4) Innovation and Learning. Measures of the firm's ability to develop and utilize human
resources to meet the strategic goals now and into the future.
Objectives Measures
Increase employee capabilities Employee satisfaction ratings
Employee turnover percentages
Employee productivity (revenue/employee)
Hours of training
Strategic job coverage ratio (percentage of
critical job requirements filled)
Suggestions per employee
Increase motivation and alignment
Suggestions implemented per employee
Increase information systems capabilities
Percentage of processes with real-time
feedback capabilities
Percentage of customer-facing employees
with on-line access to customer and product
information
184 Chapter 7
.

Figure 7-2: Illustrative Balanced Scorecard


The Balanced Scorecard for XYZ Inc., for the Year 20X3
Target Actual
Objectives Measures Initiatives
Performanc Performan
•Financial e ce
Perspective •Operating •Manage costs •40,000,000 •42,000,000
income from and unused
productivity capacity
gain •30,000,000 •68,400,000
•Increased •Build strong
shareholder •Operating customer
value income from relationships
growth
•Build strong •6% •6.48%
•Revenue customer
Growth relationships
•Customer Perspective
•Increase •Market share in •Identify future •6% •7%
market share communication needs of
networks segment customers
•New customers •Identify new •5% •6%
target customer
segments

•Increase •Customer •Increase •90% of •87% of


customer satisfaction customer focus customers give customers give
satisfaction survey of sales top two ratings top two ratings
organization

•Internal Business Process Perspective

•Improve •Percentage of •Organize R&D •75% •75%


manufacturin processes with / manufacturing
g capability advanced teams to
controls implement
advanced
controls
•Improve •Yield •Identify root •78% •79.3%
manufacturin causes of
g quality and problems and
productivity improve quality

•Reduce delivery •Order delivery •Reengineer •30 days •30 days


time to time order delivery
customers process
The Balanced Scorecard: A Tool to Implement Strategy 185
•Meet specified •On-time •Reengineer •92% •90%
delivery dates delivery order delivery
process
•Learning and Growth Perspective

•Percentage of •Employee •90% •92%


•Develop process employees training
skill trained in programs
process and
quality
management
•Empower •Percentage of •Have •85% •90%
workforce frontline supervisors
workers act as coaches
empowered to rather than
manage decision
processes makers
•Align •Employee •Employee •80% of •88% of
employee and satisfaction participation employees employees
organization survey and suggestions give top two give top two
goals program to ratings ratings
build teamwork
•Enhance •Percentage of •Improve off- •80% •80%
information manufacturing line data
system processes with gathering
capabilities real-time
feedback
•Organize R&D •5 •5
•Improve •Number of
/ manufacturing
manufacturing major
teams modify
processes improvements
processes
in process
controls
a (Revenues in 20X1– Revenues in 20X0 + Revenues in 20X0 = (P28,750,000)-P27,000,000) ÷
27,000,000 = 6.48%
b Customers increased from 40 to 46 in the year 20X1
c Yield = Units of CM1 produced + Units of CM1 started x 100 = 1,150,000 1,450,000 x 100 =
79.3%
Features of a Good Balanced Scorecard
1. The balanced scorecard should tell the story of a company's strategy by articulating a
sequence of cause-and-effect relationships. For example, if the objective of XYZ Manufacturing
Co. is to be a low-cost producer with emphasis on growth, the balanced scorecard describes the
specific objectives and measures in the learning and growth perspective that lead to
improvements in internal business processes. These would lead to increased customer
satisfaction and market share as well as higher operating income and shareholder wealth. Each
The Balanced Scorecard: A Tool to Implement Strategy 185

measure in the scorecard is part of a cause-and- effect chain, a linkage from strategy formulation
to financial results.

2. It helps to communicate the strategy to all members of the organization by translating the
strategy into a coherent and linked set to understandable and measurable operational targets.
Managers and employees are guided by the scorecard and take actions and make decisions that
aim to achieve the company's strategy.
3. In for-profit companies, the balanced scorecard places strong emphasis on financial objectives
and measures. When financial and nonfinancial performance measures are linked, many of the
nonfinancial measures serve as leading indicators of future financial performance.
4. The balanced scorecard should focus only on key measures to be used by identifying only the
most critical ones.
5. The scorecard should highlight suboptimal tradeoffs that managers may make when they fail
to consider operational and financial measure together.

Pitfalls in Implementing a Balanced Scorecard

Pitfalls to avoid in implementing a balanced scorecard include the following:

1. Don't assume the cause-and-effect linkages are precise. They are merely hypotheses. Over
time, a company must gather evidence of the strength and speed of the linkages among the
nonfinancial and financial measures.

2. Don't seek improvements across all of the measures all of the time. Trade- off may need to be
made across various strategic goals. For example, strive for quality and on-time performance but
not beyond a point at which further improvement in these objectives may be inconsistent with
long-run profit maximization.

3. Don't use only objective measures in the balanced scorecard. The balanced scorecard should
include both objective measures (such as operating income from cost leadership, market share
and manufacturing yield) and subjective measures (such as customer and employee satisfaction
ratings). When using subjective measures, though, management must be careful to trade off the
benefits of the richer information these measures provide against the imprecision and potential
for manipulation.

4. Don't fail to consider both costs and benefits of initiatives such as spending on information
technology and R&D before including these objectives in the balanced scorecard. Otherwise,
management may focus the organization on measures that will not result in overall long-run
financial benefits.
184 Chapter 7
.

5 . Don't ignore nonfinancial measures when evaluating managers and employees. Managers tend
to focus on what their performance is measured by. Excluding nonfinancial measures when
evaluating performance will reduce the significance and importance that managers give
nonfinancial measures.

6. Don't use too many measures. It clutters the balanced scorecard and takes attention away from
the measures that are critical for implementing strategy.

Evaluating the Success of a Strategy

To evaluate the success of a company's strategy, we analyze changes in operating income into
components that can be identified with growth, product differentiation, and cost leadership.
Subdividing the change in operating income to evaluate the success of a company's strategy is
similar to variance analysis. The focus here, however, is on comparing actual operating
performance over two different time periods and explicitly linking it to strategic choices. A
company is considered to be successful in implementing its strategy when the amounts of the
product differentiation, cost leadership, and growth components align closely with its strategy.

The following analytical relationships may be used:

1. Growth component
The calculations for the growth component are similar to Sales-Volume or Quantity
Factor
Revenue effect of growth component (Quantity Factor)
Actual units of output sold this year Pxx
Less: Actual units of output sold last year xx
Increase (Decrease) Pxx
Multiply by: Output price last year xx
Favorable (Unfavorable) Pxx
Cost effect of growth component*
Actual units of input or capacity that would
have been used to produce this year's output
assuming the same input-output relationship
that existed last year Pxx
Less: Actual units of inputs or capacity
to produce last year's output xx
Increase (Decrease) Pxx
Multiply by: Input prices last year xx
(Favorable) Unfavorable Pxx
The Balanced Scorecard: A Tool to Implement Strategy 185

*This will be computed for each cost element such as direct materials cost, conversion costs,
selling and customer-service cost.

2. Price-Recovery component
Revenue effect of price-recovery component (Price Factor)
Output price this year Pxx
Less: Output price last year xx
Increase (Decrease) in Output price Pxx
Multiply by: Actual units of output sold this year xx
Favorable (Unfavorable) Pxx
Cost effect of price-recovery component*
Input prices this year Pxx
Less: Input prices last year xx
Increase (Decrease)* Pxx
Multiply by: Actual units of inputs or capacity
that would have been used to produce this
year's output assuming the same input-output
relationship that existed last year xx
(Favorable) Unfavorable*. Pxx

* To be computed for each cost element.

3. Productivity component
Actual units of inputs or capacity
used to produce this year's output Pxx
Less: Actual inputs or capacity that would
have been used to produce this year's
output assuming the same input-output
relationship that existed last year xx
Increase (Decrease) Pxx
Multiply by: Input price last year xx
Favorable (Unfavorable)* Pxx
*Favorable if it increases operating income. Unfavorable if it decreases operating
income.
The Balanced Scorecard: A Tool to Implement Strategy 185

Illustrative Problem 7-1: Strategy; Balanced Scorecard; Strategic Analysis of Operating


Income; Identifying and Managing Unused Capacity

Metro Corporation makes a special-purpose machine OM used in the textile industry. Metro has
designed the OM machine for 20X3 to be distinct from its competitors. It has been generally
regarded as a superior machine. Metro presents the following data for the years 20X2 and 20X3.

20X2 20X3

1. Units of OM produced and sold 200 210

2. Selling price 40,000 42,000

3. Direct materials (kilograms) 300,000 310,000

4. Direct materials cost per kilogram P8 P8.50

5. Manufacturing capacity in units of OM 250 250

6. Total conversion costs P2,000,000 P2,025,000

7. Conversion costs per unit of capacity P8,000 P8,100

8. Selling and customer-service capacity 100 customers 95 customers

9. Total selling and customer-service costs P1,000,000 P940,500

10. Selling and customer-service capacity

cost per customer P10,000 P9,900

11. Design staff 12 12

12. Total design costs P1,200,000 P1,212,000

13. Design costs per employee P100,000 P101,000


The Balanced Scorecard: A Tool to Implement Strategy 185

Metro produces no defective machines, but it wants to reduce direct materials usage per OM
machine in 20X2. Conversion costs in each year depend on production capacity defined in terms
of OM units that can be produced, not the actual units of OM produced. Selling and customer-
service costs depend on the number of customers that Metro can support, not the actual number
of customers Metro serves. Metro has 75 customers in 20X2 and 80 customers in 20X3. At the
start of each year, management uses its discretion to determine the number of design staff for the
year. The design staff and costs have no direct relationship with the quantity of OM produced or
the number of customers to whom OM is sold.

Required:

1. Is Metro’s strategy one of the product differentiation or cost leadership? Explain briefly.

2. Describe briefly key elements that you would include in Metro's balanced scorecard and the reasons for
doing so.

3. Calculate the operating income of Metro Corporation in 20X2 and 20X3.

4. Calculate the growth, price-recovery; and productivity components that explain the change in operating
income from 20X2 to 20X3.

5. Comment on your answer in requirement 4. What do these components indicate?

6. Where possible, calculate the amount and cost of unused capacity for (a) manufacturing, (b) selling and
customer service, and (c) design at the beginning of the year 20X3 based on year 20X3 production. If you
could not calculate the amount and cost of unused capacity, indicate why not.

7. Suppose Metro can add or reduce its manufacturing capacity in increments of 30 units. What is the
maximum amount of costs that Metro could save in 20X3 by downsizing manufacturing capacity?

8. Metro, in fact, does not eliminate any of its unused manufacturing capacity. Why might Metro not
downsize?

Answer:

1. Metro Corporation follows a product differentiation strategy in 20X3. Metro's OM machine is distinct
from its competitors and generally regarded as superior to competitors' products. To succeed, Metro must
continue to differentiate its product and charge a premium price.

2. Balanced Scorecard measures for 20X3 follow:

Financial Perspective

(1) Increase in operating income from charging higher margins

(2) Price premium earned on products


200 Chapter 4
.

These measures indicate whether Metro has been able to charge premium prices and achieve operating
income increases through product differentiation.

Customer Perspective

(1) Market share in high-end special-purpose textile machines

(2) Customer satisfaction

(3) New customers

Improvements in these customer measures are leading indicators of superior financial


performance.

Internal Business Process Perspective

(1) Manufacturing quality

(2) New product features added

(3) Order delivery time

Improvements in these measures are expected to result in more satisfied customers and in turn
superior financial performance.

Learning and Growth Perspective

(1) Development time for designing new machines

(2) Improvements in manufacturing processes

(3) Employee education and skill levels

(4) Employee satisfaction

Improvements in these measures have a cause-and-effect relationship with improvements in


internal business processes, which in turn lead to customer satisfaction and financial
performance.

3. Operating income for each year is as follows:

20X2 20X3
Revenue (40,000×200; 42,000×210 8,820,000

Costs

Direct materials costs (8×300,000;8.50×310,000)

Manufacturing conversion costs (8,000×250; P8,100×250)


The Balanced Scorecard: A Tool to Implement Strategy 185

8,000,000

2,400,000 2,635,000

2,000,000 2,025,000

1,000,000 940,500
1,200,000 1,212,000
6,600,000 6,812,500
P1,400,000 P2,007,500

607,500F

4. The Growth Component


Actual units of Output
Revenue effect of output sold in Actual units of price in
growth = 20X3 - output sold in × 20X2
component 20X2
= (210-200) x P40,000 = P400,000 F*
Alternative presentation:
Revenue effect of growth component (Quantity Factor)
Actual units of output sold this year 210
Less: Actual units of output sold last year 200
Increase (Decrease) 10
Multiply by: Output price last year P40,000
Favorable P400,000
Actual units of input or Actual units of
Cost effect of capacity that would have been Input
inputs or capacity
growth used to produce year 20X3 - prices in
= used to produce ×
component 20X2
output assuming the same 20X2 output
input-output relationship that
existed in 20X2

Direct materials costs that would be required in 20X3 to produce 210 units instead of the 200
units produced in 20X2, assuming the 20X2 input-output relationship continued into 20X3, equal
315,000 kilogram [(300,000 + 200) x 210]. Manufacturing conversion costs and selling and
customer-service costs will not change since adequate capacity exists in 20X2 to support year
200 Chapter 4
.

20X3 output and customers. R & D costs would not change in 20X2 if Metro had to produce and
sell the higher 20X3 volume in 20X2.

The cost effects of growth component are:

Direct materials costs (315,000-300,000) × P8 = P120,000U

Manufacturing conversion costs (250-250) × P8,000 = 0

Selling and customer-service costs (100-100) × P25,000 = 0

Design costs (12-12) × P100,000 = 0

Cost effect of growth component P120,000U

In summary, the net increase in operating income as a result of the growth component equals:

Revenue effect of growth component P400,000 F

Cost effect of growth component 120,000 U

Increase in operating income due to growth component P280.000 F

The Price-Recovery Component

Revenue effect of Output Output Actual units


price-recovery = price in - price in × of output
20X3 20X2 sold in 20X3
component
=( P 42,000-P40,000)×210=P420,000F
Actual units of inputs or
Input Input
Cost effect of price- capacity that would have
= prices in - prices in ×
recovery been used to produce year
component year 20X3 year 20X2
20X3 output assuming the
same input-output
relationship that existed in
20X2

Direct materials costs (P8.50-P8) × 315,000 = P157.50U


Manufacturing conversion costs (P8,100-P8,000) × 250 = 25,000U
Selling and customer-service costs (9,900-P10,000) × 100 = 10,000U
Design costs (P101,000-P100,000) × 12 = 12,000U
Total cost effect of price-recovery component P184,500U
200 Chapter 4
.

In summary, the net increase in operating income as a result of the price- recovery
component equals:

Revenue effect of price-recovery component P420,000 F

Cost effect of price-recovery component 184,500 U

Increase in operating income due to price-recovery component P235,500 F

The Productivity Component


Actual units of Actual units of inputs or
inputs or capacity that would have
Input
capacity used been used to produce
Productivity prices
to produce year year 20X3 output
= - × in 20X3
component 20X3 output assuming the same
input-output relationship
that existed in 20X2

The productivity component of cost changes are:

Direct materials costs

Manufacturing conversion costs

Selling and customer-service costs

Design costs

Change in operating income due to productivity component

The change in operating income between 20X2 and 20X3 can be analyzed as follows:
Cost Effect Income
Income Revenue and Revenue and
of Statement
Statement Cost Effects Cost Effects of
Amounts In
Amounts of Growth Price- Recovery Productivity 20X3 (5)
in 20X2 Component Component in Component
in 20X3 (4) (1)+(2)+(3)
(1) in 20X3 (2) 20X3 (3) +(4)
200 Chapter 4
.

Revenues P8,000,000 P400,000 F P420,000 F - P8,820,000

Cost 6,600,000 120,000 U 184,500 U 92,000 F 6,812,500

Operating P1,400,000 P280,000 F P235,500 F 92,000 F P2,007,500


income
P607,500 F

Change in operating income

5. The analysis of operating income indicates that a significant amount of the increase in
operating income resulted from Metro's product differentiation strategy. The company was able
to continue to charge a premium price while growing sales. Metro was also able to earn
additional operating income by improving its productivity.

6. The amount and cost of unused capacity at the beginning of year 20X3 based on year 20X3
production follows:

Amount of Unused Cost of Unused


Capacity Capacity

Manufacturing, 250-210; (250-210) x P8,100 40 P324,000

Selling and customer service, 100-80;

(100-80) x P9,900 20 198,000

Design Discretionary cost, Discretionary


so cannot cost so cannot
determine unused be calculated*
capacity*
200 Chapter 4
.

• The absence of a cause-and-effect relationship makes identifying unused capacity for


discretionary costs difficult. Management cannot determine the R & D resources used for the
actual output produced to compare R & D capacity against.
The Balanced Scorecard: A Tool to Implement Strategy 185

7. Metro can reduce manufacturing capacity from 250 units to 220 (250-30) units. Metro will
save 30 x P8,100 P243,000. This is the maximum amount of costs Metro can save in 2013. It
cannot reduce capacity further (by another 30 units to 190 units) because it would then not have
enough capacity to manufacture 210 units in 20X3 (units that contribute significantly to
operating income).

8. Metro may choose not to downsize because it projects sales increases that would lead to a
greater demand for and utilization of capacity. Metro may have also decided not to downsize
because downsizing requires a significant reduction in capacity. For example, Metro may have
chosen to downsize some more manufacturing capacity if it could do so in increments, of say, 10,
rather than 30 units. Also, Metro may be focused on product differentiation, which is key to its
strategy, rather than on cost reduction. Not reducing significant capacity also helps to boost and
maintain employee morale.

Internal Business Process Performance

Most of the performance measures are self-explanatory, However, three are not delivery cycle
time, throughput time, and manufacturing cycle efficiency (MCE) These three important
performance measures are discussed below.

Figure 7-3: Delivery Cycle Time and Throughput (Manufacturing Cycle) Time

Customer's Production Goods


Order Received Started Shipped

Wait Time Process Time + Inspection Time + Move Time + Queue Time

Throughput (Manufacturing Cycle) Time


200 Chapter 4
.

Delivery Cycle Time


Value-Added Time Non-Value-Added Time
Process Time Wait Time
Inspection Time
Move Time
Queue Time

Delivery Cycle Time. The amount of time from when an order is received from a customer to
when the completed order is shipped is called delivery time cycle. This time is clearly a key
concern to many customers, who would like the delivery cycle time to be as short as possible.
Cutting the delivery cycle time may give a company a key competitive advantage - and may be
necessary for survival. Consequently, many companies would include this performance measure
on their balanced scorecard.

Throughput (Manufacturing Cycle) Time. The amount of time required to turn raw materials
into completed products is called throughput time, or manufacturing cycle time. The relation
between the delivery cycle time and the throughput (manufacturing cycle) time is illustrated in
the diagram above.

As shown in the diagram, the throughput time, or manufacturing cycle time, is made up of
process time, inspection time, move time, and queue time process time is the amount of time
work is actually done on the product. Inspection time is the amount of time spent ensuring that
the product is not defective. Move time is the time required to move materials or partially
completed products from Workstation to workstation. Queue time is the amount of time a
product spends waiting to be worked on, to be moved, to be inspected, or to be shipped.

As shown at the bottom of the diagram, only one of these four activities adds value to the
product - process time. The other three activities - inspecting, moving, and queuing - add no
value and should be eliminated as much as possible.

Manufacturing Cycle Efficiency (MCE). Through concerted efforts to eliminate the non-value-
added activities of inspecting, moving, and queuing, some companies have reduced their
throughput time to only a fraction of previous levels. In turn, this has helped to reduce the
delivery cycle time from months to only weeks or hours. Throughput time, which is considered
to be a key measure in delivery performance, can be put into better perspective by computing the
manufacturing cycle efficiency (MCE). The MCE is computed by relating the value-added time
to the throughput time. The formula is:
200 Chapter 4
.

Value-added time
MCE =
Throughput (manufacturing cycle) time

If the MCE is less than 1, then non-value-added time is present in the production process. An
MCE of 0.5, for example, would mean that half of the total production time consisted of
inspection, moving, and similar non-value-added activities. In many manufacturing companies,
the MCE is less than 0.1 (10%), which means that 90% of the time a unit is in process is spent on
activities that do not add value to the product. By monitoring the MCE, companies are able to
reduce non-value- added activities and thus get products into the hands of customers more
quickly and at a lower cost.

Illustrative Problem 7-2: Measures of Internal Business Process Performance

Southwest Company keeps careful track of the time relating to orders and their production.
During the most recent quarter, the following average times were recorded for each unit or order.
Days
Wait time 17.0

Inspection time 0.4

Process time 2.4

Move time 0.6

Queue time 5.0

Goods are shipped as soon as production is completed.

REQUIRED:
200 Chapter 4
.

1. Compute the throughput time, or velocity of production.

2. Compute the manufacturing cycle efficiency (MCE).

3. What percentage of the production time is spent in non-value-added activities?

4. Compute the delivery cycle time.

Solution:

1. Throughput time = Process time + Inspection time + Move time + Queue time
= 2.0 days + 0.4 days + 0.6 days + 5.0 days
= 8.0 days
2. Only process time is value-added time; therefore, the computation of the MCE would be as
follows:
MCE = Value-added time = 2.0 days
= 0.25
Throughput time = 8.0 days

Thus, once put into production, a typical unit is actually being worked on only 25% of the
time.

3. Since the MCE is 25%, the complement of this figure, or 75% of the total production time, is
spent in non-value-added activities.

4. Delivery cycle time = Wait time + Throughput time


=17.0 days + 8.0 days
=25.0 days
REVIEW QUESTIONS AND PROBLEMS

Questions

1. Give the major weakness of each of the three competitive strategies: (1) cost leadership, (2)
differentiation, and (3) focus.

2. What is a balanced scorecard? What is the primary objective when using a balanced
scorecard?

3. Contrast using the balanced scorecard with using only financial measures of success.

4. How can an analyst incorporate the industry-market-size factor and the interrelationships
between the growth, price-recovery, and productivity components into a strategic analysis of
operating income.
The Balanced Scorecard: A Tool to Implement Strategy 185

5. Why does balanced scorecard differ from company to company?

6. What is the difference between the delivery cycle time and the throughput time? What four
elements make up the throughput time? Into what two classes can these four elements be
placed?

7. Why does the balanced scorecard include financial performance measures as well as
measures of how well internal business processes are doing?

8. If a company has a manufacturing cycle efficiency (MCE) of less than 1, what does it mean?
How would you interpret an MCE of 0.40?

Problem (Measures of Internal Business Process Performance)

Melody Fabrications, Ltd., of Dasmariñas, Cavite, has recently begun a continuous improvement
campaign in conjunction with a move toward JIT production and purchasing. Management has
developed new performance measures as part of this campaign. The following operating data
have been gathered over the last four months:

Month

1 2 3 4
Throughput time, or velocity…. ? ? ? ?

Manufacturing cycle efficiency.. ? ? ? ?

Delivery cycle time…… ………. ? ? ? ?

Percentage of on-time deliveries.. 72% 73% 78% 85%

Total sales (units)…………......... 10,540 10,570 10,550 10,490


200 Chapter 4
.

Management would like to know the company's throughput time, manufacturing cycle efficiency,
and delivery cycle time. The data to compute these measures have been gathered and appear
below:

Month
1 2 3 4
Move time per unit, in days…….. 0.5 0.5 0.4 0.5

Process time per unit, in days….. 0.6 0.5 0.5 0.4

Wait time per order before start of

production, in days…………. 9.6 8.7 5.3 4.7

Queue time per unit, in days……. 3.6 3.6 2.6 1.7

Inspection time per unit, in days.... 0.7 0.7 0.4 0.3

As part of its continuous improvement program, the company is planning to move toward a JIT
purchasing and production system.

Required:

1. For each month, compute the following operating performance measures:

a. The throughput time, or velocity of production.

b. The manufacturing cycle efficiency (MCE).

c. The delivery cycle time.

2. Using the performance measures given in the problem and those you computed in (1) above,
identify whether the trend over the four months is generally favorable, generally unfavorable,
or mixed. What areas apparently require improvement and how might they be improved?

3. Refer to the move time, process time, and so forth, given above for month 4.

a. Assume that in month 5 the move time, process time, and so forth, are the same as for
month 4, except that through the implementation of JIT, the company is able to completely
eliminate the queue time during production. Compute the new throughput time and MCE.
b. Assume that in month 6.the move time, process time, and so forth, are the same as for
month 4, except that the company is able to completely eliminate both the queue time during
production and the inspection time. Compute the new throughput time and MCE.
200 Chapter 4
.

Problem 2 (Balanced Scorecard; Non-value-Added Activities; Strategy Translation)


The Balanced Scorecard: A Tool to Implement Strategy 185

At the beginning of the last quarter of 20X5, Stardust, Inc., a large consumer products firm, hired
Leon Santos to take over one of its divisions. The division manufactured small home appliances
and was struggling to survive in a very competitive market. Leon immediately requested a
projected income statement for 20X5. In response, the controller provided a statement that shows
operating loss P1,000,000.

After some investigation, Leon soon realized that the products being produced had a serious
problem with quality. He once again requested a special study by the controller's office to supply
a report on the level of quality costs. By the middle of November, Leon received the following
report from the controller:

Inspection costs, finished product P 400,000


Rework costs 2,000,000
Scrapped units 600,000
Warranty costs 3,000,000
Sales returns (quality-related) 1,000,000
Customer compliant department 500,000
Total estimated quality costs P7,500,000

Leon was surprised at the level of quality costs. They represented 30 percent. of sales, certainly
excessive. He knew that to survive the division had to produce high-quality products. The
number of defective units produced needed to be reduced dramatically. Thus, Leon decided to
pursue a quality- driven turnaround strategy. Revenue growth and cost reduction could both be
achieved if quality could be improved. By increasing revenues and decreasing costs, profitability
can be increased.
CHAPTER
COST PLANNING FOR PRODUCT

8 LIFE LONG-TERM PRICING;


TARGET COSTING AND THEORY
OF CONSTRAINT

EXPECTED LEARNING OUTCOMES


After studying this chapter, you should be able to…
1. Describe the concepts of the cost life cycle and sales life concept

2. Explain and apply the methods in analyzing strategic cost management issues of

the cost-life cycle such as

 Life-cycle costing

 Target costing

 Theory of constraints

3. Distinguish between upstream costs, manufacturing and downstream costs of a

product’s life cycle

4. Realize the importance of decision making at the design stage of a product

5. Describe the characteristics of the common design model.

6. Explain the strategic cost management over the product’s sales life cycle

7. Describe Target Costing and how it is applied in the cost-life cycle

8. Know the techniques in reducing costs to a target cost level

9. Explain the steps in implementing a target cost approach

10. Describe the concept of theory of constraints and how it is applied to

improve the speed in improving speed in the manufacturing process.


CHAPTER 8
COST MANAGEMENT FOR
PRODUCT LIFE CYCLE:
LIDE-CYCLE COSTING AND LONG-TERM
PRICING; TARGET COSTING AND
THEORY OF CONSTRAINTS

COST MANAGEMENT FOR PRODUCT LIFE CYCLE

Life Cycle Costing

This chapter focuses on the time dimension of cost management. Consideration is given both to
(1) the effect of the timeliness of operations on total costs and (2) the way in which costs change
over the life cycle of the product. Product life cycle is consideration in each of two aspects

a) The cost life cycle


b) The sales life cycle

Cost life cycle is the sequence of activities within the firm that begins with research and
development, followed by design, manufacturing, marketing/distribution and customer service.
Sales life cycle is the sequence of phases in the product’s or service’s life in the market-from the
introduction of the product or service to growth in sales and finally maturity, decline and
withdrawal from the market.
Important strategic cost management issues arise in each activity of the cost life cycle. The
methods helpful in analyzing the cost life cycle are
A. Life-Cycle Costing
B. Target Costing and
C. Theory of Constraints

Life-Cycle Costing is used throughout the cost life cycle to minimize overall cost.
Target Costing is used for managing cost primarily in the design activity
Theory of Constraints is a method for managing manufacturing costs.
Two of the methods, target costing and the theory of constraints are particularly applicable to
manufacturing firms because they deal primarily with product design and manufacture. However,
each method also can be applied to service firm, to improve the efficiency and speed of the
processes involved in providing service.

A. COST MANAGEMENT FOR THE PRODUCT LIFE-CYCLE

Life-Cycle Costing is a management technique used to identify and monitor the cost of the
product or service throughout its life cycle. It provides a long-term perspective of product costs
and product or service profitability. For instance, a product that is designed quickly and
carelessly, with little investment in design costs, may have significantly higher marketing and
service costs later in the life cycle. Managers are interested in the total cost, over the entire life
cycle, and not manufacturing costs only.

Total cost over the product’s life cycle often is broken down into three components -upstream
costs, manufacturing cost and downstream costs. See Figure 8-1.

Figure 8-1:Life-Cycle Costing

Marketing & Customer


R&D Design Manufacturing
Distribution Service

LIFE-CYCLE COSTING
The sub-components of these costs follow:

Upstream Costs
Research and development
Design: prototyping, testing, concurrent engineering and quality development
Industries with high upstream costs include computer software, specialized
industrial and medical equipment

Manufacturing costs
Purchasing
Direct manufacturing costs
Indirect manufacturing costs

Downstream costs
Marketing and distribution-packaging, shipping, samples, promotion, advertising
Service and warranty-recalls, service, product liability, customer support Industries
with high downstream costs include pharmacratic, performer, cosmetics and toiletries

Why Design is Important

Decision making at the design stage is critical. Although the costs incurred at the design stage
may be very small in relation to the total costs over the entire life cycle the decision stage
decisions are important because they lock in most of the remaining life-cycle costs.

The critical success factors at the design stage include:


1. Reduced time-to-market.
2. Reduced expected service costs.
3. Improved ease-of-manufacture.
4. Process planning and design.

Reduced time-to-market
The speed of product development and the speed of delivery and efforts to reduce time-to-
market are critical for a business firm to sustain its competitiveness.
Reduced expected service costs
By careful simple design and the use of interchangeable or modular components can reduce
expected service costs.
Improved ease-of-manufacture
The design must be easy to manufacture in order to reduce production costs and speed
production.
Process planning and design
The plan for the manufacturing process should be flexible, allowing for fast setups and
product changeovers, using computer-integrated manufacturing computer assisted design and
concurrent engineering.

Common Design Models


The four common design methods are
a. Basic engineering
b. Prototyping
c. Templating and
d. Concurrent engineering
Basic engineering
This is a method in which product designers work independently from marketing and
manufacturing to develop a design from specific plans and specifications.

Prototyping
This is a method in which functional models of the product are developed and tested by
engineers and trial customers.
Templating
This is a design method in which an existing product is scaled up or down to fit the
specifications of the desired new product.
Concurrent engineering
Concurrent engineering or simultaneous engineering, is an important new approach in
which product design is integrated with manufacturing and marketing throughout the
product’s life cycle.
Figure 8-2 summarizes the characteristics of the Four Design Methods.
Figure 8-2:Characteristics of the Four Design Methods
Design Method Design Speed Design Cost Effect on
Downstream Costs
Basic engineering Fast Depends on desired Can be very high: as
complexity and marketing and
functionality; should production are not
be relatively low integral to the design
process
Prototyping Slow Significant; materials, Potentially a
labor and time significant
reduction in
downstream costs
Templating Fast Modest Unknown, can have
costly unexpected
results if the scaling
does not work in the
market or in
production
Concurrent Continuous Significant; design is The best method for
engineering an integral, ongoing reducing downstream
process costs

Illustrative Case I: Life-Cycle Costing and Pricing

Star Communications Technologies, Inc. has introduced a new phone so small that it can be
carried in a wallet. Star invested P400,000 in research and development for the technology, and
another P800,000 to design and test the prototypes. Star predicts a four-year life cycle for this
model and gathered this cost data for the wallet phone:

Monthly Fixed Cost Variable Cost


Manufacturing costs P25,000 P20
Marketing costs 20,000 5
Customer costs 3,000 8
Distribution costs 5,000 15

Sales prediction:
For price of P150 – average annual rates of 80,000 units.
For price of P180 – average annual rates of 60,000 units
For price of P225 – average annual rates of 48,000 units
If the price of a wallet phone is P225, Star will have to increase the research and the model for
the higher price. Fixed customer service costs also would increase by level of P150, fixed
marketing costs would be reduced by P5,000 per month because the low-price would be the
principal selling feature.

Required:
1. Determine the life-cycle costs for each pricing decision.
2. What price will produce the most profit for Star for the wallet phone’s life cycle?
Solution to Illustrative Case I: Life-Cycle Costing and Pricing
Requirement 1:Life-Cycle costs for each decision
Price P 150.00 P 180.00 P 225.00
Units Sold 80,000 80,000 48,000
Revenues P 12,000,000 P 10,800,000 P 10,800,000

Costs
R&D P 400,000 P 400,000 P 500,000 More at P225
Prototypes P 800,000 P 800,000 P 1,200,000 More at P225

Manufacturing
Fixed P 1,200,000 P 1,200,000 P 1,200,000 = 25,000 × 12 months × 4
years
Variable P 1,600,000 P 1,200,000 P 960,000 = 20 per unit
Marketing
Fixed P 720,000 P 960,000 P 960,000 = 20,000 × 12 × 4 (15,000
at P150)
Variable P 400,000 P 300,000 P 240,000 = 5 per unit
Customer
Service
Fixed P 144,000 P 144,000 P 168,000 = 3,000 × 12 × 4 (3500 at
P225)
Variable P 640,000 P 480,000 P 384,000 = 8 per unit
Distribution
Fixed P 240,000 P 240,000 P 240,000 = 5,000 × 12× 4
Variable P 1,2000,000 P 900,000 P 960,000 = 15 per unit (20 at P225
price)

Total Cost P 7,344,000 P 4,176,000 P 3,988,000


Requirement 2
The P150 price renders the highest expected profit.
COST MANAGEMENT OVER THE SALES LIFE CYCLE

The sales life cycle is the sequence of phases in the product’s or service’s life in the market from
the introduction of the product and services to growth in sales and finally, maturity, decline and
withdrawal from the market: Sales are at first small, then peak in the maturity phase and decline
thereafter. Figure 8-3 illustrates the sales life cycle of a product.

Figure 8-3: The Sales Life Cycle of a Product

Phases of The Sales Life Cycle

Phase 1: Product Introduction


In the first phase there is little competition, and sales rise slowly as customers become
aware of the new product or service. Costs are relatively high because of high R&D
expenditures and capital costs for setting up production facilities and marketing efforts.
Process are relatively high because of product differentiation and the high costs at this
phase. Product variety is limited.
Phase 2: Growth
Sales begin to grow rapidly and product variety increases. The product continues to enjoy
the benefits of differentiation. There is increasing competition and prices begin to soften
Phase 3: Maturity
Sales continue to increase but at a decreasing rate. There is a reduction in the
differentiation is no longer important. Competition is based on cost, given competitive
quality and functionality.

Phase 4: Decline
Sales begin to decline, as do the number of competitors. Prices stabilize. Emphasis on
differentiation returns. Survivors are able to differentiate their product, control costs, and
deliver quality and excellent service. Control of costs and an effective distribution
network are key to continued survival.

Management Focus

In the first phase, the focus of management is on design, differentiation, and marketing. The
focus shifts to new product development and pricing strategy as competition develops in the
second phase. In the third and fourth phases, management’s attention turns to cost control,
quality and service as the market continues to become more competitive. Thus, the firm’s
strategy for the product or service changes over the sales life cycle, from differentiation in the
early phases to cos leadership in the final phases.

Strategic Pricing Strategy


The strategic pricing approach changes over the life cycle of the product or service. In the first
phase, pricing is set relatively high to recover development costs and to take advantage of
product differentiation and the new demand for the product. In the second phase, pricing is likely
to stay relatively high as the firm attempts to build profitability in the growing market.
Alternatively, to maintain or increase market share at this time, relatively low prices (penetration
pricing) might be used. In the latter phases, pricing becomes more competitive, and target costing
and life-cycle costing methods are used, as the firm becomes more of a price taker rather than a
price setter and makes efforts to reduce upstream(for product enhancement)and downstream
costs.

Cost Management System


Together with the change in strategy and pricing, there is a change in the cost management
system. At the introduction and into the growth phases, the primary need is for value chain
analysis, to guide the design of products in a cost-efficient manner. Master budgets also are used
in these early phases to manage cash flows; there are large developmental costs at a time when
sales revenues are still relatively small. As the strategy shifts to cost leadership in the latter
phases, the goal of the cost management system is to provide the detailed budgets and activity-
based costing tools for accurate cost information.

Illustrative Case II: Sales Life-Cycle Analysis

The management accountant at the Aeron Manufacturing Company has collected these data in
preparation for a sales life-cycle analysis on one of its products, a leaf blower:

Item This Year Change over Last Average Annual


Year Change over the
Last Four Years
Annual sales P 2,000,000 1.5% 19.6%
Unit sales price P 400 2.0 6.9
Unit profit P 180 (0.8) 2.5

Required: Determine what stage of the sales life cycle the leaf blower is in.

Solution to Illustrative Case II: Sales Life-Cycle Analysis

It seems that sales are stabilizing since they only grew 1.5% over the past year and the average
annual growth over the past four years was 19.6%. The unit sales price has also slowed, and the
unit profit is beginning decline. As a result, total profit is starting to level off. Because of these
signs, it seems that the leaf blower is in the early maturity stage.

Illustrative Case III: Strategic Costing and Pricing


Optic Care Inc. (OCI) manufactures specialized equipment for polishing optical lenses. There are
two models – one principally used for fine eye wear ( L-25) and another for lenses used in
binoculars, cameras and similar equipment (BL-10).

The manufacturing cost of each unit is calculated by activity-based costing*,using these


manufacturing cost pools:
Cost Pools Allocation Base Costing Rate
1. Materials handling Number of parts P1.85 per part
2. Manufacturing supervision Hours of machine time P11.40 per hour
3. Assembly Number of parts P2.55 per part
4. Machine Setup Each setup P 43.30 per setup
5. Inspection and testing Logged hours P35 per hour
6. Packaging Logged hours P 15 per hour

OCI currently sells the BL-10 model for P1,050 and the L-25 model for P725.Manufacturing
costs and activity usage for the two products are:
BL- 10 L-25
Direct materials 125.50 58.19
Number of parts 121 88
Machine hours 6.1 3.2
Inspection time 1.3 0.6
Packing time 0.7 0.4
Setups 2 1
Required:
1. Calculate the product cost and product margin for each product.
2. A new competitor has entered the market for lens polishing equipment with a superior
product at significantly lower prices-P750 for the BL-10 model and P550 for the L-25
model. To try to compete, OCI has made some radical improvements in the design and
manufacturing of its two products. While the costing rates have stayed the same, the
materials costs and activity usage rates have been decreased significantly:
BL- 10 L-25
Direct materials 111.50 48.30
Number of parts 96 77
Machine hours 5.7 2.9
Inspection time 1.0 0.5
Packing time 0.7 0.4
Setups 1 1
*Activity-based costing is discussed in Chapter 9-Cost Accounting and Control,
2018Edition,E.B.CABRERA
Calculate the total product cost with the new activity usage data.
Can OCI make a profit with the new costs, assuming that OCI must meet the price set by
the new competitor?
3. What cost management method might be useful to OCI at this time and why?

Solution to Illustrative Case III: Strategic Costing


Requirement 1 and 2
Cost Pool Allocation Bas Costing
Rate
Materials Handling Number of parts 1.85
Mfg Supervision Machine hours 11.40
Assembly Number of parts 2.55
Set-ups 43.50
Inspection and Hours 35.00
Test Hours 15
Packaging

Costs and Activity Usage for Each Product Current Revised


BL-10 L-25 BL-10 L-25
Direct Materials 125.50 58.19 111.5 48.3
Number of parts 121 88 96.00 77.00
Machine hour 6.1 3.2 5.70 2.90
Inspection time 1.3 0.6 1.00 0.50
Packing time 0.7 0.4 6.75 0.40
Set-ups 2 1 1.00 1.00

Activity-based
costs
Rate
Materials 126.50 58.19 111.50 48.30
Materials handling 1.85 223.85 162.80 177.60 142.45
Mfg Supervision 11.40 69.54 36.48 64.98 33.06
Assembly 2.55 308.55 224.40 244.80 196.35
Set-ups 43.50 87.00 43.50 43.50 43.50
Inspection and 35.00 45.50 21.00 35.00 17.50
Test 10.50 6.00 10.50 6.00
Packaging 871.44 552.37 687.88 487.16
1050.00 725.00 750.00 550.00
Price 178.56 172.63 62.12 62.84
Margin
Profit will be earned even if prices are reduced as shown in the above schedule

Requirement 3

Target costing should be useful to OCI to assist the firm in meeting the new competition by
finding new ways to cut costs without reducing product quality or functionality.

B. TARGET COSTING
Target costing is a technique in which the firm determines the desired cost for the product or
service, given a competitive market price so the firm can earned a desired profit.
Target Cost = Competitive Price – Desired Profit
Target costing is a very useful way to manage the needed trade-off between increased
functionality and higher cost.

Figure 8-4shows the target costing in the cost life cycle.


Figure 8-4:Target Costing in the Cost Life Cycle

Marketing & Custo


R&D Design Manufacturing
Distribution mer
Servi

TARGET COSTING

With its positioning in the early, upstream phases of the cost life cycle, Target Costing can
clearly help a firm reduce total costs.

How to Reduce Costs to a Target Cost Level


1. Integrate new manufacturing technology using advanced cost management techniques
such as activity-based costing and seeking higher productivity through improved
organization and labor relations.
2. Redesign the product or service. This approach is more common than the first one
because it recognizes that design decisions account for mush of the product life cycle
costs.

Many firms employ both methods-operational control to achieve productivity gains and target
costing to determine low-cost design.
Steps in Implementing a Target Cost Approach
1. Determine the market price.
2. Determine the desired profit.
3. Calculate the target cost at market price less desired profit.
4. Use value engineering to identify ways to reduce product cost.
5. Use kaizen costing and operational control to further reduce costs.

The first three steps do not require additional explanation.

The following sections explain the fourth and the fifth steps.
a. The role of value engineering
b. Kaizen costing and operational control

a. Role of Value Engineering


Value engineering is used in target costing to reduce product cost by analyzing the trade-
offs between(1) different types and levels of products functionality and (2) total product
cost. An important first step in value engineering is a consumer analysis performed
during the design stage of the new or revised product. The consumer analysis identifies
critical consumer preferences that define the desired functionality for the new product.

The type of value engineering used depends on the functionality of the product. For one
group of products including camera, video equipment, functionality can be added or
deleted relatively easily. These are products that have frequent new models or updates
and customer preferences change frequently. On the other hand, for another group of
products such as construction equipment and heavy trucks, the functionality of the
product must be designed into the product rather than added on. In contract to the first
group customer preferences here are rather stable.

Target costing is more useful for products in the first group because there are a large
number of features about which the firm has some discretion.
A common type of value engineering employed in these firms is functional analysis in
which the performance and cost of each major function or feature of the product is
examined.
An overall desired level of achievement of performance for each function is obtained
while keeping the cost of all functions below the target cost.

Another technique is benchmarking which is used to determine which features give the
firm a competitive advantage. Its objective is to come up with an overall bundle of
features for the product that achieve the desired balance of meeting consumer preferences
while keeping the costs below targeted level.

Design Analysis is the common form of value engineering for products in group two,
industrial and specialized products. The design team prepare several possible designs of
the product, each having similar features, that have different levels of performance and
different levels of performance and different costs. The design team works with cost
management personnel to select the one design that best meets customer preferences
while not exceeding the target cost.

Other cost reduction approaches Include cost tables and group technology.

Cost tables are computer-based databases that include comprehensive information about
the firm's drivers. Cost drivers include, for example, the size of the product, the materials
used in its manufacture, and the number of features. Firms that manufacture different
sized parts from the same design (pipe fittings, tools and so on) use cost tables to show
the difference in cost for parts of different sizes and different types of materials.

Group technology is a method of identifying similarities in the parts of products a firm


manufactures, so the same parts can be used in two or more products, thereby reducing
costs. Large manufacturers of diverse product lines, such as in the automobile, industry,
use group technology in this way. A point of concern in the use of group technology is
that, while manufacturing costs are reduced, service and warranty costs might be
increased if a failed part is spread over many different models, with the result that a
product recall will affect many more customers.
b. Target Costing and Kaizen Costing

The fifth step in target costing is to use kaizen costing and operational control to further
reduce costs. Kaizen costing occurs at the manufacturing stage, so that the effects of
value engineering and improved design are already in place; the role for cost reduction at
this phase is to develop new manufacturing methods (such as flexible manufacturing
systems) and to use new management techniques such as operational control, total quality
management and the theory of constraints to further reduce costs. Kaizen means
“continual improvement", that is, the ongoing search for new ways to reduce costs in the
manufacturing process of a product with a given design and functionality.

Illustrative Case IV: Target Costing

MotoDrive manufactures a wide variety of parts for recreational boating, including part a
and part b component for high-powered outboard boat engines. The component is
purchased by original equipment manufacturers such as Mercury and Honda, for use in
large, more powerful outboards. The units sell for P510, and sales volume averages
25,000 units per year.

Recently, MotoDrive's major competitor reduced the price of its equivalent part to P450.
The market is very competitive, and MotoDrive realizes it must meet the new price or
lose significant market share. The controller has assembled these cost and usage data for
the most recent year for MotoDrive's production of 25,000 units:

Standard Cost Actual Quantity Actual Cost


Materials P5,125,000 P5,500,000
Direct labor 1,750.000 1.670,000
Indirect labor 2,500,000 2,359,000
Inspection (hours and cost) - 2,000 350,000
Materials handling (number - 56,000 245,000
of purchases and cost)
Machine setups (number and - 3,500 980,000
cost)
Returns and rework (number - 500 65,000
of times and cost)
TOTAL P11,169,000
Required:
1. Calculate the target cost for maintaining current market share and profitability.
2. Can the target cost be achieved? How?

Solution to Illustrative Case IV: Target Costing

Requirement 1

Current unit cost P11,169,000


2,5000 unit = P446.76

Current profit per item:


Current selling price P510
Current unit cost 446.76
Profit per item P 63.24

Target cost to meet the competitive price:


Competitor’s price P450
Less: Desired Profit 63.2
Target cost P386.76

Requirement 2:
The target cost can probably be achieved by efforts in two areas:
a. The standard cost analysis shows an unfavorable materials variance of P375,000
P5,500,000-P5,125,000) or P15 per unit, a very significant variance. Efforts to reduce or
eliminate this variance will make the firm much more competitive. Notice that the labor
usage variances, both for direct and indirect labor, are favorable, so it appears no
additional work is needed here, assuming the standards are properly set.
b. The manufacturing costs except for direct materials and direct labor can be considered
non-value adding costs, since they do not add to the functionality or quality of the
product. Efforts can be made to reduce the total cost of these manufacturing costs, which
now total a significant P3,999,000 or P159.96 per unit

D. THEORY OF CONTRAINTS

Most strategic initiatives undertaken by firm today focus on improving the speed of their
operations throughout the cost life cycle. For many companies speed is a competitive edge.
Shorter sales life cycle in many industries mean that manufacturers are working to reduce
product development time.
Theory of constraints is a process of identifying and managing constraint in the making of
products or in the providing of services. It also describes methods to maximize operating income
when faced with some bottleneck and some no bottleneck operations.
This section presents one of the methods to improve speed, Theory of Constraints (TOC) a
technique used to improve speed in the manufacturing process and thus speed.
In contrast to target costing, which focuses on the early phases of the cost life cycle, the Theory
of Constraints focuses on manufacturing activity. This theory focuses the manager’s attention on
the constraints, or bottlenecks that slow the production process. TOC emphasis the improvement
of throughput (overall all rate of manufacturing output) by removing or reducing the bottlenecks
in the production process that slow the rate of output.
Manufacturing and distribution processes that do not affect throughout are nonbinding
constraints that receive less attention that bottlenecks or binding constraints. Fast throughput
enables firms to be better prepared for quick product changeovers and changes in customer
preferences.

The Theory of Constraints define three measurement.


1. Throughput Contribution:

Revenues

2. Investments:
Sum of materials costs in direct materials, work-in-process, and finished goods
inventories; R&D costs; and costs of equipment and buildings.
3. Operating Costs:
All costs of operations (other than direct materials) incurred to earn throughout
contribution. Operating costs include salaries and wages, rent, utilities and depreciation.
Steps in Theory of Constraints Analysis
Step 1:Identify the Binding Constraint(s)

In the first step in the management accounts works with manufacturing managers and engineers
to identify binding constraints by developing a network diagram of the flow of production. A
network diagram is a flowchart of the work that shows the sequence of processes and the amount
of time required for each. The purpose of the network diagram is to help the management
accountant look for large amounts of inventory accumulating, or where there are long lead times.
Task analysis, which describes the activity of each process in detail, also could be used to
identify binding constraints.

Step 2:Determine the Most Efficient Utilization for Each Binding Constraint

In this step, the management accountant determines how to most effectively utilize the firm’s
resources. The approach differs somewhat depending on whether there is one product, or two or
more (as SPI has). If there is one product, the management accountant looks for ways to
maximize the flow of production through the constraint.

Step 3:Manage the Flows Through the Binding Constraint

In step 3,the objective is to manage the flow of production in and out of the binding constraint to
smooth the flow of production throughout the plant. The orderly scheduling of production
prevents the building of materials or work-in-process inventory at various processes. An
important tool for managing product flow in this context is the drum-buffer-rope (DBR) system,
which is a system for balancing the flow of production through a binding constraint.
Step 4: Add Capacity to the Constraint

As a longer-term measure to relieve the constraint and improve cycle time, management should
consider adding capacity to the constraints by adding new or improved machines and/or
additional labor.
Step 5: Redesign the Manufacturing Process for Flexibility and Fast Cycle Time

The most complete strategic response to the constraint is to redesign the manufacturing process,
including the introduction of new manufacturing technology, deletion of some hard-to-
manufacture products, and redesign of some products for greater ease of manufacturing. Simply
removing one or more minor features on a given product might speed up the production process
significantly. The use of value engineering as described earlier might help at this point.

The problems requiring the application of “Theory of Constraints” may also be resolved using
Linear Programming technique.

Illustrative Problem 8-1: Theory of Constraints, Throughput Contribution, Quality,


Relevant Costs

Basic data on Columbia Industries follow:

Columbia Industries manufactures electronic testing equipment. Columbia also installs the
equipment at customer’s sites and ensures that it functions smoothly. Additional information on
the Manufacturing and installation departments is as follows (capacities are expressed in terms of
the number of units of electronic testing equipment):

Equipment Equipment Installed


Manufactured
Annual capacity 400 units per year 300 units per year
Equipment manufactured and installed 300 units per year 300 units per year
Columbia manufactures only 300 units per year because the Installation Department has only
enough capacity to install 300 units. The equipment sells for P40,000 per unit (installed) and has
direct materials costs of P15,000.All costs other than direct materials costs are fixed.

Case I.
Columbia’s engineers have found a way to reduce equipment manufacturing time. The new
method would cost an additional P50 per unit a d would allow Columbia manufacture 20
additional units a year. Should Columbia implement the new method?
Answer:
It will cost Columbia P50 per unit to reduce manufacturing time. But manufacturing is not a
bottleneck operation; installation is. Therefore , manufacturing more equipment will not increase
sales and throughput contribution. Columbia Industries should nor implement the new
manufacturing method.

Case II.
Columbia’s designers have proposed a change in direct materials that would increase direct
materials costs by P2,000 per unit. This change would enable Columbia to install 320 units of
equipment each year. If Columbia makes the change, it will implement the new design on all
equipment sold. Should Columbia use the new design?
Answer:
Additional relevant costs of new direct materials,
P2,000x320 units P640,000
Increase in throughput contribution, P25,000 x 20 units P500,000
The additional incremental costs exceed the benefits from higher throughput contribution by
P140,000, so Columbia Industries should not implement the new design.
Alternatively, compare throughput contribution under each alternative.
Current throughput contribution is
P25,000x300 P7,500,000
With the modification, throughput contribution is
P23,000x320 P7,360,000
The current throughput contribution is greater than the throughput contribution resulting from the
proposed change in direct materials. Hence, Columbia Industries should not implement the new
design.

Case III.
A new installation technique has been developed that will enable Columbia’s engineers to install
10 additional units of equipment a year. The new method will increase installation costs by
P50,000 each year. Should Columbia implement the new technique?
Answer:
Increase in throughput contribution, P25,000x10 units P250,000
Increase in relevant costs P 50,000
The additional throughput contribution exceeds incremental costs by P200,000, so Columbia
Industries should implement the new installation technique.

Case IV.
Columbia is considering how to motivate workers to improve their productivity (output per
hour). One proposal is to evaluate and compensate workers in the Manufacturing and Installation
departments on the basis of their productivities. Is the new proposal a good idea?

Answer:
Motivating installation workers to increase productivity is worthwhile because installation is a
bottleneck operation, and any increase in productivity at the bottleneck will increase throughput
contribution. On the other hand, motivating workers in the manufacturing department to increase
productivity is not worthwhile. Manufacturing is not a bottleneck operation, so any increase in
output will result only in extra inventory of equipment. Columbia Industries should encourage
manufacturing to produce only as much equipment as the installation department needs, not to
produce as much as it can. Under these circumstances, it would not be a good idea to evaluate
and compensate manufacturing workers on the basis of their productivity.
Illustrative Case V: Theory of Constraints

Kable Inc., manufactures a part, XX3, used in automobiles. Three processes are involved in the
production of XX3: drilling, inserting and packaging. Each process performed at a separate
workstation and has these performance characteristics:
 The drilling function can drill 30,000 parts per hour.
 The inserting function can insert 3,000 parts per 5 minutes.
 The packaging function can package 10,000 parts per half hour.

Required: How many units of XX3 can be manufactured in a week, and which process is the
binding constraint?

Solution to Illustrative Case V: Theory of Constraints


The packaging function is the constraint because only 20,000 parts can be packaged an hour
whereas 30,000 can be drilled and 36,000 can be inserted.
Assuming a 40-hour work week, the number manufactured/week =20,000×40hours = 20,000 x
40hours = 800,000/week
REVIEW QUESTIONS AND PROBLEMS

Questions

1. What is life-cycle costing, and why is it used?


2. Do cost management practices change over the product’s sales life cycle? Explain how.
3. For what types of firms is life-cycle costing most appropriate and why?
4. Explain the difference in intended application between sales life-cycle analysis and
life-cycle costing.
5. What are the different methods of product engineering used in product design and life-
cycle costing?
6. What is meant by the sales life cycle? What are the phases of the sales life cycle? How
does it differ from the cost life cycle?
7. Do pricing strategies change over the different phases of the sales life cycle? Explain
how.
8. What is target costing, and what type of firms use it?
9. For what types of firms is target costing most appropriate and why?
10.What is meant by the concept of value engineering? How is it used in target costing?
11. Explain the two methods for reducing total product costs to achieve a desired target
cost. Which is most common in the consumer electronics industries? In the specialized
equipment manufacturing industries?
12.What is the main difference between activity-based costing and the theory of
constraints? When it is appropriate to use each one?
13. What is the role of the network diagram in the theory of constraints analysis?
14. What is meant by a binding constraint in the theory of constraint analysis? A
nonbinding constraint?
15. Name the five steps of the theory of constraints and explain the purpose of each.
Which is the most important step and why?
16. For what types of firms is the theory of constraints analysis most appropriate and
why?
Problems
Problem 1(Matching Market Characteristics with Sales Life-Cycle Stages)
Activities and Market Characteristics Sales Life-Cycle Stage
Decline in sales
Advertising
Boost in production
Stabilized profits
Competitor’s entrance into market
Market research
Market saturation
Start production
Product testing
Termination of product
Large increase in sales

Required:
Insert the appropriate life-cycle stage in the space provided after each activity.

Problem 2 (Life-Cycle Costing)


The following revenue and the cost data are for Round Manufacturing’s to radial saws.
The RM 200 is for the commercial market and the RM 800 is for industrial customers.
Both products are expected to have three-year life cycles.
RM200
Year 1 Year 2 Year 3
Revenue Costs P 500,000 P 2,000,000 P 2,500, 000
Research and 1,000,000 -0- -0-
development
Prototypes 300,000 50,000 -0-
Marketing 60,000 320,000 475,000
Distribution 80,000 120,000 130,000
Manufacturing 20,000 800,000 1,000,000
Customer service -0- 60,000 85,000
Income P (960,000) P 650,000 P 810,000
RM800
Year 1 Year 2 Year 3
Revenue Costs P 900,000 P 1,800,000 P 2,000, 000
Research and 1,150,000 -0- -0-
development
Prototypes 550,000 30,000 10,000
Marketing 124,000 200,000 260,000
Distribution 170,000 300,000 410,000
Manufacturing 85,000 600,000 700,000
Customer service -0- 20,000 10,000
Income P (1,179,000) P 650,000 P 610,000

Required:
1. How would a product life-cycle income statement differ from this calendar-year income
statement?
2. Prepare a three-year life-cycle income statement for both products. Which product appears to
be more profitable?
3.Prepare a schedule showing each cost category as a percentage of total annual costs. Pay
particular attention to the research and development and customer service categories. What do
you think this indicates about the profitability of each product over the three-year life cycle?

Problem 3 (Target Costing in a Service Firm)


TARA Alarm Systems installs home security systems. Two of TARA’s Systems, the MCU 100
and the MCU 900,have these characteristics: Design Specifications MCU 100 MCU 900
Design Specifications MCU 100 MCU 900
Video Cameras 1 3
Video monitors 1 1
Motion detectors 5 8
Floodlights 3 7
Alarms 1 2
Wiring 700 ft. 1,100 ft.
Installations 16 hrs. 26 hrs.
Cost Data for both systems
Video cameras P150/ea
Video monitors P75/ea
Motion detectors P15/ea
Floodlights P8/ea
Alarms P15/ea
Wiring P0.10/ea
Installation P20/hr

The MCU 100 sells for P810 installed and the MCU 900 sells for P1,520 installed.
Required:
1.What are the current profit margins on both systems?
2. TARA’s management believes it must drop the price of the MCU 100 to P750 and the MCU
900 to P1,390 to remain competitive in the market. Recalculate profit margins for both products
at these price levels.
3. Describe two ways that TARA could cut its costs to get the profit margins back to their
original levels.

Problem 4 (Target Costing, Strategy)


Benchmark Industries manufactures large workbenches for industrial use. Wally Garcia, the vice
president for marketing at Benchmark, has concluded from his market analysis that sales are
dwindling for Benchmark’s standard table because of aggressive pricing by competitors.
Benchmark’s table sells for P875 whereas the competition’s comparable table is selling in the
P800range.Garcia has determined that dropping price to P800 is necessary to regain the firm’s
annual market share of 10,000 tables. Cost data based on sales of 10,000 tables are:
Budgeted Amount Actual Amount Actual Cost
Direct materials 400,000 sq. ft. 425,000 sq. ft. P 2,700,000
Direct labor 85,000 hrs. 100,000 hrs. 1,000,000
Machine setups 30,000 hrs. 30,000 hrs. 300,000
Mechanical assembly 320,000 hrs. 320,000 hrs. 4,000,000
Required:
1. Calculate the current cost and profit per unit.
2. How much of the current cost per unit is attributable to non-value-added activities?
3. Calculate the new target cost per unit for a sales price of P800 if the profit per unit is
maintained.
4. What strategy do you suggest for Benchmark to attain the target cost calculated in requirement
3?

Problem 5 (Target Costing; Warehousing)

Yanny Ceramic, a wholesaler, has determined that its operations have three primary activities:
purchasing, warehousing and distributing. The firm reports the following pertinent operating data
for the year just completed:
Activity Cost Driver Quantity of Cost Cost per Unit of
Driver Cost Driver
Purchasing Number of purchasing orders 1,000 P100 per order
Warehousing Number of moves 8,000 20 per move
Distributing Number of shipments 500 80 per order

Yanny buys 100,000 units at an average cost of PS per unit and sells them at an average unit
price of P10. The firm also has a fixed operating cost of P100,000 for the year.

Yanny’s customers are demanding a 5 percent discount for the coming year. Yanny expects to sell
the same quantity if the demand for price reduction can be met. Yanny’s suppliers, however, are
willing to give only a 4 percent discount.

Required:
Yanny has estimated that the number of purchasing orders can be reduced to 800 and a P5
decrease in the cost of each shipment can be achieved with minor changes in operations, Any
further cost saving has to come from reengineering the warehousing process, What is the
maximum cost(i.e. Target cost) for warehousing if the firm desires to earn the same amount of
profit next year?
Problem 6 (Theory of Constraints, Throughput Contribution, Relevant Cost)

The Zashi Corporation manufactures filing cabinets in two operations machining and finishing. It
provides the following information.

Machining Finishing

Annual capacity 100,000 units 80,000 units


Annual production 80,000 units 80,000 units
Fixed operating cost (excluding direct
Materials) 6,400,000 4,000,000
Fixed operating cost per unit produced
(P6,400,00 ÷ 80,000,000; P4,000,000
÷ 80,000,000) P80 per unit P50 per unit

Each cabinet sells for P720 and has direct materials costs of P320 incurred at the start of the
machining operation. Zashi has no other variable costs. Zashi can sell whatever output it
produces. The following requirements refer only to the preceding data. There is no connection
between the requirements

Required:
1. Zashi is considering using some modern jigs and tools in the finishing operation that would
increase annual finishing output by 1,000 units. The annual cost of these jigs and tool is
P300,000. Should Zashi acquire these tools? Show your calculations.

2. The production manager of the Machining Department has submitted a proposal to do faster
setups that would increase the annual capacity of the Machining Department by 10,000 units and
cost P50,000 per year. Should Zashi implement the change? Show your calculations
Problem 7 (Theory of Constraints, Throughput Contribution, Relevant Costs)

Refer to the information in Problem 6 in answering the following requirements. There is no


connection between the requirements.

Required:

1. An outside contractor offers to do the finishing operation for 12,000 units at P100 per unit,
double the P50 per unit that it costs Zashi to do the finishing in-house. Should Zashi accept the
subcontractor's offer? Show your calculations.

2. The Rainee Corporation offers to machine 4,000 units at P40 per unit, half the P80 per
unit that it costs Zashi to do the machining in-house. Should Zashi accept the
subcontractor's offer? Show your calculations.

Problem 8 (Theory of Constraints, Throughput Contribution, Quality)

Refer to the information in Problem 6 in answering the following requirements. There is


no connection between the requirements.

Required:

1. Zashi produces 2,000 defective units at the machining operation. What is the cost to
Zashi of the defective items produced? Explain your answer briefly.

2. Zashi produces 2,000 defective units at the finishing operation. What is the cost to
Zashi of the defective items produced? Explain your answer briefly.
Problem 9 (Cost-plus Target Return on Investment Pricing)

John Bleu is the managing partner of a partnership that has just finished building 60-room motel.
Beck anticipates that he will rent these rooms for 16,000 nights next ear (or 16.000 room-nights).
All rooms are similar and will rent for the same price. Beck estimates the following operating
costs for next year

Variable operating cost P30 per room night


Fixed costs

Salaries and wages P1,750,000


Maintenance of building and pool 370,000
Other operating and administration
Cost 1,400,000
Total P3,520,000

The capital invested in the motel is P9.600.000. The partnership's target return on investment is
25%. Beck expects demand for rooms to be about uniform throughout the year: He plans to price
the rooms at cost plus a markup to earn the target return on investment.

Required;

1. What price should Beck charge for a room-night? What is the markup over the full cost of a
room-night?

2. Beck's market research indicates that if the price of a room-night determined in Requirement I
was reduced by 10%. Should Beck make the 10% cut?

Problem 10 (Life-Cycle Product Costing, Activity-Based Costing)

Destiny Products make digital watches. Destiny is preparing a product lifecycle budget for a new
watch, MX3. Development on the new watch with features such as a calculator and a daily diary
is to start shortly. Destiny expects the watch to have a product life cycle of 3 years. Estimates
about MX3 are as follows:

Year 1 Year 2 Year 3


Units manufactured and sold 50,000 200,000 150,000
Price per watch P450 P400 P350
R & D and design cost P9,000,000 P1,000,000 -
Manufacturing
Variable cost per watch P160 P150 P150
Variable cost per batch P1,700 P6,000 P6,000
Watches per batch 400 500 500
Fixed costs P6,000,000 P6,000,000 P6,000,000
Marketing
Variable cost per watch P36 P32 P28
Fixed costs P4,000,000 P3,000,000 P3,000,000
Distribution
Variable cost per watch P10 P10 P10
Variable cost per batch P1,200 P1,200 P1,000
Watches per batch 200 160 120
Fixed costs P2,400,000 P2,400,000 P2,400,000
Customer service costs per watch P20 P15 P15

Ignore the time value of money in your answers

Required:

1. Calculate the budgeted life-cycle operating income for the new watch.
2. What percentage of the budgeted product life-cycle costs will be incurred at the end of the
R&D and design stages?
3. An analysis reveals that 80% of the total product life-cycle costs of the new watch will be
locked in at the end of the R&D and design stages.

Problem 11 (Cost-Plus and Market-Based Pricing)

Temps. a large labor contractor, supplies contract labor to building construction companies. For
20XI, Temps has budgeted to supply 80,000 hours of contract labor. Its variable cost is P120 per
hour and its fixed costs. are P2,400,000. Roger Mason, the general manager, has proposed a cost-
plus approach for pricing labor at full cost plus 20%.

Required:

1. Calculate the price per hour that Temps should charge based on Mason's proposal.
2. Sheila Ragos, the marketing manager, has supplied the following information on demand
levels at different prices:

Price per hour Demand (hours)


P160 120,000
170 100,000
180 80,000
190 70,000
200 60,000

Temps can meet any of these demand levels. Fixed costs will remain unchanged for all
the preceding demand levels. On the basis of this additional information, what price per
hour should Temps charge?

3. Comment on your answers to Requirements I and 2. Why are they the same or not the same
Multiple Choice

1. The critical success factors for a business today are all:


a. planning-oriented. c. sales-oriented.
b. production-oriented. d. customer-oriented.

2. The Theory of Constraints (TOC) focuses on improving cycle time, the rate at which raw
materials are converted to finished product. This strategic management technique is primarily
concerned with the critical success factor of:
a. energy. c. originality.
b. quality. d. speed.

3. The key concept in TOC is:


a. benchmarking. c. the bottleneck
b. throughout. d. reengineering.

4. Which of the following determines the desired cost for a product based upon a given
competitive price?
a. benchmarking. c. reengineering.
b. target costing. d. life-cycle costing.

5. Target costing forces the firm to become more competitive, like:


a. reengineering c. activity-based costing
b. life-cycle costing. d. benchmarking.

6. Which of the following is not one of the steps in the life cycle of a product?
a. Manufacturing, inspecting, packaging and warehousing
b. Research and development
c. Purchasing and receiving
d. Marketing, promotion and distribution

7. In comparison to the Cost Life Cycle of a product, the Sales Life Cycle of a product is:
a. Much shorter
b. Much longer
c. Exactly parallel, except that it is expressed in sales terms.
d. different because it represents a sequence of phases relating to sales, not production

8. In each of the phases of a product’s sales life cycle, management’s focus will be:
a. Parallel c. different
b. On the next phase as well as the current one. d. undifferentiated

9. generally, firms will price a product more competitively at which stage of the products sales
life cycle?
a. Product Introduction c. Maturity
b. Growth d. Decline

10. Because of the four stages of a product’s sales life cycle has a different emphasis, the cost
management system be expected to provide data that is:
a. different at each stage. c. lesser in amount in the later stages.
b. common to all stages. d. lesser in amount in the early stages.

11. The sequence of activities within the firm which begins with research and development,
followed by design, and manufacturing, marketing/distribution, and customer service is the:
a. sales life cycle c. market life cycle
b. target life cycle d. critical life cycle

12. The sequence of phases in the product or service’s life in the market-from the introduction of
the product or service to the growth in sales and finally maturity, decline, and withdrawal from
the market is the:
a. sales life cycle c. market life cycle
b. target lifecycle d. cost life cycle

13. When a firm determines the desired cost for a product or service, given a competitive market
price, in order to earn a desired profit, the firm is exercising:
a. Target costing c. variable costing
b. Life cycle costing d. absorption costing

14. Which one of the following is used in target costing to reduce product cost by analyzing the
tradeoffs between (1) different types and levels of product functionality and (2)total product
cost?
a. Benchmarking c. productivity analysis
b. functional analysis d. value engineering

15. Which one of the following is a common type of value engineering in which performance
and cost?
a. Benchmarking c. productivity analysis
b. Functional analysis d. functional engineering

16. Which one of the following is a common form of value engineering in which the designing
team prepares several possible designs of the product?
a. Benchmarking c. productivity analysis
b. functional analysis d. design analysis

17.Which one of the following is not one of the five steps in TOC analysis?
a. Identify the binding constraint(s).
b. Determine the most efficient utilization for each binding constraint.
c. Manage the flow through the binding constraint.
d. Deduct capacity from the constraint.
18. Which one of the following is true concerning TOC?
a. No No Ye
b. No Yes No
c. Yes No No
d. No Yes Yes

19. Which one of the following is a downstream cost?


c. Research and development c. purchasing
d. Packaging d. prototyping

20. Which one of the following industries has high upstream costs?
a. Retail c. cosmetics
b. Perfumes d. computer software

21.Which of the following is not a critical success factor at the design stage?
a. Improved ease-of-manufacture
b. Reduced time-to-market
c. Reduced expected service costs
d. Enhanced quality

22. Which one of the following is not a common design method?


a. concurrent engineering. c. templating
b. design engineering d. prototyping

23. Sales begin to grow rapidly and product variety increases in:
a. phase three c. phase five
b. phase two d. phase four

24. Sales continue to increase but at a decreasing rate is:


a. phase three c. phase five
b. phase two d. phase four

25.Sales begin to decline, as does the number of competitors, in:


a. phase three c. phase five
b. phase two d. phase four

CHAPTER
DECENTRALIZED OPERATIONS

9
AND SEGMENT REPORTING

EXPECTED LEARNING OUTCOMES


After studying this chapter, you should be able to...
1. Describe a decentralized type of organization
2. Explain the importance of decentralization in a responsibility accounting system
3. Enumerate the advantages of decentralization
4. Prepare a segmented income statement using the contribution margin format
5. Explain the difference between traceable and common fixed costs

CHAPTER 9
DECENTRALIZED OPERATIONS
AND SEGMENT REPORTING
DECENTRALIZED OPERATIONS
Underlying Concept of Decentralized Operations
Management must accomplish its objectives by working through people. Presidents of
companies could not possibly execute all of their company's strategies alone and therefore must
rely on other people. The process of delegating the decision-making authority throughout an
organization is called decentralization.
Before designing strategic performance measurement systems, top managers determine
when delegation of responsibility (called decentralization) is desirable.
The strategic benefit of the centralized approach is that top management retains control
over key business functions, ensuring a desired level of performance. Additionally, with top
management involvement in most decisions, the expertise of top management can be effectively
coordinated for many firms, however, a decentralized approach is preferable. The main reason is
that top management cannot effectively manage the operations at a very detailed level; it lacks
the necessary local knowledge. Decisions at lower levels in the firm must be made on a timely
basis using the information at hand to make the firm more responsive to the customer. For
example, the retail store manager must often make quick changes in inventory, pricing, and
advertising to respond to local competition and changing customer buying habits and tastes.
ln a decentralized organization, managers are allowed at various operating levels the
authority to make decisions relating to their area of responsibilities. This is usually presented in
an organization chart which shows not only the formal lines of reporting and communicating or
chain of command but also how responsibility has been divided among managers. It is also
possible that informal relationships and channels of communication can develop outside the
formal reporting relationships on the organization chart as a result of personal contact among
managers. The informal structure does not appear on the organization chart but is often vital to
effective operations,

Advantages of Decentralization
1. Create greater responsiveness to local needs. Information is the key to wise decisions. Subunit
managers compared with top managers are better informed about their customers, competitors,
suppliers, and employees, an well as about local factors that affect the performance of their jobs
such as ways to decrease costs and improve quality. Sony Company reports that two advantages
of decentralization are an "increase in the company's knowledge of the marketplace and
improved service to customers."
2. Leads to gains from quicker decision making. Decentralization speeds decision making,
creating a competitive advantage over centralized organizations. Centralization slows decision
making as responsibility for decisions creeps upward through layer after layer of management.

3. Increases motivation of subunit managers. Subunit managers are usually more highly
motivated when they can exercise greater individual enterprise. Johnson & Johnson, a highly
decentralized company, maintains that "Decentralization = Creativity Productivity."

4. Aids management development and learning. Giving managers more responsibility promotes
the development of an experienced pool of management talent a pool to draw from for higher-
level management positions. The organization also leams which people are not management
material. An electronics instruments company, expressed this benefit as follows: "Decentralized
units provide a training ground for general managers, and a visible field of combat where product
champions fight for their ideas."

5. Sharpens the focus of subunit managers. In a decentralized setting, the manager of a small
subunit has a concentrated focus. A small subunit is more flexible and nimble than a larger
subunit is better able to adapt itself quickly to a fast-opening market opportunity. Also, top
management.relieved of the burden of day-to-day operating decisions, can spend more time and
energy on strategic planning for the entire organization.

6. Decisions are best made at that level in an organization where problems and opportunities
arise.

7. Management is relieved of much day-to-day problem solving and is left free to concentrate on
long-range planning and on coordination of efforts

8. Segment managers obtain more job satisfaction and are encouraged to pa forth their best
efforts by giving them added responsibility and decision making authority.

9. It provides excellent training to managers by giving them greater decision- making control
over their segments.

10. Better and faster performance evaluation. Performance of managers are better measured and
evaluated because through decentralization he has more latitude for employing his or her skills
and efforts.

Limitations of Decentralization

1. Dysfunctional decision making may result to suboptimal or incongruent decision making. This
will arise when a decision's benefit to one subunit is more than offset by the costs or loss of
benefits to the organization as a whole. This cost arises because top management has given up
control over decision making.
Suboptimal decision making may occur (1) when there is a lack of harmony or congruence
among the overall organization goals, the subunit goals, and the individual goals of decision
makers, or (2) when no guidance is given to subunit managers concerning the effects of their
decision on other parts of the organization. Suboptimal decision making is most likely to occur
when the end product of one subunit is used or sold by another subunit.

2. Manager's attention may be focused only on the subunit rather than the organization as a
whole. Individual subunit managers may regard themselves as competing with managers of other
subunits in the same organization as if they were external rivals. Consequently, managers may be
unwilling to share information or to assist when another subunit faces an emergency. Also,
subunit managers may use information they have about local conditions to further their own self-
interest rather than the organization's goals.

3. Cost to gather information is increased.Managers may spend too much time obtaining
information about different subunits of the organization in order to coordinate their actions.

4. Activities may be duplicated. Several individual subunits of the organization may undertake
the same activity separately. For example, there may be a duplication of staff functions
(accounting, human resources, and legal) if an organization is highly decentralized. Centralizing
these functions helps to consolidate, streamline, and downsize these activities.

SEGMENT REPORTING

To operate effectively, managers need more information at their disposal than is available
in a single companywide income statement.

Statements of income designed to focus on various segments of the company known as


segment reporting. A segment is any part or activity of an organization about which manager
seeks costs, revenue or profit data. Examples of segments include division of a company sales
territories, individual stores, service centers manufacturing plants, marketing departments
individual customers and product lines.

The purpose of segment reporting is to provide information needed by the manager to


determine profitability of product lines, divisions, sales territories and other segments of a
company. It emphasizes performance of a profit or investment centers rather than the
performance of the company as a whole.

Illustrative Problem 4-1. Preparation of Segmental Income Statement

From the following data, prepare a segmental income statement for the X'OR Company for year
2018 and evaluate the results

Divisions
Total Pharmaceutical Agricultural
Sales P1,000,000 P600,000 P400,000
Fixed Cost:
Controllable by division
Manager 250,000 130,000 120,000
Controlled by others 120,000 70,000 50,000
Variable Costs:
Manufacturing 420,000 220,000 200,000
Selling and administrative 140,000 70,000 70,000
Unallocated fixed costs:
Manufacturing 40,000
Selling and administrative 20,000

X’OR COMPANY
Income Statement by Segments

Divisions
Total Pharmaceutical Agricultural
Sales P1,000,000 P600,000 P400,000
Variable Cost:
Manufacturing 420,000 220,000 200,000
Selling and administrative 140,000 70,000 70,000
Total variable costs 560,000 290,000 270,000
Contribution Margin P440,000 P310,000 P130,000
Less: Fixed Costs Controlled by
Division Managers 250,000 130,000 120,000
Contribution Controllable by
Division Managers P190,000 P180,000 P10,000
Less: Fixed costs controlled by
others 120,000 70,000 50,000
Divisional Segment Margin P70,000 P110,000 P(40,000)
Less: Unallocated common costs
Manufacturing 40,000
Selling and administrative 20,000
Total 60,000
Net Income P10,000

Analysis:
Both divisions showed positive results as far as the Contribution Controllable by
Division Managers are concerned. However, the Agricultural Division showed a much
smaller contribution so that it was not able to fully absorb the other fixed costs allocated to
it. The manager of this division should strive to have a bigger contribution margin either by
adjusting the selling price of his products or controlling the variable costs to enable it to
absorb fixed costs and still have profit.
Levels of Segmented Statement

Segmented income statements can be prepared for activities at many levels in a


company. To provide more information to the company's divisional manager, the report
could further segment the divisions according to their major product lines, and the product
lines could be segmented as to how they are sold - in retail computer stores or by catalog
sales. As one goes from one segmented statement to another, he/she looks at smaller and
smaller pieces of the company. This is illustrated in Figure 9-1.

Figure 9-1
Luzon Products Inc.
Segmented Income Statement in the Contribution Format
Segments Defined as Divisions Divisions
Business Consumer
Total Company Products Division Products Division
Sales P500,000 P330,000 P200,000
Less variable expenses:
Variable cost of goods sold 180,000 120,000 60,000
Other variable expenses 50,000 30,000 20,000
Total variable expenses 230,000 150,000 80,000
Contribution margin 270,000 150,000 120,000
Less traceable fixed expenses 170,000 90,000 80,000
Divisional segment margin 100,000 P 60,000 P 40,000
Less common fixed expenses not
traceable to the individual divisions 85,000
Net operating income P 15,000

Segments Defined as Product Lines


of the Consumer Products Division
Consumer Product Line
Products Division Clip Art Computer Games
Sales P200,000 P75,000 P125,000
Less variable expenses:
Variable cost of goods sold 60,000 20,000 40,000
Other variable expenses 20,000 5,000 15,000
Total variable expenses 80,000 25,000 55,000
Contribution margin 120,000 50,000 70,000
Less traceable fixed expenses 70,000 30,000 40,000
Product-line segment margin 50,000 P 20,000 P 30,000
Less common fixed expenses not
traceable to the individual product lines 10,000
Divisional segment margin P 40,000

Segments Defined as Sales Channels for One Product Line,


Computer Games, of the Consumer Products Division

Sales Channels
Computer Games Retail Stores Catalog Sales

Sales P125,000 P100,000 P 25,000


Less variable expenses:
Variable cost of goods sold 40,000 32,000 8,000
Other variable expenses 15,000 5,000 10,000
Total variable expenses 55,000 37,000 18,000
Contribution margin 70,000 63,000 7,000
Less traceable fixed expenses 25,000 15,000 10,000
Sale-channel segment margin 45,000 P 48,000 P( 3,000)
Less common fixed expenses not
traceable to the individual sale channels 15,000
Product-line segment margin P 30,000

Significant benefits are received from a series of statements such as those presented in
Figure 9-1. A manager is able to gain considerable knowledge into the company's operations
viewed from many different angles by examining trends and results in each segment.
Through advance computer information system, such statements can easily be
constructed and be kept continuously current.

Sales and Contribution Margin

In the preparation of an income statement for a particular segment, variable expenses a


deducted from sales to yield the contribution margin for the segment. The contribution margin
particularly useful in determining what happens to profit as volume changes, assuming that the
segment's capacity and fixed costs are constant. Decisions concerning
the most effective, temporary or short-run uses of existing capacity such as special orders can be
ma using the contribution margin information.
Traceable and Common Fixed Costs

The report in Figure 9-1 shows two kinds of fixed costs-traceable and common.

Only the traceable or direct fixed costs are charged to the segments in the report. Traceable
fix costs of a segment is a fixed cost that is incurred as a consequence of the existence of the
segment and could be easily identified or traced to the particular segment. Examples are:
-Salary of the division manager
-Insurance and maintenance cost of the division building
Deducting traceable fixed costs from the segment contribution margin would yield the
segment margin or contribution to indirect or common costs.

Segment margin represents the margin available after a segment has covered all of its
own cost and the best gauge of the long-run profitability of a segment. It is also most useful in
ma decisions that affect capacity such as dropping a segment.

Finally, deducting the allocated common fixed expenses not traceable to the individual
segment from the segment margin would result to the net operating income or loss of the
segment.
Problems Related to Proper Cost Assignment
For segment reporting to accomplish its objective, costs must be properly assigned
segments. If the purpose is to determine the profits being generated by a particular division,
then all of the costs attributable to that division and only those costs-should be assigned to it.
The problem however is there are certain practices that hinder proper cost assignment. These
are:

a. omission of some costs in the assignment process,


b. the use of inappropriate methods for allocating costs among segments of a
company, and
c. the assignment of costs to segments when they are really common costs.
Omission of Costs

For financial reporting purposes, only manufacturing costs are included in product costs.
The costs assigned to a segment should include all costs attributable to that segment from the
company's entire value chain. Value chain consists of all the major business functions that add
value to a company's products and services. These business functions include research and
development, product design, manufacturing, marketing, distribution and customer service, are
required to bring a product or service to the customer and generate revenues. If companies
omit from their profitability analysis part or all of the “upstream costs" (research and
development and product design) and the "downstream costs" (marketing, distribution and
customer service), then the product is under costed and management may unintentionally
develop and maintain products that in the long run result in losses rather than profits for the
company.

Inappropriate Methods for Allocating Costs Among Segments


Cost distortion or cross-subsidization occurs when costs are improperly assigned among the
company's segments. This can occur in two ways:
(1) When the company fails to track costs directly to segments in those situations where
it is feasible to do so.
(2) When the company uses inappropriate bases to allocate costs.
Arbitrarily Dividing Common Costs Among Segments

The practice of assigning non traceable or common costs to segments is another business
practice that leads to distorted segment costs. For example, some companies allocate or divide
the costs of the corporate headquarters building equally to products on segment reports. While it
is true that common costs must be covered, arbitrarily allocating common costs to segment may
produce results that could be used in making erroneous decisions. For example, adding a share
of common costs to the real costs of a segment may make an otherwise profitable segment
appear to be unprofitable. If a manager erroneously eliminates the segment, the revenues will
be lost, the real costs of the segment will be avoided, but the common costs will still be
there. This will result to a reduction in the profits of the company as a whole and make it even
more difficult to “cover the common costs.”

REVIEW QUESTIONS AND PROBLEMS

Questions
1. What is meant by the term decentralization?
2. What benefits are felt to result from decentralization in an organization?
3. Identify three business practices that hinder proper cost assignment to segments of a
company.
4. Explain how the segment margin differs from the contribution margin. Which concept is most
useful to the manager? Why?
5. What is a segment of an organization? Give several examples of segments.
6. What costs are assigned to a segment under the contribution approach?
7. 7.Distinguish between traceable cost and a common cost. Give several
examples of each.

Problems
Problem 1(Working with a Segmented Income Statement)

Purple Associates is a consulting firm that specializes in information systems for


construction and landscaping companies. The firm has two offices-one in Manila and one in
Cebu. The firm classifies the direct costs of consulting jobs as variable costs. A segmented
income statement for the company's most recent year is given below:
Segment
Total Company Manila Cebu
Sales P750,000 100% P150,000 1 0 0 % P600,000 100%
Less variable expenses 405,000 54.0 45,000 30 360,000 60
Contribution margin 345,000 46.0 105,000 70 240,000 40
Less traceable fixed expenses168,000 22.4 78,000 52 90,000 15
Office segment margin 177,000 23.6 P27,000 18% P150,000 25%
Less common fixed expenses not
traceable to segments 120,000 16.0
Net operating income P57,000 7.6%

Required:

1. By how much would the company's net operating income increase if Cebu increased
it sales by P75,000 per year? Assume no change in cost behavior patterns.
2. Refer to the original data. Assume that sales in Manila increase by P50,000 next year and
that sales in Cebu remain unchanged.Assume no change in fixed costs.
a. Prepare a new segmented income statement for the company using the format above:
Show both amounts and percentages.
b. Observe from the income statement you have prepared that the CM ratio for Manila
has remained unchanged at 70% (the same as in the data above) but that the segment
margin ratio has changed. How do you explain the change in the segment margin ratio?

Problem 2 (Segmented Income Statement)


Sterling Company, a wholesale distributor of DVDs, has been experiencing losses for some
time, as shown by its most recent monthly income statement below:
Sales P1,500,000

Less variable expenses 588,000

Contribution margin 912,000

Less fixed expenses 945,000


Net operating loss. P (33,000)

In an effort to isolate the problem, the president has asked for an income statement
segmented by geographic market. Accordingly, the Accounting Department has developed the
following data:
Geographic Market
East Central West
Sales P400,000 P600,000 P 500,000
Variable expenses as a
percentage of sales 52% 30% 40%
Traceable fixed expenses P240,000 P330,000 P200,000

Required:
1. Prepare an income statement segmented by geographic market, as desired by the
president. Show both Amount and Percent columns for the company as a whole and for
each geographic market. Carry percentage computations to one decimal place.
2. The company's sales manager believes that sales Central geographic market could
be increased by 15% if advertising were increased by P25,000 each month. Would
you recommend the increased advertising? Show computations to support your
answer.

Problem 3(Basic Segmented Income Statement)


SCG,Inc.,produces and sells recordable CD and DVD packs. Revenue and cost
information relating to the products follow:

Product

CD DVD

Selling price per pack P 8.00 P 25.00


Variable expenses per pack P 3.20 P 17.50
Traceable fixed expenses per year P138,000 P45,000

Common fixed expenses in the company total P105,000 annually. Last year the company
produced and sold 37,500 CD packs and 18,000 DVD packs.

Required: Prepare a contribution format income statement for the year segmented by
product lines.

Problem 4(ROI Calculations with Varying Assumptions)

Knix Products is a division of Park Textiles,Inc. During the coming year.it expects
to earn a net operating income of P310,000 based on sales of P3.45million; without any
new investments, the division will have average net operating assets of P3 million. The
division is considering a capital investment project-adding knitting machines to produce
gaiters - that requires an additional investment of P600,000 and increases net operating
income by P57,500 (sales would increase by P575,000). If made, the investment would
increase beginning net operating assets by P600,000 and ending net operating assets by
P400,000. Assume that the minimum rate of return required by the company is 7 percent.

Required:
1. Compute the ROI for the division without the investment.

2. Compute the margin and turnover ratios without the investment. Show that the product
of the margin and turnover ratios equals the ROI computed in Requirement 1.

3. Compute the ROI for the division with the new investment. Do you think the divisional
manager will approve the investment?

4. Compute the margin and turnover ratios for the division with the new investment.
Compare these with the old ratios,

5. Assume that a JIT purchasing and manufacturing system is installed,reducing average


operating assets by P800,000. Compute the ROI with and without the investment under
this new scenario.Now do you think the divisional manager will accept the new
investment? Should he accept it? Explain your answer.

6. Refer to Requirement 5. Compute the margin and turnover ratios without the
investment.Use these ratios to explain why the ROI increases.

Problem 5(Product Line Analysis with Complementary Effects)

Funtime Company produces three lines of greeting cards: scented, musical and regular.
Segmented income statements for the past year are as follows:

Scented Musical Regular Total

Sales P10,000 P15,000 P25,000 P50,000

Less:Variable expenses 7,000 12,000 12,500 31,500

Contribution margin P3,000 P3,000 P12,500 P18,500


Less:Direct fixed expenses 4,000 5,000 3,000 12,000

Segmented margin P(1,000) P(2,000) P9,500 P6,500

Less:Common fixed expenses 7,500

Net profit(loss) P(1,000)

Kathy Reyes, president of Funtime, is concerned about the financial performance of her firm
and is seriously considering dropping both the scented and musical product lines.However,
before making a final decision,she consults Jim Diaz,Funtime's vice- president of marketing.

Required:

1. Jim believes that by increasing advertising by P1,000 (P250 for the scented line and
P750 for the musical line),sales of those two lines would increase by 30 percent. If you
were Kathy,how would you react to this information?
2. Jim warns Kathy that eliminating the scented and musical lines would lower the sales of
the regular line by 20 percent. Given this information,would it be profitable to eliminate the
scented and musical lines?
3. Suppose that eliminating either line reduces sales of the regular cards by 10 percent.
Would a combination of increased advertising (the option described in Requirement 1) and
eliminating one of the lines be beneficial? Identify the best combination for the firm.

Problem 6(Segmented Income Statement; Product Line Analysis)


Coco Company produces blenders and coffee makers. During the past year, 100,000
blenders and 25,000 coffee makers were produced and sold. Fixed costs for Coco totaled
P250,000, of which P90,000 can be avoided if the blenders are not produced and P45,000 can be
avoided if the coffee makers are not produced. Revenue and variable cost information follow:

Blenders Coffee Makers

Variable expenses per appliance P20 P43

Selling price per appliance 22 45

Required:

1. Prepare product-line income statements. Segregate direct and common fixed costs.
2. What would the effect be on Coco's profit if the coffee-maker line is dropped? the
blender line?
3. What would be te effect on firm profits if an additional 10,000 blenders could be
produced (using existing capacity) and sold for P20.50 on a special- order basis? Existing
sales would be unaffected by the special order.

Multiple Choice
1. A good example of a common cost which normally could not be assigned to products on a
segmented income statement except on an arbitrary basis would be:
a. productadvertising outlays.
b. salary of a corporation president.
c. direct materials.
d. the product manager's salary.

2. All other things being equal, if a division's traceable fixed expenses increase:
a. the division's contribution margin ratio will decrease.
b. the division's segment margin ratio will remain the same.
c. the division's segment margin will decrease.
d. the overall company profit will remain the same.

3. Devlin Company has two divisions,C and D. The overall company contribution margin
ratio is 30%,with sales in the two divisions totaling P500,000. If variable expenses are
P300,000 in Division C,and if division C's contribution margin ratio is 25%,then sales in
Division D must be:

a. P50,000 c. P150,000

b. P100,000 d. P200,000

4. Walsh Company has three stores:X,Y,and Z. During August,the variable expenses in Store
X were P90,000 and the contribution margin ratio was 25%. Store Y had a contribution margin
of P27,000 and a contribution margin ratio of 20%. Store Z had variable expenses of P120,000
and a variable expense ratio of 60% of sales. For August,Walsh Company's sales were:

a. P318,000 c. P485,000

b. P455,000 d. P555,000

5. Channing Company has two divisions, S and T. The company's overall contribution margin
ratio is 30% when sales in the two divisions total P750,000. If variable expenses are P405,000
in Division S,and if Division S's contribution margin ratio is 25%, then sales in Division T
must be:
a. P75,000. c. P225,000
b. P150,000 d. P300,000

Questions 6 and 7 are based on the following information.


JTC Company has two sales areas: East and West. During last year, the contribution margin in the
East area was P50,000, or 20% of sales. The segment margin in the West area was P15,000, or 8%
of sales. Traceable fixed costs are P15,000 in the East and P10,000 in the West. During last year,
the company reported total net income of P26,000.
6. The total fixed costs (traceable and common) for JTC Company for the year were:
a. P49,000 c. P24,000
b. P25,000 d. P50,000
7. The variable costs for the West Area for the year were:
a. P230,000 c. P162,500
b. P185,000 d. P65,000
Questions 8 through 11 are based on the following information.
Okinawa Company has two stores: D and S. During November, Okinawa Company reported a
net income of P30,000 and sales of P450,000. The contribution margin in Store D was
P100,000, or 40% of sales. The segment margin in Store S was P30,000, or 15% of sales.
Traceable fixed expenses are P60,000 in Store D, and P40,000 in Store S.

8. Sales in Store D totaled:


a. P400,000 c. P150,000
b. P250,000 d. P100,000
9. Variable expenses in Store S totaled:
a. P70,000 c. P200,000
b. P110,000 d. P130,000
10. Okinawa Company's total fixed expenses for the year were:
a. P40,000 c. P140,000
b. P100,000 d. P170,000
11. The segment margin ratio in Store D was:
a. 16% c. 40%
b. 24% d. 60%
12. Mamee Company has two divisions, 1 and 2. During July, the contribution margin in
Division 1 was P60,000. The contribution margin ratio in Division 2 was 40% and its sales
were P250,000. Division 2's segment margin was P60,000. The common fixed expenses were
P50,000 and the company net income was P20,000. The segment margin for Division 1 was:
a. P0 c. P50,000
b. P10,000 d. P60,000

CHAPTER
VARIABLE COSTING: A TOOL FOR
EVALUATING MANAGEMENT
PERFORMANCE
10
EXPECTED LEARNING OUTCOMES
After studying this chapter, you should be able to…

1. Explain the meaning and underlying concept of variable costing

2. Enumerate the advantages and disadvantages of using variable costing as a method of

product costing

3. Describe absorption costing

4. Compare variable costing with absorption costing

5. Prepare income statements under variable costing and absorption costing

6. Reconcile net income computed under absorption costing and net income computed under

variable costing.

7. Explain the difference in net operating income, inventory values and treatment of operating

costs between variable costing and absorption costing

8. Know why managers prefer direct costing to absorption costing

CHAPTER 10

VARIABLE COSTING: A TOOL FOR EVALUATING


MANAGEMENT PERFORMANCE
Inventory Costing and Capacity Analysis

Planning decisions typically analysis of how alternative inventory-costing choices would affect reported
income. Reported income is considered crucial in evaluating performance of mangers.

The inventory costing choices relate to which manufacturing costs are treated as inventoriable costs.
There are three of inventory costing

• Absorption costing

• Variable costing and

• Throughput costing

For manufacturing companies, the two common methods of costing inventories are absorption costing
and variable costing,

Absorption Costing

(also known as full, traditional, conventional and normal costing) is a method of product costing, in which
all manufacturing costs, fixed and variable are treated as product inventoriable costs.

This method is the required method for external reporting and ta reporting in most countries including the
Philippines.

Variable costing

(or direct costing) is a method of inventory costing in which all variable manufacturing costs are included
as inventoriable costs. All fixed manufacturing costs are excluded from inventoriable costs, fixed
manufacturing costs are instead treated as costs of the period in which there are incurred.

Variable Costing…. 255

Underlying Concept of Variable Costing

Proponents of this product costing method maintain that the fixed part of factory overhead is more closely
related to the capacity to produce than to the production of specific units and therefore should be charged
off as expense in the period incurred. Furthermore, the use of this system will permit construction of an
income statement which highlights the contribution margin of the product and therefore facilitates
managerial decision-making process. The use of variable costing for external reporting is, however, still
the center of considerable controversy. It is contended that under this method, assets (inventory) are being
understated and that it is not an accepted accounting practice.

Until variable costing becomes a generally accepted accounting practice, companies who wish to use it
must convert inventory and cost of goods sold figures to an absorption costing basis for external
reporting. This conversion is a relatively simple process in most cases and is no deterrent to the use of
variable costing for internal management purpose.

Advantages of Using Variable Costing

On advantage of variable costing is that it meets the three objectives of management control systems by
showing separately those costs that can be traced to, and controlled by each strategic business unit (SBU).
Also, net income using variable costing is not affected by changes in inventory levels because all fixed
costs are deducted from income in the period in which they occur. For this reason appraisal of
performance of product line or other segments of the business can be facilitated without the need for
arbitrary allocations of fixed cost.

Furthermore, cost-volume-profit relationship data needed for profit planning purposes is readily obtained
from the regular accounting statements. Analysis of costs relevant to pricing is likewise simplified and
enhanced. Variable costing ties in with effective plans for cost control as standard costs and flexible
budget.

Disadvantages of Using Variable Costing

1. Variable costing may encourage a shortsighted approach to profit planning at the expense of the long-
run situation.

2. Variable costing tends to give the impression that variable costs, are recovered first, that fixed costs are
recovered later and that finally profits are realized.

3. Variable costing is not acceptable for external reporting and tax purpose.

256 Chapter 10

Comparison between Variable Costing and Absorption Costing

1. As to treatment of the various operating costs:


Absorption Variable
Costing Costing

Product Direct materials Product


Costs Direct labor Costs
Variable manufacturing overhead
Fixed manufacturing overhead
Period Period
Costs Variable selling and administrative expenses
Costs
Fixed selling and administrative expenses

It will be noted that it is only in the treatment of Fixed Factory Overhead that the two costing
methods differ. Under variable costing, it is considered as period cost while under absorption costing, it is
treated as product cost.

2. As to net operating income

Net income is not affected by changes in production under variable costing. Net income, however is
affected by changes in production when absorption costing is in use. Net income goes up under the
absorption approach in response to the increase in production for a particular year and goes down when
production goes down. The reason for this effect can be traced to the shifting of fixed manufacturing cost
between periods under the absorption costing method as explained below:

Relationship between
Production (P) and Sales (S) Net Income

a) P=S AC = VC
b) P>S AC > VC
c) P<S AC < VC

a. When production volume equals sales volume, net income reported under absorption costing and
variable costing are the same. The reason is that the amount of fixed overhead charged off to
operations is the same under each method and also because there is no change in the amount of
fixed overhead in the absorption inventory.

b. When production exceeds sales volume, net income reported under absorption costing will be
greater than that under variable costing.

Variable Costing…. 257

This result occurs because part of the period’s production would go to increase in inventory, and
under absorption costing, part of the period’s fixed overhead would be deferred along with it.
c. When sales exceed production volume, net income reported under absorption costing will be
lesser than that under variable costing. The reason is that part of the period’s sales would come
from the beginning inventory, which, under absorption costing, carries with it a portion of the
prior period’s fixed overhead.

3. As to amount of inventory

Inventory value under absorption costing would be higher in amount than that under variable
costing. The inventory amount would carry a portion of fixed overhead incurred during the period under
absorption costing.

Reconciliation of Net Income under Variable Costing with Net Income under Absorption Costing

The reconciliation of the net income figures under the two product costing methods may be done as
follows:

Net Income, Absorption Costing Pxx


Add: Fixed overhead in beginning inventory xx
Total Pxx
Less: Fixed overhead in ending inventory xx
Net Income, Variable Costing Pxx

Illustrative Problem 10-1: Preparation of Income Statements under Variable Costing and
Absorption Costing

Assume the following facts for Karen Corporation for the months of January and

January February
Units:
Beginning Inventory 0 100
Production 1,000 1,000
Sales 900 1,100
Finished Goods Ending Inventory 100 0
(There is no work in process inventory)

258 Chapter 10

Costs:

Variable manufacturing costs


(per unit produced):
Direct materials P10 P10
Direct labor 5 5
Variable manufacturing overhead 3 3
Fixed manufacturing costs (per month) 8,000 8,000
Variable marketing costs (per unit sold) 2 2
Fixed manufacturing and administrative
costs (per month) 12,000 12,000
Price per unit sold P50 P50

REQUIRED:

1. Prepare Income Statements for each month and two-months combined under
a. Variable Costing
b. Absorption Costing
2. Reconcile the net income under absorption costing with net income under variable costing.

Solution: Karen Corporation

Requirement 1.
Karen Corporation
Income Statement
January and February
Variable Costing

January February Total


Sales P45,000 P25,000 P100,000
Less: Variable Cost of Sale*
Direct Materials P 9,000 P 11,000 P 20, 00
Direct Labor 4,500 5,500 10,000
Factory Overhead 2,700 3,300 6,000
Total P16,200 P19,800 P36,000
Manufacturing P28,800 P35,200 P64,000
Less: Variable marketing cost 1,800 2,000 4,000
Contribution margin-final 27,000 33,000 60,000
Less: Fixed cost
Manufacturing P 8,000 P 8,000 P 16,000
Selling and administrative 12,000 12,000 24,000
Total P20,000 P20,000 P 40,000
Net Income P 7,000 P13,000 P 20,000

Variable Costing…. 259

• Variable cost of sales may also be computed as follows:

January February Total


Inventory, beginning P 0 P 1,800 P 0
Add: Cost of goods manufactured
Direct materials P10,000 P10,000 P20,000
Direct labor 5,000 5,000 10,000
Factory overhead – variable 3,000 3,000 6,000
Total P18,000 P18,000 P36,000
Total available for sale P18,000 P19,800 P36,000
Less: Inventory, ending 1,800 - -
Cost of Sales P16, 200 P19,800 P36,000

Karen Corporation
Income Statement
January and February
Absorption Costing

January February Total


Sales P45,000 P55,000 P100,000
Less: Cost of goods sold
Inventory, beginning P 0 P 2,600 P -
Cost of goods manufactured
(1,000 x P26) 26,000 26,000 52,000
Total available P26,000 P28,000 P 52,000
Less: Inventory, end 2,600 - -
Cost of goods sold P23,400 P28,600 P 52,000
Gross Profit P21,600 P26,400 P 48,000
Les: Operating expenses
Selling and administrative
Variable P 1,800 P 2, 200 P 4,000
Fixed 12,000 12,000 24,000
Total P13,800 P14,200 P 28,000
Net Income P 7,800 P12,200 P 20,000

Requirement 2. Reconciliation of Net Income


January February Total
Net income, absorption costing P 7,800 P12,200 P20,000
Add: Fixed overhead in beginning inventory 0 800 0
Total P 7,800 P13,000 P20,000
Less: Fixed overhead in ending inventory 800 - -
Net Income, variable costing P 7,000 P13,000 P20,000

260 Chapter 10

Illustrative Problem 10-2: Computation of Inventory values under Variable


Costing and Absorption Costing
Questions 1 and 2 are based on the following information:

For the year 2018, the following cost data are available for DEF Company:
Direct material used P335,000
Direct labor P500,000
Variable manufacturing overhead P365,000
Fixed manufacturing overhead P,300,000

The company produced 150,000 units and sold 135,000 units. There are no beginning and ending
inventories of work in process and no beginning inventory of finished goods.

1. The value to be assigned to the finished goods inventory at the end under the variable costing is:
a. P100,000 d. P180,000
b. P120.000 e. None of the above
c. P150,000

2. The value to be assigned to the finished goods inventory at the end under
absorption costing is:
a. P175,000 d. P250,000
b. P200,000 e. None of the above
c. P225,000

Solution: DEF Company


1. Value to be assigned to the finished goods inventory at the end under the variable
Costing.
Direct material used P 335,000
Direct labor 500,000
Variable manufacturing overhead 365,000
Total manufacturing cost F1,200,000

P1,200,000
X 15,000 = P120,000
150,000 units

Answer: b

Variable Costing…. 261

2. Value to be assigned to the finished goods inventory at the end under absorption
costing:
Direct materials used P 35,000
Direct labor 500,000
Manufacturing overhead
Fixed P365,000
Variable 300.000 665,000
Total manufacturing cost P1,500,000

P1,500,000 X 15,000 = P150,000


150,000 units

Answer: e

Illustrative Problem 10-3: Conversion of Income Statement from Variable Costing method to
Absorption Costing method

An income statement for the manufacturing operations of Luzon Industries, Inc. for 2018 is given below.
The company operated at 75% of normal capacity during 2018 and applied the fixed manufacturing costs
to the products at a standard rate per unit of product. The inventory at the beginning of the year consisted
of 40,000 units of product and the inventory at the end of the year consisted of 30,000-units. The
company sold 88,000 units of product during the year. Inventories and production are stated on a standard
cost basis.

Luzon Industries Inc.


Income Statement - Manufacturing
For the Year 2018

Sales P1,056,000
Cost of goods sold:
Inventory, January 1 280,000
Current production cost 546,000
Cost of merchandise available for sale 826,000
Less: Inventory, December 31 210,000
Cost of goods sold 616,000
Income from manufacturing operations (standard) 440,000
Less: Capacity variance 130,000
Income from manufacturing operations (actual) P 310.000

REQUIRED:

Recast the statement on a Variable Costing basis.

262 Chapter 10

Solution: Luzon Industries, Inc.

Luzon Industries, Inc.


Income Statement for 2018
(Variable Costing)
Sales P1,056,000
Less: Cost of Sales
Inventory, beginning (40,000 x P2) P 80,000
Production costs (78,000 X P2) 158,000
Total available P 236,000
Less: Inventory,end (30,000 X P2) 60,000 176,000
Contribution margin P 880,000
Less: Fixed costs 520,000
Net Income P 360,000

Supporting Computations:
Capacity variance P130,000
Fixed costs = = 25%
25%

= 78,000 units P520,000


Normal capacity =
75%

P520,000
Fixed costs per unit = = P5.00/unit
104,000

Why Managers Prefer Direct Costing to Absorption Costing

In variable costing, only variable manufacturing costs are included in a unit's product costs, and thus in
the value of inventory and cost of goods sold. Fixed manufacturing overhead is excluded. Fixed
manufacturing overhead is excluded. It is reported as a separate expense and deducted from the
contribution margin along with fixed selling and administrative expense in determining operating income.

Managers generally prefer variable costing because it separates fixed from variable costs as in cost-
volume-profit analysis. As a result, it is easier to compare actual operating income to planned operating
income. With absorption costing, actual operating income corresponds well with planned operating
income only when inventory levels remain unchanged. With variable costing, income is more closely
associated with sales while absorption costing is influenced by units produced and
units sold.

Variable Costing... 263

Variable Costing and Performance Evaluation of Managers

The evaluation of managers is often to the profitability of the units they control. How income changes
from one period to the nest and how actual income compares to planned income are frequently used as
signals of managerial ability. To be meaningful signals, however, income should reflect managerial effort.
For example, if a manager has worked hard and increased sales while holding costs in check, income
should increase over the prior period, signaling success. In general terms, if income performance is
expected to reflect managerial performance, then managers have the high to expect the following:
1. As sales revenue increases from one period to next, all other things being equal, income should
increase.
2. As sales revenue decreases from one period to the next, all other things being equal, income
should decrease.
3. As sales revenue remains unchanged from one period to the next, all other things being equal,
income should remain unchanged.

Interestingly, income under variable costing always follows this expected association between and
income; under absorption costing, at times, it does not.

The important point is that variable costing is not affected by the change in inventory because all fixed
costs are deducted from income in the period in which they occur, fixed costs are not included in
inventory so that inventory changes do not affect net income. For this reason, variable costing net income
can be considered a more reliable measure and is preferable for use in strategic performance
measurement. When full costing is used (as is required for financial reporting), the management
accountant must use special caution in interpreting the amount of net income and attempt to determine
what portion of profit, if any, might be due to inventory changes. This is especially important of net
income is used as a basis for performance evaluation, as it is in profit Strategic Business Units (SBUs).

Variable Costing and Segmented Reporting

The usefulness of variable costing for performance evaluation extends beyond evaluating managers.
Managers need to be able to evaluate the activities over which they have responsibility. For example,
managers must continuously evaluate the profit contributions of plants, product lines, and sales territories.

Theseparation of fixed and variable costs basic to variable costing is critical for making accurate
evaluations. Implicit in an evaluation is an associated decision whether to continue to operate a plant or
not, or whether to keep or drop a product line.

264 Chapter 10

Without a distinction between fixed and variable costs, the evaluation of profit- making activities and the
resulting decision may both be erroneous. Reporting the profit contributions of activities or other units
within an organization is called segmented reporting.
Segmented reports prepared on a variable costing basis produce better evaluations and decisions than
those prepared on an absorption-costing basis. Let's take a closer look
at segmented reporting and see why this is true.

To evaluate many different activities within a firm, a manager needs more than the summary information
that appears in a firm's income statement. For example, in a company with several divisions operating in
different markets, the manager would certainly want to know how profitable each division has been. This
knowledge may lead to greater overall profit by eliminating unprofitable divisions, giving special
profitable attention to problem divisions, allocating additional funds to the more divisions, and so on.
Divisional income statements, however, are not all that a good managerial accounting system should
apply. Even finer segmentation is needed for managers to carry out their responsibilities properly.
Divisions are made up of different plants. Plants produce products and information on product
profitability is critical. Some products may be profitable; some may not be. Similarly, profit information
on sales territories, special projects, individual sales persons, and so on, is important.

Managers need to know the profitability of various segments within a firm in order to be able to make
evaluations and decisions concerning each segment's continued existence, level of funding, and soon. A
segment is any profit-making entity within the organization. A segment report can provide valuable
information on costs controllable by the segment manager. Thus, a manager who has no responsibility for
a cost should not be held accountable for that cost. For example, divisional managers have no power to
authorize corporate-level costs, such as research and development and salaries of top managers.
Therefore, divisional manager should not be held accountable for the incurrence of those costs. If
noncontrollable costs are included in a segment report, they should be separated from controllable costs
and labeled as noncontrollable. For example, fixed costs common to two or more plants within a division
would not be allocated to each plant but instead would be shown as a common cost for the division.

Segmented Reporting: Variable-Costing Basis

Segmented income statements using variable costing have one feature in addition to the variable-costing
income statements already shown. Fixed expenses are broken down into two categories: direct fixed
expenses and common fixed expenses. This additional subdivision highlights controllable versus
noncontrollable costs and enhances the manager's ability to evaluate each segment's contribution to
overall firm
performance.

Variable Costing... 265

Direct fixed expenses are fixed expenses that are directly traceable to segment (a product line in this
example). These are sometimes referred to as avoidable fixed expenses or traceable fixed expenses
because they vanish if the segment is eliminated. These fixed expenses are caused by the existence of the
segment itself.
Common fixed expenses are jointly caused by two or more segments. These expenses persist even if one
of the segments to which they are common is eliminated.

Figure 10-1 shows a segmented income statement using variable costing.

Figure 10-1: Segmented Income Statement, 2000, Variable-Costing Basis

FELCOM, INC.
Segmented Income Statement, 2000
Variable – Costing basis
Product M Product N Total
Sales P400,000 P290,000 P690,000
Less: Variable expenses
Variable cost of goods sold (300,000) (200, (500,000)
Variable selling and administrative (5,000) 000) (15,000)
Contribution margin P95,000 (10,000) P175,000
Less: Direct fixed expenses P80,000
Direct fixed overhead (30,000) (50,000)
Direct selling and administrative (10,000) (20,000) (15,000)
Segment margin P55,000 (5,000) P110,000
Less: Common fixed expenses P55,000
Common fixed overhead (100,000)
Common selling and administrative (20,000)
Net income (loss) (P10,000)

Note: Segments are defined as product lines.

Now let's examine Figure 10-1 to see whether this form of segmented income statement is more useful
than the absorption-costing format. Notice that both Product M and Product N have large positive
contribution margins (P95,000 for stereos and P80,000 for video recorders). Both products are providing
revenue above variable costs that can be used to help cover the firm's fixed costs. However, some of the
firm's fixed costs are caused by the segments themselves. Thus, the real measure of the profit contribution
of each segment is what is left over after these direct fixed costs are covered.

266 Chapter 10

The profit contribution each segment makes toward covering a firm's common fixed costs is called
segment margin. A segment should at least be able to cover both its own variable costs and direct fixed
costs. A negative segment margin drags down the firm's total profit, making it time to consider dropping
the product.
Ignoring any effect, a segment may have on the sales of other segments, the segment margin measures the
change in a firm's profits that would occur if the segment were eliminated.

From Figure 10-1 we see that the Product N line contributes P55,000 toward by P55,000. Dropping the
product N line was a disastrous decision, and now we covering Felcom's common fixed costs. If the line
is dropped, total profit decreases know why.

The correct decision is to retain both product lines. Both are making equal contributions to the firm's
profitability. Dropping either product simply aggravates the problem, unless it is replaced by a product
with a higher segment margin. Since both products have large, positive contribution margins, other
solutions to the net loss are needed. Accounting can help by focusing on a more detailed analysis of costs
using activity-based costing.

Variable Costing for Planning and Control

Financial planning requires managers to estimate future sales, future production levels, future costs, and
so on. Because sales forecasts, the basis of the budget, are not certain, management may wish to look at
several different levels of sales to assess the range of possibilities facing the firm. Knowledge of cost
behavior is fundamental to achieving this outcome. Fixed costs do not vary with volume changes; so,
distinguishing between fixed and variable costs is essential to making an accurate cost assessment at the
different possible sales and production volumes.

Once management has chosen one expected sales and production level for the coming year, the costs that
should occur can also be determined. The financial plan, then, consists of the expected activity levels and
the associated expected costs. This plan can be used to monitor the actual performance as it unfolds.

If actual performance is different from what was expected, corrective action may be necessary. By
comparing actual outcomes with the expected outcomes and taking corrective action when necessary,
managers exercise control. For the control process to work, though, cost behavior must be known.

Variable Costing... 266A

SUPER-VARIABLE COSTING
Super-variable costing is a variation on variable costing in which direct labor and manufacturing
overhead costs are considered to be fixed. Super-variable costing classifies all direct labor and
manufacturing overhead costs as fixed period costs and only direct materials as a variable product cost.

Super-Variable Costing and Variable Costing – An Illustration

To illustrate the difference between treating direct labor as a fixed cost (as in super-variable costing) and
treating direct labor as a variable cost (as in variable costing), we will use the following data from Exotic
River Boats, Inc. To simplify, this illustration we also assume that selling and administrative expenses are
entirely fixed.

Per Boat Per Month


Selling price P1,000,000
Direct materials P 190,000
Direct labor P200,000
Fixed manufacturing overhead P740,000
Fixed selling and administrative expense P400,000

January February March


Beginning inventory 0 0 1
Units produced 2 2 2
Units sold 2 1 3
Ending inventory 0 1 0

The key thing to notice here is that direct labor is a fixed cost - P200,000 per month. Also, notice that
Exotic River Boats, Inc., has no variable manufacturing overhead costs and no variable selling and
administrative expenses. For the months of January, February, and March, the company's selling price per
aircraft, variable cost per aircraft, monthly production in units, and total monthly fixed expenses never
change. The only thing that changes in this example is the number of units sold (January = 2 units sold;
February = 1 unit sold; March = 3 units sold).

Required:
1. Construct the company's super-variable costing income statements (using the contribution
format) for January, February, and March.
2. Construct in Variable Costing Income Statement.

266B Chapter 10

Super-Variable Costing Income Statements


To prepare the company's super-variable costing income statements for each
month we follow four steps:

1 First, we compute sales by multiplying the number of units sold by the selling price per unit,
which in this example is P1,000,000per unit.
2. Second, we compute the variable cost of goods sold by multiplying the number of unit sold by the
unit product cost, which in this example is the direct materials cost P190,000 per unit.
3. Third, we compute the contribution margin by subtracting variable cost of goods sold from sales.
4. Fourth, we compute net operating income by subtracting total fixed expenses, which in this

example is P1,340,000 per month ( = P200,000+ P740,000+ P400,000), from the contribution
margin.

Using these four steps. Exotic's super-variable costing income statements for each month would appear as
shown in Answer below. Notice that the only variable expense is variable cost of goods sold, which is the
P190,000 of direct materials per unit sold. For example, in March, the unit product cost of P190,000 is
multiplied by three units sold to obtain the variable cost of goods sold of P57,000. The total monthly
fixed manufacturing expenses of P940,000 include P200,000 of direct labor and P74,000 of fixed
manufacturing overhead.

Super-Variable Costing Income Statements

January February March


Sales (@P1,000,000 per unit) P2,000,000 P1,000,000 P3,000,000
Variable cost of gods sold
(sP190,000 per unit). 380,000 190,000 570,000
Contribution margin 1,620,000 810,000 2,430,000
Fixed expenses:
Fixed manufacturing expenses 940,000 940,000 940,000
Fixed selling and administrative expenses 400,000 400,000 400,000
Total fixed expenses 1,340,000 1,340,000 1,340,000
Net operating income (loss) P 280,000 P (530,000) P1,090,000

266C Chapter 10

Variable Costing Income Statements


The variable costing income statements in this example differ from the super- variable costing income
statements in one important respect - we will assume that direct labor is incorrectly classified as a variable
cost and is included in unit product costs. Because the monthly direct labor cost is P20,000 and two
aircraft produced each month, if direct labor costs are included in unit product costs, then Exotic River
Boats, Inc. will assign P100,000 of direct labor cost to each aircraft that it produces. Thus, the company's
unit product costs under variable costing would be computed as follows:

January February March


Direct materials P190,000 P190,000 P190,000
Direct labor 1000,000 1000,000 1000,000
Unit product cost P290,000 P290,000 P290,000

Given these unit product cost figures, the company's variable costing income statements would be
computed as shown below. For example, in March, the unit product cost of P290,000 is multiplied by
three units sold to obtain the variable cost of goods sold of P870,000. The total fixed manufacturing
overhead of P740,000 and total fixed selling and administrative expenses of P400,000 are both recorded
as period expenses.

Variable Costing Income Statements

January February March


Sales (@P1,000,000 per unit) P2,000,000 P1,000,000 P3,000,000
Variable cost of gods sold
(@P290,000 per unit) 580,000 290,000 870,000
Contribution margin 1,420,000 710,000 2,130,000
Fixed expenses:
Fixed manufacturing expenses 740,000 740,000 740,000
Fixed selling and administrative expenses 400,000 400,000 400,000
Total fixed expenses 1,140,000 1,140,000 1,140,000
Net operating income (loss) P 280,000 P (430,000) P 990,00

Reconciliation of Super-Variable Costing and Variable Costing Income

The super-variable costing and variable costing net operating incomes are both P280,000 in January.
However, in February, the super-variable costing income is P100,000 lower than the variable costing
income and the opposite holds true in March. In other words, the super-variable costing income in March
is P100,000 higher than the variable costing income.

266D Chapter 10
Why do these two costing methods produce different net operating incomes? The answer can be found in
the accounting for direct labor cost. Super-variable costing treats direct labor as a fixed period expense
whereas variable costing month's income statement. Conversely, variable costing assigns P100,000 of
costing records the entire direct labor cost of P200,000 as an expense on each treats direct labor as a
variable product cost. In other words, super-variable direct labor cost to each unit produced. The
P100,000 assigned to each unit produced remains in inventory on the balance sheet until the unit is sold-
which point the P100,000 assigned to it is transferred to variable cost of goods sold on the income
statement. Given this background, the super-variable costing and variable costing incomes for each month
can be reconciled as follows:

January February March


Direct labor cost in ending inventory
(@P100,000 per unit) P 0 P100,000 P 0
Deduct: Direct labor cost in beginning
inventory (@P100,000 per unit) 0 0 100,000
Direct labor cost deferred in
(released from) inventory P 0 P100,000 P(100,000)

January February March


Super-variable costing net operating
income (loss) P280,000 P(530,000) P1,090,000
Direct labor deferred in (released from)
inventory 0 100,000 (100,000)
Variable costing net operating income
(loss) P280,000 P(43,000) P 990,000

In summary, the key issue considered in this illustration is how a company treats direct labor costs. If a
company treats direct labor as a variable cost, the cost system may encourage managers to treat labor
costs as an expense, to be minimized when sales decline and this may result in reduced morale and
eventual problems when business picks up. Second, in practice, management may have little ability to
adjust the direct labor force even if they wanted to, resulting in a situation in which direct labor costs are
in fact fixed. In either case, treating direct labor costs as variable can lead to bad decisions. The super-
variable costing approach overcomes this problem by treating labor costs as fixed costs.

Variable Costing... 267


REVIEW QUESTIONS, EXERCISES AND PROBLEMS
Questions

1. "The main trouble with variable costing is that it ignores the increasing importance of fixed
costs in modern business." Do you agree? Why
2. *The central issue in variable costing is timing." Explain.
3. Why is "direct costing" a misnomer? Explain.
4. How are marketing and administrative costs treated under variable costing? Under full-
absorption costing?
5. Explain how fixed manufacturing overhead costs are shifted from one period to another under
absorption costing.
6. What arguments can be advanced in favor of treating fixed manufacturing overhead costs as
product costs?
7. If fixed manufacturing overhead costs are released from inventory under absorption costing,
what does this tell you about the level of production in relation to the level of sales?
8. Under absorption costing, how is it possible to increase net operating income without
increasing sales?
9. How is the use of variable costing limited?
10. If production exceeds sales, which method would you expect to show the higher net operating
income, variable costing or absorption costing? Why?

Exercises

Exercise 1 (Variable and Absorption Costing Unit Product Costs and Income Statements)

Max Company manufactures and sells a single product. The following costs were
incurred during the company's first year of operations:

Variable costs per unit:


Production:
Direct materials…………………………. P 18
Direct labor……………………………… 7
Variable manufacturing overhead.............. 2
Variable selling and administrative…………... 5

2668 Chapter 10
Fixed costs per year:
Fixed manufacturing overhead……………………... P160,000
Fixed selling and administrative
expenses…………………………………………… 110,000

During the year, the company produced 20,000 units and sold 16,000 units. The selling price of the
company's product is P50 per unit

Required:
1. Assume that the company uses the absorption costing method:
a. Compute the unit product cost.
b. Prepare an income statement for the year.
2. Assume that the company uses the variable costing method:
a. Compute the unit product cost.
b. Prepare an income statement for the year.

Exercise 2 (Variable and Absorption Costing Unit Product Costs)


Super Bicycle produces an inexpensive yet rugged bicycle for P5,000. Selected data for the company's
operations last year follow:

Units in beginning inventory……………………………………. 0


Units produced………………………………………………….. 10,000
Units sold……………………………………………………….. 8,000
Units in ending inventory………………………………………. 2,000

Variable costs per unit:


Direct materials…………………………………………… P1,200
Direct labor……………………………………………….. 1,400
Variable manufacturing overhead………………………… 500
Variable selling and administrative………………………. 200
Fixed costs:
Fixed manufacturing overhead…………………………… P6,000,000
Fixed selling and administrative…………………………. P4,000,000

Required:
1. Assume that the company uses absorption costing. Compute the unit product cost for one
bicycle.
2. Assume that the company uses variable costing. Compute the unit product cost for one bicycle.

Variable Costing... 269


Exercise 3 (Variable Costing Unit Product Cost and Income Statement,
Break-even)

PC Desk, Inc., makes an oak desk specially designed for personal computers
The desk sells for P2,000. Data for last year's operations follow:

Units in beginning inventory............................................ 0


Units produced................................................................. 10,000
Units sold……………………………………………….. 9,000
Units in ending inventory………………………………. 1,000

Variable costs per unit:


Direct materials..........................................................
Direct labor………………………………………….
Variable manufacturing overhead…………………...
Variable selling and administrative………………….
Total variable cost per unit.................................................

Fixed costs:
Fixed manufacturing overhead………………………
Fixed selling and administrative…………………….
Total fixed costs……………………………………………

Required:
1. Assume that the company uses variable costing. Compute the unit product cost for one computer
desk.
2. Assume that the company uses variable costing. Prepare an income statement for the year using the
contribution format.
3. What is the company's break-even point in terms of units sold?

Problems

Problem 1
Carlos Romero has gone over the financial statements for Romero Parts, Inc. The income statement has
been prepared on an absorption costing basis and Romero would like to have the statement revised on a
variable costing basis.
The company has a normal production capacity of 1,200,000 units each year. Only one line of product is
manufactured, and the inventory is accounted for on a FIFO basis. In 20X3, the fixed factory overhead
was P6,000,000. During the year, Romero Parts, Inc. manufactured 1,100,000 units of product.

270 Chapter 10
Romero Parts, Inc.
Income Statement -- Manufacturing
For the Year Ended December 31, 20X3

Sales………………………………………………….. P20,700,000
Cost of goods sold……………………………………. P 1,980,000
Inventory, first of year…………………………… 13, 200,000
Current production ………………………………
Cost of production available
for sale…………………………………….. P15,800,000
Less inventory, end of year…………………………… 1,380,000 13,800,000
Gross margin…………………………………………. P 6,900,000
Factory overhead capacity variance…………………. 500,000
Income from manufacturing…………………………. P 6,400,000

For the current year 20X4, plans have been made to manufacture 1,400,000 units of product and to sell
1,450,000 units. The unit variable cost and the selling price are expected to be the same as they were last
year. The normal capacity level will remain unchanged but fixed factory overhead can be reduced to
P5,400,000 for the year.

Required:
1. Recast the income statement for 20X3 to place it on a variable costing basis. (In your solution, show
beginning inventory at variable cost.)
2. Prepare an estimated income statement for 20X4 on an absorption costing
basis.
4. Prepare another estimated income statement for 20X4 on variable costing basis.

Problem 2

The Honey Company is comparing its present absorption costing practices with direct costing methods.
An examination of its records produced the following information:

Maximum plant capacity 40,000 units


Normal capacity 36,000 units
Fixed factory overhead P54,000
Sales price per unit P20,000
Fixed marketing and administrative expense P 10
Variable marketing expense per unit sold P 4
Standard variable manufacturing cost per unit sold P 1

Variable Costing... 271


For the year, the following data are available:

Budgeted production 36,000 units


Actual production 30,000 units
Sales 28,000 units
Finished goods inventory, January 1 P1,000
Unfavorable variances from standard
variable manufacturing costs P5,000

All variances are written off directly at year-end as an adjustment to Cost of Goods Sold.

Required:
1. Prepare the income statement under the direct costing method.
2. Prepare the income statement under the absorption costing method.

Problem 3 (Variable Costing Income Statement; Reconciliation)

During Floppy Company's first two years of operations, the company reported net operating income as
follows (absorption costing basis):

Year 1 Year 2
Sales (at P50 per unit) P1,000,000 P1,500,000
Less cost of goods sold:
Beginning inventory 0 170,000
Add cost of goods manufactured
(at P34 per unit) 850,000 850,000
Goods available for sale 850,000 1,020,000
Less ending inventory (at P34 per unit) 170,000 0
Cost of goods sold 680,000 1,020,000
Gross margin 320,000 480,000
Less selling and administrative expenses* 3,000 340,000
Net operating income P 10,000 P 140,000

*P3 per unit variable; P250,000 fixed each year.

272 Chapter 10
The company's P34 unit product cost is computed as follows:

Direct materials…………………………………… P 8
Direct labor……………………………………….. 10
Variable manufacturing overhead............................ 2
Fixed manufacturing overhead (P350,000
+25,000 units)………………………………… 14
Unit product cost…………………………………. P34

Production and cost data for the two years are given below:

Year 1 Year 2
Units produced……………………….. 25,000 25,000
Units sold…………………………….. 20,000 30,000

Required:
1. Prepare an income statement for each year in the contribution format using
variable costing.
2. Reconcile the absorption costing and variable costing net operating income figures for each
year.

Problem 4 (Prepare and Interpret Statements; Changes in Both Sales and Production; JIT)

Nemo, Inc. manufactures and sells a unique electronic part. Operating results. for the first three years of
activity were as follows (absorption costing basis):

Sales………………………………………………… P1,000,000 P800,000 P1,000,000


Cost of goods sold
Beginning inventory………………………………… 0 0 280,000
Add cost of goods manufactured…………………… 800,000 840,000 760,000
Goods available for sale……………………………. 800,000 840,000 1,040,000
Less ending inventory………………………………. 0 280,000 190,000
Cost of goods sold…………………………………… 800,000 560,000 850,000
Gross margin……………………………………….. 200,000 240,000 150,000
Less selling and administrative expenses…………… 170,000 150,000 170,000
Net operating income (loss)…………………………. P 30,000 P 90,000 P (20,000)

Sales dropped by 20% during Year 2 due to the entry of several foreign competitors into the market.
Nemo had expected sales to remain constant at 50,000 units for the year; production was set at 60,000
units in order to build a buffer of protection against unexpected spurts in demand. By the start of Year 3,
management could see that spurts in demand were unlikely and that

Variable Costing... 273


the inventory was excessive. To work off the excessive inventories, Nemo cut back production during
Year 3, as shown below:

Year 1 Year 2 Year 3


Production in units…………………………….. 50,000 60,000 40,000
Sales in units....................................................... 50,000 40,000 50,000

Additional information about the company follows:


a. The company's plant is highly automated. Variable manufacturing costs (direct materials, direct
labor, and variable manufacturing overhead) total only P4 per unit, and fixed manufacturing costs
total P600,000 per year.
b. Fixed manufacturing costs are applied to units of product on the basis of each year's production.
(That is, a new fixed overhead rate is computed each year).
c. Variable selling and administrative expenses are P2 per unit sold. Fixed selling and administrative
expenses total P70,000 per year.
d. The company uses a FIFO inventory flow assumption.

Required:
1. Prepare a new income statement for each year using the contribution approach with variable
costing.
2. Refer to the absorption costing income statements above.
a. Compute the unit product cost in each year under absorption costing. (Show how much of this
cost is variable and how much is fixed).
b. Reconcile the variable costing and absorption costing net operating income figures for each
year.
3. Refer again to the absorption costing income statements. Explain why net operating income
was higher in Year 2 than it was in Year 1 under the absorption approach, in light of the fact that
fewer units were sold in Year 2 than in Year 1.
4. Refer again to the absorption costing income statements. Explain why the company suffered a
loss in Year 3 but reported a profit in Year 1, although the same number of units was sold in each
year.
5. a. Explain how operations would have differed in Year 2 and Year 3 if the company had been
using JIT inventory methods.
b. If JIT had been in use during Year 2 and Year 3, what would the company's net operating
income (or loss) have been in each year under absorption costing? Explain the reason for any
differences between these income figures and the figures reported by the company in the
statements above.

274 Chapter 10
Multiple Choice

1. Under variable costing, fixed manufacturing ovelhead is:


a. carried in a liability account.
b. carried in an asset account.
c. ignored.
d. immediately charged against sales as a period cost.

2. Which one of the following statements is true for a firm that uses variable costing?
a. The unit product cost changes because of changes in the number of units
manufactured.
b. Profit fluctuates with sales.
c. Any underapplied overhead is calculated into the product cost.
d. Product costs include variable administration costs.

3. A principal difference between variable costing and absorption costing


centers on:
a. whether variable manufacturing costs should be included as product costs
b. whether fixed manufacturing costs should be included as product costs.
c. whether fixed manufacturing costs and fixed selling and administrative costs should
be included as product costs.
d. none of these.

4. Under variable costing:


a. net income will tend to move upward and downward in response to changes in levels
of production.
b. inventory costs will always be lower than under absorption costing.
c. net income will tend to vary inversely with production changes.
d. net income will always be higher than under absorption costing.

5. When sales are constant, but the production level fluctuates, net income determined by
the variable costing method will:
a. fluctuate in direct proportion to changes in production.
b. remain constant.
c. fluctuate inversely with changes in production.
d. be greater than net income under absorption costing.
6. The costing method that treats all fixed costs as period costs is:
a. absorption costing c. variable costing
b. job-order costing d. process costing.

Variable Costing... 275


7. Under absorption costing, fixed factory overhead costs:
a. are deferred in inventory when production exceeds sales.
b. are always treated as period costs.
c. are released from inventory when production exceeds sales.
d. none of these.

8. WB Company computes net income under both the absorption costing approach and the
variable costing approach. For a given year, the absorption costing net income was
greater than the variable costing net income. This fact suggests that:
a. variable manufacturing cots were less than fixed manufacturing costs.
b. more units were produced during the year than were sold.
c. more units were sold during the year than were produced.
d. common costs were greater than variable costs for the year.

9. Net income computed using variable costing would exceed net income computed using
absorption costing if:
a. units sold exceed units produced.
b. units sold are less than units produced.
c. units sold equal units produced.
d. the unit fixed cost is zero.

10. When sales are constant, but the production level fluctuates, net income determined by
the absorption costing method will:
a. tend to fluctuate in the same direction as fluctuations in the level of production.
b. tend to remain constant.
c. tend to fluctuate inversely with fluctuations in the level of production.
d. none of these.
11. Manga, Inc. manufactured 700 units last year. The ending inventory consisted of 100
units. There was no beginning inventory. Variable manufacturing costs were P6.00 per unit and
fixed manufacturing costs were P2.00 per unit. What would be the change in the peso amount of
ending inventory if variable costing was used instead of absorption costing?
a. P800 decrease c. PO
b. P200 decrease. d. P200 increase
12. Variable production costs are P12 per unit and variable selling and administrative
expenses are P3 per unit. Fixed manufacturing overhead totals P36,000 and fixed selling and
administration expenses total P40,000. Assuming a beginning inventory of zero, production of
4,000 units and sales of 3,600 units, the peso value of the ending inventory under variable costing
would be:
a. P4,800. c. P6,000
b. P8,400. d. P3,600.
CHAPTER

11 RELEVANT COSTS FOR


NON-ROUTINE DECISION MAKING

EXPECTED LEARNING OUTCOMES

After studying this chapter, you should be able to..

. 1. Describe the decision-making process


2. State the general rule for distinguishing between relevant and irrelevant costs in a decision-
making situation
3. Identify sunk costs and explain why they are not relevant in decision making
4. Identify opportunity costs as well as out-of-pocket costs
5. Apply the incremental analysis approach in decision
6. Apply the total project analysis approach in decision making
7. Prepare an analysis whether a part should be manufactured or purchased
8. Prepare an analysis showing whether a product line or other segments should be dropped or
retained
9. Prepare an analysis showing whether joint products should be sold at the split-off point or
processed further
10. Evaluate whether a special order should be accepted or rejected
11. Make appropriate computations to determine the optimum utilization of scarce resources 12.
Evaluate whether the company continue operations or temporarily shutdown
13. Determine selling price using cost-plus pricing approach
14. Apply target costing
CHAPTER 11 RELEVANT COSTS FOR NON-ROUTINE DECISION MAKING

Managers must constantly make decisions. In making these decisions, they must estimate how
each decision could affect operating income.

The management accountant's role in this process is to supply information on changes in costs
and revenues to facilitate the decision process. How does the accountant decide which
information to present?

Managers often select the course of action that maximizes expected operating income over the
period affected by the decision. To do this, they analyze relevant information. Relevant
information is the expected future data that differ among alternative courses of action.

In decision making, revenue and costs are often the key factors. These revenues and costs of one
alternative must be compared against the revenues and costs of other alternatives as one step in
the decision making process. The problem is that some costs associated with an alternative may
not be relevant to the decision to be made. A relevant cost can be defined as a cost that is
applicable to a particular decision in the sense that it will have a bearing on which alternative the
manager selects.

THE DECISION MAKING PROCESS

Decision making is the process of studying and evaluating two or more available alternatives
leading to a final choice. This selection process is not automatic; rather, it is a conscious
procedure. Intimately involved with planning for the future, decision making is directed toward a
specific objective or goal.

Although there are innumerable variables or factors that exist and should be considered in
making decisions in the real word, in textbook problems only a few variables that affect decision
results will be taken into account. Therefore, an organized and systematic approach may be
helpful to managers in making decisions.
The steps are outlined as follows:
1. Define strategies: business goals and tactics to achieve them.
2. Identify the alternative choices or courses of action.
3. Collect and analyze the relevant data on the choices.
4. Choose the best alternative to achieve goals.

Consideration should also be given not only to quantitative analysis but also major qualitative
issues in applying the above steps

. IDENTIFYING RELEVANT COSTS

Any cost that is avoidable is relevant for decision purposes

. Avoidable cost can be defined as a cost that can be eliminated (in whole or in part) as a result
of choosing one alternative over another in a decision-making situation. All costs are considered
avoidable, except:

1. Sunk costs
2. Future costs that do not differ between the alternatives at hand.

Relevant costs are expected future costs which differ between the decision alternatives. These
are costs that will be increased or decreased as a result of a decision.
Under the concept of relevant cost, decision-making process involves. the following analytical
steps:

1. Determine all costs associated with each alternative being considered.


2. Drop those costs that are sunk or historical.
3. Drop those costs that do not differ between alternatives.
4. Make a decision based on the remaining costs. These costs will be the future differential or
avoidable costs, and hence the costs relevant to the decision to be made.

Sunk or historical costs are never relevant in decisions because they are not avoidable and
therefore they must be eliminated from the manager's decision framework. Depreciation relating
to the book value of old equipment is not relevant in decision making. However, it is not correct
to assume that depreciation of any kind is irrelevant in the decision making process.
Depreciation is irrelevant in decision only if it relates to a sunk cost. Depreciation on a new
machine is relevant because the investment in the new machine has not yet been made and
therefore it does not represent depreciation of a sunk cost.

The resale of disposal value of an existing asset will be relevant in any decision that involves
disposing of the asset.

Lastly, any future cost that does not differ between the alternatives in a decision situation is not
a relevant cost so far as that decision is concerned.

Opportunity costs are the profits lost by the diversion of an input factor from one use to
another. They are the net economic benefit given up when an alternative is rejected. They are
relevant when a company is considering eliminating one activity and using plant facilities
advantageously in another activity. Usually formal accounting systems do not record
opportunity costs because such costs do not involve cash receipts or outlays and only data
concerning the alternative selected are recorded. However, these rejected alternatives do have
significance in decision making. For example, a single proprietor has foregone the opportunity
to earn a salary elsewhere by owning a company. In deciding to own a business, the proprietor
weighs the salary that would have been earned if he worked elsewhere.

Out-of-pocket costs involve either an intermediate or near-future cash outlay; they are usually
relevant to decisions. Frequently, variable costs fall into this classification. For example, the
direct materials needed to fill additional order's are both relevant and out-of-pocket cost while
depreciation on the existing manufacturing facilities is not. Out-of-pocket costs are important in
decision making because management should determine whether a proposed project would, at
the minimum return is initial cash outlay.
Consider the following example:

On December 31, 20X3, Company A completed the construction of a new P900,000 machine.
On cost January 3, 20X4, a salesman from an equipment supplier offered to sell the company
an P800,000 machine that can replace the constructed machine and provide operating savings
of P200,000 per year for the next five years (the life of both machines). The machine built by
Company A has no salvage value.Which costs are relevant?

The relevant costs in this example are the P800,000 potential outlay for the new machine and
the resulting operating savings of P200,000 per year. The book value of the old machine is
irrelevant. However, it is used in determining the gain or loss on disposal for tax purposes and
the effect of taxes on cash flow is a relevant cost. For Company A, a 32% tax rate on the loss of
P900,000 could mean a tax benefit of P288,000; it is the tax benefit that would be the relevant
amount.
Illustrative Problem 11-1: Identification of Relevant and Irrelevant Costs

Rosal Company owns a rice milling machine that was purchased three years ago for P250,000
with five years remaining life. Its present book value is P156,250 and resale value is P100,000.
The company is contemplating replacing this machine with a new one which will cost
P500,000 and have a five-year useful life with no salvage value. The new machine will
generate the same amount of revenue as the old one but will substantially decrease the variable
operating costs. Based on normal sales volume of 20,000 units, the annual sales and operating
costs of the old machine and the proposed replacement are estimated as follows:
OLD MACHINE NEW MACHINE
Sale 30 P600,00 P600,000
Variable Costs 350,000 200,000
Contribution Margin 250,000 400,000
Fixed Costs:
Depreciation (straight-line) 31,250 100,000
Insurance,taxes,salaries,etc. 40,000 40,000
Total 71,250 140,000
Net Operating Income P178,950 260,000

At first glance, it appears that the new machine will provide an increase in net income of
P81,050 annually. The book value of the old machine however, is a sunk cost and is not
relevant to this decision. In addition, sales and fixed costs (insurance taxes, salaries, etc.) are
also not relevant since they do not differ between the two alternatives being considered. If the
irrelevant costs, taxes and time value of money can be disregarded, the alternatives can be
analyzed as follows:

Savings in variable cost for 5 years (50.000 x 5). P750,000


Purchase cost of new machine (500,000)
Resale value of old machine 100,000
Net cash inflow. P350,000

The above computation will indicate that it would be a good move to buy the new machine
because it would result to a net cash flow of P350,000 for the 5-year period.
Approaches in Analyzing Alternatives in Nonroutine Decision Making

The two commonly used approaches in evaluating alternative courses of action are

1) Incremental or Differential analysis approach


2) Total Project Analysis approach or Comparative Statements approach.

Incremental, Differential, or Relevant Cost analysis contrasts choices by comparing differential


revenues, differential costs and differential contribution margins. It has the advantage of
showing only relevant amounts. All sunk and nondifferential items are disregarded.

The following steps are followed in using this approach:


1. Gather all costs associated with each alternative.
2. Drop the sunk costs and non-differential costs.
3. Select the best alternative based on the remaining cost data.

Total Project Analysis approach shows all the items of revenue and cost data (whether they are
relevant or not) under the different alternatives and compares the net income results.
Comparative income statements under this approach are prepared in a Contribution format.
These approaches are illustrated in the following case problem on Special Sales Order.

Illustrative Problem 11-2: Special Sales Order


Parton, Inc., a manufacturer of rattan baskets, ordinarily sells regular baskets for P32.00 each.
At the beginning of the year 20X4, an exporter has offered Parton P350,000 for 50,000 baskets
or P17.50 per basket. This sale will not affect regular business in any way. Furthermore, it will
not change fixed costs nor require additional variable selling and administrative expense and it
will put to use idle manufacturing capacity. Parton's manufacturing product cost of a basket is
P20 of which P12 is variable cost.
The income statement for the year just ended, December 31, 20X4 showed the
following results:

Sales (250,000 x P32). P8,000,000


Less: Variable expenses
Manufacturing. 3,000,000
Selling and Administrative. 750,000
Total. 3,750,000
Contribution Margin P4,250,000
Less: Fixed expenses
Manufacturing. 2,000,000
Selling and Administrative. 1,250,000
Total. 3,250,000
Operating Income. P1.000.000
Should Parton accept the special order at a sale price of P17.50?

A. Using the Differential Analysis approach, the following can be gathered:

Incremental sales revenue (50,000 x P17.50). P875,000


Incremental variable manufacturing costs (50,000 x P12) 600,000
Expected increase in operating income P275,000
B. Under the Total Project Analysis approach, the outcome will be as follows:

Without Special With Special Order Special


Order
Order 250 000 units 300,000 units 50,000 units
Total
per unit
Sales P8,000,000 P8,875,000 P875,000
P17.50
Variable Expenses
Manufacturing 3,000,000 3,600,000 600,000
12.00
Selling & administrative 750,000 750,000
Total 3,750,000 4,350,000 600,000
12.00
Contribution Margin 4,250,000 4,525,000 275,000
5.50
Fixed Expenses
Manufacturing 2,000,000 2,000,000
Selling & administrative 1,250,000 1,250,000
Total 3,250,000 3,250,000
Operating Income P1,000,000 P1,275,000 P275.000
P 5.50

Based on the analyses made (differential/total project), the Parton Company may accept the
special sales order because it will yield incremental income of P275,000.

SHORT RUN VS. LONG RUN: OTHER FACTORS TO CONSIDER

Our special sales order analysis focused only on short-run factors, the expected effect on
operating income. In making the final decision, however, long-run factors should be such as:
1. What will be the impact on customers?
2. Should regular customers find out about the special price? Will they complain, at paying
more?
3. How will competitors react?

It is therefore possible that although the order will yield additional income of P275,000, the
sales manager may reject the order to protect its long-run market position. Rejecting the order is
like "investing" P275,000 in the company's long- run future.

TYPES OF DECISIONS
This Chapter examines groups of decisions that require particular decision rules, relevant data
and formats. These decisions that commonly occur in all business activities are as follows:

1. Make or Buy
2. Add or Drop a Product or Other Segments
3. Sell Now or Process Further
4. Special Sales Pricing
5.Utilization of Scarce Resources
6. Shut-down or Continue Operations
7. Pricing

Make or Buy Decision

Differential cost analysis is appropriate for short-run make-or-buy decisions involving the
construction of plant assets or component parts of the finished product on the company premises
rather than acquiring them outside. The make-or-buy decision is a management decision about
whether an item should be made internally or bought from an outside supplier. To put idle
capacity to use, firms often consider manufacturing a part or subassembly they are currently
purchasing. For example, a watch company might use its idle capacity to produce
its own watch bands or bracelet. Or a company that manufactures cars might use its idle capacity
to manufacture its own stock absorbers instead of buying them from an outside supplier.

When these opportunities arise, the managerial accountant is often asked to compare the cost of
manufacturing a part internally with the cost of purchasing it.
Illustrative Problem 11-3: Make or buy decision

Assume that KLM Company is purchasing 2,000 parts from an outside suppliers for P170 a part.
If the company makes the part internally, costs will be assigned to the part as follows:

Direct materials. P120,000


Direct labor. P100,000
Variable overhead. P60,000
Fixed overhead. 80,000
P 360,000
Manufacturing costs per unitwill amount to P180 [ P360,000 ]

Should the company manufacture the parts or buy them from an outside supplier?

If KLM managers simply compare the total manufacturing costs of P360,000 with total purchase
costs of P340,000, they will in most probability decide to buy the part.

But looking more closely at the individual cost components, we will find the inclusion of
overhead which will be incurred regardless of whether or not it makes or buys the part. Hence,
these costs are not relevant.
For 2,000 units, the relevant costs under "make" alternative will be as follows:
Per Unit Total
Direct materials. P60. P120,000
Direct labor. P50. 100,000
Variable overhead. 30. 60,000
Total. P140. P280,000

While relevant purchase costs remain at P170 per unit or a total of P340,000. All things being
equal therefore, it would be advisable for the KLM Company to manufacture the part internally
because the company will realize cost savings of P60,000.

Before making the final decision, the company should consider other factors, both quantitative
and qualitative.
The other quantitative factor to be considered is the effect on the company's required
production level. The analysis done was based on a 2,000-part level. What if the company feels
that the needed production level is different? To guide The other quantitative factor to be
considered is the effect on the company's from the 2,000 units originally used, the accountant can
determine the point of management in making decision should the production requirement be
different indifference cost volume. This is the production level at which the cost of buying an
item equals the cost of making it. In KLM Company's case the point ofindifference cost volume
is calculated as follows:

Total costs to make Total costs to buy

40,000 + 140X 170x

where X representing the production volume = 1,333 units.

The indifference cost volume is 1,333 units. If expected production volume is below 1,333,
purchasing the part will be advisable because it is the least costly alternative. If expected
production volume is above 1,333, making the part is less costly.

Opportunity costs or earnings the company could have made if it had applied the capacity to
some alternative use are relevant and should therefore be added to the relevant costs of "make"
alternative. Examples are rent offered for the use of idle facilities, avoidable fixed costs if parts
are purchased from outside suppliers.

Adding or Dropping Products/Segments

Over time, consumers' preferences change. Some products become obsolete and are dropped
from product lines, others are developed to replace them. When management is considering
dropping a product line or customer group, the only relevant costs are those that a company
would avoid by dropping the product or customer. An important factor in deciding whether to
add or drop a product is the decision's effect on operating income.

Illustrative Problem 11-4: Eliminate or Retain a Product line

Suppose a company furnishes the following recent operating statement for its three product lines,
A, B and C.
A B C
Total
Sales P400,000 P360,000 P300,000
P1,060,000
Variable expenses 280,000 (70%) 216,000 (60%) 240,000 (80%)
736,000
Fixed expenses:
Salaries of product
line supervisors 30,000 32,000 40,000
102,000
Marketing cost allocated
to product lines on
basis of sales 8,000 7,200 6,000
21,200
Administrative costs
allocated equally 22,000 22,000 22,000
66.000
Total expenses 340,000 277,200 308,000
925.200
Operating income (loss) P 60,000 P 82.800 P (8.000)
P134,800

Management is considering discontinuing Product C operations.

The company can sell assets used in Product C operations at book value. They would lay off the
Product C supervisor with no termination pay.

a.Assuming no other changes are expected, should the company drop Product
C?
Analysis: Product C has a positive contribution to indirect costs of P20,000 (P300,000 -P240,000
- P40,000) and therefore should not be eliminated. Overall income will decrease by P20,000 if
the company will drop Product C.

b. Assuming that in addition to the data given, the following changes are expected:
1. Sales of Product A and Product B increase by 10% and 15%,
respectively.
2. Marketing costs will remain unchanged.
3. Salaries of Product A and B's product line supervisors would increase by 8% and 10%
respectively due to the increased sales.
4. No increase in total assets is required.
Should the company drop Product C?
Sell Now or Process Further
In some industries, a number of end products are produced from a single or common raw
material input. For example, in the meat-packing industry, a great variety of end products - ham,
bacon, spare ribs, pork roasts, and so on are produced from a common input are referred to as
joint products. Firms that produce several end products from a common input are faced with the
problem of deciding how the joint product cost of that input is going to be divided among the
joint product.

Joint product costs is used to describe those manufacturing costs that are incurring is producing
the joint products up to the split-off point. The split- off point is that point in the manufacturing
process at which the joint product can be recognized as separate products. Joint product costs are
irrelevant in decisions regarding what to do with a product from the split off point forward
because they have already been incurred and therefore are sunk costs.

Costs incurred after the split-off point for the benefit of only one particular product are called
separable costs. They are relevant costs in the sell-or-process-further decision.
.In sell-or-process-further decision, it will always be profitable to continue processing a joint
product after the split-off so long as the incremental revenue from such processing exceeds the
incremental processing costs.

SPECIAL SALES PRICING

Managers often must evaluate whether a special order should be accepted, or if the order is
accepted, the price that should be charged. A special order is a one- time order that is not
considered part of the company's ongoing business. Managers may be asked to consider
accepting a special order for their product at a reduced price to make use of the excess, or idle
facilities. Such orders are worth considering, provided they will not affect regular sales of the
same product.
UTILIZATION OF SCARCE RESOURCES

Choosing which products to manufacture and sell is a common managerial resources are going to
be utilized. For example, a department store has a limited decision. Managers are routinely faced
with the problem of deciding how scarce amount of flow space and therefore cannot stock every
product that may be available. A small CPA firm, due to a shortage of personnel may have to
choose limited number of machine hours and a limited number of direct labor-hours at its
between performing work for client A or for client B. A manufacturing firm has a
disposal.
When capacity becomes pressed because of a scarce resource, the firm is said to have a
constraint. Because of the constrained scarce resource, the company cannot fully satisfy demand,
so the manager must decide how the scarce resource should be used. Fixed costs are usually
unaffected by such choices, so the manager should select the course of action that will maximize
the firm's total contribution margin. This is based on the assumption that the product choices as
short-run decisions because we have adopted the definition that in the short run, capacity is
fixed, while in the long-run, capacity can be changed.

Contribution in Relation to Scarce Resources

To maximize total contribution margin, a firm should not necessarily promote those products that
have the highest contribution margins per unit. With a single constrained resource, the important
measure of profitability is the contribution margin per unit of scarce resource used.
Management is concerned with the fact that a further drop in sales volume will create a loss. This
concern has been intensified by the sales manager's opinion that the selling price of the
company's product will soon have to be adjusted to meet the increasing pressure of competition.
Since all costs, as the president puts it, have been cut to the bones, management has under
consideration a recommendation that operations be suspended until favorable conditions can be
attained and a better selling price can be set.
Before making their final decision, the company executives must recognize that not all of the
non-variable costs will be eliminated by a temporary closing of the plant. Key personnel cannot
be discharged lest they seek employment elsewhere: a skeleton staff must be maintained;
maintenance costs of building and equipment will continue, taxes and insurance premiums must
be paid during the shut-down period. As a first step, an estimate of the shutdown costs must be
made.
Assume that a conservative estimate of costs if plant operations are suspended indicates a shut-
down cost of P2,000 per month. Since there is no immediate possibility of profit under present
conditions, the problem of the company is the possibility of minimizing the loss.

PRICING PRODUCTS AND SERVICES

Some businesses have no pricing problems at all. They may be making a product for which a
market price already exists. Under these circumstances, no price calculations are necessary
because every firm charges whatever is the prevailing market price. This usually is true for basic
raw materials such as farm products, minerals, etc.

In many situations however, the firm is faced with the problem of selling its own prices. The
pricing decision can be critical because

1. the prices charged for a firm's products largely determine the quantities customers are willing
to purchase and
2. the prices should be high enough to cover all the costs of the firm.
Cost-Plus Pricing

The most basic approach in pricing decision is that the price of the product or service should
cover all the costs that are traceable to the product and service, variable as well as fixed. If
revenues are not sufficient to cover these traceable costs, then the firm would be better off
without the product or service. In addition to the traceable costs, all products and services must
assist in covering the common costs of the organization. These common costs may include
general factory, selling and administrative costs. And of course, the selling price should not only
cover all the costs of the organization but also provide a return on invested capital.

In practice, the most common approach to pricing of products is to use some type of cost-plus
pricing formula. The formula is expressed as follows:

Target selling = [mark up percentage x cost]

Target costing

This pricing approach is used when company will already know what price should be charged
and the problem will be to produce the product that can be marketed profitably. Target costing is
the process of determining the maximum allowable cost for a new product and then developing a
sample that can be profitably manufactured and distributed for that maximum target cost figure.
The target cost
is computed as follows:

Target cost = Anticipated selling price - Desired Profit


REVIEW QUESTIONS, EXERCISES AND PROBLEMS Questions

1. Distinguish briefly between quantitative and qualitative factors in decision making.


2. Why are historical costs irrelevant?
3. What is a differential cost? Distinguish it from a relevant cost.
4. A truck, costing P10,000 and uninsured, is wrecked the first day in use. It can be either
(a) disposed of for P1,000 cash and replaced with a similar truck costing P10,200, or
(b) rebuilt for P8,500 and be brand new as far as operating characteristics and looks are
concerned.
What should be done?
5. Are variable costs always relevant costs? Explain.
6. SH Company is considering dropping one of its product lines. What costs of the product line
would be relevant to this decision? Irrelevant?
7. "If a product line is generating a loss, then it should be discontinued." Do you agree? Explain.
8. What is the danger in allocating common fixed costs among product lines or other segments of
an organization?
9. What is cost-plus pricing?
10. What does the price clasticity of demand measure? What is inelastic demand? What is elastic
demand?
11. According to the economists' approach to setting prices, the profit- maximizing price should
depend on what two factors?
12. Which product should have a larger markup over variable cost, a product whose demand is
elastic or a product whose demand is inelastic?
You sent
Exercises

Exercise 1 (Identifying Relevant Costs)


A number of costs are listed below that may be relevant in decisions faced by the management of
Amarillo Co., a furniture manufacturer:
Item
a.Sales revenue....
b. Direct materials...
c. Direct labor........
d. Variable manufacturing overhead.. e. Book value - Model E7000 machine.......
f. Disposal value - Model E7000 machine........
g. Depreciation - Model E7000 machine........
h. Market value - Model F5000 machine (cost).......
i. Fixed manufacturing overhead (general) .........
j. Variable selling expense.
k. Fixed selling expense....
1. General administrative overhead....

Required
Copy the information above onto your answer sheet and place an X in the appropriate column to
indicate whether each item is relevant or not relevant in the following situations. Requirement 1
relates to Case 1 above, and requirement 2 relates to Case 2. Consider the two cases
independently. 1. The company chronically runs at capacity and the old Model E7000 machine is
the company's constraint. Management is considering the purchase of a new Model F5000
machine to use in addition to the company's present Mode! E7000 machine. The old Model
E7000 machine will continue to be used to capacity as before, with the new Model F5000 being
used to expand production. The increase in volume will be large enough to require increases in
fixed selling expenses and in general administrative overhead, but not in the general fixed
manufacturing overhead.

2. The old Model E7000 machine is not the company's constraint, but management is considering
replacing it with a new Model F5000 machine because of the potential savings in direct materials
cost with the new machine. The Model E7000 machine would be sold. This change will have no
effect on production or sales, other than some savings in
direct materials costs due to less waste.

The variable operating costs consist of gasoline, oil, retires, maintenance, and repairs. Ingrid
estimates that at her current rate of usage, the car will have zero value in five years, so the annual
straight-line depreciation is P2,000. The car is kept in a garage for a monthly fee.

Required:
1. Ingrid drove the car 10,000 kms. last year. Compute the average cost per km. of owning and
operating the car.
2. Ingrid is unsure about whether she should use her own car or rent a car to go on an extended
country trip for two weeks during summer break. What costs above are relevant in this decision?
Explain.
3. Ingrid is thinking about buying an expensive sports car to replace the car she bought last year.
She would drive the same number of kms. irrespective of which car she owns and would rent the
same parking space. The sport car's variable operating costs would be roughly the same as the
variable operating costs of her old car. However, her insurance and automobile tax and license
costs would go up. What costs are relevant in estimating the incremental cost of owning the more
expensive car? Explain.
Exercise 3 (Make or Buy a Component)
Callada Inc., manufactures a variety of heating and air-conditioning units outside supplier has
offered to sell a thermostat to Calada for P200 per unit. The company is currently manufacturing
all of its own component parts. An To evaluate this offer, Cañada, Inc.. has gathered the
following information relating to its own cost of producing the thermostat internally:
Direct materials
Direct labor
Variable manufacturing overhead..
Fixed manufacturing overhead, traceable.. Fixed manufacturing overhead, common,but
allocated.

Required:
1. Assuming that the company has no alternative use for the facilities now being used to produce
the thermostat, should the outside supplier's offer be accepted? Show all computations.
2. Suppose that if the thermostats were purchased, Cañada, Inc., could use the freed capacity to
launch a new product. The segment margin of the new product would be P650,000 per year.
Should Cañada, Inc., accept the offer to buy the thermostats from the outside supplier for P200
each? Show computations.
You sent

Exercise 4
(Evaluating Special Order)

Glamour Jewelers is considering a special order for 10 handcrafted gold bracelets to be given as
gifts to members of a wedding party. The normal selling price of a gold bracelet is P3,899.50 and
its unit product cost isP2,640.00 as shown below:

Most of the manufacturing overhead is fixed and unaffected by variations in how much jewelry
is produced in any given period. However, P70 of the overhead is variable with respect to the
number of bracelets produced. The customer who is interested in the special bracelet order would
like special filigree applied to the bracelets. This filigree would require additional materials
costing P60 per bracelet and would also require acquisition of a special tool costing P4,650 that
would have no other use once the special order is completed. This order would have no effect on
the company's regular sales and the order could be fulfilled using the company's existing
capacity without affecting any other order.

Required:

What effect would accepting this order have on the company's net operating income if a special
price of P3,499.50 is offered per bracelet for this order? Should the special order be accepted at
this price?
Exercise 5 (Utilization of a Constrained Resource)
Jaycee Company produces three products: X, Y, and Z. The selling price, variable costs, and
contribution margin for one unit of each product follow

Multiple Choice

1. Cost that do not appear in accounting records and that do not require peso outlays but do
involve a foregone opportunity by the entity whose costs are being measured are
a.conversion costs.
b. differential costs.
c. imputed costs.
d. prime costs.

2. pear Company temporarily has unused production capacity. The idle plant facilities can be
used to manufacture a low-margin item. The low- margin item should be produced if it can be
sold for more than its
a.fixed costs.
b. variable costs.
C.variable costs plus any opportunity cost of the idle facilities.
d. indirect costs plus any opportunity cost of the idle facilities.

3. As part of the data presented in support if a proposal to increase the production of clock-
radios, the sales manager of Whittaker Electronics reported the total additional cost required for
the proposed increased production level. The increase in total cost is known as
a. controllable cost.
b. differential cost.
c. opportunity cost..
d. out-of-pocket cost.

4. An item whose entire amount is usually a differential cost is


a. factory overhead.
b. direct cost.
c.conversion cost.
d. period cost.

5. In the development of accounting data for decision-making purpose, relevant costs are defined
asa.future costs which will differ under each alternative course of action.
b. the change in prime cost under each alternative course of action.
C.standard costs which are developed by time and motion study techniques because of their
relevance to managerial control.
D. historical costs which are the best available basis for estimating future costs.

6.Which of the following costs are always irrelevant in decision


making?
a.avoidable costs.
b.sunk costs.
C.opportunity costs.
d.fixed costs.

7.Consider the following statements:

I. Assemble all costs associated with each alternative beingconsidered.


II. Eliminate those costs that are sunk.
III.Eliminate those costs that differ between alternatives.

Which of the above statements does not represent a step in identifying the relevant costs in a
decision problem?
a. Only I.
B.Only III.
C. Only I and III.
D.Only II.

8.The acceptance of a special order will improve overall net operating income so long as the
revenue from the special order exceeds:

a.the contribution margin on the order.


b.the incremental costs associated with the order.
C.the variable costs associated with the order.
d.the sunk costs associated with the order.

9.Allocated common fixed costs:


A. can make a product line appear to be unprofitable.
B.are always incremental costs.
C.are always relevant in decisions involving dropping a product line.
D.responses a, b, and c are all correct.

Consider the following statements:


I. A division's net operating income, after deducting both traceable and allocated common
corporate costs, is negative.
II. The division's avoidable fixed costs exceed its contribution
margin.

III. The division's traceable fixed costs plus its allocated common corporate costs exceed its
contribution margin.
10.Which of the above statements give an economic reason for eliminating the division?

a.Only I.
B. Only II.
c.Only III.
d.Only I and II.

11.In a make or buy decision:


A.only the variable costs are relevant.
B.only the fixed costs are relevant.
C.both the variable costs and the fixed costs which will continue regardless of the decision are
relevant.
D.both the variable costs and the fixed costs which are avoidable are relevant.

12.Which of the following best describes an opportunity cost?

A.It is a relevant cost in decision making, but it is not part of the traditional accounting records.
B.It is not a relevant cost in decision making, but is part of the traditional accounting records.
C.It is a relevant cost in decision making, and is part of the traditional accounting records.
D.It is not a relevant cost in decision making, and is not part of the traditional accounting
records.

13. The opportunity cost of making a component part in a factory with excess capacity for which
there is no alternative use is:
the variable manufacturing cost of the component. the total manufacturing cost of the
component. the fixed manufacturing cost of the component. zero.

14. In a sell or process further decision, consider the following costs:

I. A variable production cost incurred prior to split-off.


II. A variable production cost incurred after split-off.
III. An avoidable fixed production cost incurred after split-off.
a.Only I.
b. Only III.
c.Only I and II.
d.Only I and III.

15. Consider the following statements: regarding

I. A firm that decides to produce its own parts runs the risk of destroying long-run relationships
with suppliers.
II. An integrated firm is less dependent on its suppliers. III. Changing technology often makes
continued production of one's own parts more costly than buying them from the outside.
Which of these statements indicate hazards to a firm that arise from being vertically integrated?

a.Only I.
b. Only II.
c Only I and II.
d.Only I and III.

16. Kala Company prepared the following tentative forecast concerning


product A for 20X3. Study made by the sales manager disclosed that the unit selling price could
be increased by 20%, with an expected volume decrease of only 10%. Assuming that Kala
incorporates these changes in its 20X3 forecast, what should be the operating income from
product A?

a. P66,000
b. P90,000
c. P120,000
d. P145,000

17. Wiggle Company sells product A at a selling price of P21 per unit. Wiggle's cost per unit
based on the full capacity of 200,000 units is as follows:

Direct materials 4
Direct labor 5
Overhead (two-thirds of which is fixed) 6
total : 15
A special order affecting to buy 20,000 units was received from a foreign distributor. The only
selling costs that would be incurred on this order would be P3 per unit for shipping. Wiggle has
sufficient existing capacity to manufacture the additional units. In negotiating a price for the
special order, Wiggle should consider that the minimum selling price per unit should be
a. P14.
b. P15.
c. P16.
d. P18.

18. Plainfield Company manufactures Part G for use in its production cycle. The costs per unit
for 10,000 units for Part G are as follows:
Direct materials 3
Direct labor 15
Variable overhead 6
Fixed overhead 8
total: 35

Verona Company has offered to sell Plainfield 10,000 units of Part G for P30 per unit. If
Plainfield accepts Verona's offer, the released facilities could be used to save P45,000 in relevant
costs in the manufacture of Part H. In addition P5 per unit of the fixed overhead applied to Part G
would be totally eliminated. What alternative is more desirable and by what amount is it more
desirable?
Alternative

a. Manufacture P10,000
b. Manufacture Amount 15,000

c. Buy 35,000
d. Buy 65,000

19. Relic Corp. manufactures batons. Relic can manufacture 300,000 batons a year at a variable
cost of P750,000 and a fixed cost of P450,000. Based on Relic's predictions, 240,000 batons will
be sold at the regular price of P5.00 each. In addition, a special order was placed for 60,000
batons to be sold at a 40% discount off the regular price. By what amount would income before
taxes be increased or decreased as a result of the special order?
a. P60,000 decrease.
b. P30,000 increase.
c. P 36,000 increase
d. P180,000 increase.

20. Three companies are each manufacturing and selling annually 10,000 units of a similar
product at a sales price of P20 per unit. The companies
have fixed and variable costs as follows:

Company RST P 40,000


Fixed Cost 80,000
Variable Cost per Unit 120,000

Each company contemplates a price decrease from P20 to P16 per unit in the expectation that
sales will increase from 10,000 to 15.000 units per Year. The contribution margin for each
company at the present sales level is a. 21. Refer to No. 0.contemplated price and sales levels
are:

22. Refer to No. 20. The increase (decrease) in operating income for R Company resulting from
the price decrease and the sales volume increase
is:
a. P(20,000) decrease.
b. P20,000 increase.
C. P 5,000 increase.
d. No increase or decrease.
E.None of the above.

23. From the accounting records of Sta. Barbara Company, the following data on costs for the
quarter ended September 30, 20X3 were
determined:

P300,00
Variable costs 0
Fixed costs 400,000
Direct materials 80,000
Direct labor P 50,000
Factory overhead 70,000
Marketing expenses 30,000
Administrative expenses 50,000
Sales for the quarter totaled
P1,200,000.

The company is considering two alternative proposals that would change certain cost items.
Proposal A would increase fixed costs by P10,000 with sales and variable costs remaining the
same. Proposal B would involve acquiring modern equipment at an annual increase of fixed
costs of P25,000, with the expectation of saving the same amount in each of the direct materials
and direct labor costs.If Proposal A is adopted, the company's profit would be:

a.P110,000.
b. P120,000.
C.P190,000..
D.P175,000.

24. Refer to Question No. 23. If Proposal B is adopted, the company's profit
would be:
a. P110,000.
b. P120,000.
c. P175,000.
d. P190,000.
e. None of the above.

25. Scully, Inc. has been manufacturing 5,000 units of Part 20561 which is used in the
manufacture of one of its products. At this level of production, the cost per unit of manufacturing
Part 20561 is as follows:
Direct materials 8
Direct labor 4
Variable overhead 46
Fixed overhead applied Total 58

Mulder Company has offered to sell Scully 5,000 units of Part 20561 for P19 a unit. Scully has
determined that it could use the facilities presently used to manufacture Part 20561 to
manufacture Product X and generate an operating profit of P4,000. Scully has also determined
that two thirds of the fixed overhead applied will continue even if Part 20561 is purchased from
Mulder. To determine whether to accept Mulder's offer, the net relevant manufacturing costs to
Scully are:

a. P70,000.
b. P80,000.
c. P90,000.
d. P95,000.

26. Dipper Company needs 20,000 of a certain part to use in its production
cycle. The following information is available:
Cost to Dipper to make the part:
Direct materials 16
Direct labor 18
Variable overhead 10
Fixed overhead applied P48
Cost to buy the part from the Orion C.
If Dipper buys the part from Orion instead of making it, Dipper could not use the released
facilities in another manufacturing activity. 60% of the fixed overhead applied will continue
regardless of what decision is made
In deciding whether to make or buy the part, the total relevant costs to make the part are:
a.P560,000.
b. P640,000.
c. P720,000.
d. P840,000.

27. The Blade Division of Dana Company produces hardened steel blades. One third of the Blade
Division's output is sold to the Lawn Products Division of Dana; the remainder is sold to outside
customers. The Blade Division's estimated sales and standard cost data for the fiscal year ending
June 30, 20X3, are as follows:

P15,00
Lawn Products 0
P40,00
Outsiders 0
Sales -10,000
Variable costs -20,000
Fixed costs -3,000
Gross margin -6,000
Unit sales P 2,000

The Lawn Products Division has an opportunity to purchase 10,000 blades of identical quality
from an outside supplier at a cost of P1.25 per unit on a continuing basis. Assume that the Blade
Division cannot sell any additional products to outside customers. Should Dana allow its Lawn
Products Division to purchase the blades from the outside supplier, and why?
You sent
a.Yes, because buying the blades would save Dana Company P500.
b. No, because making the blades would save Dana Company P1,500.
c. Yes, because buying the blades would save Dana Company P2,500.
d. No, because making the blades would save Dana Company P2,500.

28. Sta. Helena Company manufactures men's caps. The projected income
statement for the year before any special order is as follows:
Sales P400,000
Cost of goods sold 320,000
Gross margin P 80,000
Selling expenses 30,000
Operating income P 50.000

Fixed costs included in above projected income statement are P80,000 in cost of goods sold and
P9,000 in selling expenses.
A special order offering to buy 2,000 caps for P17 each was made to Sta. Helena. No additional
selling expenses will be incurred if the special order is accepted. Sta. Helena has the capacity to
manufacture 2,000 more caps. As a result of the special order, the operating income would
increase by:

a. P34,000.
b. P24,000.
C. P10,000.
d.No increase or decrease.
e.None of these.

29. Petey Company is considering a proposal to replace existing machine used for the
manufacture of product A. The new machines are expected to cause increased annual fixed costs
of P120,000; however, variable costs should decrease by 20% due to a reduction in direct labor
hours and more efficient usage of direct materials. Before this change was under consideration,
Petey had budgeted product A sales and costs for 20X3 as follows:
Sales Variable costs Fixed costs P2,000,000 70% of sales P400,000
Assuming that Petey implemented the above proposal by January 1 20X3. What would be the
increase in budgeted operating profe Product A for 20X37
A.P160,000
b. P280,000
C. P360,000
D. P480,000
30. Laney Appliance Company makes and sells electric fans. Each fam regularly sells for P42.00
The following cost data per fan is based on a fullcapacity of 150,000 fans produced each period.

Direct materials. P8
Direct labor.. 9
Manufacturing overhead (70% variable and 10
30% unavoidable fixed)
A special order has been received by Laney for a sale of 25,000 fans to an overseas customer.
The only selling costs that would be incurred on this order would be P4 per fan for shipping.
Laney is now selling 120,000 fans through regular channels each period. What should Laney use
as a minimum selling price per fan in negotiating a price for this special order?
a.p28
b.P27
c. P31
d.P24

31. Zach Company produces and sells 8,000 units of Product X each year. Each unit of Product
X sells for P10 and has a contribution margin of P6. It is estimated that if Product X is
discontinued, P50,000 of the P60,000 in fixed costs charged to Product X could be eliminated.
These data indicate that if Product X is discontinued, overall company operating income should:

a.increase by P2,000 per year.


b.decrease by P2,000 per year.
C.increase by P38,000 per year.
d.decrease by P38,000 per year.

32. The Siller Company has two divisions - East and West. The divisions have the following
revenues and expenses:
Sales.
Variable costs
Traceable fixed costs....
East
West
P720,000
P350,000
370,000
240,000
130,000
80,000
Allocated common corporate costs P70,000
Operating income (loss).. 120.000

The management at Siller is pondering the elimination of the West division since it has shown an
operating loss for the past several years. If the West division were eliminated, its traceable fixed
costs could be avoided. The total common corporate costs would be unaffected by this decision.
Given these data, the elimination of the West Division would result in an overall company
operating income of:

a.P100,000.
b. P 80,000.
C. P120,000.
d. P 50,000.

33. ADD Realty manages five apartment complexes in its region. Shown below are summary
income statements for each apartment complex:
Included in the expenses is P1,200 of common corporate expenses that have been allocated to the
apartment complexes based on rental income. These common corporate expenses would have to
be incurred regardless of how many apartment complexes ADD Realty manages. The apartment
complex(s) that ADD Realty should consider dropping is (are):

a.V, W, X, Y.
b.W, X, Y.
C.X, Y.
d.X.

34. Mott Company manufactures 10,000 units of Part EM each year for use in its production.
The following total costs were reported last year:

Direct materials P 20,000


Direct labor 55,000
Variable manufacturing overhead 45,000
Total manufacturing cost 70,000
P190.00
Fixed manufacturing overhead 0
Volvo Company has offered to sell Mott 10,000 units of Part EM for used to manufacture Part
EM could be rented to a third party at an P18 per unit. If Mott accepts the offer, some of the
facilities presently annual rental of P15,000. Additionally, P4 per unit of the fixed overhead
applied to Part EM would be totally eliminated. Should Mott accept Volvo's offer, and why?

a..No, because it would be P5,000 cheaper to make the part.


b.Yes, because it would be P10,000 cheaper to buy the part.
c.No, because it would be P15,000 cheaper to make the part.
d. Yes, because it would be P25,000 cheaper to buy the part.

35. Hollie Company produces three products, with costs and selling prices as
shown below:
Products A particular machine is a bottleneck. On that machine, 3 machine hours are required to
produce each unit of Product A, 1 hour is required to produce each unit of Product B, and 2 hours
are required to produce each unit of Product C. In which order should it produce its products?
a. C, A, B.
b.A, C, B.
C.B, C, A.
d.the order of production doesn't matter.
CHAPTER

12 QUANTITATIVE TECHNIQUES FOR


DECISION MAKING

EXPECTED LEARNING OUTCOMES

After studying this chapter, you should be able to…

1. Explain the rationale in using quantitative models for planning, control and
decision making.
2. Apply probabilities in decision making.
3. Use payoff tables in decision making.
4. Determine the expected value of perfect information.
5. Describe and prepare a decision tree.
6. Discuss the benefits and limitations of decision tree analysis.
7. Properly apply the concept of decision tree analysis.
8. Describe other quantitative techniques such as simulation techniques, Monte
Carlo technique, sensitivity analysis and queuing
9. State the nature and application of linear programming
10. Apply linear programming techniques using the graphic and simplex methods
11. Define Program Evaluation and Review Techniques (PERT)
12. Discuss and properly apply the basic underlying concept of PERT
13. Enumerate the benefits and limitations of PERT
14. Discuss the benefits of Gantt Chart
15. Apply EOQ model for inventory
16. Compute reorder point and safety stock
CHAPTER 12
QUANTITATIVE TECHNIQUES FOR DECISION MAKING

RATIONALE IN USING QUANTITATIVE TECHNIQUES

Businessmen are continually faced with making important decisions which consider a
number of variables namely;

(a) The factors that are relevant to the problem they face
(b) The various alternatives that are available to them
(c) The logical consequences of the possible alternatives
(d) The best alternative in terms of profits and realization of objectives

Increasingly, management accountants are devoting more attention to providing


management and other interested partis with data they can use in cost control, planning
and decision making rather than emphasizing cost accumulation and determination.
Emphasis is now given on the predictive ability of data rather than solely emphasizing the
past. Accountans are now integrating planning and control models within the accounting
system for monitoring actual results against plan to provide feedback for corrective
action.

Accountants must adapt to the change and equip themselves with a background in
mathematical methods if they are to supply the kind of information management
currently demands.

The quantitative analysis is a proble-solver which attempts to formulate the decision


problem in mathematical terms.

A mathematical model requires (1) specification of a complete list of a variable factors


that are relevant to the problem at hand and (2) specific quantifiable relationships among
those variables. The model can help the decision maker in making a choice which
compatible with his goals as well as enabling him to consider the variables which are
relevant in making an appropritate decision.

The accountant must concern also himself with the decision-making process since he is
the one who mustdesign the accounting information system. To be effective in this role,
the accountant must acquaint himself with the objectives, assumptions and requirement of
the decision models employed. A decision model is not intended to be the sole basis for
making a decision. Rather, it is a tool that the decision maker uses in addition to other
inputs in arriving at a decision. The use of a decision model helps to ensure that the
alternative are logically evaluated in the light of a specific criterion and explicit
assumptions.

The more commonly used quantitative models for planning, control and decision making
are as follows:

1. Probability
2. Payoff (Decision) Tables
3. Value of Perfect Information
4. Decision Tree
5. Learning Curve
6. Simulation Technique
7. Monte Carlo Technique
8. Sensitivity Analysis
9. Queuing
10. Linear Programming
11. Program Evaluation and Review Technique – Critical Path Method (PERT –
CPM)
12. Gantt Chart
13. Inventory Modeling
14. Regression Analysis
15. Present Value Concept (will be discussed in chapter 13)

An exhaustive treatment of the above-mentioned quantitative techniques is beyond the


scope or intent of this Chapter. Rather a fair understanding and appreciation of the
concepts are expected from the leader.

PROBABILITY

Decision Making under Certainty

Decision making under certainty means that for each decision action there is only one
event and therefore only a single outcome for each action. When an event is certain, there
is a 100% chance of occurrence, hence the probability is 1.0.

Decision Making under Uncertainty

Decision making under Uncertainty, which is more common in reality, involves several
events for eavch action with its probability of occurrence. The decision maker may know
the probability of occurrence of each of the events because of mathematical proofs or the
compilation of historical evidence. In the absence of these two bases, he may resort to the
subjective assignment of probabilities.

Management may know enough about the likelihood of each environment to attach
probabilities of occurrence to each alternative. If so, management certainly wants to
select the alternative that appears to produce the largest outcome as long as that
alternative does not expose the company to a high probability of a large loss. The payoffs
can be reduced using alternative to one figure by weighing the possible payoffs according
to the relative probabilities that the various conditions will occur.

Payoff is the value assigned to different outcomes from a decision and may be positive or
negative.

Briefly, information is deemed to meet the cost-benefit test if the expected value of a
decision (net of the costs of the information) increases as a result of obtaining additional
information. The process in deciding whether the cost-benefit criterion has been met is
called information economics.

Assigning Probabilities
Because decision makers normally deal with uncertainty, rather than certainty,they must
estimate the probability of various outcomes. It is necessary to assign probabilities that
represent the likelihood of a various events occurring. A probability distribution describes
the chance or likelihood of each of the collectively exhaustive and mutually exclusive set
of events. The probability distribution can be based on past data if management believes
that the same forces will continue to operate in the future. Probability provides a method
for mathematically expressing doubt or assurance about the occurrence of a chance event.
The probability of an event from 0 to 1.

(a) A probability of 0 means the event cannot occur, whereas a probability of 1 means
the event is certain to occur.
(b) A probability between 0 and 1 indicates the likelihood of the event’s occurrence,
e,g., the probability that a fair coin will yield heads is 0.5 on any single toss.
Types of Probabilities

a. Objective Probabilites are calculated from either logic or actual experience.


For example, in rolling dice one would logically expect each face on a single
die to be equally likely to turn up at a probability of 1/6. Alternatively, the die
could be rolled a great many times, and the fraction of times each face turned
up could then be used as the frequency or probability of occurrence.
b. Subjective Probabilites are estimates, based on judgment and past experience,
of the likelihood of future events. Weather forecasts often include the
subjective probability of rain. In business, subjective probability can indicate
the degree of confidence a person has that a certain outcome will occur, e.g.,
future performance of a new employee.

Basic Terms Used with Probability

(1) Two events are mutually exclusive if they cannot occur simultaneously (e.g.,
heads and tails cannot both occur on a single toss of a coin).
(2) The joint probability for two events is the probability that both will occur.
(3) The conditional probability of two events is the probability that one will occur
given that the other has already occurred.
(4) Two events are independent if the occurrence of one has no effect on the
probability of the other (e.g., rolling two dice)
(a) If one event has an effect on the other event, they are dependent.
(b) Two events are independent if their joint probability equals the product of
their individual probabilities.
(c) Two events are independent if the conditional probability of each event equals
its unconditional probability.

Rules in Combining Probabilites


The joint probability for two events equals the probability (Pr) of the first event
multiplied by the conditional probability of the second event, given that the first has
already occurred.

Example: If 60% of the students at a university are male, Pr (male) is 6/10. If 1/6 of the
male students have a B average, Pr (B average given male) is 1/6. Therefore, the
probability that any given student (male or female) selected at random, is both male and
has a B average is

Pr(male) × Pr(B| male) ¿ Pr(male ∩ B)

6/10 × 1/6 ¿ 1/10


Pr(male∩B) is 0.10; that is the probability that the student is male and has a B average
is 10%.

The probability that either one or both of two events will occur equals the sum of their
separate probabilities minus their joint probability.

Example; If two fair coins are thrown, the probability that at least one will come up
heads is Pr(coin#1 is heads) plus Pr(coin#2 is heads) minus Pr(coin #1 and #2 are both
heads), or

(0.5) + (0.5) – (0.5 × 0.5¿=0.75

Example: if in the earlier example 1/3 of all students, male or female, have a B
average [Pr(B average) is 1/3], the probability that any given student either is male or has
a B average is

Pr(male) + Pr(B avg.) – Pr(B∩ male) = Pr (male or has B avg.)

6/10 + 1/3 _ 1/10 = 25/30

The term Pr(B∩male ¿ must be substracted to avoid double counting those students who
belong to both groups.

The probabilities for all possible mutually exclusive outcomes of a single experiment
must add up to one.

Example: Flipping two coins (H= heads, T= tails)

If coin #1 is If coin #2 is Probability of this combination

H H 0.25

H T 0.25

T H 0.25

T T 0.25

Probability that one of the four possible combinations will occur 1.00 (certainty)
Probability Distributions
A probability distribution specifies the values of the variables and their respective
probabilities. Certain standard distributions seem to occur frequently in nature and have
proven useful in business.

Discrete distributions include the following:

(1) Uniform distribution. All outcomes are equally likely, such as the flipping of one
coin, or even of two coins, as in the example above.
(2) Binomial distribution. Each trial has only two possible outcomes, e.g., accept or
reject, heads or tails. This distribution shows the likelihood of each of the possible
combinations of trial results. It used in equally control.
(3) The Bernoulli distribution. Involves only one trial, whereas the binomial
distribution deals with as many as necessary. Thus, the binomial distribution
reduces to the Bernoulli distribution when is 1.
(4) The hypergeometric distribution. Is similar to the binomial distribution. It is used
for sampling without replacement.
(a) For infinite populations, sampling without replacement removes each item
sampled from the population, thus changing the composition of the population
from trial to trial.
(b) For large populations and small samples, the binomial distribution
(c) approximates the hypergeometric distribution and is computationally more
convenient.
(5) The Poisson distribution. Is useful when the event being studied may happen
more than once with random frequency during a given period.

Continious distributions include the following:

(1) Normal distribution. The most important and useful of all probability
distributions, it describes many physical phenomena. In sampling, it describes the
distribution of the sample mean regardless of the distribution of the population. It
has a symmetrical, bell-shaped curve centered about the mean. For the normal
distribution, about 68% of the area (or probability) lies within plus or minus 1
standard deviation of the mean, 95.5% lies within 2 standard deviations, and 99%
lies within 3 standard deviations of the mean.
(2) The exponential distribution. Is related to the poisson distribution. It is the
probability of zero occurrence in a time period T.
(3) The t-distribution (also known as student’s distribution) is a special distribution
used with small samples of the population, usually less than 30, with unknown
population variance.
(a) For large sample sizes (n>30) the t-distribution is almost identical to the standard
normal distribution
(b) For small sample sizes (n>30) for which only the sample standard deviation is
known, the t-distribution provides a reasonable estimate for tests of the population
mean if the population is normally distributed.
(c) The t-distribution is useful in business because large samples are often too
expensive. For a small sample, the t-statistu=ic (from a t-table) provides a better
estimate of the variance thsn thst from a table for the normal distribution.
(4) The chi-square distribution. Is used in testing the goodness of fit between actual
data and thereoretical distribution. In other words, it tests whether the sample is
likely to be from the population,based on a comparison of the sample variance
and the population variance.

Illustrative Problem 16.1 Decision making under uncertainty


Payong Corporation is considering two new colors for their umbrella products- Sky blue
and Baby pink. Either can be produced using the present facilities. Each the same selling
price of P100 and the same variable cost per unit of P80.

After studying past experience with similar products, management has prepared the
following probability distribution:

Event Probability For

(units demanded) sky blue umbrella baby pink umbrella

10,000 0.1 0.1

20,000 0.2 0.1

30,000 0.4 0.2

40,000 0.2 0.4

50,000 0.1 0.1

1.0 1.0

Management would like to know

a. The breakeven point for each product


b. Which product should be chosen, assuming the objective is to maximize expected
operating income?
Solution

a. Since both product has the same contributions margin per unit of P20 (P100-P80)
break-even point for each product will be the same coumputed as follows.

Break-even point = P400,000/P20

= 20,000 units

b. (1) Determine the expected demand for the two umbrellas:

Event Sky Blue Baby Pink


Demand Probability Units Probability Units

5,000 0.0 0 0.1 500


10,000 0.1 1,000 0.1 1,000
20,000 0.2 4,000 0.1 2,000
30,000 0.4 12,000 0.2 6,000
40,000 0.2 8,000 0.4 16,000
50,000 0.1 5,000 0.1 5,000
1.0 30,000 units 1.0 30,500 units

(2) Compute the expected operating income of the two umbrellas.

Sky Blue Baby Pink


Sales P3,000,000 P3,050,000
Variable Costs 2,400,000 2,440,000
Contribution Margin P 600,000 P 610,000
Fixed Costs 400,000 400,000
Operating Income P 200,000 P 210,000

The Baby Pink umbrellas should be chosen because they have the higher expected
income.
PAYOFF (DECISION) TABLES

Payoff (decision) tables are helpful tools for identifying the best solution given
several decision choices and future conditions that involve risk.
A payoff table presents the outcomes (payoffs) of specific decisions when certain
states of nature (events not within the control of the decision maker) occur.

Example: A dealer in luxury yachts may order 0, 1, or 2 yachts for this season's
inventory. The cost of carrying each excess yacht is P50,000, and the gain for
each yacht sold is P200,000. The situation may be described by a payoff table as
follows:

State of nature= Decision= Decision= Decision=


Seasons actual demand Order 0 Order 1 Order 2

0 yachts 0 P(50,000) P(100,000)


1 yacht 0 200,000 150,000
2 yachts 0 200,000 400,000

The probabilities of the season’s demand are

Pr Demand

0.10 0
0.50 1
0.40 2

The dealer may calculate the expected value of each decision as follows:

Order 0 Order 1 Order 2

0.1 × 0 0.1 × P(50,000) 0.1 × P(100,000)


0.5 × 0 + 0.5 × 200,000 + 0.5 × 150,000
0.4 × 0 + 0.4 ×200,000 + 0.4 ×400,000

EV (0)= EV (1)= P175,000 EV (2) =P225,000

The decision with the greatest value is to order tho yachts, so, in the absence of
additional information, the dealer should order two.
EXPECTED VALUE OF PERFECT INFORMATION

Even though the P210,000 expected value of operating income of the Baby Pink
umbrellas is higher than that of the Sky Blue umbrellas, management and/or owners may
resist exposure to the percentage involved in making a decision under risk. The
probabilities associated with which environmental conditions will actually occur are
based on existing information. The company may decide to hire marketing analysts to
obtain additional information on the environmental situation.

The expected value of perfect information is the amount the company is willing to pay
for the market analysts' errorless advice. Assuming that the market analyst could indicate
with certainty which condition would occur, a manager would decide with complete
certainty. Of course, "perfect information" is not perfect in the sense of absolute
predictions.

Perfect information is the knowledge that a future state of nature will occur with
certainty, i.e., being sure of what will occur in the future.

The expected value of perfect information (EVPI) is the difference between the expected
value without perfect information and the return if the best action is taken given perfect
information.

Example (from the yacht dealer problem on pages 674 to 675): If the yacht dealer were
able to poll all potential customers and they truthfully stated whether they would
purchase a yacht this year (i.e., if perfect information about this year's yacht sales could
be purchased), what is the greatest amount of money the dealer should pay for this
information? What is EVPI?

If the dealer had perfect knowledge of demand, he/she would make the best decision for
each state of nature. The cost of the other decisions is the conditional cost of making
other than the best choice. This cost may be calculated by subtracting the expected value
from the expected value given perfect information. This difference measures how much
better off the decision maker would be with perfect information. From the payoff table on
page 675, we find the expected value of the best choice under each state of nature.

Pr State of Nature Best Action Best Action Expected


Value

Payoff (Pr × Payoff)

0.1 Demand=0 Buy 0 P 0 P 0


0.5 Demand=1 Buy 1 200,000 100,000
0.4 Demand=2 Buy 2 400,000 160,000
P 260,000
The dealer expects to make P260,000 with perfect information about future
demand, and P225,000 if the choice with the best expected value is made. The
expected value of perfect information (EVPI) is then

Expected value with perfect information P260,000


Expected value of the best choice (225,000)
EVPI= P 35,000

The dealer will not pay more than P35,000 for information about future demand
because it would then be more profitable to make the expected value choice than
to pay more for information.

DECISION TREE

Underlying Concept

A decision tree is an analytical tool used in a problem in which a series of


decision has to be made at various time intervals, with each decision influenced
by the information that is available at the time it is made.

In its simplest form, a decision tree is a diagram that shows the several decisions
or acts and the possible consequences called events of each act. In a more
elaborate form, the probabilities and the revenue and costs of each event's
outcome are estimated and these are combined to give an expected value for the
event.

Decision trees provide a systematic framework for analyzing a sequence of


interrelated decisions the managers may make over time. Stemming from the
present investment decisions are alternative scenarios that depend on the
occurrence of future events and consequences of those events. Decision tree
analysis encourages the study and understanding of these scenarios.

Advantages of Decision Tree Analysis

Some benefits that may be derived from the use of Decision Tree Analysis are

1. Decision tree is an effective means of presenting the relevant information


needed by management in an investment problem. Such relevant information
includes choices, risks, monetary gains, and objectives.

2. Combination of action choices with different events or results of action that


chance or other uncontrollable circumstances partially affect can be better
presented and studied.
3. The interactions of the impact of future events, decision alternatives, uncertain
events and their possible payoffs can be shown with greater ease and clarity.
4. Data are presented in a manner that enables systematic analysis and better
decisions.

Limitations of Decision Tree Analysis

1. A decision tree does not give management the answers to an investment


problem.
2. It does not identify all the possible events or does it list all the decisions that
must be made on a subject under analysis.
3. The interactions of such decision with the objective of other parts of the
business organization would be too complicated to compute manually. The use of
computers will be suitable when studying the effect of variations in figures and/or
the events involved.
4. Decision tree analysis treats uncertain alternatives as if they were discrete well-
defined possibilities. It should be remembered that uncertain situations depend not
only on one variable but on several independent or partially related variables
subject to such chance influences.

Steps in Making a Decision Tree

The requirements for the decision tree preparation are


1. Identification of the points and decision and the alternatives available at each
point.
2. Determination of the points of uncertainty and the type or range of alternative
outcomes at each point.
3. Estimates of the probabilities of different events or results of actions.
4. Estimates of the costs and gains of various events and actions.
5. Analysis of the alternative values in choosing a course of action.

Because the time between successive decision stages on a decision tree may be long, it
would be more realistic to consider the time value of future earnings and other cash
flows. In others the discounted cash flow approach may be applied on analyzing various
investment alternatives. This is discussed exclusively in Chapter 12.
Illustrative Problem 12-2: Preparation of Decision Tree

After several years of supplying gelatin bars to several supermarket chains, the Castillo
family decided it was time to shift to another venture in the light of increasing
competition from other gelatin manufacturers. They would want to introduce a type of
gelatin ice cream.

Based on the family accountant's estimates, if the sales are high, the total contribution
margin will be P300,000. If sales are low, the total contribution margin will be P50,000.
Fixed costs will be P150,000. The accountant attaches a probability of 0.5 for high sales
and 0.5 for low sales.

The Castillo family can conduct a survey of various health food outlets to determine the
true demand for the new product. The reliability of the survey is such that it will signal
high sales 70 percent of the times when actual sales will be high, and signal low sales 90
percent of the time when actual sales will be low. The costs of such a survey are P20,000.

Assuming that the Castillo family bases its decisions on expected value:

a) What action will they take without the survey?

b) Should the Castillo family take the survey? What should their decision

be?

c) How much will they be willing to pay for perfect information?

Solution:

High sales P300,000- P150,000

Pr=0.5

a. produce

EV= -P25,000
Low sales P50,000- P150,000

EV =P0
Do not produce P0

Decision: accept project


B. Let

H= actual high-sales
L= actual low-sales
Y Η=¿ survey signal for high state
Y Ι =¿ survey signal for low state
Pr= Probability

Pr Y Η = 0.1 (0.5) + 0.7 (0.5)


= 0.4

Pr Y Ι = 0.9 (0.5) + 0.3 (0.5)


= 0.6

Pr (H/ Y Η ) = 0.7 (0.5)


0.4

= 0.875

Pr (L/ Y Η ) = 1- 0.875
= 0.125

Pr (H/Y Ι ) = 0.3 (0.5)

= 0.6

= 0.25

Pr (L/ Y Ι ) = 1-0.25

= 0.75
LEARNING CURVE
Learning curves reflect the increased rate at which people perform tasks as they gain experience.
The time required to perform a given task becomes progressively shorter, but this technique is
only applicable to the early stages of production or of any new task.
Ordinarily, the curve is expressed as a percentage of reduced time to complete a task for each
doubling of cumulative production. Research has shown learning curve percentages to be
between 60% and 80%. In other words, the time required is reduced by 20% to 40% each time
cumulative production is doubled, with 20% being common.
One common assumption made in a learning curve model is that the cumulative average time per
unit is reduced by a certain percentage each time production doubles.
Example given an 80% learning curve model based on the first assumption stated above, the
following performance is expected during the early stages of the manufacture of a new product.

Cumulative Cumulative
Number of tasks average time per unit
100 3.0
200 2.4 (3.0 × 80 % ¿
400 1.92 (2.4× 80 % ¿
800 1.536 (1.92× 80 % ¿
1,500 1. 228 (1.536× 80 % ¿
Graphical presentation

Productivity

Experience
If the average time for 100 units in the example above were 3 minutes per unit, the total time
would be 300 minutes. At an average time of 2.4 minutes for 200 units, the total time would be
480 minutes. In other words, the additional 100 units required only 180 minutes (480-300), or
1.8 minutes per unit.
SIMULATION TECHNIQUES
Simulation is a technique for experimenting with logical and mathematical models using a
computer. Despite the power of mathematics, many problems cannot be solved by known
analytical methods because of the behavior of the variables and the complexity of their
interactions, e.g.,
a. Corporate planning models
b. Financial planning models
c. New product marketing models
d. Queuing system simulations
e. Inventory control simulation

Experimentation is neither new nor uncommon in business. Building a mockup of a new


automobile, having one department try out new accounting procedures, and test-marketing a new
product are all forms of experimentation. In effect, experimentation is organized trial and error
using a model of the real world to
obtain information prior to full implementation.
Models can be classified as either physical or abstract.
(a) Physical models include automobile mockups, airplane models used for wind-tunnel tests,
and breadboard models of electronic circuits.
(b) Abstract models may be pictorial (architectural plans), verbal (a proposed procedure), or
logical-mathematical. Experimentation with logical-
mathematical models can involve many time-consuming calculations. Computers have
eliminated much of this costly drudgery and have led to the growing interest in simulation for
management.
The simulation procedure has five steps:
a. Define the objectives. The objectives serve as guidelines for all that follows. The objectives
may be to aid in the understanding of an existing system (e.g., an inventory system with rising
costs) or to explore ne alternatives (e.g., the effect of investments on the firm's financial
structure). A third type of objective is estimating the behavior of some new system such as a
production line. Thus, a simulation can be designed to ask "what-if" questions, such as whether
modifying the actual system will result in better performance.
b. Formulate the model. The variables to be included, their individual behavior, and their
interrelationships must be defined in precise logical- mathematical terms. The objectives of the
simulation serve as guidelines in deciding which factors are relevant. Moreover, inputs reflected
in the model are of two kinds: controllable and probabilistic. The former are those subject to the
decision-makers' influence, and the latter involve circumstances beyond their control, such as
general economic conditionsor the acts of competitors.
c. Validate the model. Some assurance is needed that the results of the experiment will be
realistic. This assurance requires validation of the model - often using historical data. If the
model gives results equivalent to what actually happened, the model is historically valid. Some
risk remains, however, that changes could make the model invalid for the future.
d. Design the experiment. Experimentation is sampling the operation of a
system. For example, if a particular policy is simulated on an inventory model for two years, the
results are a single sample. With replication, the sample size can be increased and the confidence
level raised. The number of runs to be made, length of each run, measurements to be made, and
methods for analyzing the results are all part of the design of the experiment. The experiments
also may take the form of asking "what-if questions, that is, varying an input or assumption to
ascertain the effect on the results.
e. Conduct the simulation - evaluation results. The simulation should be conducted with care.
The results are analyzed using appropriate statistical methods. These results constitute outcomes
that permit evaluation of the probabilities of real-world performance.

Advantages and Limitations of Simulation


The advantages of simulation are as follows:
a. Time can be compressed. A corporate planning model can show the results of a policy for 5
years into the future, using only minutes of computer time.
b. Alternative policies can be explored. With simulations, managers can ask what-if questions to
explore possible policies, providing management with a powerful new planning tool.
c. Complex systems can be analyzed. In many cases, simulation is the only possible quantitative
method for analyzing a complex system such as a production or inventory system, or the entire
firm.
The limitations of simulation are as follows:
a. Cost. Simulation models can be costly to develop. They can be justified only if the
information to be obtained is worth more than the costs to arlova develop the model and carry
out the experiment.
b. Risk of error. A simulation results in a prediction of how an actual system would behave. As
in forecasting, the prediction may be in error.

MONTE CARLO TECHNIQUE


Quantitative Techniques for Decision Making
The Monte Carlo technique is often used in simulation to generate the individual values for a
random variable. A random number generator is used to produce numbers with a uniform
probability distribution (equal likelihoods of occurrence). The second step is to transform these
numbers into values consistent with the
desired distribution.
The performance of a quantitative model may be investigated by randomly selecting values for
each of the variables in the model (based on the probability distribution of each variable) and
then calculating the value of the solution. If this s is performed a large number of times, the
distribution of results from the
Process model will be obtained.
Example: A new marketing model includes a factor for a competitor's introduction of a similar
product within 1 year. Management estimates a 50% chance that this event will happen. For each
simulation, this factor must be determined, perhaps by flipping a coin, or by putting two numbers
in a hat and selecting one number. Random numbers between 0 and 1 could be generated.
Numbers under 0.5 would signify introduction of a similar product; numbers over 0.5 would
indicate the nonoccurrence of this event.
SENSITIVITY ANALYSIS
Sensitivity analysis describes how sensitive the linear programming optimal solution is to a
change in any one number. Sensitivity analysis answers what-if questions about the effect of
change in prices or variable costs; changes in value; addition or deletion of constraints, such as
available machine hours; and changes in industrial coefficients, such as the labor-hours required
in manufacturing in a specific unit.
After a problem has been formulated into any mathematical model, it may be subjected to
sensitivity analysis. This approach is especially useful and significant when probabilities of states
of nature and decision payoffs are derived subjectively rather than by using objectively
quantifiable information.
A trial-and-error method may be adopted in which the sensitivity of the solution to changes in
any given variable, parameter, or other assumption is calculated. (1) The risk of the project being
simulated may also be estimated. (2) The best project may be one that is least sensitive to
changes in probabilistic (uncertain) inputs.
(3) A sensitivity analysis may indicate whether expending additional resources to obtain better
forecasts of future conditions is cost justified. In linear programming problems, sensitivity is the
range within which a constraint changing the optimal solution. Shadow price is the synonym for
sensitivity in that value, such as a cost efficient or any other variable, may be changed without
context.
In the application of discounted cash flow methods (e.g., net present value), a sensitivity analysis
might be performed to ascertain the effects of variability of the discount rate or periodic cash
flows.
Financial planning models, including those for cash flows and capital budgeting, are other
significant applications of sensitivity analysis. For example, changes in selling prices or resource
costs may affect available cash and require more or less short-term borrowing.
Still another application is the calculation of the margin of safety in a CVP analysis.
QUEUING
Queuing is often referred to as Waiting line analysis. Queuing can occur in many situations and
in a wide variety of forms. However, every queuing problem involves four basic issues.
1) Input mechanisms
2) Line or queue discipline
3) Service facilities, and
4) Output
The characteristics of these four items describe the queuing model of concern.
With adequate data on the population to be serviced, the queuing and service characteristics,
analytic solution of the queuing problem may be feasible. Many queuing problems are so
complex that simulations must be used to "solve" them. The mathematical deviation of systems
is moderately complex even for the simpler queuing problems and is beyond the scope of this
chapter.
LINEAR PROGRAMMING
Nature and Applications provide
Linear programming is a mathematical technique that permits the determination optimum use of
the available resources, namely, money, personnel, materials, facilities and time. It is a valuable
aid to management because it a systematic and efficient procedure which can be used as a guide
in decision making. Managers are often faced with problems of selecting the most or least costly
way to use available resources. The advantage of linear sensitivity analysis. A major
disadvantage is the restrictiveness of its linear programming is its applicability to many types of
problems and its usefulness for assumptions, for example, that all costs are variable or fixed.
Example: A manufacturer should minimize production costs while satisfying production
requirements, maintaining required inventory levels, staying within production capacities, and
using available employees. The objective function is the production cost to be minimized; the
constraints are production requirements, inventory levels, production capacity, and available
employees.
This technique may be used to maximize revenue, contribution margin or profit function, or to
maximize a cost function subject to constraints. Specific applications include:
1) Determination of the product mix to maximize contribution margin.
2) Selection of an investment mix.
3) Determination of materials mix to minimize costs.
4) Assignment of jobs to machines.
5) Determination of transportation routes.
Steps in the Formulation of Linear Program
1. The first step is to define the decision variables.
2. The next step is to express the objective function and constraints in terms
of these decision variables.

Computational Methods of Linear Programming


To solve Linear Programming problems, several solution methods are available
and they include:
1) Graphic Method
2) Algebraic Method
3) Simplex Method
The graphical method, although the easiest technique, is limited to problems with
two variables.
The algebraic method is a trial-and-error technique. Pairs of constraints are solved algebraically
to find their intersection. The values of the decision variables are then substituted into the
objective function and compared to find the best combination.
(1) The basic rule is that the optimal solution will be at the intersection of two or more constraint
equations.
(2) Thus, all intersections can be computed and each solution evaluated in the objective function
to determine which solution is optimal.

The simplex method is the technique most commonly used to solve linear programming
problems. It is an algorithm to move from one corner solution to a better corner solution. When a
better solution cannot be found, the optimal solution has been reached.
(1) The simplex method relies on an area of mathematics called matrix algebra. The equations
that form the constraints are arranged in a matrix of coefficients and manipulated as a group with
matrix algebra.
(2) Almost all practical applications of linear programming require the use of computers. Most
computer facilities have a linear programming package that uses the simplex algorithm to find
the optimal solution.
Graphic Method
When a linear programming problem involves only two variables, a two- dimensional graph can
be used to determine the optimal solution.

The steps are:


1) Determine the objective function.
2) Determine basic relationships
3) Compute the optimum solution in the situation, especially the constraints
a) Change inequalities to equalities. b) Graph the equalities.
c) Identify the correct side for the original inequalities.
d) Identify the area of feasible solution. Feasible solutions are values of decision variables that
satisfy all the constraints simultaneously.
e) Determine the contribution margin at all the corners in the feasible region.

Illustrative Problem 12-1: Linear Programming - Graphic Method


applied to Product Mix Problem

A firm produces two products, A and B. Product A requires two hours of grinding and four hours
of polishing. Product B requires five hours of grinding and two hours of polishing. The
manufacturer has 3 grinders and 2 polishers; therefore in a 40-hour week there are 120 hours of
grinding capacity and 80 hours of polishing capacity.
There is a ready market for both products and the contribution margin per unit of Product A and
Product B are P3 and P4, respectively.
To maximize the total contribution margin, the management must decide on
1) the allocation of the available production capacity to Products A and B,
and org to an
2) the number of units of each product to produce

Solution
a.) Definition of decision variables
LET A= the number of units of product A to be produced
B= the number of units of product B to be produced

b.) Objective function


TOTAL CM= P3A + P4B

c.) Explicit contraints


2A + 5B ≤ 120
4A + 2B ≤ 80

Implicit constraint:
A, B ≤ 0

The LP model is therefore


Maximize: total CM = P3A + P4B
Subject to:
2A + 5B ≤ 120
4A + 2B ≤ 80
A, B ≤ 0

d.) Prepare graph with A axis representing the number of units of product A to be produced
and B axis respresenting the number of units of product B to be produced

e.) Plot all the constraints on the graph by connecting the points that represent the extremes
of production of each product. These points are;

Grinding:
If A=0; B=24
B= 0; A=60
Polishing:
If A=0; B=40
B=0; A=20
f.) Evaluate the corner points in the feasible region in terms of their contribution margin:

Corner product mix (units) contribution margin


points (P3A+P4B)
A B
A 20 0 3 (20) + 4 (0) = 60
B 10 20 3 (10) + 4 (20)= 110
C 0 24 3 (0) + 4(20)=96
D 0 0 3 (0) + 4(0)= 0

ANSWER

Produce 10 units of A and 20 units of B. The total contribution margin associated with
this combination is P110 which is the highest among the four combination.
Limitations of PERT
1. Reliable cost data are difficult to obtain, particularly for jobs that are done infrequently or
projects that are being undertaken for the first time.
2. Inclinations of supervisors to overstate their needs and cost requirements
3. Time estimates are more likely to be overly conservative - motivated in part by the realization
that when ones' performance is being rated, it is are inflated in order to maintain budgets. better
to have slack than to miss a deadline.
GANTT CHART
A Gantt chart is a bar chart with time shown on the horizontal axis and the duration of the task
represented as a bar running from the starting date to the ending date. Gantt charts are considered
very useful tools in planning projects and recording progress toward goal. The chart also shows
(1) how expected performance of a specific task compares with actual performance, (2) which
tasks should be in progress on a specific date, and (3) how close to completion a task should be
given on a given date. A Gantt chart is also a control technique because it allows the comparison
of actual production with scheduled production to identify variations and initiate corrective
action.
Steps in Preparing a Gantt Chart
The preparation of a Gantt chart involves the following steps: say
1. Identify and determine the sequences of activities.
2. Schedule the work by periods.
3. Prepare the chart using the Y-axis to represent the tasks or activities workers must perform
while the X-axis represents the time available for work.
4. Plot the lines representing the work to be done.
a) Horizontal broken line is used to represent work scheduled by periods within the departments
b) Use short vertical broken lines to represent work unfinished ofcarried over from previous
periods.
c) The large shaded bar line is a summation of the individual horizontal broken lines and
represents cumulative work to be performed.
REVIEW QUESTIONS AND PROBLEMS
1. a. What is the purpose of decision tree analysis?
b. List three advantages and three weaknesses of decision tree analysis. 2. Discuss some of the
factors that must be considered in decision making
that decision tree analysis does not incorporate.
3. Identify two ways that PERT is superior to Gantt chart techniques for
complex projects.
4. Discuss how PERT can be used in planning a complex project.
5. Should management analyze time slippage on all activities?
6. Define the term critical path and explain why it is considered critical.
7. What is meant by crashing the network and what is differential crash cost of an activity?
8. What is slack and how can management utilize it?
9. If we want to maximize profit, why do we use unit contribution margins in our analysis
instead of unit gross margin?
10. Management notes that the contribution margin from one product is greater than the
contribution from a second product. Hence, they conclude that the company should concentrate
on production of the first product. Under what, if any, conditions, will this approach result in
maximum profits?
11. What do we mean by the opportunity cost of a constraint?
12. What is the feasible production region?
13. What is the role of the accountant in the management decision process that I uses linear
programming models (or other mathematical programming
techniques)?
14. For each of the following costs, indicate whether the cost would be an out- of-pocket
carrying cost (C), or a cost of placing an order (P). If the item does not qualify for either of these
categories, note that it as none of the above (N). Assume that wages vary with the level of work
while salaries are fixed for a monthly or longer time period.
a. Hourly fee for inventory audit.
b. Salary of purchasing supervisor.
c. Costs to audit purchase orders and invoices, on a per-order basis.
d. Taxes on inventory.
e. Stockout costs.
f. Storage costs charged per unit in inventory.
g. Fire insurance on inventory.
h. Fire insurance on warehouse. i. Obsolescence costs on inventory.
j. Shipping costs per shipment.
15. Since the operations research specialists develop and maintain inventory models, why does
the accountant become concerned with inventory policy decisions?
16. Why is the cost of capital included as a carrying cost of inventory?
17. A review of the inventories of a company indicates the following cost data for a given item:
Invoice price P102.25
Processing invoice and other documents P 21.45 per order +
P147.80 per month
Permit fees for shipping P201.65 per truckload
Excise tax 4% of invoice price
Inventory tax 2% of the invoice price
Insurance on shipments P 1.50 per unit
Insurance on inventory P 2.80 per unit
Warehouse rental P 985.00 per month
Stockout costs P 122.00 per order
Cost of capital 25%
Unloading per order P 80.20 per order
Show differential costs that would be included in an EOQ model

1. The probability of a 3% increase in the cost of both Material 1 and Material 2 is


a.15%. c. 80%
b. 40%. d 20%
(CIA, Adapted)
Questions 3 and 4 are based on the following information
A computer store sells four computer models designated as N201, 1201, C201, and 0201. The
store manager has made random number assignments to represent customer choices based on
past sales data. The assignments are shown below.
Model Random Numbers
N201 0-1
I201 2-6
C201 7-8
O201 9
2. The probability that a customer will select model N201 is.
a.10%.
b. 20%.
C. 50%
D. Some percentage other than those given. (CMA, Adapted)
4.oll In running a simulation of the computer demand, the following numbers are drawn in
sequence: 2, 8, and 6. The simulation indicates that the buot third customer will purchase
a. Model N2010
b. Model 1201.
C. Model C201.
D’ Model 0201.(CMA, Adapted)
5. Rico Buenavista is contemplating whether to investigate a labor inefficiency variance in the
Assembly Department. It will cost P6,000 to undertake the investigation and another P18,000 to
correct operations if the department is found to be operating improperly. If the department is
operating improperly and Buenavista fails to investigate, operating tot costs from the various
inefficiencies are expected to amount to P33,000. Buenavista will be indifferent between
investigating and not investigating the variance if the probability of improper operation is
a. 0.29.
b. 0.40.
c.0.60.
d. 0.71.
(CMA, Adapted)
18. Fil Enterprises, distributor of compact disks (CDs), is developing its range of sales estimates
and associated probabilities for the year: budgeted cost of goods sold for 2013. Fil has developed
the following
Sales Estimate
P 60,000 85,000 100,000
Probability
25%
40
35
Fil's cost of goods sold averages 80% of sales. What is the expected value of Fil's 2013 budgeted
cost of goods sold?
a.P85,000
B P84,000
C. P68,000
d. P67,20 (CMA, Adapted)
19. The modeling technique to be used for situations involving a sequence of events with several
possible outcomes associated with each event is
a.queuing theory. c.the critical path method.
b. dynamic programming. d. decision tree analysis. (CMA, Adapted)

20. Which of the following statements does not apply to decision tree analysis?
a.The sum of the probabilities of the events is less than one.
b.All of the events are mutually exclusive.
C.All of the events are included in the decision.
d. The branches emanate from a node from left to right. (CMA, Adapted)

21. The modeling technique to be employed in a situation involving a sequence of events with
several possible outcomes associated with each event is
a. network analysis. C. Monte Carlo simulation.
b. decision tree analysis. d. linear programming. (CMA, Adapted)
Questions 22 through 24 are based on the following information:
A beverage stand can sell either soft drinks or coffee on any given day. If the stand sells soft
drinks and the weather is hot, it will make P2,500; if the weather is cold, the profit will be
P1,000. If the stand sells coffee and the weather is hot, it will make P1,900; if the weather is
cold, the profit will be P2,000. The probability of cold weather on a given day at this time is
60%.
22. The expected payoff for selling coffee is
a. P1,360.
b. P2,200.
c. P3,900.
d. P1,960. (CMA, Adapted)
23. The expected payoff if the vendor has perfect information is
a.3,900
b. P2,200.
c. P1,360.
d. P1,960. (CMA, Adapted)
24. Considering only the information given in the fact pattern, if the probability of hot weather
given a hot weather forecast is 50%, how much would the vendor be willing to pay for the
forecast?
a.P600
b. P300
c. P1,000
d. P500
25. The expected value of perfect information is the
a. same as the expected profit under certainy
b. sum of the conditional profit (loss) for the best event of each act times the probability of each
event occurring.
c. difference between the expected profit under certainty and the expected opportunity loss.
d. difference between the expected profit under certainty and the expected monetary value of the
best act under certainty. (CMA, Adapted)
26. One limitation of the linear programming technique is that it is effective only for
a.two-product situations.
b. manufacturing resource constraint situations.
C.straight-line relationship situations.
d.income maximization situations. (CMA, Adapted)

27. In order to solve a linear programming problem, slack, surplus, and artificial variables must
be employed. A slack variable represents
a.opportunity costs.
b. unused capacity.
C outside variables with high cost.
d.the variable with the most negative value. (CMA, Adapted)
28. The procedure employed to solve linear programming problem is
A differential calculus
b.integral calculus.
C Simulation
D .matrix algebra. (CMA, Adapted)

46. A Gantt chart


ashows the critical path for a project.
b. is used for determining an optimal product mix..
c. shows only the activities along the critical path of a network.
D does not necessarily show the critical path through a network. (CMA, Adapted)

47. Lucky Corporation is preparing a proposal for the government to produce a steam generator
to be used in nuclear submarines. Lucky has three factories that can initiate and complete the
generator; however, completion times vary due to different technologies and older equipment
within each of the three factories. The decision tool that would best assist Lucky in its factory
selection is.
A queuing theory.
B time series analysis.
C PERT-cost analysis.
d. linear programming. (CMA, Adapted)

48. When using PERT (Program Evaluation Review Technique), the expected time for an activity
when given an optimistic time (A), a pessimistic time (B), and a most likely time (m) is
calculated by which one of the following formulas?
a.(b-a)÷2
b. (a+b)÷2
C.(a + 4m + b) ÷ 6÷
D (4abm) 6 (CMA, Adapted)

49. Luzon Building Corporation uses the critical path method to monitor construction jobs. The
company is currently 2 weeks behind schedule on Job #123, which is subject to a P10,500-per-
week completion penalty. Path A-B-C-F-G-H-I has a normal completion time of 20 weeks, and
critical path A-D-E-F-G-H-I has a normal completion time of 22 weeks. The following activities
can be crashed:
Luzon Building desires to reduce the normal completion time of Job #123 and, at the same time,
report the highest possible income for the
year. Luzon Building should crash
a.Activity BC 1 week and activity EF 1 week.
b. Activity BC 2 weeks.
C.Activity EF 2 weeks.
d.Activity DE 1 week and activity EF 1 week.
(CMA, Adapted)
Questions 50 through 52 are based on the following information.
The following are the times required for various sequences of activities (paths through the
network) shown in a PERT diagram:
PATH TIME IN DAYS
A-B-C 27
A-D-E-C 33
A-F-C 34
A-F-G-C 36

50. Which is the critical path?


a.A-B-C
b. A-D-E-C
c A-F-C
d A-F-G-C
51.How much slack time is there in path A-D-E-C?
a. 1 day.
b. 3 days.
C.6 days.
d. 33 days.
52. Path A-B-C is expected to require 27 days to complete. This estimate was calculated using
the PERT formula based on a most likely completion time of 25 days and a most optimistic
completion time of 24 days. What estimate was used for the most pessimistic completion time?
a. 25 days. c 32 days.
b. 27 days d. 38 days.
53. The critical path through a network is the
a.shortest path.
b.path with the most nodes.
c. longest path to reach.
d. path with the most slack. (CMA, Adapted)
Luzon Building desires to reduce the normal completion time of Job #123 and, at the same time,
report the highest possible income for the year
.Luzon Building should crash
a. Activity BC 1 week and activity EF 1 week.
b. Activity BC 2 weeks.
C.Activity EF 2 weeks.
d.Activity DE 1 week and activity EF 1 week.
(CMA, Adapted)
58 If an 80% learning curve is applicable, Grosse Point's total cost on this order would be
estimated at
a.P26,400.
b. P32,000.
c.P38,000.
d. P41,800. (CMA, Adapted)
59. A particular manufacturing job is subject to an estimated 90% learning curve. The first unit
required 50 labor hours to complete. What is the cumulative average time per unit after four units
are completed?
a.50.0 hours
c.40.5 hours
d.40.0 hours
b. 45.0 hours
60. Grosse Point Manufacturing recently completed and sold an order of 50 units that had costs
as follows:
If Grosse Point had experienced a 70% learning curve, the bid for the 150 units would

a.show a 30% reduction in the total direct labor hours required with no learning curve.
b. include increased fixed overhead costs.
C.be 10% lower than the total bid at an 80% learning curve.
d. include 6.40 direct labor hours per unit at P8.50 per hour. (CMA, Adapted)
61. A widely used approach that managers use to recognize uncertainty about individual items
and to obtain an immediate financial estimate of the consequences of possible prediction errors is
a. expected value analysis.
b. learning curve analysis.
c. sensitivity analysis.
d. regression analysis.
(CMA, Adapted)

62. When simulating with the Monte Carlo technique, the average simulated demand over the
long run should approximate the
a. actual demand.
B, real demand
C.sampled demand.
d.expected demand.
(CMA, Adapted)

63. Through the use of decision models, managers thoroughly analyze many alternatives and
decide on the best alternative for the company. Often the actual results achieved from a particular
decision are not what was expected when the decision was made. In addition, an alternative that
was not selected would have actually been the best decision for the company. The appropriate
technique to analyze the alternatives by using expected inputs and altering them before a
decision is made is
a expected value analysis.
b. linear programming.
C. Program Evaluation Review Technique (PERT).
d. sensitivity analysis. (CMA, Adapted)
CHAPTER

13 CAPITAL INVESTMENT

DECISION

EXPECTED LEARNING OUTCOMES


After studying this chapter, you should be able to...
1. Define capital budgeting
2. State the characteristics of a capital investment decision
3. Describe the categories ad capital investment
4. State and explain the elements of capital budgeting
5. Compute initial investment, annual net cash returns/savings of an investment proposal
Discuss the process of capital budgeting
Enumerate and explain the categories of project cash flows
8. Apply the techniques in evaluating capital investment projects such as payback period.
accounting rate of return and discounted cash flow analysis (net present value, internal rate of
return, profitability index)
9. Explain the process of ranking investment projects
10. Explain the effect of inflation in capital investment analysis
Capital budgeting is the process of deciding whether or not to commit resources to projects
whose costs and benefits are spread over several time periods. It
involves:
1) the preparation of annual budget for capital investment
2) the assessment of funding capacities and
3) the allocation of resources to renewal and expansion projects which most clearly conform with
the company's priorities.
Capital budgeting is used to describe actions relating to the planning and financing capital
outlays for such purposes as the purchase of new machinery, the modernization of plant facilities
or the introduction of new product lines. Capital budgeting is an investment concept, since it
involves a commitment of funds now in order to receive some desired return in the future in the
form of additional cash
inflows or reduced cash outflows.
Characteristics of a Capital Investment Decision
benefits and budgeting for them is one of the most important areas of managerial Capital
expenditures are long-term commitments of resources to realize future decision. They deserve
penetrating analysis and attention of top management
because of the following reasons:
1) Substantial amount of funds are required in capital projects.
2) Because of the length of time spanned by a capital investmentdecision, the element of
uncertainty becomes more critical.
3) The effect of managerial errors will be difficult to reverse.
4) Plans must be made well into an uncertain future.
5) Success or failure of the company may depend upon a single or relatively few investment
decisions.
the identification of costs relevant to the problem. Because the alternatives lie in One of the most
difficult steps involved in the decision-making process relates to the future, the only costs which
are relevant are future costs. Historical costs arising from past decisions are sunk costs and so
cannot affect future alternatives. Such considerations as monetary advantage of an alternative, its
effect on employee relations, company image and relations with other companies are usually
evaluated in choosing from among the alternatives.
Categories of Capital Investments
The two general types of capital investment decisions are:
A. Independent capital investment projects or Screening decisions
- these are projects which are evaluated individually and reviewed against predetermined
corporate standards of acceptability resulting in an "accept" or "reject" decision.
Examples are:
1. Investment in long-term assets such as property, plant and equipment.
2. New product development.
3. Undertaking a large scale advertising campaign.
4. Introduction of a computer.
5. Corporate acquisitions (such as purchase of shares in subsidiaries or affiliates).
B. Mutually exclusive capital investment projects or Preference decisions
these are projects which require the company to choose from among specific alternatives. The
project to be acceptable must pass the criteria of acceptability set by the company and be better
than the other investment alternatives. Examples are:
1. Replacement against renovation of equipment or facilities.
2. Rent or lease against ownership of facilities.
3. Manual bookkeeping system against computerized system.
4. Preventive maintenance against periodic overhaul of machineries.
5. Purchase of machinery from an outside supplier against assembly of the machinery by the
company's own staff.

Elements of Capital Budgeting


The elements or factors to be considered in evaluating capital investment proposals are:
1. The net amount of the investment.
2. The minimum acceptable rate of return on the investment/
3. The operating cash flows or returns from the investment.
4.Cost of capital.
5.Net Initial Investment or Project Cost
6.Net investment represents the initial cash outlay that is required to obtain future returns or the
net cash outflow to support a capital investment project.

In certain cases, the net investment is the sacrifice of an inflow of cash, that is, the opportunity
cost that arises when a benefit is rejected. An example is when a company has in its possession
fixed assets no longer used in operation and are awaiting disposal through sale. If it should
happen that these assets can be put to good use on a proposed capital project rather than be
disposed of, then the estimated project cost or investment should include the net amount to be
realizedfrom the sale of the assets.
Illustrative Problem 13-1: Determination of Net Initial Investment
The management of Maingat Company plans to replace a sorting machine that was acquired
several years ago at a cost of P60,000. The machine has been depreciated
to its residual value of P10,000.
A new sorter can be purchased for P96,000. The dealer will grant a trade-in allowance of
P16,000 on the old machine. If a new machine is not purchased,

Maingat Company will spend on trade-in transactions are n machine can be deducted in 40% of
the income subj 20,000
REQUIRED: Compute th n Solution: Maingat Compa
Purchase price of new sorte Add Additional working C Total Less: Trade-in allowance Avoidable
repairs (net of increase)
Net investment Net Cash Returns
The cash returns are the cash cost that can be dire
Annual incremental re Less: Incremental ca Annual cash inflow be Less: Taxes
[Tax rate (An Annual net cash inflcw)]
Annual incremental Less: Incremental Annual cash inflow Less: Incremental Net income before
Less: Income taxe Net income after tax Add: Incremental Annual net cash in
Maingat Company on trade-in transactions P50,000. will spend P10,000 to repair the old
machine. Gains and losses are not subject to income taxes. The cost to repair the old machine can
be deducted in computing income taxes. Income taxes are estimated at 40% of the income
subject to tax. Additional working capital required is REQUIRED: Compute the net initial
investment in this project.

Solution: Maingat Company


Purchase price of new sorter P 96,000
Add: Additional working capital Total P50,000
Less: Trade-in allowance on old P146,000
Avoidable repairs cost on old sorter P16,000
(net of increase in income taxes of P4,000) P 6,000
Net investment P 22,000
Net Cash Returns P124.000
Alalay Company is considering the acquisition of a machine which will cost P120,000. It has an
expected useful life of five years at the end of which its scrap value will be P20,000. The
company expects to be able to generate annual cash flow before taxes of P40,000. Estimated
income tax rate is 30%. What is the
annual cash flow after taxes on this investment?

Solution: Alalay Company

Annual cashflow before taxes P40,000


Less: Depreciation 5 years 20,000
Net Income before taxes P20,000
Less: Income taxes (30%) 6,000
Net Income after taxes P14,000
Add: Depreciation 20,000
Annual cashflow after taxes P34,000

The minimum or low the average rate of re opportunities or the c must pay to attract in be
computed as folla 1. Cost of Debt: Interest r 2. Cost of Prefe
Market

Capital Investment Decisions 419


Illustrative Problem 13-3: Determination of Annual Cash Savings
The Visayan Division of Marlow Supply Company has been considering a new production
method that can reduce materials costs by an estimated amount of P52,000 a year. The new
method is also expected to result in an annual savings years. Income taxes are estimated at 30%
of income before income taxes. What of labor and overhead methods is estimated at P40,000 a
year over a period of ten are the annual net returns (or savings) expected from the new
production method?
Solution: Marlow Supply Company,Annual savings and direct materials costs
Annual savings in direct labor and overhead costs.
Total savings before depreciation
Less: Depreciation Net increase in income
Savings after depreciation Add: Depreciation
Less: Incremental income taxes (30%) Net cash returns (savings)
Minimum or Lowest Acceptable Rate of 40,000
P 56,000 16,800
P 52,000 P 39,200
64,000 40,000
P116,000
The minimum or lowest acceptable rate of return or opportunity cost may equal the average rate
of return that the company will earn from alternative investment opportunities or the cost of
capital which is the average rate of return that the firm must pay to attract investment fund. The
cost of capital according to source may be computed as follows:
1. Cost of Debt: Expected Cash
Interest rate (1 - Corporate Tax Rate) Dividends per share
2. Cost of Preference Shares: Current price per share of
Dividends per share Ordinary Shares
Market value per share of Preference Shares b. Book-value based*
3. Cost of Ordinary Shares: Dividend growth rate
Stock price-based COMPUTE

4. Cost of Retained Earnings - same as cost of ordinary equity


* This is used when dividend growth rate is not known.
To compute for the overall or weighted average cost of capital, multiply the cost of each type of
capital by their respective weights (percentage of each source to the firm's total capital structure)
and add up the individual weighted cost of capital.
Illustrative Problem 13-4: Computation of Weighted Average Cost of Capital
The following information on Bettina Corporation's capital structure is available
from the latest financial statement:
PROCESS OF CAPITAL BUDGETING
The capital budgeting can be divided into seven significant phases:
1. Finding Investment Opportunities
Many capital expenditures proposals can be identified during the
strategic
or long-term planning process. Since the long-term profitability of most companies depends on
the nature and quality of their capital investments, these investment opportunities should be
carefully analyzed and
evaluated.
2. Collect Relevant Information about Opportunities To effectively evaluate any investment
opportunity, the expected cash flows from the project must be estimated and the total cash outlay
necessary to place the investment in operative form must be determined. A plan for
implementing the opportunity must be developed and other nonfinancial information must be
assembled.
3. Select Discount Rate Before the cash flow can be evaluated, the discounted (cost of capital)
(These techniques will be discussed later). must be established if the discounted cash flow
approach is to be applied
4. Financial Analysis of Cash Flows developed in the second phase. The techniques of capital
budgeting are applied to the estimated cash flows
5. Decision Many factors, quantitative as well as qualitative, should be given consideration
before the final decision is made as to the selection of a particular investment. They will include
among others, relationship of this opportunity to other aspects of the company operations and
long-term goals, the timing of the cash flows, the availability of funds for investment purposes,
the impact on the financial structure of the company, social impact of the opportunity, and legal
ramifications.
6. Project Implementation Once the decision has been made to invest funds, more detailed plans
for making the project operational are developed.
7. Project Evalua This last phase actually is. Th of the project, difficult this project be det
appraised for
This section of the investment projects. follows:
Cash Inflows:
1.Periodic
2. Investment
3. Proceed
4. Avoidable
5.Return.
6.Cash in and Cash Outflows
7. Acquis equipment
8. Additicnal charges
9. Other restoration
Capital Investment Decisions

1. Project Evaluation and Appraisal This last phase involves the assessment of how effective the
investment of the project, so that corrective action can be taken. Regardless of how actually is.
The evaluation may be in the form of continuous monitoring difficult this may be, it is important
that not only the effectiveness of the project be determined, but that the overall decision-making
process be appraised for possible improvement.
The stages just described give us only some indication or summary from, of the of the project,
size of the capital outlay and length of time to place the project in capital investment process.
The way they are implemented depends on the nature operation.
Categories of Project Cash Flows
This section of the Chapter outlines a method of estimating cash flows for investment projects.
The major categories of cash flows for a project are as follows:
Cash Inflows:
1. Periodic cash inflows from operations, net of taxes
2. Investment tax credit
3.Proceeds from sale of old asset being replaced, net of taxes
4.Avoidable costs,' net of taxes
5. Return of some working capital invested in the project*
6. Cash inflow from salvage of the new long-term asset at the end of its useful life. This will be
net of tax consequence."
Cash Outflows:
7. Acquisition cost of purchasing and installing assets (e.g., new
equipment or machinery)
8. Additional working capital
9. Other cash flows such as severance payments, relocation costs,
restoration costs and similar costs.
*The end of a project's life will usually result in some cash flows. These cash flows are referred
to as disinvestment flows.
A brief discussion of the foregoing items follows:
1. Periodic net cash returns or cash inflows from operations, net of taxes. To generate positive
periodic operating cash flows is usually the primary reason for acquiring long-term assets. These
positive flows
may result from such revenue generating activities as new they may stem from cost-saving
programs.
2. Investment-Tax Credit Products or The investment tax credit allows a credit against a
company's income tax liability based on the cost of an acquired asset. If the present Cost.
income tax laws allow investment tax credit, it would reduce the of making investments by
giving companies a credit against their corporate income taxes equal to (say) 10% of the cost of
assets.
3. Proceeds from sale of old assets being replaced, net of taxes If an old equipment is to be sold,
the proceeds from such sale is treated as a reduction from cost of initial investment. If the old
asset is sold at a gain, the incremental income tax should be deducted from the proceeds. If the
old asset is sold at a loss that is, its book value exceed the selling price, the tax savings will be
added to the proceeds.
4. Avoidable costs, net of taxes In some instances, purchase of new asset may result to the
avoidance of incurring expenses to repair the old asset. The avoidable repairs cost, net of
incremental tax will be treated as a deduction in computing the cost of initial investment.
5. Return of working capital When a project ends, there are usually some leftover inventory, cash
of other working capital items that were used to support operations. These working capital items
are then freed for use elsewhere and treated as a cash inflow.
6. Cash inflow from salvage of the new long-term asset at the end of its useful life Ending a
project will usually require disposal of its assets. In some cases, more money is spent in
disassembling the assets and disposing these than is gained from their sale. Any net outflows
from the disposal of a project's assets become tax deductions in the year of disposal. The net
salvage value of an asset is listed as a cash inflow at the time it is expected to be realized. If the
net salvage value of the asset is negative, then it is listed as cash outflow also at the time it is
expected to be incurred,
7. Acquisition costs of purchasing and installing assets These acquisition costs represent the
primary outflows for most capital investments. They are listed as cash outflows in the years in
which they are incurred.
8. Additional working capital Many projects require fund for working capital needs (for example,
to build up inventories, additional cash balance to handle increased level of activities). These
cash flows often occur before the project is in operation.
SCREENING CAPITAL INVESTMENT PROPOSALS
Several methods are available for the evaluation of alternative capital investment proposals. One
method may be used exclusively or in combination with another. The most commonly used
methods of evaluating capital investment projects are:
A. Non-discounted cash flow (unadjusted) approach
1. Payback period
2. Accounting rate of return (book value rate of return)
3. Payback reciprocal
B. Discounted cash flow (time-adjusted) approach
1. Net present value
2. Discounted rate of return or internal rate of return
3. Profitability index
4. Discounted payback period
Payback Period
Payback period (also known as payoff and payout period), measures the length of time required
to recover the amount of initial investment. It is the time interval between time of the initial
outlay and the full recovery of the investment.
When the periodic cash flows are uniform, payback period is computed as follows
Net Investment Annual cash returns
When the periodic cash flows are not uniform, payback period is computed by
cumulating the estimated annual cash inflows and determining the point in time at which they
equal the investment outlay.
Decision Rule: payback
The desirability of the project is determined by comparing the project's period against the
maximum acceptable payback period as predetermined by accepted. In short: management. The
project with shorter payback period than the maximum will be
If: PB period < Maximum allowed PB period; Accept If: PB period > Maximum allowed PB
period; Reject
Advantages of Payback period method:
1. It is easy to compute and understand.
2. It is used to measure the degree of risk associated with a project.
3. Generally, the longer the payback period, the higher the risk.
4. It is used to select projects which provide a quick return of invested funds.
Disadvantages of the payback period method:
1. It does not recognize the time value of money.
2. It ignores the impact of cash inflows after the payback period.
3. It does not distinguish between alternatives having different economic lives.
4. The conventional payback computation fails to consider salvage value, if any.
5. It does not measure profitability - only the relative liquidity of the investment.
6. There is no necessary relationship between a given payback and investor payback is
acceptable Illustration Assume.
Illustrative Problem 13-5: Determination of Payback Period
Assume the following cash flows for two alternative investment proposals:
A Proposal P150,000
B Net Investment in equipment P300,000
Annual cash returns: Years 1 to 3 75,000
Years 4 to 5 75,000

Economic life
REQUIRED: Determine the payback period of the two proposals.
Solution:
Proposal A:Payback period =P150,000 P 75,000= 2 years
Proposal B:Payback period = 3 years P300,000-P225,000
(a) P100,000
(b) 1 year]
= 3.75 years Bail-out Period
(a) cumulative returns for 3 years
(b) cash returns in the 4th year
In conventional payback computations, investment salvage value is usually An approach which
incorporates the salvage value in payback computations is the "Bail-out period". This is reached
when the cumulative cash earnings plus the salvage value at the end of a particular year equals
the original ignored.
Illustrative Problem 13-6: Determination of Bail-out period
An investment of P150,000 is expected to produce annual cash earnings of P50,000 for 5 years.
Its estimated salvage value is P70,000 during the first year and this is expected to decrease by
P15,000 annually.
REQUIRED: What is the bail-out payback period?
Cash returns during the 1st year + Salvage value;
2nd year
cash returns during the 2nd year
ACCOUNTING RATE OF RETURN OR SIMPLE RATE OF RETURN
Simple rate of return or Accounting rate of return (ARR) also known as book value rate of return,
measures profitability from the conventional accounting standpoint by relating the required
investment to the future annual net income. This is
computed as follows:
AAR =Average Annual Net Income Initial Investment or Average Investment
or, if a cost reduction project is involved, the formula becomes AAR
1Cost savings Depreciation on new equipment Initial Investment or Average Investment
Decision Rule:
the project if the ARR is greater than the cost of capital. Thus: Under the ARR method, choose
the project with the highest rate of return. Accept If: ARR > Required rate of return; Accept If:
ARR < Required rate of return; Reject
Advantages of using the ARR:
1. It is easily understood by investors acquainted with financial statements. 2. It is used as a
rough preliminary screening device of investment proposals. Disadvantages of using the ARR:
It ignores the time value of money by failing to discount the future cash inflows and outflows.
2. It does not consider the timing component of cash inflows.
3. Different averaging techniques may yield inaccurate answers.
4. It utilizes the concepts of capital and income primarily designed for the purposes of financial
statements preparation and which may not be relevant to the evaluation of investment proposals.
Illustrative Problem 13-7: Determination of Accounting Rate of Return
Consider the following information about a proposed project:
 Initial investment required
 Estimated life
 Annual cash inflows
 Salvage value of the asset at the end of 20 years Straight-line method of depreciation will
be used.
REQUIRED:
Compute the Accounting rate of return (ARR)
a) based on initial investment P65,000
b) based on average investment P10,0000
Solution:
investment=P10,000-P3,250 P65,000 10.38%
P10,000-P3,250 P65,000+ PO 20.8%
Average investment =Initial investment+Salvage value of the asset at the end of economic life
DISCOUNTED CASH FLOW TECHNIQUES
Under the discounted cash flow decision criterion, also frequently called the present-value
approach, cash outlays and cash inflows are both discounted back to the present period using an
appropriate discount rate. The variations in the DCF techniques are as follows:
1) Net present value or excess present method.
2) Discount rate of return or internal rate of return.
3) Profitability index.
4) Discounted payback period.
The Table for Present Value of P1 is shown in Appendix A-1 and the Table for Present Value of
an Annuity of P1 in Arrears is presented in Appendix A-2.
Advantages of the DCF Method:
1. It is more reliable because the time value of money is taken into account.
2. Income over the entire life of the project is considered.
3. It is more objective and relevant because it focuses on cashflow.
Disadvantages of the DCF Method are:
1. It is not easily understood.
2. It is more complex and difficult to apply.
3. It requires detailed long-term forecasts of incremental cash flow data.
4. It requires pre-determination of the cost of capital or the discount rate to
be used.
NET PRESENT VALUE (NPV)
the project over the amount of the initial investment. This is computed as follows: Net Present
Value is the excess of the present value of cash inflows generated by
Present value of cash inflows computed based on minimum desired discount rate
Less: Present value of investment Net Present Value
For independent project proposal, accept it if NPV is positive or zero and reject if NPV is
negative. If the NPV is positive, it means that the project will earn a return greater than the
discount rate also known as the hurdle rate. If the projects do not meet the hurdle rate, they
should be rejected because the funds that would be invested in them can earn a higher rate in
some other investment. In short:
If:NPV ≥ 0; Accept
If:NPV < 0; Reject
Net Present Value Application, Uniform Cash Inflows
ABC wants to invest in a machine costing P80,000 with a useful life of six years and no salvage
value. The machine will be depreciated using the straight-line method and is expected to produce
annual cash inflow from operations, net of income taxes, of P22,000. The present value of an
ordinary annuity of P1 for six periods at 10% is 4.355. The present value of P1 for six periods at
10% is 0.564. Assuming that ABC wants a minimum rate of return of 10%, what is the net
present value of this proposed investment? Is the proposal acceptable?
Solution: ABC Company
Present value of annual cash inflows for six periods at 10% (P22,000 x 4.355)
Less: Present value of net investment Net present value
Yes. The ABC Company should invest in the new machine because it could earn more than the
minimum return that they desire as indicated by the positive net present value
in the above schedule.
Illustrative Problem 13-9: Net Present Value Application:
Uneven Cash Inflow
Desdet Corp. plans to invest in a four-year project that will cost P750,000. Detdet's cost of
capital is 8%. Additional information on the project is as follows:
Year 1
Cash Flow from Operations, net of taxes

REQUIRED:
Using the net present value method, determine whether the project is acceptable or
not.
Solution: Detdet Corp.
Present value of cash inflow after taxes at 8%:
Less: Present value of net investment
Conclusion: The project is acceptable because it will yield a return exceeding the
minimum desired rate of 8%.
Illustrative Problem 13-10: Application of Net Present Value Method
Under a special licensing arrangement, Santos Company has an opportunity to market a new
product in Southern Luzon for a five-year period. The product would be purchased from the
manufacturer, with Santos Company responsible for all costs of promotion and distribution. The
licensing arrangement could be renewed at the end of the five-year period at the option of the
manufacturer. After careful study, Santos Company has estimated that the following costs and
revenues would
be associated with the new product:
Cost of equipment needed. Working capital needed.
Overhaul of the equipment in four years. Salvage value of the equipment in five years.
Annual revenues and costs:
Sales revenues.
Cost of goods sold
Out-of-pocket operating costs (for salaries, advertising, and other direct costs).
Capital Investment Decisions
At the end of the five-year period, the working capital would be released for investment
elsewhere if the manufacturer decided not to renew the licensing arrangement. Santos Company's
cost of capital is 20%. Would you recommend that the new product be introduced? Ignore
income taxes.
Solution: Santos Company

From Table for the Present Value of Pl


* From Table for the Present Value of an Annuity of P1 in arrears
Sales revenues.
Less cost of goods sold..
Gross margin..
Less out-of-pocket costs for salaries, advertising, etc..
Annual net cash inflows.
DISCOUNTED RATE OF RETURN/INTERNAL RATE OF RETURN
Discounted Rate of Return, also known as internal rate of return (IRR) and time- adjusted rate of
return, is the rate which equates the present value of the future cash inflows with the cost of the
investment which produces them. It is also the equivalent maximum rate of interest that could be
paid each year for the capital employed over the life of an investment without loss on the project.
Steps in the Computation of the Discounted Rate of Return
A. Cash inflows are evenly received:
If the cash returns or inflows are evenly received during the life of the project, the computational
procedures are as follows:
1. Compute the Present Value Factor by dividing Net Investment by
Annual Cash Returns.
2. Trace the PV factor in the Table for Present Value of P1 received annually using the life of the
project as point of reference.
3. The column that gives the closest amount to the PV factor is the "Discounted rate of return".
4. To get the exact Discounted rate of return, interpolation is applied.
B. Cash inflows are not evenly received:
The steps in computing for the discounted rate of return are:
1. Compute the Average Annual Cash Returns by dividing the sum of the returns to be received
during the life of the project by the total economic life of the project.
2. Divided Net Investment by the Average Annual Cash Returns to get the Present Value Factor.
3. Refer to the Table for Present Value of P1 received annually to determine the rate that will
give the closest factor to the computed
present value factor.
4. Using the rate obtained in Step No. 3, refer to the Table for Present Value of P1. If the returns
are increasing, use a discount rate lower than the rate obtained in Step No. 3, if the returns are
decreasing, use a higher rate. Compute the present value of the annual cash returns. 5. Add the
present value of the annual returns and compare with the Net
Investment.
Capital Investment Decisions 435
returns and net investment, try at another rate.
6. If the result in Step No. 5 does not give equality of present value of
7. Interpolate to get the exact discounted rate of return.
Decision Rule:
Accept the proposed investment if DCR or IRR is equal to or greater than minimum desired rate
of return or cost of capital. Reject the proposal if IRR is
lower than the minimum desired rate of return. In short:
If IRR > Required rate of return; Accept
If IRR Required rate of return; Reject
Tlustrative Problem 13-11: Discounted or Internal Rate of
Return Computation, Uniform Cash Returns
An investment of P50,000 will yield an average annual cash return of P7,500 a year for a period
of 10 years. What is the discounted rate of return?
Annual Cash Returns (PV factor) 7,500 X 6.6667
2. Referring to the Table for Present Value of Pl received annually for 10 years, the column that
gives the nearest value to 6.6667 is the column for 8%.
3. To get the exact rate of return, interpolate between 8% and 10%.
Illustrative Problem 13-12: Discounted or Internal Rate of
Return Computation; Uneven Cash Returns
An investment amounting to P100,000 is expected to yield cash returns as follows:
Year1 2
REQUIRED: Compute the discounted rate of return.
. Average cash returns Referring to the Table for Present Value of Pi received annually period 3,
the column that will give the nearest value to 2 is the column for 22%. discounted as follows: 4.
Using the Table for Present Value of P1, column 22%, the cash returns are Trial et 22%
 Amount of Cash
 PV of Cash Returns
Discounted rate of return is 22%. If the exact discount rate of return is required,
PAYBACK RECIPROCAL
Payback reciprocal is the rate of recovery of investment during the payback period. When a
project is at least twice the payback period and the annual cash flows are approximately equal,
the payback reciprocal may be used to estimate the discounted rate of return. A project with an
infinite life would have a discounted rate of return exactly equal to its payback reciprocal.
To compute for the payback reciprocal, the following may be used.
 Payback reciprocal
 Annual Cash Inflows
 Net Investment
 PROFITABILITY INDEX
 Payback Period
The Profitability Index, (also known as present value index, benefit-cost rate, desirability index),
is the ratio of the total present value of future cash inflows to the initial investment. The index
expresses the present value of cash benefits as to an amount per peso of investment in a project
and is used as a measure of ranking projects in a descending order of desirability. This is
computed as follows:
PV index
PV of Cash Inflows PV of Net Investment
438 Chapter 13
Decision Rule:
The higher the profitability index, the more desirable the project. Projects with
index of less than I are rejected. Thus:
If: PV Index > 1; Accept
If PV Index < 1; Reject
Illustrative Problem 13-13: Profitability Index
Company XYZ has P200,000 funds available for investment. It is considering the following
projects:
 Present value of annual cash inflows
 Less: Investment required
 Net Present Value
a) The PV indexes of Projects B and C are higher than Project A.
The company should invest in Projects B and C for the following reasons:
b) The combined net present value of Projects B and C is higher than that of
Project A.
c) The company can afford to invest in both A and B.
DISCOUNTED PAYBACK PERIOD
A method that recognizes the time value of money in a payback context is the discounted
payback method. This is used to compute the payback in terms of discounted cash flows received
in the future. That is, the periodic cash flows are discounted using an appropriate cost of capital
rate. The payback period is computed using the discounted cash flow values rather than the
actual cash flows.
Illustrative Problem 13-14: Discounted Payback Period
A project requiring an investment of P70,000 is expected to generate the following cash inflows:

REQUIRED:
1. If the cost of capital is 15%, what is its discounted payback period? 2. Should the profit be
accepted if the maximum allowable DPB is 3 years?
Solution:
The discounted payback period is determined as follows:

Cash P(170,000)
Present Value P(170,000)
Discounted 52,000
Year 60,000
Flow 52,000
factor at 15% (118,000)
Cash Flow 60,000
Balance 45,000

1. The discounted payback period is 4 years.


2. If the maximum allowable DPB is 3 years, the project should be rejected.

PREFERENCE DECISIONS - THE RANKING OF INVESTMENT


PROJECTS
Preference decisions come after screening decisions and attempt to resolve the question of "How
do the investment proposals, all of which have been screened and provide an acceptable rate of
return, rank in terms of preference?". The preference decisions are much more difficult to make
than screening decisions because investment funds are usually limited and that some or many
other profitable investment opportunities may have to be foregone.
Basically, either the internal rate of return method or the net present value method can be used in
making preference decisions.
Internal Rate of Return Method
The preference rule when using the internal rate of return method to rank competing investment
projects is:
"The higher the internal rate of return, the more desirable the project.
This method is widely used for two main reasons, namely
1) No additional computations need to be made beyond those already performed in making the
initial screening decisions.
2) The ranking data are easily understood by management.
Net Present Value Method
The net present value method can be used to rank competing investment projects if the projects
are of equal size, that is, investment funds required are the same. If the competing projects
require different amount of funding, it may be necessary to compute the profitability index.
Profitability index is computed by dividing the present value of the cash inflows by the
investment required.
The preference rule to rank competing investment projects using the profitability
index is
"The higher the profitability index, the more desirable the project.
27

Comparing the Preference Rates


lead to the same accept/reject decision.
If an independent project is being evaluated, then the NPV and IRR criteria always For mutually
exclusive projects (choosing among acceptable alternative) especially those that differ in scale
(project size) and/or timing a conflicts of ranking may arise. That is, the IRR method may favor
one alternative over another while the NPV method may indicate otherwise. If conflicts arise, the
NPV method should be used. The NPV method assumes the cash flows will be reinvested at the
firm's cost of capital while the IRR method assumes reinvestment at the project's IRR. Because
reinvestment at the cost of capital is generally a better (closer to reality) assumption, the NPV is
superior to the IRR.
The profitability index is conceptually superior to the internal rate of return as method of making
preference decisions. The reason is that the profitability index will always give a correct
indication as to the relative desirability of alternatives, even if the alternatives have different
lives and different pattern of earnings. On the other hand, if lives are unequal, the internal rate of
return method can lead the manager to make incorrect decisions.
Comparing Projects with Unequal Lives
In previous examples, replacement decisions involved comparing two mutually exclusive
projects: retaining the old asset versus buying a new one. It is also assumed that the new
equipment had a life equal to the remaining life the old equipment. However, if we were
deciding between two mutually exclusive alternative with significantly different lives, an
adjustment would be necessary. This problem may be dealt with using any one of these
procedures,
1. The replacement chain method and
2. The equivalent annual annuity method (EAA)
3.Replacement Chain (Common Life) Approach
This method compares project of unequal lines which assumes that each project Present Values
(NPVS) over this life span are then compared, and the project with can be repeated as many
times as necessary to reach a common life span. The Net the higher common life NPV is chosen.
To illustrate the application of this method we shall assume the following data on mutually
exclusive projects, Project N and Project M.

Observations and Analysis


(a) Based on the NPV, Project N appears to the better project. This analysis is, however,
incomplete and the decision to choose it, may not be correct. (b) If Project M is chosen, there
will be an opportunity to make a similar investment in 3 years and if cost and revenue conditions
continue, this investment will also be profitable. If Project N is chosen, there is no second
investment opportunity.
(c) To make a proper comparison of Project M and Project N, the replacement chain (common
life) approach could be applied.
•For Project M, add in a second project to extend the overall life of the combined project to 6
years.
• Assuming that Project M's investment cost and annual cash inflows will not change if the
project is repeated in 3 years (Year 1: P70,000, Year 2: P130,000 and Year 3: P120,000) cost of
capital will remain at 12%. The new NPV of this project will be
P88,240 and IRR will be at 25.2%.
Since the P88,240 extended NPV of Project M over the common life of 6 years is greater than
the P64910 NPV of Project N, Project M should be
selected.
Although the above illustrative case shows why an extended analysis is necessary
if there are
mutually exclusive project with different lines, the arithmetic is generally more complex in
practice. However, even for mutually exclusive projects it is not always appropriate to extend the
analysis to a common life. This repeated at the end of their initial life. should only be done if
there is a high probability that the projects will actually be
Equivalent Annual Annuity (EAA) Approach
Another procedure, known as the equivalent annual annuity (EAA) method may also be used in
evaluating mutually exclusive projects with different lives.
Equivalent Annual Annuity (EAA) Method is a method which calculates the annual payments a
project would provide if it were an annuity. Generally, when comparing projects of unequal lives,
the one with the higher equivalent annual annuity should be chosen.
To illustrate how this procedure is applied, let us assume the same data for the two projects,
Project M and Project N on page 442.
1. It is noted that
Project M's NPV = P51,550
Project N's NPV = P64,910
2. To find the value of EAA for Project M use the equation 51,550 EAA (PV of an ordinary
annuity for 3 periods at 12%) 51,550 EAA x 2.40183
EAA = P21,463
3. To find the value of EAA for Project N, the equation will be: 64,910 = EAA x (PV of an
ordinary annuity for 6 periods at 12%)
EAA = 64,910 4.11
EAA = P15,793
4. The project with the higher EAA will always have the higher NPV when extended out to any
common life. Therefore, since Project M's EAA larger than Project N, we would choose Project
M.
The EAA method is often easier to apply then the replacement chain method.
(1) If inflation is expected, the static conditions built into the analysis
would be invalid;
(2) Replacements that occur down to the road would probably employ new technology which in
turn might change the cash flows.
a speculation.
(3) Estimating the lines of a series of projects is different and is often just
Given all the uncertainties in the estimation process, such projects would, for all practical
purposes be assumed to have the same life while the cash flow estimation is complicated, the
concepts involved are exactly the same in the
two approaches.
Inflation and Capital Budgeting
Does inflation affect a capital budgeting analysis? The answer is a qualified yes in that inflation
affects the number that are used in the analysis but does not affect the results of the analysis of
certain conditions are satisfied:
This is best illustrated using the following data:
Marvex Corporation, wants to purchase a new equipment that costs P360,000. The equipment
would provide annual net cash flows from operations of P200,000 and it would have a three year
life with no salvage value. For each of the next three years, the company expects a 10% inflation
rate in the cash flows associated with the new machine. If the company's real cost of capital is
12% or market based cost of capital of 23.2%, should the equipment be purchased.

Analysis:
When performing a new present value analysis, the following should be observed:
1. If a "market-based cost of capital" is used to discount cash flows, then the cash flow should be
adjusted upwards to reflect the effects of inflation in forthcoming period.
2. If the "real cost of capital" is used in the analysis, there is no need to adjust the cash flows
upward since the inflationary effects have been taken out
of the discount rate.

These amounts are different only because of rounding error.


It will be noted that the net present value obtained in Solution B, where inflation is explicitly
taken into account is the same within rounding error to the obtained in Solution A where the
inflation effects were not considered. This result may seem surprising but it is logical because we
have adjusted both the cash flow and the discount rate so that they are consistent and these
adjustments cancel each other out across the two solutions.
REVIEW QUESTIONS, EXERCISES AND PROBLEMS
Questions
1. Define a Capital Investment.
2. Distinguish between cost of capital and discounted rate of return.
3. What are the basic principles applied in capital budgeting?
4. What are the classifications of projects based on purpose?
may enhance overall economic returns. How?
5. In the case of mutually exclusive investments, smaller profitability indexes
6. "Cost of capital is the out-of-pocket interest charge on any debt arising from an undertaking."
Do you agree? Why?
7. Should retained earnings bear a cost of capital? Why?
8. What is the difference between capital budgeting screening decisions and capital budgeting
preference decisions?
9. What is meant by the term time value of money?
10. What is meant by the term discounting?
11. Why isn't accounting net income used in the net present value and internal rate of return
methods of making capital budgeting decisions?
12. Identify two simplifying assumptions associated with discounted cash flow methods of
making capital budgeting decisions.
13. If a company has to pay interest of 14% on long-term debt, then its cost of capital is 14%. Do
you agree? Explain.
14. What is meant by an investment project's internal rate of return? How is the internal rate of
return computed?
15. How is the project profitability index computed, and what does it measure? 16. What is the
major criticism of the payback and simple rate of return methods of making capital budgeting
decisions?
Matching Type. Match the definition in the right column with the terms on the
left column.
1. Economic life
2. Direct costing
3. Incremental analysis
4. Capital budgeting
5. Average cost of capital
6. Accounting rate of return method
7. Payback period
8. Present value method
9. Sales mix analysis
10. Capital expenditure decision
A. Period within which returns areexpected.
B. The process of identifying a facility need, analyzing alternative, and rationing available funds
among competing needs.
C. The assignment of direct material, direct labor, and variable factory overhead costs to
products.
D. The capital budgeting method that determines the minimum length of time it would take to
recover the initial investment in an asset.
E. Determining when and how much to spend on capital facilities. F. The comparison of cost and
revenue data that differ among alternatives.
G. The capital budgeting method that discounts all net cash inflows to the present.
on
H. The capital budgeting method that divides a proposed project's net income by the average
investment cost.
I. The cost of financing the company's activities.
J. Determining the most profitable combination of product sales.

Exercise 1 (Simple Rate of Return Method)


The management of Ann Gee MicroBrew is considering the purchase of an automated bottling
machine for P80,000. The machine would replace an old piece of equipment that costs P33,000
per year to operate. The new machine would cost P10,000 per year to operate. The old machine
currently in use could be sold now for a scrap value of P5,000. The new machine would have a
useful life of 10 years with no salvage value.
Required:
Compute the simple rate of return on the new automated bottling machine.
Exercise 2 (Basic Present Value Concepts)
Each of the following parts is independent. (Ignore income taxes.)
1. Amano Freightlines plans to build a new garage in three years to have more space for repairing
its trucks. The garage will cost P400,000. What lump-sum amount should the company invest
now to have the P400,000 available at the end of the three-year period? Assume that the
company can invest money at:
a. Eight percent.
b. Twelve percent.
2. Lorna Products, Inc., can purchase a new copier that will save P5,000 per year in copying
costs. The copier will last for six years and have no salvage value. What is the maximum
purchase price that Lorna Products would be willing to pay for the copier if the company's
required rate of
return is:
a. Ten percent?
b. Sixteen percent?
3. Tom has just won the million-peso slot machine jackpot at a gambling casino. The casino will
pay her P50,000 per year for 20 years as the payoff. If Tom can invest money at a 10% rate of
return, what is the present value of her winnings? Did she really win a million pesos? Explain.
Exercise 3 (After-Tax Costs)
Solve each of the following parts independently.
a. Sugar Company has hired a management consulting firm o review and make recommendations
concerning Sugar's organizational structure. The consulting firm's fee will be P100,000. What
will be the after-tax cost of the consulting firm's fee if Sugar's tax rate is 30%?
b. The Royal Riding Club has redirected its advertising toward a different sector of the market.
As a result of this change in advertising, the club's annual revenues have increased by P40,000. If
the club's tax rate is 30%, what is the after-tax benefit from the increased revenues?
c. The Sparkling Stars Basketball Team has just installed an electronic scoreboard in its playing
arena at a cost of P210,000. For tax purposes, the entire original cost of the electronic scoreboard
will be depreciated over seven years, using the straight-line method. Determine the yearly tax
savings from the depreciation tax shield. Assume that the income tax rate is 30%.
Exercise 4 (Basic Net Present Value Analysis)
On January 2, Pebbles Cruz paid P18,000 for 900 ordinary shares of Rainbow Company. Ms.
Cruz received an P0.80 per share dividend on the shares at the end of each year for four years. At
the end of four years, she sold the share for her investments. P22,500. Ms. Cruz has a goal of
earning a minimum return of 12% on all of
Required:
(Ignore income taxes.) Did Ms. Cruz earn a 12% return on the share? Use the the nearest whole
peso. net present value method and the general format. Round all computations to
Capital Investment Decisions 451
Exercise 5 (Internal Rate of Return and Net Present Value)
machine for cleaning window blinds. The machine would cost P136,700, Honeylette's Cleaning
Service is investigating the purchase of an ultrasound including invoice cost, freight, and training
of employees to operate it. Honey lette's has estimated that the new machine would increase the
company's cash flows, net of expenses, by P25,000 per year. The machine would have a 14-year
useful life with no expected salvage value.
Required:
(Ignore income taxes.)
1. Compute the machine's internal rate of return to the nearest whole percent.
2. Compute the machine's net present value. Use a discount rate of 16%. Why do you have a zero
net present value?
3. Suppose that the new machine would increase the company's annual cash flows, net of
expenses, by only P20,000 per year. Under these conditions, compute the internal rate of return
to the nearest whole percent.
Exercise 6 (Basic Net Present Value and Internal Rate of Return Analysis) (Ignore income taxes.)
Consider each case below independently.
1. Edward Company's required rate of return is 15%. The company can purchase a new machine
at a cost of P40,350. The new machine would generate cash inflows of P15,000 per year and
have a four-year life with no salvage value. Compute the machine's net present value. Is the
machine an acceptable investment? Explain.
2. Isabella Products, Inc., is investigating the purchase of a new grinding machine that has a
projected life of 15 years. It is estimated that the machine will save P20,000 per year in cash
operating costs. What is the machine's internal rate of return if it costs P111,500 new?
3. Nessie Press has just purchased a new trimming machine that cost P14,125. The machine is
expected to save P2,500 per year in cash operating costs and to have a 10-year life. Compute the
machine's internal rate of return. If the company's required rate of return is 16%, did it make a
wise investment? Explain.
Problems
Problem 1 (Equipment Replacement Sensitivity Analysis)
P50,000 molding machine for automatically producing a special toy. The A toy manufacturer
who specializes in making fad items has just developed a machine has been used to produce only
one unit so far. It is planned to depreciate the P50,000 original cost evenly over four years, after
which time production of the toy will be stopped.
Suddenly, a machine salesman appears. He has a new machine that is ideally suited for
producing this toy. His automatic machine is distinctly superior. It reduces the cost of materials
by 10 percent and produces twice as many units per hour. It will cost P44,000 and will have zero
disposal value at the end of
four years.
Production and sales would continue to be at a rate of 25,000 per year for four years; annual
sales will be P90,000. The scrap value of the toy company's machine is now P5,000 and will be
P2,600 four years from now. Both machines will be useless after the 100,000-unit total market
potential is
exhausted.
With its present equipment, the company's annual expenses will be: direct materials, P10,000,
direct labor, P20,000; and variable factory overhead, P15,000. Fixed factory overhead, exclusive
of depreciation is P7,500 annually, and fixed selling and administrative expenses are P12,000
annually.
Required:
1. Assume that the hurdle rate of return is 18 percent. Using discounted cashflow techniques,
show whether the new equipment should be purchased. Use a total-project approach and an
incremental approach. What is the role of the book value of the old equipment in the analysis?
2. What is the payback period for the new equipment?
3. As the manager who developed the P50,000 molding machine, you are trying to justify not
buying the new P44,000 machine. You question the accuracy of the expected cash operating
savings. By how much must these cash savings fall before the point of indifference, the point
where net present value of the project is zero, is reached?

Problem 2
A medium-sized manufacturing company is considering the purchase of a small computer in
order to reduce the cost of its data-processing operations. following direct cash expenses per
month: At the present time, the manual bookkeeping system in use involves the
Existing furniture and equipment are fully depreciated in the accounts and have no salvage value.
The cost of the computer, including alterations, installation, and accessory equipment, is
P100,000. This entire amount is depreciable for income-tax purposes on a double-declining basis
at the rate of 20 percent per annum.
Estimated annual costs of computerized data processing are as follows:
Supervisory salaries P15,000
Other salaries 24,000
Payroll taxes and fringe benefits Forms and supplies P53,600

The computer is expected to be obsolete in three years, at which time its salvage value is
expected to be P20,000. The company follows the practice of treating salvage value as inflows at
the time that it is likely to be received.
Required:
1. Compute the savings in annual cash expenses after taxes. Assume a 50
percent tax rate.
2. Decide whether or not to purchase the computer, using the net present value method. Assume a
minimum rate of return of 10 percent.
Problem 3
ETC Products Company manufactures several different products. One of the firm's principal
products sells for P20 per unit. The sales manager of ETC Products has stated repeatedly that he
could sell more units of this product if they were available. In an attempt to substantiate his
claim, the sales manager conducted a market research study last year at a cost of P44,000 to
determine potential demand for this product. The study indicated that ETC Products could sell
18,000 units of this product annually for the next five years.
The equipment currently in use has the capacity to produce 11,000 units annually. The variable
production costs are P9 per unit. The equipment has a book value of P60,000 and a remaining
useful life of five years. The salvage value of the equipment is negligible now and will be zero in
five years.
A maximum of 20,000 units could be produced annually on the new machinery which can be
purchased. The new equipment costs P300,000 and has an estimated useful life of five years with
no salvage value at the end of the five years. ETC Product's production manager has estimated
that the new equipment would provide increased production efficiencies that would reduce the
variable production costs to P7 per unit.
ETC Products Company uses straight-line depreciation on all of its equipment for tax purposes.
The firm is subject to a 40 percent tax rate, and its after-tax cost of capital is 15 percent.
The sales manager felt so strongly about the need for additional capacity that he attempted to
prepare an economic justification for the equipment, although this was not one of his
responsibilities. His analysis, presented on the next page, disappointed him because it did not
justify acquiring the equipment.
1.. The controller of ETC Products Company plans to prepare a discounted cash flow analysis for
this investment proposal. The controller has asked you to prepare corrected calculations of
(a) The required investment in the new equipment
(b) The recurring annual cash flows
Explain the treatment of each item of your corrected calculations that is treated differently from
the original analysis prepared by the sales manager.
2. Calculate the net present value of the proposed investment in the new
equipment.
Problem 4
Notting Hill Hospital needs to expand its facilities and desires to obtain a new building on a
piece of property adjacent to its present location. Two options are available to Notting Hill, as
follows:
Option 1: Buy the property, erect the building, and install the fixtures at a total
cost of P600,000. This cost would be paid off in five installments:
An immediate payment of P200,000, and a payment of P100,000 at the end of each of the next
four years. The annual cash operating costs associated with the new facilities are estimated to be
P12,000 per year. The new facilities would be occupied for thirteen years, and would have a total
resale value of P300,000 at the end of the 13-year period.
Option 2: A leasing company would buy the property and construct the new facilities for Notting
Hill which would then be leased back to Notting Hill an annual lease cost of P70,000. The lease
period would run for 13 years, with each payment being due at the BEGINNING of the year.
Additionally, the company would require an immediate P10,000 security deposit, which would
be returned to Notting Hill at the end of the 13-year period. Finally, Notting Hill would have to
pay the annual maintenance cost of the facilities, which is estimated to be P4,000 per year. There
would be no resale value at the end of the 13-year period under this option.
The hospital uses a discount rate of 14% and the total-cost approach to net present value analysis
in evaluating its investment decisions. Ignore income taxes in this problem.
Required:
1. What is the net present value of all cash flows under Option 1 (rounded to the nearest thousand
pesos)?
2. What is the net present value of all the annual lease payments of P70,000 under Option 2
(rounded to the nearest hundred pesos)?
3. What is the present value of all cash flows associated with maintenance costs under Option 2
(rounded to the nearest hundred pesos)?

Problem 5
Robert Computers, Inc. is considering the purchase of an automated etching machine for use in
the production of its circuit boards. The machine would cost P900,000. An additional P650,000
would be required for installation costs and for software. Management believes that the
automated machine would provide substantial annual reductions in costs, as shown below:

Labor costs Material costs P240,000


Annual Reduction in Costs P240,000

The new machine would require considerable maintenance work to keep it in proper adjustment.
The company's engineers estimate that maintenance costs would increase by P4,250 per month if
the machine were addition, the machine would require a P90,000 overhaul at the end of the sixth
year.
The new Capital Investment Decisions 457 etching machine would be usable for 10 years, after
which it would be sold for its scrap value of P210,000. It would replace an old etching machine
that can be sold now for its scrap value of P70,000. Robert Computers, Inc., requires a return of
at least 18% on investments of this type. Required:
(Ignore income taxes.)
1. Compute the net annual cost savings promised by a new etching machine.
2. Using the data from (1) above the other data from the problem, compute the new machine's net
present value. (Use the incremental-cost approach). Would you recommend that the machine be
purchased? Explain.
3. Assume that management can identify several intangible benefits associated with the new
machine, including greater flexibility in shifting from one type of circuit board to another,
improved quality of output, and faster delivery as a result of reduced throughput time. What peso
value per year would management have to attach to these intangible benefits in order to make the
new etching machine an acceptable investment?
Problem 6
Stewart Parcel Service has been offered an eight-year contract to deliver mail and small parcels
between army installations. To accept the contract, the company would have to purchase several
new delivery trucks at a total cost of P450,000. Other data relating to the contract follow:
Net annual cash receipts (before taxes) from the contract Cost of overhauling the motors in the
trucks in five years Salvage value of the trucks at termination of the contract the new trucks.
If the contract were accepted, several old, fully depreciated trucks would be sold at a total price
of P30,000. These funds would be used to help purchase For tax purposes, the company
computes depreciation deductions assuming zero salvage value and uses straight-line
depreciation. The trucks would be depreciated over eight years. The company requires a 12%
after-tax return on all equipment purchases. The tax rate is 30%.
Required:
Compute the net present value of this investment opportunity. Round all peso amounts to the
nearest whole peso. Would you recommend that the contract be accepted?
Problem 7
Dr. C. Blue is the managing partner of the Crest Dental Clinic. Dr. Blue is trying to determine
whether or not the clinic should move patient files and other items out of the spare room in the
clinic and use the room for dental work. She has determined that it would require an investment
of P142,950 for equipment and related costs of getting the room ready for use. Based on receipts
being generated from other rooms in the clinic, Dr. Blue estimates that the new room would have
a seven-year estimated useful life.
Required:
(Ignore income taxes.)
1. Compute the internal rate of return on the equipment for the new room to the nearest whole
percent. Verify your answer by computing the net computed as the discount rate. present value of
the equipment using the internal rate of return you have
2. Assume that Dr. Blue will not purchase the new equipment unless it promises a return of at
least 14%. Compute the amount of annual cash inflow that would provide this return on the
P142,950 investment.
3. Although seven years is the average life for dental equipment, Dr. Blue knows that due to
changing technology this life can vary substantially. Compute the internal rate of return to the
nearest whole percentage if the life of the equipment were (a) five years and (b) nine years,
rather than you would be particularly anxious to show Dr. Blue? seven years. Is there any
information provided by these computations that
greater or less than this figure.
4. Dr. Blue is unsure about the estimated P37,500 annual cash inflow from the room. She thinks
that the actual cash inflow could be as much as 20%
a. Assume that the actual cash inflow each year is 20% greater than estimated. Recompute the
internal rate of return to nearest whole
percent.
b. Assume that the actual cash inflow each year is 20% less than estimated. Recompute the
internal rate of return to the nearest whole
percent.

5. Refer to the original data. Assume that the equipment is purchased and that the room is opened
for dental use. However, due to an increasing number of dentists in the area, the clinic is able to
generate only P30,000 per year in net cash receipts from the new room. At the end of five years,
the clinic closes the room and sells the equipment to a newly licensed dentist for a cash price of
P61,375. Compute the internal rate of return to the nearest whole percent that the clinic earned
on its investment over the five-year period. Round all peso amounts to the nearest whole peso.
(Hint: A useful way to proceed is to find the discount rate that will cause the net present value of
the investment to be equal to, or near, zero). Problem 8
Seattle Amusements Corporation places electronic games and other amusement devices in
supermarkets and similar outlets throughout the country. Seattle Amusements is investigating the
purchase of a new electronic game called The Coven. The manufacturer will sell 20 games to
Seattle Amusements for a total price of P180,000. Seattle Amusements has determined the
following additional information about the game:
a. The game would have a five-year useful life and a negligible salvage value. The company uses
straight-line depreciation.
b. The game would replace other games that are unpopular and generating little revenue. These
other games would be sold for a total of P30,000. c. Seattle Amusements estimates that The
Coven would generate annual incremental revenues of P200,000 (total for all 20 games). Annual
incremental out-of-pocket costs would be (in total): P50,000, and insurance, P10,000. In
addition, Seattle Amusements would have to pay a commission of 40% of total revenues to the
supermarkets and other outlets in which the games were placed. maintenance,
Required:
Capital Investment Decisions 459B
1. Calculate the project's internal rate of return. Should the company
2. Suppose that energy savings are less than claimed. Calculate the minimum annual cash
savings that must be realized for the project to
earn a rate equal to the firm's cost of capital.
3. Suppose that the life of the new system is overestimated by two years. Repeat Requirements 1
and 2 under this assumption.
4. Explain the implications of the answers from Requirements 1, 2, and 3.
Problem 11 (NPV; Uncertainty)
Eden Airlines is interested in acquiring a new aircraft to service a new route. The route would be
from Manila to Boracay. The aircraft would fly round- trip daily except for scheduled
maintenance days. There are 15 maintenance days scheduled each year. The seating capacity of
the aircraft is 150. Flights are expected to be fully booked. The average revenue per passenger
per flight (one-way) is P200. Annual operating costs of the aircraft follow:

Fuel P14,000,000
Flight personnel 5,000,000
Foods and beverages 1,000,000
Maintenance 4,000,000
Other 1,000,000
Total P25,000,000

The aircraft will cost P100,000,000 and has an expected life of 20 years. The company requires a
14 percent return. Assume there are no income taxes.
Required:
1. Calculate the NPV for the aircraft. Should the company buy it?
2. In discussing the proposal, the marketing manager for the airline believes that the assumption
of 100 percent booking is unrealistic. He believes that the booking rate will be somewhere
between 70 percent and 90 percent, with the most likely rate being 80 percent. Recalculate the
NPV using an 80 percent seating capacity. Should the aircraft be purchased?
3. Calculate the average seating rate that would be needed so that NPV = 0. without any effect on
demand. What is the average seating rate now 4. Suppose that the price per passenger could be
increased by 10 percent needed to achieve a NPV = 0? What would you now recommend?
Problem 12 (Inflation and Capital Investment)
Lanie Santos, divisional manager, has been pushing headquarters to grant approval for the
installation of a new flexible manufacturing system. Finally, in the last executive meeting, Lanie
was told that if she could show how the new system would increase the firm's value, it would be
approved. Lanie gathered the following information:
* In order to locate its operations in the state, the company was exempted from state income
taxes for 20 years. Since the company has only operated in the state for ten years, the state tax
could be ignored in the analysis.
With the exception of the cost of capital, this information ignores the rate of inflation, which has
been 4 percent per year and is expected to continue at this level for the next decade.
Required:
1. Compute the NPV for each system.
2. Compute the NPV for each system adjusting the future cash flows for the rate of inflation.
3. Comment on the importance of adjusting cash flows for inflationary
effects.
Problem 13 (Cost of Capital; NPV)Laika Company's product engineering department has
developed a new product, that has three-year life cycle. Production of the product requires
development of a new process that requires a current P100,000 capital outlay. The P100,000 will
be raised by issuing P60,000 of bonds and by selling new stock for P40,000. The P60,000 in
bonds will have net (after-tax) interest being repaid at the end of Year 3. payments of P3,000 at
the end of each of the three years, with the principal
The stock issue carries with it an expectation of a 17.5 percent return, expressed in the form of
dividends at the end of each year ([P7,000] in dividends is expected for each of the next three
years). The sources of capital for this investment represent the same proportion and costs that the
company typically has. Finally, the project will produce after-tax cash inflows of P50,000 per
year for the next three years.
Required:
1. Compute the cost of capital for the project. (Hint: The cost of capital is weighted average of
the two sources of capital where the weights are the proportion of capital from each source).
2. Compute the NPV for the project. Explain why it is not necessary to subtract the interest
payments and the dividend payments and appreciation from the inflow of P50,000 in carrying
out this computation.
Multiple Choice
The relevance of a particular cost to a decision is determined by
a.riskiness of the decision
b. number of decision variables.
c..amount of the cost.
d. potential effect on the decision.

2. In equipment-replacement decisions, which one of the following does


not affect the decision-making process?
a.Current disposal price of the old equipment.
b.Operating costs of the old equipment.
c.Original fair market value of the old equipment.
d. Cost of the new equipment.

3.Which one of the following statements concerning cash flow determination for capital
budgeting purposes is not correct?
a.payments for taxes. Tax depreciation must be considered because it affects cash
b. Book depreciation is relevant because it affects net income.
c..Sunk costs are not incremental flows and should not be included. forecasts. d. Net working
capital changes should be included in cash flow

4. A depreciation tax shield is


a.an after-tax cash outflow.
b reduction in income taxes.
.c.the cash provided by recording depreciation.
d. the expense caused by depreciation.
5. Del Corporation is evaluating a lease that takes effect on March 1. The company must make
eight equal payments, with the first payment due the present value of an annuity due. on March 1
The concept most relevant to the evaluation of the lease is

a.the future value of an annuity due.


b.the present value of an ordinary annuity.
c.the future value of an ordinary annuity.

6.P8 million in new equipment. Cruzer's sales are expected to increase Cruzer Corporation is
expanding its plant, which requires an investment by P6 million per year as a result of the
expansion. Cash investment in current assets averages 30% of sales, and accounts payable and
other current liabilities are 10% of sales. What is the estimated total cash
investment for this expansion?
a. F6.8 million.
b. P8.6 million.
c. P9.2 million.
d. P9.8 million.
7The length of time required to recover the initial cash outlay of a capital project is determined
by using the
a. discontinued cash flow method.
b. payback method.
c.weighted net present value method.
d. net present value method. (CMA, Adapted)
8. Which one of the following statements about the payback method of investment analysis is
correct? The payback method
does not consider the time value of money.
considers cash flows after the payback has been reached.
uses discounted cash flow techniques.
generally leads to the same decision as other methods for long-term
(CMA, Adapted)
proiects.
9. The payback reciprocal can be used to approximate a project's
a. profitability index.
b. net present value.
c.. accounting rate of return if the cash flow pattern is relatively stable.
d. internal rate of return if the cash flow pattern is relatively stable. (CMA, Adapted)
10. The bailout payback method
a. incorporates the time value of money.
b. equals the recovery period from normal operations.
c. eliminates the disposal value from the payback calculation.
d. measures the risk if a project is terminated.
11. When evaluating projects, breakeven time (discounted payback period)
is best described as annual fixed costs monthly contribution margin
a. project investment + annual net cash inflows. the point where cumulative cash inflows on a
project equal total cash outflows.
b. the point at which discounted cumulative cash inflows on a project equal discounted total cash
outflows.
12. The net present value (NPV) method of investment project analysis assumes that the project's
cash flows are reinvested at the
a.computed internal rate of return.
b.risk-free interest rate.
C.discount rate used in the NPV calculation.
d. firm's accounting rate of return.
(CMA, Adapted)
13. The proper discount rate to use in calculating with certainty, equivalent
net present value is the
a.risk-adjusted discount rate.
b.cost of capital.
C.risk-free rate.
d. cost of equity capital.

14. The rankings of mutually exclusive investments determined using the internal rate of return
method (IRR) and the net present value method (NPV) maybe different when the lives of the
multiple projects are equal and the size of the required investments are equal. the required rate of
return equal the IRR of each project. the required rate of return is higher than the IRR of each
project. d. multiple projects have unequal lives and the size of the investment for each project is
different.

15 Anya Corporation has not yet decided on its hurdle rate for use in evaluation of capital
budgeting projects. This lack of information will prohibit Anya from calculating a project'

16. When determining net present value in an inflationary environment, adjustments should be
made to
a.increase the discount rate only.
b.increase the estimated cash inflows and increase the discount rate.
C.increase the estimated cash inflows but not the discount rate.
d. decrease the estimated cash inflows and increase the discount rate.
Questions 17 through 19 are based on the following information.
Harry, Inc. has purchased a new fleet of trucks to deliver its merchandise. The trucks have a
useful life of 8 years and cost a total of P500,000. Harry expects its net increase in after-tax cash
flow to be P150,000 in Year 1, P175,000 in Year 2, P125,000 in Year 3, and P100,000 in each of
the remaining years.
20. Ignoring the time value of money, how long will it take Harry to recover
the amount of investment?
a. 3.5 years.
b. 4.0 years.
c. 4.2 years.
D 5 years.
21. What is the payback reciprocal for the fleet of trucks?
a.29%
b. 25%
C.24%
d. 20%

22. Assume the net cash flow to be P130,000 a year. What is the payback
time for the fleet of trucks?
a.3 years.
b. 3.15 years.
C.3.85 years.
d. 4 years.

23. Westlife Co. is considering the acquisition of a new, more efficient press. The cost of the
press is P360,000, and the press has an estimated 6-year life with zero salvage value. Westlife
uses the straight-line depreciation for both financial reporting and income tax reporting purposes
and has a 40% corporate income tax rate. In evaluating equipment acquisitions of this type,
Westlife uses a goal of a 4-year payback period. To meet Westlife's desired payback period, the
press P90,000. must produce a minimum annual before-tax operating cash savings of

A..P118,000
b. P110,000.
c. P114,000.
d. P150,000.

24. Duchess Inc. is expanding its manufacturing plant, which requires an investment of P4
million in new equipment and plant modifications. Duchess' sales are expected to increase by P3
million per year as a result of the expansion. Cash investment in current assets averages 30% of
sales; accounts payable and other current liabilities are 10% of sales.. What is the estimated total
investment for this expansion?
a. P3.4 million
b. P4.3 million.
C.P4.6 million.
d. P4.9 million.

25. Use the following 8% interest rate factors for this question.
26. The fir current preferer ordinary a P1.20 to be 1 weighted percent)
a.13.
b. 8.3

27.. Queen business the estin expected next 6 investme the next Between Future Value of
PIFuture Value of Annuity of P1- Period 1

a.10%
b. 14%
c. 16%
d. 18%

28.The Lady Company has P150,000 in a bank account as of December 31, 2003. If the company
plans to deposit P8,000 in the account at the end of each of the next 3 years (20X2, 20X3, and
20X4), and all amounts in the account earn 8% per year, what will the account balance be at
December 31, 20X4? Ignore the effect of income taxes.

a. P174,000
b. P176,000
C.P192,140
d. P215,000

29. A firm's new financing will be in proportion to the market value of its
current financing, shown below:
Long-term debt
Preference shares (100,000 shares)
Ordinary shares (200,000 shares)
Carrying Amount (P000 Omitted)

30.The firm's bonds are currently selling at 80% of par, generating a current market yield of 9%,
and the corporation has a 40% tax rate. The preference share is selling at its par value and pays a
6% dividend. The ordinary share has a current market value of P40 and is expected to pay a
P1.20 per share dividend this fiscal year. Dividend growth is expected to be 10% per year, and
flotation costs are negligible. The firm's weighted-average cost of capital is (round calculations
to tenths of a percent)
a. 13.0%
b. 8.3%
c. 9.6%
d. 9.0%
31.Queen owns a computer reselling business and is expanding her business. Queen is presented
with one proposal, Proposal A, such that the estimated investment for the expansion project is
P85,000 and it is expected to produce cash flows after taxes of P25,000 for each of the An
alternative proposal, Proposal B, involves an investment of P32,000 and after-tax cash flows of
P10,000 for each of next 6 years. the next 6 years. The cost of capital that would make Queen
indifferent between these two proposals lies between

A.% and 12%.


b.14% and 16%.
c.16% and 18%.
d.18% and 20%.

22. Lady Company has a payback goal of 3 years on new equipment acquisitions. A new sorter is
being evaluated that costs P450,000 and has a 5-year life. Straight-line depreciation will be used;
no salvage is anticipated. Lady is subject to a 40% income tax rate. To meet the company's
payback goal, the sorter must generate reductions in annual cash operating costs of

a. P 60,000.
b.P100,000.
C.P150,000.
d. P190,000.

33.Dame Co. is considering an investment in a capital project. The sole outlay will be P800,000
at the outset of the project, and the annual net after-tax cash inflow will be P216,309.75 for 6
years. The present value factors at Dame's 8% cost of capital are:
What is the breakeven time (BET)?
a.3.70 years.
C.5.00 years.
b. 4.57 years.
d. 6.00 y years.
Questions 34 through 37 are based on the following information.
In order to increase production capacity, Lord Gee Industries is considering replacing an existing
production machine with a new technologically improved machine effective January 1. The
following information is being considered by Lord Gee Industries:
·
The new machine would be purchased for P160,000 in cash. Shipping, installation, and testing
would cost an additional P30,000.
The new machine is expected to increase annual sales by 20,000 units at a sales price of P40 per
unit. Incremental operating costs include P30 per unit in variable costs and total fixed costs of
P40,000 per year.
The investment in the new machine will require an immediate increase in working capital of
P35,000. This cash outflow will be recovered at the end of year 5.
Capital Investment Decisions 467
⚫ Lord Gee uses straight-line depreciation for financial reporting and
tax reporting purposes. The new machine has an estimated useful life of 5 years and zero salvage
value.
Lord Gee is subject to a 40% corporate income tax rate.
Lord Gee uses the net present value method to analyze investments and will employ the
following factors and rates:
Present Value of P1
Present Value of an Ordinary Annuity of P1 atat 10%

38. Lord Gee Industries' net cash outflow in a capital budgeting decision is
a.P190,000.
b. P195,000.
C.P204,525.
d. P225,000. (CMA, Adapted)
39. Lord Gee Industries' discounted annual depreciation tax shield for the
year of replacement is
a.P13,817.
b. P16,762.
c. P20,725.
d. P22,800. (CMA, Adapted)

40. The acquisition of the new production machine by Lord Gee Industries will contribute a
discounted net-of-tax contribution margin of
a. P242,624.
b. P303,280.
C.P363,936.
d. P454,920.

41. The overall discounted cash flow impact of Lord Gee Industries
working capital investment for the new production machine would be
a.P(7,959).
b. P(10,080).
c.P(13,265).
d. P(35,000). (CMA, Adapted)
Questions 42 through 44 are based on the following information.
Pearl Inc. uses a 12% hurdle rate for all capital expenditures and has done the following analysis
for four projects for the upcoming year:

Initial capital
Annual
outlay

Net present Profitability Internal rate


Year 1 Year 2 Year 3 Year 4
value index of return

Project 1 80,00 40,00


P 65,000 70,000 -3,798 98% 11%
P200,000 0 0

Project 2 P100,00 135,00 90,00 65,00


4,276 101% 13%
P298,000 0 0 0 0

Project 3 90,00 80,00 14,06


P 80,000 95,000 106% 14%
P248,000 0 0 4

125,00 90,00 60,00 14,66


Project 4P272,000 P 95,000 105% 15%
0 0 0 2
45. Which project(s) should Pearl Inc. undertake during the upcoming year assuming it has no
budget restrictions?
a. All of the projects.
b. Projects 1, 2, and 3.
C.Projects 2, 3, and 4.
d. Projects 1, 3, and 4.

46. Which project(s) should Pearl Inc. undertake during the upcoming year if it has only
P600,000 of funds available?
a.Projects 1 and 3.
b. Projects 2, 3, and 4.
C.Projects 2 and 3.
d. Projects 3 and 4.

47.. Which project(s) should Pearl Inc. undertake during the upcoming year
if it has only P300,000 of capital funds available?
a.Projects 1.
b. Projects 2, 3, and 4.
C. Projects 3 and 4.
d. Project 3.
Questions 48 through 55 are based on the following information.
A proposed investment is not expected to have any salvage value at the end of its 5-year life. For
present value purposes, cash flows are assumed to occur at the end of each year. The company
uses a 12% after-tax target rate of return.
56. The accounting rate of return based on the average investment is
a.84.9%.
b. 34.4%.
57.. The net present value is
aP304,060.
b. P212,320.
C .40.8%
d. 12%.
58. The traditional payback period is
a. over 5 years.
b. 2.23 years.
C.1.65 years.
d. 2.83 years

59. The profitability index is


a. 0.61.
b. 0.42.
c. 0.86.
d. 1.425.

60. Which statement about the internal rate of return of the investment is true?
a. The IRR is exactly 12%.
b. The IRR is over 12%.
C. The IRR is under 12%.
d. No information about the IRR can be determined.
CHAPTER 14
MANAGEMENT CONTROL AND STRATEGIC PERFORMANCE MEASUREMENT:
STRATEGIC INVESTMENT UNITS AND TRANSFER PRICING
EXPECTED LEARNING OUTCOMES:
After studying this chapter, you should be able to...
1. Explain the concept and objective of performance evaluation and control system
2. Enumerate the advantages of performance evaluation and control system
3. Describe the prerequisites to initiate and maintain an effective performance
evaluation and control system
4. Explain the nature of Strategic Investment Unit (SIU) and the different types of SBUs
namely, profit, cost, revenue and investment SBUS
5. Evaluate performance of a profit SBU, cost SBU. investment SBU and revenue SBU
6. Explain the rationale of transfer pricing
7. Define transfer price
8. Discuss the need for transfer price in a decentralized organization
9. Explain the general approaches in setting transfer price
10. Compute the transfer price using the various approaches
11. Explain the concept of distress prices
12. Discuss the procedure in setting transfer price for service companies
13. Describe the application of multinational transfer pricing
CHAPTER 14
MANAGEMENT CONTROL AND STRATEGIC PERFORMANCE MEASUREMENT;
STRATEGIC INVESTMENT UNITS AND TRANSFER PRICING
PERFORMANCE EVALUATION AND CONTROL
Performance Evaluation
Performance evaluation is the process by which managers at all levels gain information
about the performance of tasks within the firm and judge that performance against: pre
established criteria as set out in budgets, plans, and goals.
Performance is evaluated at many different levels in the firm: top management mid-
management, and the operating level of individual production and sales employment. In
operations, the performance of individual production supervisors at the operating level
are evaluated by plant managers, who in turn are evaluated by executives at the
management level. Similarly, individual salespersons are evaluated by sales managers
who are evaluated in turn by upper-level sales management.
Management Control
Management control refers to the evaluation by upper-level managers of the
performance of mid-level managers. Operational control means the evaluation of
operating level employees by mid-level managers.
Objectives of Management Control
The objectives of management control follow:
1. Motivate managers to exert a high level of effort to achieve the goals set
by top management. 2. Provide the right incentive for managers to make decisions
consistent with the goals set by top management.
3. Determine fairly the rewards earned by managers for their effort and skill and the
effectiveness of their decision making.
STRATEGIC PERFORMANCE MEASUREMENT
Basic Concepts
In a well-managed organization, responsibilities for specific functions among its
employees are clearly identified. A Strategic Investment Unit (SIU) also known as a
Responsibility Center is a specific unit of an organization assigned to a manager who is
held accountable for its operations and resources. Each manager's performance is
judged by how well he or she manages those items under his or her control. In a
budgeting program, each manager is assigned responsibility for those items of
revenues and costs in the budget that he or she is able to control. Each manager is then
held responsible for deviations between budgeted goals and actual results. This
concept known as performance evaluation and control is central to any effective profit
planning and control system.
Strategic performance measurement also known as responsibility accounting is a
system used by top management to evaluate SIU managers. It is used when
responsibility can be effectively delegated to SIU managers and adequate measures for
evaluating the performance of the managers exist.
Decentralization and Segment Reporting
A decentralized organization is one in which decision making is not confined to a few
top executives but rather is spread throughout the organization, with managers at
various levels making key operating decisions relating to their sphere of responsibility.
Managers have found that segment reporting is of greatest value in organizations that
are decentralized. In segment reporting, costs and revenues are assigned to segments
to enable management to see where responsibility lies for control purposes and to
measure the performance of segment managers.
Benefits and Limitations of Decentralization
Benefits of Decentralization Drawbacks of Decentralization
● Uses local knowledge ● Can hinder coordination among
● Allows timely and effective SIUs
response to customers ● Can cause potential conflict among
● Train managers SIUS
● Motivates managers
● Offers objective method of
performance evaluation

Prerequisites to the Initiation and Maintenance of an Effective Strategies


Performance Measurement or Responsibility Accounting
Decentralization and Segment Reporting
For an effective strategic performance measurement, the following basic conditions are
necessary:
1. A well-defined organization structure. This requires that the spheres of
jurisdiction which are set forth in the organization chart must be clearly
established and understood and that a manager's financial responsibilities be
defined in advance. This means that a decentralized approach to decision
making is imperative if performance evaluation and control is to achieve the
purposes for which it is intended. A decentralized organization is one in which
decision making is not confined to a few top executives but rather is spread
throughout the organization, with managers at various levels making key
operating decisions relating to their sphere of responsibility.
2. Well-defined and established standards of performance in revenues, costs and
investments. This requires that an integrated plan for the control of operations
which would provide for cost standards, expense budgets, sales forecasts, profit
planning and programs for capital investment and financing as well as the
necessary procedures to effectuate the plan should be established and
maintained.
3. A system of accounting that identifies any revenues, expenses and assets to
specific units in the organization.
4. A system that provides for the preparation of regular performance reports. This
requires that a system of preparing the regular segment reports showing the
planned results, actual results and the variances should be established. These
reports should include only the items that are controllable by the manager of the
strategic investment unit (SIU)and should highlight those items requiring
management action.
Strategic Business Units (SBUs) and their Evaluation
An important step in establishing an effective performance evaluation and control
system is to determine the range of authority and influence the manager is permitted to
have control over revenues, costs, profit and investment. This will therefore require the
establishment of responsibility SBUs (also known as responsibility centers) within the
organization.
A strategic business unit (SBU) is a unit within the organization which has control over
costs, revenues, profits and/or investment funds
The types of SBUs are Cost SBUs, Profit SBUs, Investment SBUs, and Revenue
SBUS. Figure 14-1 illustrates business segments classified as cost, profit, and
investment SBUs. Discussion of the nature, examples and performance evaluation of
these SBUs are shown on pages 476 to 491.

Superior Foods
Investment SBUs Corporation Corporate

Operations Finance Chief


Legal Personnel
Vice Financial
General Counsel Vice President
President Officer

Salty Snacks Beverage Confections


Product Product Product
Profit SBUs Manager Manager Manager

Bottling
Warehouse Distribution
Cost SBUs Plant
Manager Manager
Manager
COST SBU
This is a unit within the organization wherein the manager is responsible for minimizing
costs subject to some output constraints. A distinguishing feature of a cost SBU is that it
has no control over generating revenue or use of investment funds. Examples are (1)
maintenance department of a manufacturing company, (2) library section of a school;
(3) accounting department of a trading concern. The manager of the cost SBU is
responsible for making the projection or budget of costs in his unit based on the
expected level of operation for the period. When approved by the higher authorities
(board of directors), the budgets will serve as the basis for the transactions or activities
for the ensuing period.
Performance of a cost SBU is evaluated using the performance reports or variance
analysis reports based on standard costs and flexible budget. The performance of
responsibility cost report will show the comparison between the actual costs incurred
and the budgeted costs. The difference between the two costs if any, referred to as the
variance will be reported and if significant, will be analyzed. Responsibility for the
incurrence of the variance will be pinpointed and the manager concerned will be
required to explain or justify the variance. The efficiency of the manager of the cost SBU
to control the costs in his unit will be evaluated on the basis of whether the cause of the
variance is within or beyond his control.
A responsibility cost report may be prepared using the format in Figure 14-2.
Figure 14-2: Pro-forma Responsibility Cost Report
Variance
Unfavorable
Costs Actual Budget (Favorable) Remarks
Direct costs
Controllable
Pxx Pxx Pxx
xx xx -
Total Pxx Pxx Pxx
Noncontrollable
Pxx Pxx Pxx
xx xx xx
Total Pxx Pxx Pxx
Indirect costs xx xx xx
Total costs Pxx Pxx Pxx
Illustrative Problem 14-1: Evaluation of a Cost SBU
The department supervisors at Meredith, Inc, are authorized to purchase the direct and
indirect materials needed in production, hire and assign the production workers, and
incur various overhead costs for their department. The equipment used in the
department is acquired at a higher management level, but supervisors are responsible
for proper care and maintenance. The salaries of the supervisors are shown under the
cost of supervision.
During the year, Department 7 manufactured 40,000 units as budgeted. Budgeted and
actual costs for Department 7 are given below:
Budget Actual
Direct materials P 210,000 P 208,600
Direct labor 136,000 137,800
Department costs
Supervision 21,000 21,000
Indirect materials 14,200 14,700
Repairs and maintenance 2,100 2,200
Equipment operating cost 3,400 3,300
Depreciation, equipment 4,000 4,000
Allocated plant costs:
Superintendence 19,500 21,000
Heat, light and power 3,700 3,900
Taxes and insurance 5,400 6,100
Other plant occupancy cost 5,000 6,700
Depreciation, plant 7,000 7,000
P 431,300 P 436,300
REQUIRED:
1. Compute the budgeted unit cost of the product and the actual unit cost.
2. Prepare a responsibility cost report. Show cost variations from the budget.
3. Does it appear that the supervisor was responsible for a large part of the
variation between budgeted and actual costs?
Solution: Meredith, Inc.
Requirement 1
Budgeted cost per unit = P 431, 300
40,000
= P10.7825
Actual cost per unit = P436,300
40,000
= P10.9075
Requirement 2:
Meredith, Inc
Department 7
Responsibility Cost Report
For the Year Ended December 31, 20X
Variance
(Fav)
Actual Budget Unit
Direct costs
Controllable costs
Direct materials P 208, 600 P 210,000 P (1,400)
Direct labor 137,800 136,000 1,800
Indirect materials 14,700 14,200 500
Repairs and maintenance 2,200 2,100 100
Equipment Operating cost 3,300 3,400 (100)
Total P 366,600 P 365,700 P 900
Non-controllable costs
Supervision P 21,000 P 21,000 P -
Depreciation, equipment 4,000 4,000 -
Total 25,000 25,000 - .
Total direct costs P 391,600 P 390,700 P
900

Indirect costs allocated to the


department
Superintendence P 21,000 P 19,500 P 1,500
Heat, light and power 3,900 3,700 200
Taxes and insurance 6,100 5,400 700
Other plant occupancy cost 6,700 5,000 1,700
Depreciation, plant 7,000 7,000 - .
44,700 40,600 4,100
P 436,300 P 431,300 P 5,000

Requirement 3:
From the computations shown above, it can be observed that P900 of the total
P5.000 unfavorable cost variation are traceable to direct costs over which the
department supervisor had control. Therefore, he was responsible only for the
18% unfavorable cost variation. A larger part of the unfavorable cost variation
could be traced to the costs which were allocated to his department and over
which he did not have any control or authority. He, therefore, should not be made
answerable for the unfavorable variances arising from non-controllable and
indirect costs. Performance of the manager of a cost SIU is considered
satisfactory if he is able to provide quality goods or services within the budgeted
costs.
PROFIT SBU
This is a unit or segment within the organization wherein the manager is responsible for
the generation of revenues and control of costs incurred in that SBU.xamples
are(1)loans and discounts department of a commercial bank;(2)ladies wear section of a
department store; (3) college department of a university.As in other SBUs,the manager
of the profit SBU will likewise be responsible for preparing the budget in his unit and the
approved projections will provide the guidelines and authority for him to enter into
transactions for the budget period.
Performance of a profit SBU is measured by preparing the income statements using the
contribution approach,presenting both the actual results and budgeted figures.The
statement will show the comparative revenue,direct costs and the profit SBU's
contribution to indirect costs. The operating performance of the profit SBU is generally
considered satisfactory if it is able to generate or even exceed the expected contribution
to indirect costs or common costs of the company.Figure 14-3 shows the pro-forma
income statement of a profit SBU.
Figure 14-3
X Company - Segment A
Segmented Income Statement
For the month of July 20X3
Variance
Actual Budget (Fav) Unfav Remarks
Sales in units xx xx xx
Sales Revenues Pxx Pxx Pxx
Direct Variable Costs
Cost of goods sold xx xx xx
Sales commissions xx xx xx
Total direct variable cost xx xx xx
Contribution Margin xx xx xx
Direct fixed cost
Manufacturing xx xx xx
Selling and administrative xx xx xx
Total direct fixed cost xx xx xx
Segment margin or
Contribution to indirect costs xx xx xx
Common fixed costs
Manufacturing xx xx xx
Selling and Administrative xx xx xx
Total common fixed cost xx xx xx
Operating Income Pxx Pxx Pxx
Illustrative Problem 14-2: Evaluation of a Profit SBU
The Nicki Company, a wholesaling company, purchases and repackages three products
for resale. During recent months, the company has recommended that the profit picture
would improve if Product 3 were dropped since it has been showing loss. An income
statement of last month, which is considered to be typical, is given below:
Product Lines
Total 1 2 3 .
Net Sales……………………… P 250,000 P 120,000 P 80,000 P 50,000
Cost:
Cost of product……………… P120,000 P 60,000 P 30,000 P 30,000
Sales of commissions (5%).. 12,500 6,000 4,000 2,500
Fixed Administrative costs
allocated to product lines… 80,000 40,000 20,000 20,000
Total Costs……………………… P 212,500 P 106,000 P 54,000 P 52,500
Operating Income……………… P 37,500 P 14,000 P 26,000 P (2,500)
REQUIRED:
Do an analysis to show whether the sales manager's recommendation should be
accepted or not. Assume that Product 3 could not be replaced with some other line, and
also assume that the fixed costs would not change.
Solution: Nicki Company
Nicki Company
Profitability Analysis of Product 3
Net Sales P 50,000
Less: Direct Cost and Expenses
Costs of Product P 30,000
Sales Commission 2,500 32,500
Contribution to Indirect Expenses P 17,500
Conclusion:
Product 3 contributes P17,500 to the recovery of indirect expenses. Therefore, in
the absence of any other better alternative, it should not be dropped. \
Illustrative Problem 14-3:
Evaluation of Profit SBUs RTW Fashions Inc. operates with three divisions, A, B, and C.
Division A produces revenue of P1,200,000 for the year, Division B produces revenue of
P700,000 and Division C produces P600,000.The total costs for the year for each
division:
DIvisions
A B C Total
Materials, labor and other direct costs P680,000 P720,000 P480,000 P1,880,00
Allocated company costs 200,00 200,000 200,000 600,000
Total Costs P880,000 P920,000 P680,000 P2,480,000
REQUIRED:
1. List the costs by division that can be directly attributed to that division.
2. Do all three divisions provide an amount over their direct costs to the total
operation? Identify any division that does not.
3. Is there any division that covers direct costs but does not bear its full share of
costs of the total operations? Identify that division,if any.
4. Which division(s) can bear all of its share of the allocated cost?
Solution: RTW Fashions Inc
Supporting Analysis: Divisions
A B C Total
Revenue P1,200,000 P700,000 P600,000 P2,500,000
Less: Direct costs 680,000 720,000 480,000 1,880,000
Contribution to indirect cost P 520,000 P (20,000) P 120,000 P 620,000
Less: Allocated company costs 200,000 200,000 200,000 600,000
Net income (loss) 320,000 P(220,000) P(80,000) P2,480,000
Answer:
(1).Direct costs of the divisions:
Division A P680,000
Division B P720,000
Division C P480,000
(2).No, Not all divisions provide an amount over their direct costs to the total
operation, Division B does not cover its direct cost by P20,000.
(3) Division C contributes but cannot cover P80,000 of the allocated cost.
(4) Division A.
INVESTMENT SBU
This is a unit or segment within the organization where the manager is responsible for
the control of revenues, costs and investments made in that SBU. Examples include
corporate headquarters or division of a large decentralized organization such as (1)
Magnolia Products Division of San Miguel Corporation; (2) Pharmaceutical Division of
Novartis (Phils.) Inc.; (3) Subsidiary companies
The objectives of an investment SBU or business unit are:
(a) motivate managers to exert a high level of effort to achieve the goals of the
firm.
(b) provide the right incentive for managers to make decisions that are consistent
with the goals of top management.
(c) determine fairly the rewards earned by the managers for their effort and skill.
Advantage of ROI
(1) It is easily understood and has gained wide usage.
(2) It is comparable to interest rates of returns of alternative investments.
Return on Investment (ROI) Formula
ROI= Net Operating Income
Average Operating assets
OR
ROI= Net Operating Income x Sales .
Sales Average operating asset
ROI = Operating Profit Margin x Asset Turnover (Return on Sales)
Net operating income [sometimes referred to as EBIT (earnings before interest and
taxes)] is generally used because it is consistent with the base to which it is applied,that
is, operating assets.
Operating assets include cash,accounts receivable, inventory, plant and equipment
(net), and all other assets held for productive use in the organization.Examples of
assets that generally would not be included in the operating assets category are land
held for future use,as investment in another company or a factory building rented to
someone else.
Limitations of ROI
(1) Although ROI is widely used in evaluating performance, this method is subject
to some criticisms. One of these criticisms is that ROI tends to emphasize short-
run performance rather than long-run profitability.Managers may be motivated to
reject profitable investment opportunities if the expected rate of return is lower
than the current ROI. ROI may not be fully controllable by the division manager
due to the presence of committed costs. Hence,the ROI makes it difficult to
distinguish between a performance of the division manager and the performance
of the division as an investment SBU.
(2) It results to disincentive for high ROI units to invest in projects with ROI
greater than the minimum rate of return but less than unit's current ROI.
It is therefore advisable to use multiple criteria in evaluating performance rather than
relying on ROI as a sole measure. These other criteria include(1)growth in market share
(2) increase in productivity (3) product innovation (4)peso profit(5) receivable and
inventory turnover and (6) ability to venture into new and profitable areas.
In investment SBUs,ROI can be improved by either increasing sales, by reducing
expenses or by reducing assets.
Residual Income
Another approach to measuring performance in an investment SBU is a concept known
as Residual Income.
Residual Income is the net operating income that an investment SBU is able to earn
above some minimum return on the operating assets. Generally,the larger the residual
income figure,the better is the performance rating received by the division's manager.
Advantages of Residual Income
1. A unit pursues an investment opportunity costs as long as the return from the
investment exceeds the minimum rate of return set by the firm.
2. The firm can adjust the required rates of return for differences in risk and types
of assets. For example, units with higher business risk can be evaluated at a
higher minimum rate of return. The increased risk might be due to obsolete
products, increased completion in the industry or other economic factors affecting
the business unit.
3. It is possible to calculate a different investment charge for different types of
assets. For example, one might use a higher minimum rate of return for long-
lived assets than 484 Chapter 14 more likely to be specialized in use and thus
not as readily salable.
Limitations of Residual Income
While the residual income measure deals effectively with the disincentive
problem of ROI it has also certain limitations. These are
1. Since residual income is not a percentage, it suffers the same problem of profit
SBUs in that it is not useful for composing units of significantly different sizes. It
forms larger unit that would be expected to have larger residual income,even with
relatively poor performance.
2. It is not as intuitive as ROI.
3. It may be difficult to obtain a minimum rate of return.
Economic Value Added
Economic value added (EVA) is a business unit's income after taxes and after
deducting the cost of capital. The idea is very similar to what we have explained as
residual income. The objectives of the measures are the same - to effectively motivate
investment SBU managers and to properly measure their performance. In contrast to
residual income, EVA uses the firm's cost of capital instead of a minimum rate of return.
The cost of capital is usually obtained by calculating a weighted average of the cost of
the firm's two sources of funds - borrowing and selling stock. For many firms the
minimum desire rate of return and the cost of capital are very nearly the same, with
small differences due to adjustments for risk and for strategic goals such as the desired
growth rate for the firm.
Another difference is that users of EVA do not follow conventional, conservative
accounting policies. Expenses that contribute to the long term value of the company are
capitalized. These expenses usually are expensed under generally accepted
accounting policies. Such expenses include research and development, certain types of
advertising, and training and employee development.
The main advantage of using EVA is that it focuses manager's attention of creating
value for shareholders by earning profit greater than the firm cost of capital.
Illustrative Problem 14-4: Performance Evaluation of an Investment SBU
Case 1. MNO, division of Aeon Manufacturing, has assets of P450,000 and an
operating income of P10,000
a. What is the division's ROI?
b. If the minimum rate of return is 12%, what is the division's residual
income?
Solution: Aeon Manufacturing
a. ROI = Operating income
Total assets
= P110,000
P450,000
= 24.44%
b. Operating income P110,000
Less: Minimum required return
(12% x P450,000) 54,000
Residual income P 56,000
Case 2. Consider the following
(000’s omitted)
Division A Division B
Operating assets P5,000 P12,500
Operating income P1,000 P 2,500
ROI 20% 18%
1. Which is the more successful division in terms of ROI?
2. Using 16 percent as the minimum required rate of return, compute the
residual income for each division. Which division is more successful under
this rate?
Solution:
1. Based on the information given, the more successful division in terms of ROI
division A because it has a higher ROI.
2. ( 000’s omitted)
Division A Division B
Operating assets P1,000 P 2,250
Less:Minimum required return:
Division A (0.16 x P5,000) 800
Division B (0.16 x P12,500) 2,000
Residual Income P200 P250
Case 3. The following data are given for the Blade division for 19A:
Return on investment (ROI) 25%
Sales P1,200,000
Margin 10%
Minimum required rate of return 18%
1. Compute the division's operating assets.
2. Compute the division's residual income (RI).
Solution: Blade Division
Operating income
a. ROI = Operating assets
25% = (P1,200,000 x 10%)
Operating assets
Operating assets = P480,000
b. Operating income P120,000
Less: Minimum required return
(18% x P480,000) 86,400
Residual income P 33,600

More and more companies are using residual income as a measure of performance.
They argue that the residual income approach encourages managers to make profitable
investments that would be rejected by managers who are evaluated by the ROI formula.

Illustrative Problem 14-5: Comparison of Performance of Profit SBU


Consider the following data for two comparable divisions, Division A and Division B of
Melo Corporation.
Average operating assets P100,000
Net Operating Income P 20,000
Minimum required rate of return 15%

Division A is evaluated using the ROI while Division B's performance is assessed
according to how large or how small the Residual Income is.
Division A's ROI is 20% while Division B's Residual Income is positive P5,000.

Assume that each of the divisions is presented with an opportunity to make an


investment of P25,000 in a new project that would generate 18% on invested assets.

REQUIRED: Determine if the project is acceptable to Divisions A and B. Explain the


reason for your answer.

Solution: Melo Corporation


1. Division A manager in most probability will reject the project because it will reduce his

current ROI figure from 20% to 19.6% P24,500


P125,000
2. Division B manager will be very anxious to accept the new investment opportunity
because her concern is maximizing her residual income. Any project that provides a
return greater than the maximum rate required (15%) will be attractive since it will add

to the total amount of the residual income figure. The residual income will increase
from P5,000 to P5,750.
Furthermore, the well-being of both the manager of Division B and the company as a
whole will be maximized by accepting all investment opportunities down the 15%
cutoff rate.

REVENUE SBU
This is a unit or segment within an organization where the manager is responsible for
selling budgeted quantities of various products or services at budgeted price.
Examples of managers of revenue SBUs are: A sales representative selling bread to
supermarkets, a sales manager distributing automobile to dealers in specific geographic
areas and the manager of the toys department in a local department store.
Such managers may also be responsible for travel, entertainment, and other marketing
expenses. If so, they are liable to use flexible budgets to control these expenses. But
expense control is a secondary goal in a revenue SBU.
Managers of revenue SBUs use variance in sales price and sales mix to monitor or
control their operations. Managers of revenue SBUs are responsible for achieving
budgeted levels of contribution margin by controlling the number of units sold, product
mix, and selling prices.
Three types of variances and their formulas are useful to revenue SBU managers in
meeting their goals:

1. Sales Price Variance

= Actual - Master Budget x Actual Unit


Sales Price Sales Price Sales

This variance shows how much of the difference between actual and budgeted
contribution margin is caused by the difference between actual and budgeted
sales prices.

2. Sales Volume Variance

= Actual - Master Budget x Master Budget Average


Unit Sales Unit Sales Contribution Margin per unit*

*Master budget average = Master budget Total contribution margin


contribution margin per unit Master budget sales

This variance measures the difference between actual unit sales and budgeted
unit sales.

3. Sales Mix Variance


Flexible Master
Budget Budget
= Average - Average x Actual Unit Sales
Contribution Contribution
Margin per unit *** Margin per unit

** Flexible budget average = Flexible budget Total Contribution margin


contribution margin per unit Actual unit sales

This variance is a measure of the change in contribution margin caused by


selling products in proportions (mix) different from those that were budgeted.

Illustrative Problem 14-6: Evaluation of a Revenue SBU


Karen Company's actual and budgeted sales and expense data for April are as follows:

Actual Budgeted
———————— ————————

Product Product Product Product


Zim Zoom Zim Zoom

Sales in units 4800 5300 5000 5000


Selling price per unit P12.50 P10.00 P13.00 P10.00
Variables expenses per unit P 6.90 P 5.65 P 7.00 P 5.50
Contribution Margin per unit P 5.60 P 4.35 P 6.00 P 4.50
Fixed expenses for the month for
products P40,360 P40,000

REQUIRED: Determine the following variances and indicate whether they are favorable
or unfavorable
1. Sales Price Variance
2. Sales Volume Variance
3. Sales Mix Variance

Solution: Karen Company


1. Sales Price Variance:
Product Zim = (P12.50 - P13) x 4,800 units
= P2,400 unfavorable
Product Zoom = (P10 - P10) x 5,300 units
= P0

Total Sales Price Variance


Zim P2,400 unfavorable
Zoom 0
P2400 unfavorable

Karen’s management may have a pricing problem with Product Zim as shown in the
computation of sales price variance.

2. Sales Volume Variance


= (10,000 units - 10,000 units) x P5.25 per unit*
= P525 favorable

* Master budget average = (5,000 x P6) + (5,000 x P4.50)


contribution margin per unit 10,000 units
= P5.25
The variance is favorable because the total actual number of units sold was greater
than the total budgeted sales volume.

3. Sales Mix Variance


= (P5.21287** - P5.25) x 10,100 units
= P375 unfavorable

** Flexible budget average = (4,800 x P6) + (5,300 x P4.50)


contribution margin per unit 10,100 units
= P5.21287
The sales mix variance is unfavorable because the flexible budget average contribution
margin per unit is less than the master budget average contribution margin per unit.

TRANSFER PRICING

Rationale

A problem common to most companies operating with decentralized segments is that of


placing a fair value of exchange of goods and services bete segments within the
company - the problem of Transfer pricing.
When goods or services are transferred from one unit of an organization to another, the
transaction is recorded in the accounting records. In the days when all companies were
small and management was centralized, accountants transferred goods and services
from one cost SIU to another at the cost of production. Today many companies are giant
conglomerates having multiple divisions. Simply transferring goods and services at cost
no longer serves the needs of these decentralized organizations. The problems revolve
around the question of what transfer price to charge between the segments.
A transfer price is the price charged when one segment of a company provides goods
or services to another segment of the company. For example, when the Bakery Division
of Rustan's Inc. transfers bread to the Supermarket Division, some transfer price must
be agreed upon.

Definition

Transfer price is the value assigned to goods and services transferred between
segments within the company.
The transfer price of interdivisional sales will affect the selling divisions' sales and the
buying divisions' costs but will not have any direct effect on the company's profit.
However, the transfer price policy of the company can have an indirect effect on
company profit by influencing decisions of the division manager.

The Need for Transfer Price

Transfer pricing becomes complex because of the need to evaluate an organization's


segments.
To the department selling goods and services, the transfer price is its revenue. To the
department buying the goods and services, the transfer price is its cost. Therefore,
transfer prices have a direct bearing on segment margin. Corporate managers should
set transfer pricing policies ensuring that divisions do not purchase outside when
internal facilities with high fixed cost can provide the
product. Allowing these facilities to be idle is detrimental to the overall company. A
particular transfer pricing basis may also be an excellent management tool (1) for
motivating division managers, (2) for establishing and maintaining cost control systems
and for measuring internal performance. The company should likewise establish a
transfer pricing policy that encourages decentralized managers to make an
economically optimal decision for the company without significantly reducing their
autonomy. The transfer pricing policy should allow divisional autonomy yet encourage
managers to pursue corporate goals consistent with their own personal goals.

Alternative Transfer Pricing Schemes


In practice, four general approaches are used on setting transfer price
1. Minimum Transfer Price
2. Market-based Transfer Price
3. Cost -based Transfer Price
a. Variable Cost
b. Full Cost
c. Alternative Cost Measures
4. Negotiated Transfer Price

Discussion:
1. MINIMUM TRANSFER PRICE
A general rule for making transfers to maximize a company's profits in either
perfect or imperfect market uses the formula:
Lost Contribution margin
Transfer Price = Differential costs + per unit on outside sales
per unit (or opportunity cost per unit)

The price set by the transfer pricing formula is equal to the differential costs
(generally the variable costs) of the goods being transferred, plus the contribution
margin per unit that is lost to the selling division as a result of giving up outside
sales.
It also represents the lower limit since the selling division must receive at least
the amount shown by the formula in order to be as well of as if it sold only to
outside customers. The transfer price can be more than the amount shown by
the formula but for an internal transfer to take place, the transfer price should not
exceed the purchase price from the outside supplier.
If the selling division has sufficient idle capacity to meet the demand of another
division without cutting into the sales of its regular customers, then it does not
have any opportunity costs. Hence, the lowest acceptable transfer price will be
equal to the differential or variable costs per unit. From the perspective of a
buying division, the maximum acceptable transfer price is equivalent to the price
offered by the outside supplier.
2. MARKET-BASED TRANSFER PRICE
Under this approach, the transfer price is the price at which the goods are sold
on the open market.
The market price approach is designed for situations in which there is an outside
market for the transferred product or service; the product or service is sold in its
present form to outside customers. If the selling division has no idle capacity, the
market price in the outside market is the perfect choice for the transfer price. The
reason for this is that if the selling division can sell a transferred item on the
outside market instead, then the real cost of the transfer as far as the company is
concerned is the opportunity cost of the lost revenue on the outside sale.
Whether the item is transferred internally or sold on the outside market, the
production costs are exactly the same. If the market price is used as the transfer
price, the selling division manager will not lose anything by making the transfer,
and the buying division manager will get the correct signal about how much it
really costs the company for the transfer to take place.
This is considered the best transfer price because it dovetails well with the profit
SIU concept and makes profit-based performance evaluations feasible at many
levels of the organization. By using market prices to control transfers, all divisions
or segments are able to show profits for their efforts - not just the final division in
the chain of transfers.

This market price approach is particularly useful in highly decentralized


organizations. As a general rule, this policy should contain the following
guidelines:
1. The buying division must purchase internally so long as the selling
division meets all bona fide outside prices and wants to sell internally.
2. The selling division must be free to reject internal business if it prefers
to sell outside.
3. If the selling division does not meet all bona fide outside prices, then the
buying division is free to purchase outside.
4. As independent and impartial body must be established to settle
disagreements between divisions over transfer prices.

3. COST-BASED TRANSFER PRICE


a. Variable Cost Transfer Price
Under this approach, the transfer price is based only on variable or
differential costs. Variable costs approximate differential costs in many
situations. But when fixed costs increase because of a transfer of goods
between segments, they are differential costs and therefore should be
included in the transfer price cost.

The advantage of using this basis is that it ensures in the short-run the
best use of total corporate facilities because it focuses attention on the
contribution margin a transfer generates and on how it increases short-
run profitability.

Among the disadvantages of using Variable Costs or Differential Costs


basis in setting transfer price are:
(1) A company must cover all costs before earning a profit. If fixed costs
are ignored, a variable cost transfer price may be profitable in the short
run, but not in the long run.
(2) It allows one segment manager to make a profit at the expense of
another segment manager because the receiving segment receives all the
profit.
(3) The use of this method could lead to dysfunctional decisions if a
segment must forgo outside sales to make products for other internal
segments.
(4) Transfer prices based on differential costs diminish the autonomy in
decision-making of the profit SIUs. If differential cost varies with volume,
the selling segment is dependent on the total demands of the buying
division and its external customers. This will mean that neither segment
can make its output decisions independently,
b. Full Cost Transfer Price
Full cost includes actual manufacturing costs (variable and fixed) plus
portions of marketing and administrative costs. Many companies use full
cost because of the following reasons:
(1) It is easy and convenient to apply.
(2) It leaves no intracompany profits in inventory to eliminate when
preparing consolidated statement.
(3) It allows simple and adequate end-product costing for profit
analysis by product lines.

This approach however is not suitable for companies with decentralized


structures that measure the profitability of autonomous units. Segments
tend to become complacent and less concerned about controlling costs
when they know their costs are merely passed along to the next segment.
Another criticism of full cost is that it does not create incentives for
segment managers to control or reduce costs. It likewise does not provide
management with a divisional profit figure for the selling division.

Lastly, full-cost method departs from goal-congruence. Goal congruence


means the correspondence or consistency of individual manager's
subgoals with the company's overall goals. The use of full-cost transfer
price can lead to decisions that are not goal congruent when the supplying
division is not operating at capacity. For instance, a division may want to
purchase outside the company as an apparent savings. However, a
reduction of the full-cost transfer price to the outside market purchase
price would recover all variable costs and a portion of fixed costs. The
company fails to cover these fixed costs because of the decision to
purchase outside. To avoid such suboptimization, top management must
order the lowering of transfer prices or require internal purchasing. These
solutions will both dilute the authority of individual divisions.

Standard full cost may also be used instead of the historical average cost
because it eliminates the negative effect of fluctuations in production
efficiency in one division on the reported income of another division.
Division managers can determine in advance what price they will receive
or what price they will pay for transferred goods. The disadvantage
mentioned in the previous section will likewise apply in standard full cost
basis.
c. Alternative Cost Measures
a) Full Absorption Cost-based Transfer Price
Many manufacturing firms use full-absorption costs basis because
of the difficulty in determining the opportunity cost to the company
of making internal transfer. It must be noted that only the
manufacturing costs, variable and fixed, should be included in full
absorption cost. Some advantages of this approach are
1. Costs are available in the company's records.
2. They provide the selling division with a contribution equal
to the excess of full-absorption costs over variable costs,
which gives the selling division an incentive to transfer
internally.
3. This may be a better measure of the differential costs of
transferring internally than the variable costs because other
costs such as unknown engineering and design cost are
included.
b) Cost-Plus Transfer
Some companies use cost-plus transfer pricing based on either
variable costs or full absorption cost. These methods generally
apply a normal markup to costs as a substitute for market prices
when intermediate market prices are not available.

4. NEGOTIATED TRANSFER PRICE


Under this system, managers are permitted to negotiate the price for internally
transferred goods and services. Managers act much the same as the managers
of independent companies and they use negotiation strategies similar to those
employed when trading with outside markets.

A negotiated price is an attempt to simulate an arm's-length transaction


between supplying and buying segment. If companies give segment managers
autonomous authority to buy and sell as they think necessary and if they bargain
in good faith, the result of this bargaining is the equivalent of a market price.
The major advantage of negotiated transfer prices is that they preserve the
autonomy of the division manager. However, negotiation may be very time-
consuming and require frequent re-examination and revision of prices.
Furthermore, negotiated prices eliminate the objectivity necessary
to ensure maximization of companywide profits. As a result, the negotiated price
may distort segment financial statement and mislead top management in its
attempt to evaluate performance and make decisions. Performance measure
may depend more on the manager's ability to negotiate than on other factors.

Distress Prices

When supply outstrips demand, market prices may drop well below their historical
average. If the drop in prices is expected to be temporary, these low market prices are
sometimes called "distress prices." Deciding whether a current market price is a distress
price is often difficult. The market prices of several agricultural commodities, such as
wheat and oats, have stayed for many years at what observers initially believed were
temporary distress levels.

Which transfer price should be used for judging performance if distress prices prevail?
Some companies use the distress prices themselves, but others use long-run average
prices, or "normal" market prices. In the short-run, the manager of the selling division
should meet the distress price as long as it exceeds the incremental costs of supplying
the product or service. If not, the selling division should stop selling the product or
service to the buying division, which should buy the product or service from an outside
supplier. These actions would increase overall company wide operating income. If the
long-run average market price is used, forcing the manager to buy internally at a price
above the current market price will hurt the buying division's short-run performance and
understate its profitability. Using the long-run average market price, however, provides a
better measure of the long-run viability of the supplier division. If price remains low in
the long run, though, the company should use the distress price as the transfer price.
The manager of the selling division must then decide whether to dispose of some
manufacturing facilities or shut down and have the buying division purchase the product
from an outside supplier.

Transfer Price for Services

Departments of many large organizations may sell services for customers and for each
other internally. The department performing the services to a second department
generates revenues from such activity. The same transfer is the second department's
purchase of services. For example, a company typically bill administrative services,
such as computer processing, accounting, payroll personnel to the departments they
support. In each of the cases, equitable transfer and prices must be established to
appraise the department's performance for its own return on invested capital.
The following steps may be followed in setting transfer price for services:
1. Identify the different departments contributing various services.
2. Evaluate the corresponding skill and experience of personnel involved in
delivering services.
3. Estimate the cost involved in providing the service. Factors such as time
requirements, qualifications, cost of the facilities needed to provide the service
should be considered.
4. Adopt one or any of the principles applied to the transfer of products discussed
in this chapter (e.g. cost-based transfer price, market-based).

Multinational Transfer Pricing

Transfer pricing is used worldwide to control the flow of goods and services between
segments of organizations. However, the objective of transfer pricing change when a
multinational corporation is involved and the goods and services being transferred must
cross international borders. The objectives of international transfer pricing focus on
minimizing taxes, duties, and tariffs, foreign exchange risks along with enhancing a
company's competitive position and improving its relation with foreign governments.

Corporations may change a transfer price that will reduce its total tax bill or that will
strengthen a foreign subsidiary. For example, a division in a high-income-tax-rate
country produces a subcomponent for another division in a low-income-tax rate country.
By setting a low transfer price, most of the profit from the production can be recognized
in the low-income-tax-rate country, thereby minimizing taxes. On the other hand, items
manufactured by divisions in a low-income-tax-rate country and transferred to a division
in a high-income-tax-rate country should have a high transfer price to minimize taxes.

Sometimes import duties offset income tax effects. Usually import duties are based on
the price paid for an item, whether bought from an outside company or transferred to
another division. Therefore, low transfer prices will be used to lessen the import duties.

Managers should be sensitive to the geographics, political and economic circumstances


in which they are operating, and set transfer prices in such a way as to optimize total
company performance and at the same time conform with the laws in various countries
where they operate.
Illustrative Problem 14-7
The Lewis Company has two divisions, Production and Marketing. Production
manufactures designer pants, which it sells to both the Marketing Division and to other
retailers (to the latter under a different brand name). Marketing operates numerous
pants stores, and it sells both Lewis pants and other brands. The following facts also
pertain to the Lewis Company:
● Sales price to retailers if sold by Production: P380 per pair.
● Variable cost to produce: P190 per pair.
● Fixed costs: P2,000,000 per month,
● Production is operating far below its capacity.
● Sales price to customers if sold by Marketing: P500 per pair.
● Variable marketing costs: 5 percent of sales price.

Marketing has decided to reduce the sales price of Lewis pants. The company's variable
manufacturing and marketing costs are differential to this decision. whereas fixed
manufacturing and marketing costs are not.
a. What is the minimum price that can be charged for the pants and still cover
differential manufacturing and marketing costs?

b. What is the appropriate transfer price for this decision?

c. What if the transfer price were set at P380? What effect would this have on the
minimum price set by the marketing manager?

d. How would you answer to questions a and b change if the Production Division
had been operating at full capacity?

Solution: Lewis Company


a. From the company's perspective, the minimum price would be the variable cost
of producing and marketing the goods. It would solve for this minimum price as follows:
Let X = minimum transfer price
X = 190 + .05X
= 190
0.95
= P200
The minimum price the company should accept is P200. If the company were
centralized, we would expect this information to be conveyed to the manager of
Marketing, who would be instructed not to set a price below P200.
b. The transfer price that correctly informs the marketing manager about the
differential costs of manufacturing is P190 Production is operating below capacity, so
there is no opportunity cost of transferring internally.

c. If the production manager set the price at P380, the marketing manager would
solve for the minimum price:
Let X1 = minimum price
X1 = P380 + .05X1
= P380
0.95
= P400
So the marketing manager sets the price in excess of P400 per pair, when, in fact,
prices greater than P200 would have generated a positive contribution margin from the
production and sale of pants.

d. For question a:
If Production is operating at full capacity, the minimum price is
X1 = P380 + .05X1
X1 = P380
= P400
For question b:
If Production Division had been operating at capacity, there would have been an implicit
opportunity cost of internal transfer. Production would have foregone a sale in the
wholesale market to make the internal transfer. The implicit opportunity cost to the
company is the lost contribution margin (P 80 - P190 = P190) from not selling in the
wholesale market.
Thus, if Production had sufficient sales in the wholesale market so that it would have
had to forego those sales to transfer internally, the transfer price should have been
Implicit Opportunity Cost
Differential Cost + to Company if Goods Are = P190 + P190
Transferred internally
= P380
Illustrative Problem 14-8
Assume the Speakers Division Mega Audio of Company supplies speakers outside
customers at a price of P350 each. The company has just acquired a radio assembly
company. The president believes this newly acquired Radio Division should purchase
speakers from the company's own Speakers Division because this division has excess
capacity. Until the acquisition, the radio assembly company had purchased transistors
for P350 less a 10 percent discount.

Assume no additional machines or supervisors will be acquired for the internal transfers
and the Speakers Division's unit cost is

Direct materia………………………………………………………….l………………P100
Direct labor........................................................................................................... 115
Variable overhead……………………………………………………………………… 50
Fixed overhead (now operating at 1,000,000 units activity level)......................... 30
Total cost…………………………………………………………………………………P295

Various members of management have proposed the following transfer prices:

1. Differential cost - P265. This is an appropriate transfer price for guiding top
management in deciding whether there should be transfers between the two divisions
as long as the total differential costs are less than the outside purchase price of the
buying division. This transfer price would be appropriate only when the selling division
has excess capacity as it does in this example. This transfer price also would be
appropriate in situations where there is no outside market. All benefits of using
differential costs as the basis for transfer price accrue to the buying division.

2. Full cost - P295. This transfer price would be appropriate if the company treats both
divisions as cost SIUs rather than as independent, autonomous profit SIUs. Speakers
Division profits are reflected in the profits of the Radio Division when using this transfer
price.

3. Market price - P315 (P350-P35 discount P315). This transfer price represents the
price that the Radio Division pays an independent, outside supplier and is appropriate if
the company treats both divisions as independent units.
4. Full cost plus markup - P350. This price is not appropriate because it exceeds the
price that the Radio Division pays an outside supplier. In addition, the Speaker Division
has excess capacity, and its opportunity costs are zero because it loses no contribution
margin if the internal transfer occurs. However, if the Speaker Division receives more
outside orders than it can fill and has no excess capacity, P85 represents the Speakers
Division's opportunity cost (P350-P265 P85). Using the general formula of adding
opportunity cost to differential costs results in P350 transfer price as follows: P265
differential costs per unit P85 lost contribution margin on outside sales P350.

5. Prime cost-P215. This price would not be appropriate because it does not cover all
the differential costs of the Speaker Division.

6. Negotiated price P290. A negotiated price of P290 would be appropriate if the


company treats both divisions as profit SIUs and both divisions share in the benefits.
The P50 (P315 P265) difference between the Speakers Division's differential cost and
the net outside purchase price is divided between the two divisions.

As just illustrated, the transfer price selected should depend on the capacity level at
which the selling division is operating, as well as on other factors unique to the situation.

Illustrative Problem 14-9

NY Company's Electrical Division produces a high-quality transformer. Sales and cost


data on the transformer follow:

Selling price per unit on the outside market…………….. P40


Variable costs per unit…………………………………….. P21
Fixed costs per unit (based on capacity)........................ P9
Capacity in units............................................................. 60,000

NY Company has a Motor Division that would like to begin purchasing this transformer
from the Electrical Division. The Motor Division is currently purchasing 10,000
transformers each year from another company at a cost of P38 per transformer, NY
Company evaluates its division managers on the basis of divisional profits.

REQUIRED:
1. . Assume that the Electrical Division is now selling only 50,000 transformers each
year to outside customers.
a. From the standpoint of the Electrical Division, what is the lowest
acceptable transfer price for transformers sold to the Motor Division?
b. From the standpoint of the Motor Division, what is the highest acceptable
transfer price for transformers acquired from the Electrical Division?
c. If left free to negotiate without interference, would you expect the division
managers to voluntarily agree to the transfer of 10,000 transformers from
the Electrical Division to the Motor Division? Why or why not?
d. From the standpoint of the entire company, should a transfer take place?
Why or why not?
2. Assume that the Electrical Division is now selling all of the transformers it can
produce to outside customers.
a. From the standpoint of the Electrical Division, what is the lowest
acceptable transfer price for transformers sold to the Motor Division
b. From the standpoint of the Motor Division, what is the highest acceptable
transfer price for transformers acquired from the Electrical Division?
c. If left free to negotiate without interference, would you expect the division
managers to voluntarily agree to the transfer of 10,000 transformers from
the Electrical Division to the Motor Division? Why or why not?
d. From the standpoint of the entire company, should a transfer take place?
Why or why not?

Solution:

Requirement 1

a.The lowest acceptable transfer price from the perspective of the selling division, the
Electrical Division, is given by the following formula:
Total contribution margin
Transfer price ≥ Variable cost per unit + on lost sales .
Number of units transferred
Since there is enough idle capacity to fill the entire order from the Motor Division, there
are no lost outside sales. And since the variable cost per unit is P21, the lowest
acceptable transfer price as far as the selling division is concerned is also P21.
P0 .
Transfer price ≥ P21 + 10,000 = P21

b. The Motor Division can buy a similar transformer from an outside supplier for P38.
Therefore, the Motor Division would be unwilling to pay more than P38 per transformer.

Transfer price ≥ Cost of buying from outside supplier = P38

c. Combining the requirements of both the selling division and the buying division, the
acceptable range of transfer prices in this situation is
P21 ≤ Transfer Price ≤ P38
Assuming that the managers understand their own businesses and that they are
cooperative, they should be able to agree on a transfer price within this range and the
transfer should take place.

d. From the standpoint of the entire company, the transfer should take place. The cost of
the transformers transferred is only P21 and the company saves the P38 cost of the
transformers purchased from the outside supplier.

Requirement 2

a. Each of the 10,000 units transferred to the Motor Division must displace a sale to an
outsider at a price of P40. Therefore, the selling division would demand a transfer price
of at least P40. This can also be computer using the formula for the lowest acceptable
transfer price as follows:

Transfer price ≥ P21 + ( P40-P21 ) x 10,000


10,000
=P21 + (P40-P21)= P40

b. As before, the Motor Division would be unwilling to pay more than P38 per
transformer.

c. The requirements of the selling and buying divisions in this instance are incompatible.
The selling division must have a price of at least P40 whereas the buying division will
not pay more than P38. An agreement to transfer the transformers is extremely unlikely.

d. From the standpoint of the entire company, the transfer should not take place. By
transferring a transformer internally, the company gives up revenue of P40 and saves
P38, for a loss of P2.

REVIEW QUESTIONS, EXERCISES AND PROBLEMS

Questions

1. How is performance in a cost strategic business unit (SBL) generally measured?


Performance in a profit center? Performance in an investment center?

2. When the ROI formula is being used to measure performance, what three
approaches to improving the overall profitability are open to the manager?
3. In what way can ROI lead to dysfunctional decisions on the part of the investment
SBU manager? How does the residual income approach overcome this problem?

4. Distinguish between a cost SBU, a profit SBU, and an investment SBU

5. Define the term transfer price, and why are transfer pricing systems needed?

6. If a market price for a product is determinable, why is it generally considered to be


the best transfer price?

7. Under what situation might a negotiated price be a better approach to pricing


transfers between divisions than the actual market price?

8. In what ways can suboptimization result if divisional managers are given full
autonomy in setting, accepting, and rejecting transfer prices?

Exercises

Exercise 1 (Evaluation of a Profit SBU)

The president of J. Hermosa Appliance Company notes that Department 3 has


been operating at a loss. The president reasons that profits will be increased by
P9,000 by elimination of this department in as much as costs of this department
are in excess of revenue by this amount.
Department
.
1 2 3 4
Total .
Revenue P 132,00 P168,00 P125,000 P98,000
P523,000
Direct labor cost of
department 82,000 108,000 104,000 61,000
355,000
Allocated company costs 30,000 36,000 30,000 25,000
121,000
Total costs P112,000 P144,000 P134,000 P86,000
P476,000
Net Income (loss) P 20,000 P 24,000 P (9,000) P12,000
P47,000
Required: Is the president correct? Prepare an analysis to indicate what the
results would be if Department 3 were eliminated.

Exercise 2 (Evaluation of an Investment SBU)

The Cling division has the following operating data:


Operating assets P400,000
Operating income P100,000
Minimum required rate of return 16%

Required :
(1) Compute the return ROI and RI for this division.
(2) Assume that the Cling division is presented with an investment product
yielding a 20 percent return on its investment requiring a cash outlay of P60,000.
Would the manager of the Cling division accept this investment under the ROI
approach? How about under the Rl approach?

Exercise 3 (ROI, Comparison of Three Divisions)

Consider the following sales and operating data for the three divisions of a
conglomerate
Division X Division Y
DIvision Z
Sales P280,000 P360,000
P500,000
Operating income P 10,000 P 12,600
P28,800
Operating assets P40,000 P 70,000
P180,000
Minimum required rate of return 10% 19%
20%

Required:
(1) Compute the return on investment (ROI) for each division.
(2) Assume that each division is provided with an investment opportunity that
could produce 20 percent return on investment. Which divisions would accept or
reject it?
Exercise 4 (ROL, RI, Comparisons of Two Divisions)

Cattleya Company has two divisions, A and B, that operate in similar markets.
Selected data on the two divisions follow:
Division
.
A
B
Sales………………………………….. P9,000,000
P20,000,000
Net operating income……………….. 630,000
1,800,000
Average operating assets…………… 3,000,000
10,000,000

Required.
1. Compute the ROI for each division
2. Assume that the company evaluates performance by use of residual income
and that the minimum required return for any division is 16% Compute the
residual income for each division
3. Is Division B's greater residual income an indication that it is better managed?
Explain.

Exercise 5 (Evaluation of a Cost SBU)

The supervisor of Department 10 purchases supplies, authorizes repairs and


maintenance service, and hires labor for the department. Various costs for the
month of July, 20X3, are given below:

Sales salaries and commission P9,850


Salary, supervisor of Department 10 1,800
Factory heat and light 650
General office salaries 14,200
Depreciation, factory 750
Supplies, Department 10 1,430
Repairs and maintenance 820
Factory insurance 460
Labor cost, Department 10 17,220
Salary of factory superintendent 2,400
Total P49,850
Required:
(1 ) List the costs that can be controlled by the supervisor of Dept 10.
(2)List the costs that can be directly identified with Department 10.
(3)List the costs that will have to be allocated to the factory departments.
(4) List the costs that do not pertain to factory operations.

Exercise 6 (Evaluating New Investments Using Return on Investment (ROI)


and Residual Income)

Selected sales and operating data for three divisions of three different companies
are given below:
Division A Division B
Division C
Sales………………………………… P6,000,000 P10,000,000
P8,000,000
Average operating assets,.............. 1,500,000 5,000,000
2,000,000
Net operating income……………… 300,00 900,000
180,000
Minimum required rate of return..... 15% 18%
12%

Required:
1. Compute the return on investment (ROD) for each division, using the formula
stated in terms of margin and turnover.
2. Compute the residual income for each division.
3. Assume that each division is presented with an investment opportunity that
would yield a rate of return of 17%.
A. If performance is being measured by ROI, which division or
divisions will probably accept the opportunity? Reject? Why?
B. If performance is being measured by residual income, which
division or divisions will probably accept the opportunity? Reject?
Why?

Exercise 7 (Transfer Pricing from Viewpoint of the Entire Company)


Division E manufactures picture tubes for TVs. The tubes can be sold either to
Division F of the same company or to outside customers. Last year, the following
activity was recorded in Division E:

Selling price per tube………………………………….. P 175


Production cost per tube……………………………… P 130

Number of tubes:
Produced during the year.............................. 20,000
Sold to outside customers…………………… 16,000
Sold to Division F……………………………… 4,000

Sales to Division F were at the same price as sales to outside customers. The
tubes purchased by Division F were used in a TV set manufactured by that
division. Division F incurred P300 in additional cost per TV and then sold the TVs
for P600 each.

Required:

1. Prepare income statements for last year for Division E, Division F, and the
company as a whole.

2. Assume that Division E's manufacturing capacity is 20,000 tubes per year.
Next year, Division F wants to purchase 5,000 tubes from Division E, rather than
only 4,000 tubes as in last year. (Tubes of this type are not available from outside
sources.) From the standpoint of the company as a whole, should Division E sell
the 1,000 additional tubes to Division F. or should it continue to sell them to
outside customers? Explain.

Exercise 8 (Transfer Pricing Situation)


In each of the cases below, assume that Division A has a product that can be
sold either to outside customers or Division B of the same company for use in its
production process. The manages of the divisions are evaluated based on their
divisional profits

Case .
Division X: 1 2
Capacity in units …………………………………………. 100,000 100,000
Number of units being sold to outside customers……. 100,000 80,000
Selling price per unit to outside customers …………… P50 P35
Variable costs per unit.................................................. P30 P20
Fixed costs per unit (based on capacity)...................... P8 P6

Division Y:
Number of units needed for production……………….. 20,000 20,000
Purchase price per unit now being paid to an outside
supplier................................................................ P47 P34

Required:
1. Refer to the data in case A above. Assume that P2 per unit in variable selling
costs can be avoided on intracompany sales. If the managers are free to
negotiate and make decisions on their own, will a transfer take place? If so,
within what range will the transfer price fall? Explain.
2. Refer to the data in case B above. In this case, there will be no reduction in
variable selling costs on intracompany sales. If the managers are free to
negotiate and make decisions on their own, will a transfer take place? If so,
within what range will the transfer price fall? Explain.

Exercise 9 (Transfer Pricing; Decision Making)


Nikki Inc. manufactures sports equipment. The company is comprised of several
divisions, each operating as its own profit unit. Division A has declared to go
outside the company to buy materials, since it was informed that division B was
increasing its selling price of the same materials to P200. Information for division
A and division B is as follows:

Outside price for materials P150


Division A's annual purchases 10,000 units
Division B's variable costs per unit P140
Division B's fixed costs P1,250,000
Division B's capacity utilization 100%

Required:
1.Will the company benefit if division A purchases outside the company? Assume
division B cannot sell its materials to outside buyers.
2. Assume division B can save P200,000 in fixed costs if it does not Manufacture
the material for division A. Should division A purchase from
the outside market?
3. Assume the situation in requirement 1. If the outside market value for the
materials drops P20, should A buy from the outside?
Exercise 10 (Computation of the Return on Investment (ROI)
Inilette Services Company, a division of a major oil company, provides various
services to the operators of oil fields in the Middle East. Data concerning the
most recent year appear below:

Sales P18,000,000
Net operating income P5,400,000
Average operating assets P36,000,000

Required:
1. Compute the margin for Inilette Services Company.
2. Compute the turnover for Inilette Services Company.
3. Compute the return on investment (ROI) for Inilette Services Company.

Exercise 11 (Residual Income)

Maharlika Design Ltd. is a company specializing in providing design services to


residential developers. Last year the company had net operating income of
P400,000 on sales of P2,000,000. The company's average operating assets for
the year were P2,200,000 and its minimum required rate of return was 16%.

Required:
Compute the company's residual income for the year.

Problems

Problem 1 (Evaluation of Profit SBUs)

Jadlow Manufacturing Corporation produces two (2) different product lines and
has a manager in charge of each line. The manager of product T has complained
about the Company's accounting system. He stated that he feels very strongly
that his product line had a better year than the preceding year. However, the
company's income statement for his product line showed a loss of P45,000 for
year 20X3 as follows:
Total Product S Product T
Sales P5,100,000 P2,700,000 P2,400,000
Cost of sales 4,590,000 2,370,000 2,220,000
Gross profit 510,000 330,000 180,000
Operating expenses 411,000 188,000 225,000
Net income (loss) P 99,000 P 144,000 P 145,000

You have been requested to analyze the income statement of the two product
lines to determine whether the complaint of the manager of product T is justified.
In your analysis, you find that the overall contribution margins for Product S and
T are 30% and 40%, respectively. You also discover that variable expenses
generally tend to be controllable by the managers, and that non-controllable fixed
expenses for products S and T are P600,000 and P570,000 respectively.

Required:
(a) Income statement for the two (2) product lines based on responsibility
accounting.
(b) A written explanation supporting or refuting the product line manager's
complaint. (PhilCPA Adapted)

Problem 2 (Evaluation of Profit SBUs)

Any one of these different product lines can be produced by Bubble Mills, Inc.,
with the present equipment in one of the divisions. The annual depreciation of the
equipment is P6,400, and the annual cost to operate the equipment, regardless
of product line manufactured, is P4,600.

Product A is expected to yield sales revenue of P71,000 a year with increased


costs of production amounting to P42,000. Product B should yield sales revenue
of P46,000 a year with increased costs of P15,000. Product C should yield sales
revenue of P117,000 with increased costs of P96,000.

Required:
1. Which of the three product lines seems to offer the best profit potential based
on the information given? Show computations.
2. Identify the sunk costs.
3. What is the opportunity cost of selecting only the best product fine?
(AICPA Adapted)
Problem 3 (Evaluation of Performance)

The following cost volume formula for factory overhead are given for the Ranjie
Tool Company for Year 20X3

Item Cost - Volume Formula


Utilities P1,000 per year, plus P0.80 per labor hour
Supplies P2,200 per year, plus P1.80 per labor hour
Depreciation P6,000 per year
Indirect labor P5,400 per year, plus P1.20 per labor hour
Insurance P1,200 per year

During the year, the following actual activity took place:

Budgeted labor hours 4,000 hours


Actual labor hours worked 4,200 hours
Actual overhead costs:
Utilities(P1,600 fixed) P5,200
Supplies (P2,200 fixed) 9,600
Depreciation 6,000
Indirect labor (P5,400 fixed) 10,700
Insurance 1,200

Required. Prepare a performance report for Year 20X3.

Problem 4 (Evaluation of Performance)

Rosal Corporation has three divisions. marketing, production, and personnel.


There is a manager in charge of each division. The flexible budget for each
division follows:
Marketing Production Personnel
Controllable costs:
Direct materials - P20,000 -
Direct labor - 50,000 -
Salaries P80,000 - P70,000
Supplies 20,000 6,000 4,000
Maintenance 2,000 4,000 2,000
Total P102,000 P80,000 P76,000

Actual costs by division were:


Controllable costs:
Direct materials - P24,000 -
Direct labor - P48,000 -
Salaries P102,000 - P68,000

Supplies 1,600 4,000 3,000


Maintenance 400 3,000 1,000
Total P104,000 P79,000 P72,000

Required:
1) Prepare and evaluate a performance report for the production manager.
(2) Prepare and evaluate a performance report for the vice president. Other costs
for the vice president are assumed to be budgeted P70,000 and actual P68,800.

Problem 5 (Target Sales Price: Return on Investment)

Favorite Products, a manufacturer of bicycles, uses normal volume as the basis


for the cost figures used in setting prices. That is, prices are set on the basis of
long-run volume predictions. The prices are then adjusted only for large changes
in pay rates or material prices, You are given the following information:

Materials, wages, and other variable costs P300 per unit


Fixed costs P200,000 per year
Target return on investment 20%
Normal volume 1,500 units
Investment (total assets) P800,000

Required:
1. What sales price is needed to attain the target return on investment of 20
percent?
2. What rate of return on investment will be earned at sales volumes of 2,000 and
1,000 units, respectively, given the sales price determined in requirement 1?

Problem 6 (Contrasting Return on Investment (ROI) and Residual Income)

SPF Inc. has two divisions that operate in Pasig and Quezon City. Selected data
on the two divisions follow:
Divisions .
Pasig City Quezon City
Sales P9,000,000 P20,00,000
Net operating income 630,000 1,800,000
Average operating assets 3,000,000 10,000,000

Required:

1. Compute the return on investment (ROI) for each division.


2. Assume that the company evaluates performance by use of residual income
and that the minimum required return for any division is 16%. Compute the
residual income for each division.
3.In the Quezon City Division's greater residual income an indication that it is
better managed? Explain

Problem 7 (Transfer Pricing)

Canada Products Inc. has a Valve Division the manufactures and sells a
standard valve as follows:

Capacity in units . . . . . . . . . . . . . . . . . . . . . . . . . . . . 100,000


Selling price to outside customers . . . . . . . . . . . . . . P30
Variable costs per unit . . . . . . . . . . . . . . . . . . . . . . . P18
Fixed costs per unit (based on capacity) . . . . . . . . . P9

The company has a Pump Division that could use this valve in one of its pumps.
The Pump Division is currently purchasing 10,000 valves per year from an
overseas supplier at a cost of P29 per valve.

Required:
1. Assume that the Valve Division has ample idle capacity to handle of the Pump
Division's needs. What is the acceptable range, if any, for the transfer price
between the two divisions?
2. Assume that the Valve Division is selling all of the valves that it can produce to
outside customers. What is the acceptable range, if any, for the transfer price
between the two divisions?
3. Assume again that the Valve Division is selling all of the valves that it can
produce to outside customers. Also assume that P3 in variable expenses can be
avoided on transfers within the company, due to reduced selling costs. What is
the acceptable range, if any, for the transfer price between the two divisions?
Problem 8 (Transfer Pricing)

Refer to the original data in Problem 7. Assume that the Pump Division needs
20,000 special high-pressure valves per year. The Valve Division's variable costs
to manufacture and ship the special valve would be P20 per unit. To produce
these special valves, the Valve Division would have to reduce its production and
sales of regular valves from 100,000 units per year to 70,000 units per year,

Required
As far as the Valve Division is concerned, what is the lowest acceptable transfer
price?

Problem 9 (Effects of Changes in Sales, Expenses, and Assets in ROI)

Goodies Online Corporation provides business-to-business services on the


Internet. Data concerning the most recent year appear below

Sales P8,000,000
Net operating income P800,000
Average operating assets P3200.000

Required
Consider each question below independently Carry out all computations to two
decimal places.

1. Compute the company's return on investment (ROI)


2. The entrepreneur who founded the company is convinced that sales will
increase next year by 150% and that net operating income will increase by
400%, with no increase in average operating assets. What would be the
company's ROI ?
3. The Chief Financial Officer of the company believes a more realistic scenario
would be a P2 million increase in sales, requiring an P800,500 increase in
average operating assets, with a resulting P250,000 increase in net operating
income. What would be the company's ROI in this scenario?
Problem 10 (Transfer Pricing Basics)

Jeju Company's Electrical Division produces a high-quality transformer. Sales


and cost data on the transformer follow:

Selling price per unit on the outside market P40


Variable costs per unit P21
Fixed costs per unit (based on capacity) P9
Capacity in units 60,000

Jeju Company has a Motor Division that would like to begin purchasing this
transformer from the Electrical Division. The Motor Division is currently
purchasing 10,000 transformers each year from another company at a cost of
P38 per transformer. Jeju Company evaluates its division managers on the basis
of divisional profits.

Required:
1. Assume that the Electrical Division is now selling only 50.000 transformers
each year to outside customers.
a. From the standpoint of the Electrical Division, what is the lowest acceptable
transfer price for transformers sold to the Motor Division?
b. From the standpoint of the Motor Division, what is the highest acceptable
transfer price for transformers acquired from the Electrical Division?
c.If left free to negotiate without interference, would you expect the division
managers to voluntarily agree to the transfer of 10,000 transformers from the
Electrical Division to the Motor Division? Why or why not?
d. From the standpoint of the entire company, should a transfer take place? Why
or why not?

2. Assume that the Electrical Division is now selling all of the transformers it can
produce to outside customers.
a. From the standpoint of the Electrical Division, what is the lowest acceptable
transfer price for transformers sold to the Motor Division?
b. From the standpoint of the Motor Division, what is the highest acceptable
transfer price for transformers acquired from the Electrical Division?
c. If left free to negotiate without interference, would you expect the division
managers to voluntarily agree to the transfer of 10,000 transformers from the
Electrical Division to the Motor Division? Why or why not?
d. From the standpoint of the entire company, should a transfer take place? Why
or why not?
Problem 11 (Transfer Pricing with an Outside Market)

Galaxy Products, Inc. has just purchased a small company that specializes in the
manufacture of electronic tuners that are used as a component part of TV sets.
Galaxy Products, Inc., is a decentralized company, and it will treat the newly
acquired company as an autonomous division will full profit responsibility. The
new division, called the Tuner Division, has the following revenue and costs
associated with each tuner it manufactures and sells:

Selling price P200


Expenses: P110
Variable
Fixed (based on a capacity of
100,000 tuners per year) 60 170
Net operating income P30

Galaxy Products also has an Assembly Division that assembles TV sees. This
division is currently purchasing 30,000 tuners per your from an overses supplier
at a cost of P200 per tuner, less a 10% purchase disecent The president of
Galaxy Products is anxious to have the Assembly Division begin purchasing its
tuners from the newly acquired Toner Division in order is keep the profits within
the corporate family"

Required:
For (1) through (2) below, assume that the Tuner Division can sell all of its output
to outside TV manufacturers at the normal P200 price.

1. Are the managers of the Tuner and Assembly Divisions likely to voluntarily
agree to a transfer price for 30,000 tuners each year? Why or why not?
2 If the Tuner Division meets the price that the Assembly Division is currently
paying to its overseas supplier and sells 30,000 tuners to the Assembly Division
cach year, what will be the effect on the profits of the Tuner Division, the
Assembly Division, and the company as a whole?

For (3) through (6) below, assume that the Tuner Division is currently selling only
60,000 tuners each year to outside TV manufacturers at the stated P200 price.
3. Are the managers of the Tuner and Assembly Divisions likely 10 voluntarily
agree to a transfer price for 30.000 tuners each year? Why or why not?
4. Suppose that the Assembly Division's overseas supplier drops its price (net of
purchase discount) to only P160 per tuner. Should the Tuner Division meet this
price? Explain. If the Tuner Division does not meet this price, what will be the
effect on the profits of the company as a whole?
5. Refer to (4) above. If the Tuner Division refuses to meet the P160 price, should
the Assembly Division be required to purchase from the Tuner Division at a
higher price for the good of the company as a whole? Explain.
6. Refer to (4) above. Assume that due to inflexible management policies, the
Assembly Division is required to purchase 30,000 tuners each year from the
Tuner Division at P200 per tuner. What will be the effect on the profits of the
company as a whole?

Problem 12 (Transfer Pricing: Divisional Performance)

KCC, fne, is a decentralized organization with five divisions. The company's


Electronics Division produces a variety of electronics items including a KKS
circuit board. The division (which is operating at capacity) sells the KCB circuit
board to regular customers for P125.00 each. The circuit boards have a variable
production cost of P82.50 each

The company's Clock Division has asked the Electronics Division to supply it with
large quantity of KK8 circuit boards for only P90 each. The Clock Division, which
is operating at only 60% of capacity, will put the circuit boards into a timing
device that it will produce and sell to a large oven manufacturer. The cost of the
timing device being manufactured by the Clock Division follows:

KKS circuit board (desired cost) P 90.00


Other purchased parts (from outside vendors) 300.00
Other variable costs 207.50
Fixed overhead and administrative costs 100.00
Total cost per timing device P697.50

The manager of the Clock Division feels that she can't quote a price greater than
P700 per timing device to the oven manufacturer if her division is to get the job.
As shown above, in order to keep the price at P700 or less, she can't pay more
than P90 per unit to the Electronics Division for the KK8 circuit boards. Although
the price for the KK8 circuit boards represents a substantial discount from the
normal P125.00 price, she feels that the price concession is necessary for her
division to get the oven manufacturer contract and thereby keep its core of highly
trained people.

The company uses return on investment (ROI) to measure divisional


performance.
Required
1. Assume that you are the manager of the Electronics Division. Would you
recommend that your division supply the KK8 circuit boards to the Clock Division
for P90 each as requested? Why or why not? Show all computations.
2. Would it be profitable for the company as a whole for the Electronics Division
to supply the Clock Division with the circuit boards for P90 each? Explain your
answer.
3. 1. principle, should it be possible for the two managers to agree to a transfer in
this particular situation? If so, within what range would that transfer price lie?
4. Discuss the organizational and manager behaviour problems, if srry, you
advise the inherent in this situation, what would president to do in this situation?
(CMA, adopted) Company's
Division
Queensland New South Wales
Sales P40,000,000 P70,000,000
Average operating assets P20,000,000 P20,000,000
Net operating income P3,600,000 P4,200,000
Property, plant and equipment P9,500,000 P8,000,000
Problem 13 (Computing and Interpreting Return on Investment (IOT))

Selected operating data for two divisions of Back Brewing, Lid, of Antolia are given
below:

Required:
1. Compute the rate of return for each division using the return on investment
(ROI) formula stated in terms of margin and turnover.
2. Which individual manager seems to be doing the better job? Why?

Problem 14 (Return on Investment (ROI) and Residual Income Relations)

A family friend has asked your help in analyzing the operations of three
anonymous companies operating in the same service sector industry. Supply the
missing data in the table below:

Company

A B C
Sales P90,000,000 P70,000,000 P45,000,000

Net operating income P ? P2,800,000 P. ?

Average operating assets P3,000,000 P. ? P18,000,000

Return on investment (ROI) 18% 14% ?

Minimum required rate of return:

Percentage 16% ? 15%

Peso amount P. ? P320,000 P. ?

Residual income P. ? P. ? P90,000

Problem 15 (ROI: Margin: Turnover)

The following data have been collected for the past two years for the Delta Division of
Chancellor Company:

20X0 20X1

Sales P300,000,000 P300,000,000

Net operating income 24,000,000 22,200,000

Average operating assets 150,000,000 150,000,000

Required:
1. Compute the margin and turnover ratios for each year.
2. Compute the ROI for each year.
3. Explain why the division experienced decreased ROI from 2000 to 20X1.

Problem 16
Data follow for the T-Division of Chancellor Company (see Problem 15).

20X0 20X1

Sales P500,000,000 P500,000,000

Net operating income 40,000,000 37,000,000

Average operating assets 250,000,000 200,000,000


Required:
1. Compute the margin and turnover ratios for each year.
2. Compute the ROI for the T-Division for each year.
3. How does the performance of the T-Division compare with that of the Delta Division?

Multiple Choice

1. A management decision may be beneficial for a given profit SBU, but not for the
entire company. From the overall company viewpoint, this decision could lead to
action referred to as:
a. Suboptimization.
b. Centralization.
c. Goal congruence.
d. Maximization.

2. A strategic investment unit is an organization unit headed by a responsible


manager. It is not known as a (an)
a. Cost SBU
b. Profit SBU.
c. Investment SBU.
d. Uncentralized unit
3. What term identifies an accounting system in which the operations of the
business are broken down into cost SBUs and the control function of a foreman,
sales manager or supervisor is emphasized?
a. Responsibility accounting
b. Operations research accounting
c. Control accounting
d. Budgetary accounting
4. Which of the following items of cost would be least likely to appear in a
performance report based on responsibility accounting techniques supervisor of
an assembly line in a large manufacturing situation? for the
a. Supervisor's salary
b. Materials
c. Repairs and maintenance
d. Direct labor

5. If a cost cannot be allocated to certain segments of an organization, it should be:


a. Excluded from the segmented income statement
b. Included in it.
c. Included in it but not allocated.
d. None of these.

6. A profit SBU is any submit or segment of an organization that is assigned:


a. Both revenues and expenses
b. Revenues only
c. Expenses only
d. Variable expenses only

7. Which of the following is not true about a strategic performance measurement


system?
a. It is a basis for judging management performancE
b. It is helpful in pinpointing inefficiency.
c. consists of a series of reports.
d. It holds management equally responsible for all costs.

8. In designing a strategic performance measurement system, one should keep in


mind a certain characteristic of each cost. This characteristic
a. The degree of cost controllability by the manager.
b. The degree of how the cost behaves with respect to volume.
c. The accuracy of cost allocation.
d. All of these.

9. A company that has a profit can increase its return on investment by:
a. increasing sales revenue and operating expenses by the same peso
amount.
b. increasing average operating assets and operating expenses by the same
peso amount.
c. increasing sales revenue and operating assets and sales by the same
percentage.
d. decreasing average operating assets and sales by the same percentage.

10. Given the following data:

Return on investment (ROI) 15%


Sales P120,000
Average operating assets P60,000
Minimum required rate of return 12%
Margin 7.5%

The residual income would be:


a. P1.800. c. P2,700
b. P5,400. d. P3,600.

11. An internal transfer between divisions is in the best economic interest of a


company when:
a. The variable production costs plus the opportunity cost for the selling
division is greater than the external price for the buying division.
b. There is idle capacity in the buying division.
c. There is no established market value.
d. There is no opportunity cost for the buying division.
e. The variable production costs plus the opportunity cost for the selling
division division is less than the external for price for the buying

12. Which of the following is not an appropriate use of transfer pricing?


a. Decision making
b. Product costing
c. Performance evaluation
d. Establishing cost or volume standards for production departments

13. The ideal transfer price for decision making is


a. full cost. c. market price.
b. variable cost. d. negotiated price.
14. Some managers prefer to use cost rather than market price in controlling
transfers between divisions. If cost is to be used, then it should be
a. variable cost. c. standard cost.
b. full-absorption cost. d. historical cost.

15. Transfer prices based on actual costs of the selling divisions as opposed to
standard costs
a. are preferred by the buying division.
b. often decline to provide the selling division with incentives to control costs.
c. often encourage the selling division to control costs.
d. often encourage the buying division to control costs.

16. In transfer pricing, if the selling division does not meet all bona fide outside
prices, then:
a. the buying division should accept the selling division's offer.
b. the company should drop the market price approach and use cost to
control transfers.
c. the buying division should be free to purchase outside.
d. the transfer should be made at the lowest outside price.

17. Division A has variable manufacturing costs of P25 per unit and fixed costs of P5
per unit. What is the opportunity cost of an internal transfer when the market
price is P35? Assume that Division A is operating at capacity.
a. P5 c. P25 e. P35
b. P10 d. P30

18. Josefina Corporation has two producing centers, A and B. Department A has a
variable cost of P12 per unit for its products, and a total fixed cost of P120,000.
Department A also has idle capacity for up to 50,000 units per month.
Department B would like to purchase 20,000 units of Department A's products
per month, but is unable to convince Department A to transfer units to B at P16.
Department A has consistently argued that the market price of P20 is non
negotiable. What is A's opportunity cost of transferring to B as opposed to selling
to outsiders?
a. PO c. P18
b. P12 d. P20

19. Jaycee Corporation presented the following information for its three departments
for the past month. Departments A and B are manufacturing departments,
whereas C is distribution.

Production level of A . . . . . . . . . . . . . . . . . . . significantly below capacity


Sales price to C . . . . . . . . . . . . . . . . . . . . . . . P50 per unit
A's variable cost . . . . . . . . . . . . . . . . . . . . . . . P20 per unit
Total fixed cost (A+B) . . . . . . . . . . . . . . . . . . . P120,000
C's marketing cost . . . . . . . . . . . . . . . . . . . . . 10 percent of SP
C's selling price . . . . . . . . . . . . . . . . . . . . . . . Market value

Determine the minimum transfer price from A to B.


a. P10 c. P30
b. P20 d. P50

20. With regard to information in no. 19, determine the minimum unit price that C can
sell A's product for.
a. P55 c. P30
b. P40 d. P20+ a percentage of fixed cost

Use the information below for questions 21 through 23.

Canada, Inc. operates two divisions: a management division that owns and
manages cruise ships in the Visayas; and a repair division that operates a dry
dock in Dumaguete City.The repair division works on company ships, as well as
other large-hull boats.

The repair division has an estimated variable cost of P28.50 per labor-hour. The
repair division has a two year backlog of work for outside owner ships. They
charge P48.00 per hour for labor, which is standard for this type of work. The
management division complained that it could hire its own repair workers for
P30.00 per hour (including leasing work area).

21. What is the minimum transfer price that the repair division should obtain for its
services, assuming it is operating at capacity?
a. P28.50 per hour b. P30.00 per hour c. P48.00 per hour

22. What is the maximum price the management division should pay?
a. P28.50 b. P30.00 c. P48.00
23. If the repair division had idle capacity, what is the minimum transfer price that the
repair division should obtain?]
a. P28.50
b. P30.00
c. P48.00

24. A company has two divisions, A and B, each operated as a profit SBU. A charges
B P35 per unit for each unit transferred to B. Other data follow:
A's variable cost per unit . . . . . . . . . . . . . . . . . . . . . P30
A's fixed costs . . . . . . . . . . . . . . . . . . . . . . . . . . . . . P 10,000 units
A's annual sales to B . . . . . . . . . . . . . . . . . . . . . . . . 5,000 units
A's sales to outsiders . . . . . . . . . . . . . . . . . . . . . . . . 50,000 units
A is planning to raise its transfer price to P50 per unit. Division B can purchase
units at P40 each from outsiders, but doing so would idle A's facilities now
committed to producing units for B. Division A cannot increase its sales to
outsiders. From the perspective of the company as a whole, from whom should
Division B acquire the units, assuming B's market is unaffected?
a. Outside vendors.
b. Division A, but only at the variable cost per unit.
c. Division A, but only until fixed costs are covered, then should purchase
from outside vendors.
d. Division A, in spite of the increased transfer price.

25. Cookie Company currently manufactures all component parts used in the
manufacture of various hand tools. A steel handle is used in three different tools.
The budget for these handles is 20,000 units with the following mit cost.

Direct material………………………………. P0.60


Direct labor…………………………………… 0.40
Variable overhead…………………………… 0.10
Fixed overhead………………………………. 0.20
Total unit cost…………………………. P130

Patrick Steel has offered to supply 20,000 units of the handle to Cookie
Company for P1.25 each, delivered. If Cookie currently has idle capacity that
cannot be used, accepting the offer will
a.Decrease the handle unit cost by P0.05.
b. increase the handle unit cost by P0.15.
c. decrease the handle unit cost by P0.15.
d.decrease the handle unit cost by P0.25.

Use the information below for questions 26 through 29:

Veiss Seafood, Inc. has two divisions: Wholesale Packing, and Retail Stores. Wholesale
Packing purchases seafood from local fishermen and cleans and packages the seafood.
Retail Stores operates retail fish markets. Wholesale Packing sells to the Retail Stores
Division as well as other retail operators in the Visayas region. The Retail Stores
Division purchases its goods from many sources.

Facts regarding Wholesale Packing:

Seafood......................................................Ocean trout
Production……………………………………Below capacity
Sales price to outsiders…………………….P2.00 per lb. (cleaned)
Variable costs………………………………..P1.20 per lb.
Fixed costs.................................................P15,000 per month (includes
Retail Stores Division overhead)
Facts regarding the Retail Stores Division:
Seafood ………………………………………Ocean trout
Capacity……………………………………….Sells out everyday
Sales price (retail)....................................... P3.50 pero lb.
Variable costs…………………………………10 percent of sales price
Fixed costs…………………………………….Covered by Wholesale Packing

Retail Stores Division is planning to reduce the sales price of ocean trout,

26. What is the minimum prices that can be charged by the Retail stores division to
cover the differential costs of Veiss Seafood
a. P1.33 c. P117
b. P1.20 d.P1.10

27. .What is the appropriate transfer price for this division?


a P1:20 c. P1.67
b. P1.33. d. P2.00

28. If the transfer price were set P2.00 per lb, what effect would this have on the
minimum price set by the Retail Stores Division ?
a.No change. c. Decrease to P2.00
b. Increase to P2.22 d. Increase to P3.50

29. Assume that the Wholesale Packing Division was operating at capacity Would
this affect the appropriate transfer price?
a. No. c. Not sure.
b. Yes, decrease it. d. Yes, increase it

30. Division X makes a part that it sells to customers outside of the company. Data
concerning this part appear below:

Selling price to outside customers P50


Variable cost per unit P30
Total fixed cost P400,000
Capacity in units 25,000
Division Y of the same company would like to use the part manufactured by
Division X in one of its products. Division Y currently purchases a similar part
made by an outside company for P49 per unit and would substitute the part
made by Division X. Division Y requires 5,000 of the part each period. Division X
can sell all of the units it makes to outside customers. According to the transfer
pricing formula, what is the lower limit on the transfer price?
a. P50. c. P46.
b. P49. d. P30

31. Division A makes a part that it sells to customers outside of the company. Data
concerning this part appear below:

Selling price to outside customers P40


Variable cost per unit P30
Total fixed costs P10,000
Capacity in units 20.000

Division B of the same company would like to use the pat manufactured by
Division A in one of its products. Division B currently purchases a similar part
made by an outside company for Pig per unit and would substitute the part made
by Division A. Division B requires 5,000 units of the part each period. Division A is
already selling all of the units it can produce to outside customers. If Division A
sells to Division B rather than to outside customers, the variable cost per unit
would be P1 lower. According to the transfer pricing formula, what is the lower
limit on the transfer price?
a.P40. c. P38
b. P39. d. P37.

32. The Blade Division of Dana Company produces hardened steel blades. One-third
of the Blade Division's output is sold to the Lawn Products Division of Dana; the
remainder is sold to outside customers. The Blade Division's estimated sales and
standard cost data for the next year are as follows:
Sales to
Lawn Products Outsiders
Unitsales 10,000 20,000
Sales P15,000 P40,000
Variable costs P10,000 P20,000
Fixed costs P3,000 P6,000

The Lawn Products Division has an opportunity to purchase 10,000 identical


quality blades from an outside supplier at a cost of P1.25 per unit on a continuing
basis. The Blade Division cannot sell any additional products to outside
customers because that market is saturated. This decision would have no effect
on the company's total fixed costs. If the Blade Division refuses to meet the
P1.25 price internally and the Lawn Products Division starts buying from the
outside supplier, the company as a whole will be:
a better off by P500 each period.
b.worse off by P1,500 each period.
c.better off by P2,500 each period.
d. worse off by P2,500 each period.

33. Mar Company has two decentralized divisions, X and Y Divkim X has always
purchased certain units from Division Y at 975 per Because Division Y plans to
raise the price to P100 per unit, Division X is seeking an outside supplier of the
part for the old price of P75 per une Division Y's costs follow:

Y's variable costs per unit P70


Y's annual fixed costs P15,000
Y's annual production of these units for X 1,000 units

If Division X buys from an outside supplier, the facilities Division Y uses to


manufacture these units would be idle. What would be the re if the top
management of Mar Company insists that Division X purchase from Division Y at
a transfer price of P100 per unit?

a.it would reduce the company's overall profit because Division X should buy
from outside suppliers at P75 per unit if possible
b. it would provide lower overall company net income than the old transfer price
of P75 per unit
c.it would provide higher overall company net income than the old transfer price
of P75 per unit.
d. it would be more profitable for the company then allowing X to buy from
outside suppliers at P75 per unit.

34. Decentralized firms can delegate authority and yet retain control and monitor
managers' performance by structuring the organization imo strategic business
units. Which one of the following organizational segments is most like an
independent business?

a.Revenue SBU c.Cost SBU


b. Profit SBU d. Investment SBU
(CMA, Adopted)
35. A successful responsibility accounting reporting system is dependent upon
a. the correct allocation of controllable variable costs.
b. identification of the management level at which all costs are controllable.
c. the proper delegation of responsibility and authority.
d. a reasonable separation of costs into their fixed and variable components
since fixed costs are not controllable and must be eliminated from the
responsibility report
(CMA. Adapted)
36. Pongky Manufacturing Corporation uses a strategic performance measurement
system in its operations. Which one of the following items is least likely to appear
in a performance report for a manager of one of Pongky's assembly lines?

a Direct labor
b. Materials
c. Repairs and maintenance
d. Depreciation on the manufacturing facility. (CMA, Adapted)

37. In strategic performance measurement, a unit's performance is measured by


controllable costs. Controllable costs are best described as including

a. direct material and direct labor, only.


b. only those costs that the manager can influence in the current time period.
c. only discretionary costs.
d. those costs about which the manager is knowledgeable and informed.

38. A segment of an organization is referred to as a service center if it has


a.responsibility for developing markets and selling the output of the organization
b. responsibility for combining the raw materials, direct labor, and other factors of
production into a final output
c. authority to make decisions affecting the major determinants of profit including
the power to choose its markets and sources of supply.
d. authority to provide specialized support to other units within the organization.
(CMA, Adapted)

39. The basic purpose of a strategic performance measurement system is

a. budgeting. c. authority.
b. motivation. d. variance analysis.
(CMA, Adapted)
40. The least complex segment or area of responsibility for which costs are
allocated is a(n)
a. profit center. c. contribution center.
b. investment center. d cost center.
(CMA, Adapted)
CHAPTER
FINANCIAL AND NON-FINANCIAL

15 PERFORMANCE MEASURES

EXPECTED LEARNING OUTCOMES


After studying this chapter, you should be able to...
1. Measure performance from a financial and nonfinancial perspective
2. Design measure
an accounting-based performance
3. Compute return on investment (ROI], residual income (RI), economic value added (EVA), and
return on sales (ROS)
4. Specify the difficulties when comparing the performance of divisions operating in different
countries
CHAPTER 15
FINANCIAL AND NON-FINANCIAL PERFORMANCE MEASURES
The principal focus of management control systems is strategic performance
measurement or evaluation. Performance evaluation is the process by which managers at all
levels gain information about the performance of tasks within the firm and judge that
performance against preestablished criteria as set out in budgets, plans, and goals.
The goal of top management in using strategic performance measurement is to motivate
the managers to provide a high level of effort, to guide them to make decisions that are congruent
with the goals of top management and to provide a basis for determining fair compensation for
the managers. Performance measures are a central component of management control systems
and to be effective, such measures (financial and nonfinancial) must also motivate managers and
employees at all levels of the organization to strive to achieve organization goals.
FINANCIAL AND NONFINANCIAL PERFORMANCE MEASURES
Information used in a management control system can be financial or nonfinancial.
Financial measures such as return on investment and residual income can capture
important aspects of both manager performance and organization-subunit performance.
Increasingly, companies are using internal financial measures alongside with measures based on
external financial information (e.g., stock prices), internal nonfinancial information (such as
manufacturing lead time, velocity, defect rates or even new patents) and external nonfinancial
information (such as market share, degree of customer satisfaction). Furthermore, benchmarking
against other subunits within the organization and other organizations is done.
The most commonly used instrument is the balanced scorecard (Chapter 7).
Different organizations though may use and emphasize on elements in their scorecards.
Most scorecards include
(1) profitability measures,
(2) customer-satisfaction measures,
(g) internal measures of efficiency, quality and time, and
(4) innovation measures.
Financial and Nonfinancial Performance Measures 531

In designing accounting-based performance measures, the following steps may be


followed:
1. Choose performance measures that align with top management's financial goal.
2. Determine the time horizon of each performance measure.
3. Define the components of each performance measure.
4. Choose a measurement alternative for each performance measure.
5. Decide on a target level of performance.

Step 1. Choosing among different performance measures.


There are four commonly used measures to evaluate the economic performance of
organization subunits. These are:
(a) Return on investement (ROI)

Income
¿ ∨¿
Investment
¿ Investment turnover x Returnon sales
(b) Residual Income (RI)
¿ Income−(Required rate of return x Investment )
(c) Economic value added (EVA)
After tax Weighted Total Current
= Operating – Average Cost x –
Assets Liabilities
income of Capital

(d) Return on sales (ROS)


Operating Income
¿
Sales
532 Chapter 15

Step 2. Choosing the Time Horizon of the Performance Measures


The next question that has to be resolved by the analyst is should performance measures,
such as return on assets, be calculated for one year or for a multi-year time horizon.
The ROI, RI, EVA and ROS calculations represent the results of a single time period, and in
our example, one year. To draw conclusions based on short-run changes in these measures could
conflict with the long-run interest of the organization.
It is generally desirable to evaluate subunits on the basis of ROI, RI, EVA, and ROS over
multiple years for the following reasons:
(1) to avoid using single year or short-run changes in performance measures that could run
counter with the long-run goals of the firm.
(2) the benefits of action taken in the current period may not show up in short-run
performance measures. For example, the investment, in a new hotel may adversely affect
ROI and RI in the short run but benefit ROls and RIs in the long run.
(3) If managers use the net present value method to make investment decisions. using multi-
year RI to evaluate manager's performances achieve goal congruence.
Step 3. Choosing Alternative Definitions for Performance Measures
The following are the alternative definitions of investment that companies use
(a) Total assets available = includes all assets
(b) Total assets employed = includes total assets minus the sum of idle
assets and assets purchased for future
expansion
(c) Total assets employed
Minus current liabilities = this definition excludes that portion of total assets
employed that are financed by short-term
creditors.
(d) Shareholders’ equity = use of this basis combines operating decisions
made by managers with financing decisions
regarding equity made by corporate management.
Financial and Nonfinancial Performance Measures 533

Companies generally use Total Assets available as the definition of investment in


computing ROI or RI.
Step 4. Choosing Measurement Alternatives for Performance Measures
These are different ways to measure of value assets included in the investment calculations.
These are
(1) Current cost - which could mean
a) The amount of cash or cash equivalents that would have to be paid if the same or
an equivalent asset was acquired currently, or
b) The undiscounted amount of cash or cash equivalents that would be required to
settle an obligation currently.

(2) Historical cost


a) Assels are recorded at the amount of cash or cash equivalents paid or the fair
value of the consideration given to acquire them at the time of their acquisition.
b) Liabilities are recorded at the amount of proceeds received or in exchange for the
obligation, or in some circumstances at the amounts of cash or cash equivalents
expected to be paid to satisfy the liability in the normal course of business.
Of course, measuring assets at current costs will result in different ROls and RIs compared
to the ROI and RI calculations using historical costs.
Current-cost ROI is considered a better measure of the current economic returns from the
investment compared to historical-cost ROL. A drawback, however, of using current cost is that
it can be difficult to obtain current cost estimates for some assets. This is because the estimate
requires a company to consider technological advances when determining the current cost of
assets needed to earn today's operating income.
The use of current values help to reduce the unfairness of historical cost net book value.
When comparing among business units with different aged assets, units with older assets under
the historical cost net book value have significantly higher ROls than units with newer assets
because the effect of price changes and of accumulating depreciation over the life of the assets.
The use of current values improves the use of ROI both as a measure of the performance of the
manager and of the economic performance of the unit; current values make the ROl calculation
both relevant and comparable.
534 Chapter 15
Long-term Assets: Gross or Net Book Values
Another decision to make in choosing a measure of invested capital is whether to use the
gross book value (acquisition cost) or the net book value of long-lived assets (net book value is
the acquisition cost less accumulated depreciation).
The advantages and disadvantages associated with both gross and net book value as a
measure of invested capital are as follows:
Advantages of Net Book Value; Disadvantages of Gross Book Value
1. Using net book value maintains consistency with the statement of financial position
prepared for external reporting purposes. This allows for more meaningful comparisons
of return-on-investment measures across different companies.
2. Using not book value to measure invested capital is also more consistent with the
definition of income, which is the numerator in ROI calculations. In computing income,
the current period's depreciation on long-lived assets is deducted as an expense.
Advantages of Gross Book Value; Disadvantages of Net Book Value
1. The usual methods of computing depreciation, such as the straight-line and the declining-
balance methods, are arbitrary. Hence, they should not be allowed to affect ROI or
residual income calculations.
2. When long-lived assets are depreciated, their net book value declines over time. This
results in a misleading increase in ROI and residual income across time. However, using
gross book value eliminates this problem. If an accelerated depreciation method were
used instead of the straight-line method, the increasing trend in ROI would be even more
pronounced.
Step 5. Choosing Target Levels of Performance.
a) The careful selection of benchmarks or targets can help offset shortcomings with
traditional, historical cost-based ROI, RI, or EVA measures. For example, since older
assets valued at historical cost inflate ROI (particularly if investment is defined as net
rather than gross assets), management may set higher target ROls for divisions with older
assets.
Financial and Nonfinancial Performance Measures 535
b) Many problems as asset valuation and income measurement (whether based on historical
cost or current cost) can be satisfactorily solved if top management gets everybody to
focus on what is attainable in the forthcoming budget period - regardless of whether ROI,
RI, or BVA is used and regardless of whether the financial measures are based on
historical costs or some other measure, such as current costs.

c) Another popular way to establish targets is to set continuous improvement targets. For
example, if a company is using EVA as a performance measure, top management can
evaluate operations on year-to-year changes in EVA, rather than on absolute measures of
EVA. Evaluating performance on the basis of improvements in EVA makes the initial
method of calculating EVA less important.

Step 6. Choosing the Timing of Feedback.


The final step in designing accounting-based performance measures is the timing of
feedback. The following factors should be considered in timing the report or analysis:
a) how critical is the information for the success of the organization
b) the specific level of management that is receiving the feedback
c) the sophistication of the organization's information technology.
The other consideration that should not be overlooked is that lower-level managers who are
responsible for day-to-day operations usually require more frequent feedback than senior
executives, who primarily exercise oversight over the lower level managers.
Strategic feedback should enable managers to test if the strategy produces the expected
results. If not, it could be due to one of two causes:
(1) implementation problems, or
(2) an invalid strategy.
The timely reporting of actual results and identification of the causes of the deviation could
signal that corrective action should be taken so that the plan (strategy) can be executed as
intended.
536 Chapter 15
Illustrative Problem 15-1: ROI, RI, EVA, and ROS
Consider the following data for the two geographic divisions of Luzon Electric company
that operate as profit centers:
Northern Luzon Division Southern Luzon Division
Total Assets P1,000,000 P5,000,000
Current Liabilities 250,000 1,500,000
Operating Income 200,000 750,000

REQUIRED:
1. Calculate the ROI for each division using operating income as the measure of income and
using total assets as the measure of investment.
2. Luzon Electric has used RI as a measure of management performance, the variable it wants a
manager to maximize. Using this criterion, what is the RI for each division using operating
income and total assets, if the required rate of return on investment is 12 percent?
3. Luzon Electric has two sources of funds: long-term debt with a market value of P3,500,000
and an interest rate of 10%, and equity capital with a market value of P3,500,000 at a cost of
equity of 14 percent. Luzon's income tax rate is 40 percent. Luzon applies the same weighted-
average cost of capital to both divisions, since each division faces similar risks. Calculate the
EVA for each division. Which of the measures calculated in requirements 1, 2, and 3 would you
recommend Luzon Electric to use? Why? Explain briefly.
4. Calculate the ROS for each division.

Solution:
Requirements 1 and 2 are answered together:
Northern Luzon Division Southern Luzon Division
Total Assets P 1,000,000 P5,000,000
Current Liabilities P 250,000 1,500,000
Operating Income P 200,000 ÷ P 1,000,000=20 % P 750,000÷ P 5,000,000=15 %

Residual income at
12% required rate of
return P 80,000 P 150,000

* Northern Luzon: P200,000 – (0.12 x P1,000,000) = P80,000.


Southern Luzon: P750,000 – (0.12 x P5,000,000) = P150,000.
Financial and Nonfinancial Performance Measures 537
The tabulation shows that, while the Northern Luzon Division earns the highest return on
investment, Southern Luzon Division earns the higher residual income at the 12% required rate
of return.
Requirement 3
After-tax cost of debt financing = (1–0.4) x 10% = 6%
After tax cost of equity financing= 14%

The weighted average cost of capital (WACC) is given by


( 0.06 x P 3,500,000 ) +(0.14 x P 3,500,000)
W ACC=
P 3,500,000+ P3,500,000
P 210,000+ P 490,000
¿
P7,000,000
P 700,000
¿
P 7,000,000
¿ 0.10∨10 %
Economic value added (EVA) calculations are as follows:
Northern Luzon Southern Luzon
After tax operating income
Northern (200,000 x 0.6) 120,000
Southern (750,000 x 0.6) 450,000
Less: Cost of Capital
Northern [10% x (1,000,000-250,000)] 75,000
Southern [10% x (5,000,000-1,500,000)] 350,000
EVA 45,000 100,000

Luzon should use the EVA measure for evaluating the economic performance of its
divisions for two reasons: (a) It is a residual income measure and, so, does not have the
dysfunctional effects of ROI-based measures. That is, if EVA is used as a performance evaluation
measure, divisions would have incentives to make investments whenever after. tax operating
income exceeds the weighted-average cost of capital employed. These are ROI-based
performance evaluation the correct incentives to maximize firm value. ROI-based performance
evaluation measures encourage managers to invest only when the ROI on new investments
exceeds the existing ROI. That is, managers would reject projects whose ROI exceeds the
weighted average cost of capital but is less than the current ROI of the division; using ROI as a
performance evaluation measure creates incentives for managers to reject projects that increase
the value or the her simply because they may reduce the overall ROI the division; (b) EVA
calculations incorporate tax effects that are cross to the firm. 1t, therefore, provides an after-tax
comprehensive summary of the effects of various decisions on the company and its shareholders.
538 Chapter 15
Requirement 4
ROS:
200,000
Northern Luzon= =4 %
5,000,000
750,000
Southern Luzon= =4 %
15,000,000

Illustrative Problem 15-2: Comparative Performance Analysis


Return on investment (ROI) is often expressed as follows:
Income Revenue Income
= x
Investmen Investment Revenues
Comment on the relative performance of the following companies as thoroughly as the data
permit:

Companies in Same Industry


A B C
Revenues 1,000,000 500,000 10,000,000
Income 100,000 50,000 50,000
Investment 500,000 5,000,000 5,000,000
Income as a
percentage of 10% 10% 5%
revenue
Investment 2x 1x 2x
turnover
ROI 20% 1% 1%

In this case, income and investment also shed little light on comparative performances
because of disparities in size between Company A and the other two companies. Thus it is
impossible to say whether B's low return on investment in comparison with A's is attributable to
its larger investment or to its lower income. In addition, the fact that Companies B and C have
identical income and investment may suggest that the same conditions underlie the low ROI but
this conclusion is erroneous. B has higher margins but a lower innermost turnover. Chas very
small margins (1/20 of B) but turns over investment 20 times faster.
Financial and Nonfinancial Performance Measures 539
PERFORMANCE MEASUREMENT IN MULTINATIONAL, COMPANIES

Companies that operate in several countries are called multinationals. Some


multinationals organized and operate divisions in foreign countries while others set up
corporations, transition multinationals face special problems related to evaluation of performance
and transfer pricing. It is also difficult to compare performance of a manager of a division in
another country. Even discussions that seem to be similar in terms of production may face
different economic social or political force. International environmental conditions may be very
different from and complex das, domestic conditions. Other environmental variables facing local
managers or divisions include economic, legal, political, social, and educational factors.

Economic variables include currency restrictions, taxes, transfer prices, capital markets
and inflation. Legal and political factors include quality, efficiency and effectiveness of legal
structure, defense policy; foreign policy, degree of government control of business and level of
political unrest. Educational variables vary from country to country and could refer to literary
rate, sophistication of accounting system, degree of technical training and management
development programs. Sociological and cultural variables affect how the multinational firm is
treated by the subsidiary's country. These are manifested in a cultural and social diversity, social
attitude toward material gain and cultural attitude toward authority and persons in subordinate
positions and work ethics.

Evaluation of performance of foreign units and their managers can create additional
difficulties for the following reasons:
1. Complication brought about by legal, cultural, political, economic, and social
differences across countries that direct attention to special areas or restrict a local
manager's actions.
2. Multinationals must cope with changes in exchange rates. Issues of inflation and
fluctuations in foreign currency exchange rates affect performance measures and over
which the local manager has no control.
3. Availability of materials and skilled labor, as well as costs of materials, labor and
infrastructure (power, communication, and transportation) may differ significantly
across countries.
4. Companies may have problems in setting transfer prices considering the different tax
structures and currency restrictions of the countries in which their divisions operate.
5. Import controls (6 g. tariffs and custom duties) imposed and limiting selling prices of
a company products may be imposed by government in some countries.
540 Chapter 15
Both EVA (economic value added) and ROI are important means of managerial
performance. They are however both short term measures and managers may be tempted to trade
off short-term benefits at the expense of long-term well-being or the company. To discourage this
shortsighted behavior, additional measures of performance such as market share, customer
complaints, personnel turnover rates, and personnel development should be used.

COMPUTATION OF FOREIGN DIVISION'S ROI IN THE FOREIGN


CURRENCY
Suppose Wen Phils. invests in a Hotel in Guam on December 31, 20X3. The investment
consists mainly of the costs of buildings and furnishing. The following data are also assumed:
Exchange rate on December 31, 20X3 $1=P55
Exchange rate on December 31, 20X4 $1=P56
Average exchange rate during 20X4 [(P55 + P56)/2] $1=P55.50
Investment (total assets) in Guam P55M
Operating income of the Guam Hotel in 20X4 P10M

What is the historical cost-based ROI for the Guam Hotel in 20X3?
P 10 M
ROI= =0.1818∨18.2 %
P 55 M
What is the ROI calculated using dollars?
P 10 M / P 55.50 $ 180,180
ROI= = =18.02 %
P 55 M / P 55 $ 1,000,000
Calculating the ROI in dollars adjusts for changes in exchange rates. It likewise negates
the effects of inflation (if any) because the exchange rates already incorporate whatever effect
inflation has.
Financial and Nonfinancial Performance Measures 541
DISTINGUISHING PERFORMANCE OF MANAGERS FROM PERFORMANCE OF
ORGANIZATION UNITS
Performance evaluation is performed at different levels in the firm: top management, mid -
management, and the operating level of individual employees such as production and sales.
It is important though that the performance evaluation of a manager should be distinguished
from the performance evaluation of that manger's subunit, such as a division or subsidiary of a
company.
In many cases, the most skillful division manager is assigned to a poor-performing unit to
improve or effect a turnaround in its economic return. A favorable outcome cannot be achieved
overnight and the division may take years to show improvement. It may be a mistake to conclude
from the poor performance of the division that the manager is not doing well. Furthermore, the
division's performance may be adversely affected by economic conditions and other constraints
which are beyond the control of the manager.
In evaluating the performance of the manager, the following should be given consideration.
(1) Because of uncertainty in the manager's environment, the evaluator should
recognize that other factors inside and outside the firm also influence the
outcomes of the manager's efforts and abilities. Therefore, the evaluator should
separate the outcome of the manager's actions from the effort and decision-
making skills employed by the manager; that is, separate the performance of the
manager from the performance of the unit or division. Each manager operates in
an environment that is influenced by factors beyond the manager's control -
operating factors such as unexpected and unpreventable machine breakdowns and
external factors such as fluctuations in market prices and demands. The manager's
lack of control means that there is some degree of uncertainty about the
effectiveness of the manager's actions, independent of the efforts and abilities the
manager brings to the job.
(2) The evaluator must include only factors that the manager controls. This concept is
similar to the first principle which separates the manager from the unit, this
second principle excludes known uncontrollable factors from the analysis.
(3) Because of uncertainty and lack of observability, that is, the efforts and decisions
made by the manager are not observable by top management, a risk-averse
manager is improperly biased to avoid decisions with uncertain outcomes. In
contrast, top management would prefer to see some of these relatively risky
decisions implemented because of top management’s greater tolerance for risk.
The manager supplies effort and decision-making skills on the factors and the environment
produces the outcomes. The outcomes are multi-faceted, including financial and nonfinancial
results; earnings, customer satisfaction, operating. efficiency and so on. The accountant prepares
a performance report consisting of financial and nonfinancial measures of the outcomes of the
542 Chapter 15
manager's decision and effort; the performance report goes to top management, which uses it to
determine the manager's compensation.
INTENSITY OF INCENTIVES; FINANCIAL AND NONFINANCIAL
INCENTIVES
Management compensation frequently includes incentives tied to performance.
The objective is to encourage goal congruence, so that managers will act in the best
interests of the firm. Arranging therefore managerial compensation to encourage managers to
adopt the same goals on the overall firm is an important issue. How large should the incentive
component of a manager's compensation be relative to the salary component? To answer this
question, one has to understand how much the performance measure is affected by actions the
manager takes to further the firm's objectives.
Preferred performance measures are those that are sensitive to or change significantly with
the manager's performance. Managers are evaluated based on things they can affect, even if not
completely controllable. The more that companies have sensitive performance measures
available to them, the more they can rely in incentive compensation for their managers.
The advantages of using performance measures that are directly affected by a manager's
efforts encourage the use of nonfinancial measures. The salary component of compensation
dominates when performance measures that are sensitive to manager's actions are not available.
A high salary. component however, does not mean incentives are completely absent. Salary
increases and promotions to higher ranks depend on some overall measure of performance but
the incentives are less direct. The incentive component of compensation is high unit sensitive
performance measures are available and when monitoring the employee's effort is difficult.
Financial and Nonfinancial Performance Measures 543
Questions
1. What is return on investment, and how is it calculated?
2. What are the measurement issues to be concerned about when using return on
investment?
3. What are the advantages and limitations of return on investment?
4. What are the advantages and limitations of residual income?
5. How does the concept of economic valued added compare to return on investment and
residual income?
6. Give two examples of financial performance measures and two examples of nonfinancial
performance measures.
7. What are the six steps in designing accounting-based performance measures?
8. "RI is not identical to ROI although both measures incorporate income and investment
into their computations." Do you agree? Explain.
9. Describe EVA.
10. Give three definitions of investment used in practice when computing ROI.
11. Distinguish between measuring assets based on current cost and historical cost.
12. What special problems arise when evaluating performance in multinational companies?
13. Performance measurement is perhaps the most important, most misunderstood, and most
difficult task in management accounting. What critical performance indicators are
contained in an effective system of performance measurement?
544 Chapter 15
Exercises

Exercise 1 (ROI and Residual Income)

Consider the following data from Coral Financial, Inc. Coral uses investment center
evaluation to analyze its two main divisions - mortgage loans and consumer loans (in millions):

Mortgage Loans Consumer Loans


Total assets P2,000 P10,000
Operating income P400 P1,500
Return on investment 25% 15%

Required:

1. Which division is more successful? Why?


2. Coral uses residual income as a measure of management success. What is the residual
income for each division if the minimum desired rate of return is (a) 11 percent, (b) 15
percent, (c) 17 percent? Which division is more successful under each of these rates?

Exercise 2 (Return on Investment; Comparisons of Three Companies)

Fill in the blanks for this information:

Companies in Same Industry


A B C
Sales P1,500,000 P750,000 P____________
Income 200,000 75,000 ____________
Investment (assets) 500,000 ____________ 2,500,000
Return on sales ____________ 0.5%
Assets turnover ____________ ____________ 1.5
Return on investment ____________ 1% ____________
Exercise 3 (ROI, RI, ROS, Management Incentives)

The Jump-Start Division (JSD) of Magic Industries manufactures go-carts and other
recreational vehicles. JSD is considering building a new plant in 20X3. The investment will cost
P2.5 million. The expected revenues and costs for the new plant in 20X3 are:

Revenues P2,400,000
Variable costs 800,000
Fixed costs 1,120,000
Operating income P ,480,000

JSD'S ROI in 20X2 is 24% and its return on sales (ROS) is 19 percent. ROI is defined as
operating income divided by total assets. The bonus of Jayce Tan, the division manager of JSD,
is based on division ROI.
Required:
1. Explain why Tan would be reluctant to build the new plant. Show your computations.
2. Suppose Magic Industries used RI to determine Tan's bonus. Suppose further that the
required rate of return on investment is 15 percent. Would Tan be more willing to build
the new plant? Explain.
3. Suppose Magic Industries used ROS to determine Tan's bonus. Would Tan be more
willing to build the new plant? What are the advantages and disadvantages of using ROS
to determine Tan's bonus?
Problems
Problem 1 (RI, EVA)
The Lighthouse Transport Company operates two divisions, a Truck Rental Division that
rents to individuals, and a Transportation Division that transports goods from one city to another.
Results reported for the last year are as follows:
Truck Rental Division Transportation Division
Total assets P650,000 P950,000
Current liabilities 120,000 200,000
Operating income 75,000 160,000

Required:
1. Calculate the RI for each division using operating income and investment equal to total
assets minus current liabilities. The required rate of return on investment is 12 percent.

2. The company has two sources of funds: long-term debt with a market value of P900,000
at an interest rate of 10% and equity capital with a market value of P600,000 at a cost of
equity of 15 percent. Lighthouse's income tax rate is 40 percent. Lighthouse applies the
same weighted-average cost of capital to both divisions, since each division faces similar
risks. Calculate the EVA for each division.
3. Using your answers to requirements I and 2, what would you conclude about the
performance of each division? Explain briefly.

Problem 2 (ROI, RI, Measurement of Assets)


Park Corporation recently announced a bonus plan to be awarded to the manager of the most
profitable division. The three division managers are to choose whether ROl or RI will be used to
measure profitability. In addition, they must decide whether investment will be measured using
gross book value or net book value of assets. Park defines income as operating income and
investment as total assets. The following information is available for the year just ended:

Division Gross Book Value Accumulated Division Operating


Assets Depreciation Income
S P800,000 P430,000 P94,700
P 760,000 410,000 91,700
F 500,000 280,000 61,400

Park uses a required rate of return of 10% on investment to calculate RI.


Required:
Each division manager has selected a method of bonus calculation that ranks his or her
division Number I. Identify the method for calculating profitability that each manager selected,
supporting your answer with appropriate calculations.
Problem 3 (Multinational Performance Measurement, ROI, RI)
The Angeles Corporation manufactures electric motors in the Philippines and Sweden,
The Philippine and Swedish operations are organized as decentralized divisions. The following
information is available for 2013 where ROI is calculated as operating income divided by total
assets:
Philippine Division Swedish Division
Operating Income ? 9,180,000
Total Assets 8,000,000 60,000,000
ROI 15% ?

The exchange rate at the time of Angeles investment in Sweden on December 31, 20X2
was & kronas - PI. During 20X3, the Swedish krona decline steadily in value so that the
exchange rate on December 31, 20X3 is 9 kronas =Pl. The average exchange rate during 2013 is
[(8 + 9) + 2] - 8,5 kronas = PI.
Required:
1. a. Calculate the Philippine division's operating income for 20×3.
b. Calculate the Swedish division's ROl for 20X3 in kronas.
2. Senior management wants to know which division earned a better ROl in
20X3. What would you tell them? Explain your answer.
3. Which division do you think had the better Ri performance? Explain your answer. The
required rate of return on investment (calculated in Phil. peso) is 12 percent.

Problem 4 (ROI Performance Measures Based on Historical Cost and Current Cost)
Sparkling Mineral Water Ltd. operates three divisions that process and bottle sparkling
mineral water. The historical-cost accounting system reports the following information for 20X9:
Luzon Division Visayas Division Mindanao Division
Revenues P500,000 P 700,000 P 1,100,000
Operating costs
(excluding depreciation) 300,000 380,000 600,000
Plant depreciation 70,000 100,000 120,000
Operating income P130,000 P220,000 P380,000
Current assets P200,000 P250,000 P300,000
Long-term assets – plant 140,000 900,000 1,320,000
Total assts P340,000 P1,150,000 P1,620,000

Sparkling Mineral Waters estimates the useful life of each plant to be 12 years with a zero
terminal disposal price. The straight-line depreciation method is used. At the end of 20X9, the
Luzon plant is 10 years old, the Visayas plant is 3 years old, and the Mindanao plant is I year old.

An index of construction costs for the Un-year period that Sparkling Mineral Waters has
been operating (20x0 year-end=100) is;
20X0 20X6 20X8 20X9
100 136 160 170
Given the high turnover of current assets, management believes that the historical-cost and
current-cost measures of current assets are approximately the same.

Required:
1. Compute the ROI ratio (operating income to total assets) of each division using historical-cost
measures. Comment on the results.
2. Compute the ROI of each division, incorporating current-cost estimates as of 20X9 for
depreciation and long-term assets. Comment on the results.
3. What advantages might arise from using current-cost asset measures as compared with
historical-cost measures for evaluating the performance of the managers of the three divisions?

Multiple Choice
1. A report that measures financial and nonfinancial performance measures for various
organization a. units in a single report is called a(n)
a. balanced scorecard.
b. financial report scorecard.
c. imbalanced scorecard.
d. unbalanced scorecard.

2. Customer-satisfaction measures are an example of:


a. Goal-congruence approach.
b. balanced scorecard approach.
c. financial report scorecard approach.
d. investment success approach.

3. An example of a performance measure with a long-run time horizon


a. is direct materials efficiency variances.
b. is overhead spending variances.
c. is number of new patents developed.
d. include all of the above measures.

4. Should assets be defined as total assets of net assets? This question considered part of
which step in designing an accounting-based
a. Choose performance measures that align with top management's financial goals.
b. Choose the time horizon of each performance measure.
c. Choose a definition for each performance measure.
d. Choose a measurement alternative for each performance measure.

5. Should assets be measured at historical cost or current cost? This based performance
measure question is considered part of which step in designing an accounting-based
performance measure?
a. Choose performance measures that align with top management's financial goals.
b. Choose the time horizon of each performance measure.
c. Choose a definition for each performance measure.
d. Choose a measurement alternative for each performance measure.

6. Which of the following statements about designing an accounting-based performance


measure is false?
a. The steps may be followed in a random order.
b. The issues considered in each step are independent.
c. Management's beliefs are present during the analyses.
d. Behavioral criteria are important when evaluating the steps.

7. The return on investment is usually considered the most popular approach to


incorporating the investment base into a performance measure because
a. it blends all the ingredients of profitability into a single percentage.
b. once determined, there is no need to use it with other measures of performance.
c. it is similar to the company's price earnings ratio in that a corporation's return on
investment appears every day in The Wall Street Journal.
d. of both (a) and (c).

8. During the past twelve months, the Seven Corporation had a net income of P39.200.
What is the return on investment if the amount of the investment is P280,000?
a. 10%
b. 12%
c. 14%
d. 16%

9. The Delta Corporation had the following information for 20X3.


Revenue P 900,000
Operating expenses 670,000
Total assets 1,150,000

What is the return on investment?


a. 10% c. 25%
b. 20% d. 78.2%
The following information applies to questions 10 through 12.

The HZL Corporation reported the following information for its Pasig Division:
Revenue P 1,000,000
Operating costs 600,000
Taxable income 200,000
Operating assets 500,000
Income is defined as operating income.

10. What is the Pasig Division’s investment turnover ratio?


a. 2.00 c. 2.50
b. 3.33 d. 0.80

11. What is the Pasig Division’s return on sales?


a. 0.2 c. 0.5
b. 0.4 d. 0.6

12. What is the Pasig Division’s return on investment?


a. 0.2 c. 0.5
b. 0.4 d. 0.8

13. A problem with utilizing residual income is that


a. A corporation with a high investment turnover ratio always has a higher residual
income than a corporation with a smaller investment turnover ratio.
b. A corporation with a high return on sales always has a higher residual income than a
corporation with a smaller return on sales.
c. A corporation with a larger peso amount of assets is likely to have a higher residual
income than a corporation with a smaller peso amount of assets.
d. none of the above are correct.

14. A company which favors the residual income approach


a. wants managers to concentrate on maximizing an absolute amount of pesos.
b. wants managers to concentrate on maximizing a percentage return.
c. wants managers to maximize the investment turnover ratio.
d. wants managers to maximize return on sales.
The following information applies to questions 15 through 17.
The Globe Medical Supply Company has two divisions that operate independently of one
another. The financial data for the year 20X3 reported the following results:
Uptown downtown
Sales P3,000,000 P2,500,000
Operating income 750,000 550,000
Taxable income 650,000 375,000
Investment 6,000,000 5,000,000

The company's desired rate of return is 10%. Income is defined as operating income.
15. What are the respective return-on-investment ratios for the Uptown and Downtown
Divisions?
a. 0.110 and 0.125
b. 0.108 and 0.075
c. 0.125 and 0.110
d. 0.050 and 0.150

16. What are the respective residual incomes for the Uptown and Downtown Divisions?
a. P30,000 and P50,000
b. PI50,000 and P30,000
c. P150,000 and P50,000
d. P50,000 and a negative P150,000

17. Which division has the best return on investment and which division has the best residual
income figure, respectively?
a. Uptown, Uptown.
b. Downtown, Downtown
c. Uptown, Downtown
d. Downtown, Uptown

18. A negative feature of defining finance by excluding the portion of total assets employed that
are financed by short term creditors is
a. current liabilities are sometimes difficult to define.
b. short term debt is always more expensive to finance than long dem
c. this method encourages managers to use an excessive amount of short-term debt.
d. this method encourages managers to use an excessive amount of long-term debt.
The following information applies to questions 19 through 21.
Federov Company has two sources of funds: long-term debt with a market and
book value of PI0 million issued at an interest rate of 12%, and equity capital that has a
market value of P8 million (book value of P4 million). Federov Company has profit
centers in the following locations with the following operating incomes, total assets, and
total liabilities. The cost of equity capital is 12%, while the tax rate is 25%.
Operating income Current liabilities
Assets
Tarlac P960,000 P4,000,000 P200,000
Quezon P1,200,000 P8,000,000 P600,000
Manila P2,040,000 P12,000,000 P1,200,000

19. What is the EVA for Tarlac?


a. P255,740
b. P327,460
c. P392,540
d. P720,000

20. What is the EVA for Quezon?


a. 135,580
b. P220,000
c. P234,000
d. P305,000

21. What is the EVA for Manila?


a. P 450,000
b. P1,530,000
c. P 414,360
d. P1,115,640

The following information applies to questions 22 through 24


P Jones Cleaning products, manufactures home cleaning products. The Company
has two divisions, Bleach and Cleanser. Because of different accounting methods and
inflation rates, the company is considering multiple evaluation measures. The following
information is provided for 2013;
Assets Income
Book value Current value Book value Current value
Bleach P225,000 P300,000 P150,000 P155,000
Cleanser P450,000 P250,000 P100,000 P105,000
The company is currently using 15% required rate of return.
22. What are Bleach’s and Cleanser’s return on investment based on book values,
respectively?
a. 0.22; 0.67 c. 0.52; 0.42
b. 0.42; 0.52 d. 0.67; 0.22
23. What are Bleach’s and Cleanser’s return on investment based on current values,
respectively?
a. 0.22; 0.67 c. 0.52; 0.42
b. 0.42; 0.52 d. 0.67; 0.22
24. What are Bleach’s and Cleanser’s residual incomes based on book values, respectively?
a. P116,250; P 32,500 c. P 67,500; P110,000
b. P110,000; P 67,500 d. P 37,500; P116,250
25. If a company is a multinational company with operations in several different countries,
one way to achieve comparability of historical-cost based on ROIs for facilities in
different countries is a.
a. restate the results of operations using the cash basis method of accounting.
b. use GAAP for all reporting and calculations.
c. restate the results of all operations in pesos.
d. all of the above would achieve
comparability.
26. Which of the following statements is true?
a. The economic, legal, political, social and cultural environments differ across
countries.
b. Governments in some countries may impose controls and limit selling prices of a
company's products.
c. Because of advances in telecommunications and transportation, the availability of
materials, and skilled labor does not
differ significantly across countries.
d. Both (a) and (b) are correct.
CHAPTER
EXECUTIVE PERFORMANCE

17 MEASURES AND
COMPENSATION
__________________________________________________________

27. Tying performance measures more closely to a manager's efforts


a. encourages the use of nonfinancial measures.
b. b. results in a strict use of financial ratios.
c. c. results in the salary component of compensation dominating the total
compensation package.
d. d. includes both (a) and (c).

28. Team incentives encourage cooperation by


a. forcing people to work together on difficult tasks.
b. improving morale.
c. letting individuals help one another as they strive toward a common goal.
d. rewarding all teams the same amount.

29. Many manufacturing, marketing, and design problems require employees with multiple
skills; therefore, teams are used and the members have the added encouragement of
a. individual incentives.
b. management incentives.
c. morale incentives.
d. team incentives.

30. Designers of executive compensation plans emphasize which of the following factors?
a. Achievement of organizational goals
b. Administrative case
c. The probability that the executives affected by the plan will perceive the plan as fair
d. All of the above are emphasized.

31. Rate of sales growth and number of customers are two examples of critical performance
indicators for which of the following critical success factors?
a. Service
b. Quality
c. Cost
d. Revenue
cha

◆◆◆
CHAPTER MANAGING PRODUCTIVITY
AND MARKETING
16 EFFECTIVENESS

EXPECTED LEARNING OUTCOMES


After studying this chapter, you should be able to…
1. Describe productivity

2. Compute and interpret partial operational and financial productivity

3. Compute and interpret total productivity

4. Understand the components of sales variance to assess marketing effectiveness

a. Sales rice variance


b. Sales volume variance
c. Sales mix variance
d. Sales quantity variance
e. Market size variance
f. Market share variance

CHAPTER 16
MANAGING PRODUCTIVITY AND
MARKETING EFFECTIVENESS
Managing Productivity

Sustaining profitability and maintaining or improving market share requires


effective marketing activities. Effectiveness in marketing however demands
proper consideration of factors such as selling price, sales volume, and market
price, market share and productivity.

Being able to produce more with less resources is the story behind
progress and success. This is strategic critical success factor that all firms and
organizations regardless of their chosen competitive strategy strive to attain.
Productivity has become the wealth not only of the business enterprise but of
nations as well.

Improvements in productivity is achieved when fewer workers, materials,


machines or other resources are used to manufacture and sell the same or better
product. Among the benefits that higher productivity brings about to business
firms are
(1) competitive advantages,
(2) higher-than-average returns. earnings and
(3) attainment of long-tern success,

Measuring Productivity

A productivity measure that includes all input resources used in production


is a total productivity. For example, the number of tables manufactured per
peso of manufacturing costs is a total productivity measure because the
denominator includes all manufacturing costs incurred to make the tables. Figure
16-1 summarizes productivity measures.

Figure 16-1 Productivity Measures

Partial productivity
Partial operational productivity
Productivity
Partial financial productivity
Total productivity (financial productivity)
Productivity is the ratio of output to input.

Output
Productivity =
Input

A company that spends five (5) days to manufacture 200 units has a
productivity of 40 units per day. To improve productivity, firms need to know the
productivity levels of their operations.

A productivity measure is often compared to the performance of a prior


period another firm, the industry standard or a benchmark in assessing a firm’s
productivity.

Measures of productivity are applicable to all organizations including


service firms and noy-for-profit organizations. However, imprecise measure of
output, lack of definite relationships between output and input resources, or
absence of revenue for not-for-profits may limit the usefulness of productivity
measures for service or not not-for-profit organization.

A measure of productivity can be either an operational or financial productivity measure:

Output units
Operational Productivity =
Input units

₱ Output
Financial Productivity =
₱ Input

Partial productivity relates one of part of the input factor to the output.
Basic Formula is:
Number of units or value of output manufactured
Number of units or cost of single or part of the input resources

Examples:
Output
Direct materials yield=
Input materials
Workforce productivity:
Output
Output per labor−hour=
Input of labor hours

Output
Output per person employed =
No . of labor force
Output
Process ( activity ) productivity=
Machine hours used
PARTIAL PRODUCTIVITY

A partial productivity measures the relationship between the output and one part
of the required input resources used in producing the output. The higher the ratio is, the
better. It is computed as follows:
Number of units∨value∨¿ output manuactured
∂ Productivity =
Number of units∨cost of a single
¿ part of theinput resources

The numerator is the number of units or value of the goods or services


produced. The denominator is the number or cost of a manufacturing factor such
as direct materials, direct labor-hours, or selected input resources.

A partial operational productivity is the required physical amount of an input


resource to produce one unit output, while a partial financial productivity of an input
resource is the number of units or the value of output manufactured for each peso spent
on the input resource.

Figure 16-2 presents the partial operational productivity and partial financial productivity
of Press Tool Company in 20x4 and 20x5.
Figure 16-2

Press Tool Company


Partial Productivity
Direct Materials and Direct Labor
for XOX

Partial Operational Productivity


20X4 20X5
Direct Materials 8,000 / 50,000 = 0.16 9,600 / 64,000 = 0.15
Direct Labor 8,000 / 8,000 = 1.00 9,600 / 8,000 = 1.20
Partial Financial Productivity
20X4 20X5
Direct Materials 8,000 / 1,200,000 = 0.0067 9,600 / 1,600,000 = 0.006
Direct Labor 8,000 / 320,000 = 0.0250 9,600 / 400,000 = 0.024

Partial Operational Productivity


The partial operation productivity of Press Tool Company n 20X4 indicates that
the firm manufactured 0.16 unit of output for every pound of direct materials used in the
production. Using productivity level in 20X4 as the benchmark to assess productivity in
20X5, the operating results show that the partial operational productivity of the direct
materials decreased to 0.15 or a 6.25% decrease in productivity [(0.16 – 0.15) ÷ 0.16 =
0.0625].
Partial productivity of direct labor, however, improved in 205. The firm
manufactured one unit for each direct labor hours in 2014 and 1.2 units in 20X5, a 20%
increase in productivity [(1.20 – 1.00) ÷ 1 = 0.20].
Changes in productivity also can be examined by computing the amount of input
resources that the firm would have used in 20X5 had it maintained the 20X4 partial
productivity. In this case, the 4,800 units of XOX manufactured and sold in 20X5 would
have required only 30,000 pounds of direct materials (4,800 ÷ 0.16). The decreased
partial productivity necessitated the use of an additional 2,000 pounds in 20X5 (32,000
– 30,000). Similarly, the firm would have spent 4,800 direct labor-hours in 20X5 had it
had the same direct labor partial productivity in 20X5 as in 20X4. The firm saved the
cost for 800 hours of direct labor (4,800 – 4,000) when its partial productivity in 20X5 for
direct labor increased from 1.0 to 1.2.

Partial Financial Productivity


The lower panel of Figure 10-2 reports the partial financial productivities
of direct materials and direct labor. The partial financial productivity indicates the
number of units of output manufactured for each peso the firm spent on the input
resource. The partial financial productivities of direct materials are 0.0067 in
20X4 and 0.006 in 20X5. This indicates a decrease in productivity from 20X4 to
20X5 of 10% [(0.0067 – 0.006] ÷ 0.0067]
The direct labor partial financial productivity is 0 025 in 20X4 and 0.024 for
20X5. a decrease of 4% |(0.025 - 0.024) + 0.025). This result contradicts the
direct labor partial operational productivity reported earlier (20% improvement).
These results indicate that although employee productivity per bour increased,
the cost increase due to higher hourly wages more than offset the gain in
productivity per hour. As a result, total direct labor cost increased and the partial
financial productivity decreased.
Advantages of Partial Productivity Measures
1. It allows managers to focus on the use of a particular input.
2. It is easily interpreted by all within the organization and are easy to use tor assessing
productivity of performance of operating personnel.
3. For operational control, the standard for performance are very often shout. term, say
productivity ratios of prior hatches of goods, and productivity trends within the year can
therefore be backed.
Limitations of Partial Productivity Analysis
Some of the observable limitations of the partial productivity measures are:
(1) It measures only the relationship between an input resource and the output. it
ignores any effect that changes a other manufacturing factors have on the
productivity An improved partial productivity could have been obtained by
decreasing the productivity of one or more other input resources.

(2) It ignores any effect that changes in other production factor have on productivity.
For example, improvement in materials quality are likely to raise the partial
productivity of direct materials as well as direct labor.

(3) It ignores the effects that changes in the firm's operating characteristics have on
the productivity of the input resource. Tor example, installation
of high efficiency equipment improves direct labor partial operational productivity.
The improvement in labor partial operational productivity can hardly be attributed
to increased labor productivity.

(4) An improved partial productivity does not imply that the firm division operates
efficiently. No efficiency standard is involved determination of partial productivity
measures.

Total Productivity

Total productivity shows the relationship between input resources used to


produce the output. Hence,
Units∨Sales Value of Output
Total productivity=
Total Cost of all Input Resources

Total productivity is a financial productivity measure. It can be either the


number of units or the sale value of the output obtained.
For Press Tool Company, Total Productivity for XOX for 20X4 and 20X5
can be calculated from the following data:

Press Tool Company


20X4 and 20X5

Section 1: Total Productivity in Units


20X4 20X5
(a) Total units manufactured 8,000 9,600
(b) Total variable manufacturig cost incurred ₱ 1,520,000 ₱ 2,000,000
(c) Total Productivity [(a)/(b)] 0.005263 0.004800
(d) Decrease in productivity 8.8%

Section 2: Total Productivity in Sales Pesos


20X4 20X5
(a) Total sales ₱ 4,000,000 ₱ 4,800,000
(b) Total variable manufacturig cost incurred ₱ 1,520,000 ₱ 2,000,000
(c) Total Productivity [(a)/(b)] ₱ 2.6316 ₱ 2.4000
(d) Decrease in productivity ₱0.2316
As shown in the table above, total productivity in unis and in sales pesos
both decreased in 20X5 which is not favorable.
The total productivity of all resources required to manufacture the output is
often used in assessing production operation. Achieving higher productivity by
making more units is an important first step for a successful firm. An investment
that generates higher revenue that other investments for each peso it spends on
resources is a good investment.
Benefits and Limitations of Total Productivity Measures
Benefits
Total productivity measures the combined productivity of all operating
factors. It decreases the possibility of manipulating some of the manufacturing
factors to improve the productivity measure of other manufacturing factors.
Limitations
(1) Total productivity is a financial measure and executives at the operational level
may have difficulty linking financial productivity measures to their day-to-day
operations. Furthermore, deterioration in total productivity can result from
increased costs of resources that were beyond the manager's control or
decreased productivity of some of the input resources that were outside the realm
of the manager

(2) The basis for assessing changes in productivity could vary over time, that year,
yearly measures use different years as the base.

(3) It can ignore the effects of changes in demand for the product, changes in selling
prices of the goods and services and special purchasing and selling arrangements
on productivity. For example, special arrangement to sell products at a discount
price decreases the productivity in pesos of output per input unit. On the other
hand, a special purchase of materials increases financial productivity: Neither of
these actions can be attributed to a loss or gain in productivity.
Managing Marketing Effectiveness
No entity can gain success without effective marketing activities that will
enable it to accomplish the following
(a) Earn the projected operating income.
(b) Attain the desired and budgeted market share.
(c) Adapt to market change.
Many factors affect marketing effectiveness. These include changes in selling
prices, sales quantity, product mix, market size and market share. Variances in
any of these factors affect operating results and can prevent a firm from
achieving its short-fen performance objectives and strategic goals. The difference
between the actual sales revenues of a period and the sales revenues in the
master budget is the sales variance of the period, The components of sales
variances follow:
Summary of Variance Analysis to Assess Marketing Effectiveness

Total Sales Variance


Total actual sales Pxx
Less: Total budgeted sales _xx_
(Unfavorable) Favorable Pxx_

Sales Price Variance


Formula:

Significance:
A sales price variance is the difference between the actual peso amount
received from all the units sold and the peso amount the firm would have
received had the firm sold these units at the budgeted selling price per unit.
Sales price variance measures the impact of deviations of the actual
selling prices from the master budgeted selling prices on contribution margin and
operating income. A firm computes its sales price variance by multiplying the
difference between the actual and the budgeted selling prices per unit of the
product by the actual units of the product sold. This variance also is the sales
revenue flexible budget variance.
Sales Volume Variance
Formula:

Significance:
A sales volume (activity) variance is the difference between the budgeted
contribution margin for the actual total units sold (flexible budget contribution
margin) and the budgeted contribution margin for the budgeted units (master
budget contribution margin) This variance measures the effect on contribution
margin and operating income when the quantity sold for one or more products
differs from the quantity in the master budget for the period.
Sales Mix Variance
Formula:

Significance:
The sales mix variance of a product is the product of the difference
between the actual and budgeted sales mix, the actual total units of all products
sold, and the budgeted contribution margin per unit of the product. A product's
sales mix variance measures the effect on contribution margin and operating
income due to the deviation of the actual sales mix from the budgeted sales mix
Sales Quantity Variance
Formula:

Significance:
A sales quantity variance measures the effect on the contribution margin and
operating income due to the deviation of the actual total sales units from the budgeted
total units.
Market Size Variance
Formula:

Significance:
Market size variance measures the effect of changes in the total market-
size on the firm's total contribution margin and operating income. As the size of
the total market for a firm's products changes, the total sales of the firm are likely
to change with it. When the total market size for a firm's products expands, the
total sales of the firm likely would increase A firm that failed to increase its total
sales In proportion to the increase in the total market is riot keeping up with the
market and is losing its marketing position.
Market Share Variance
Formula:

Significance:
Market share variance compares the firm's actual market share to its budgeted market
share and measures the effect of changes in the firm's market share on its to
contribution margin and operating income.
Illustrative Problem 16.1: Sales Volume, Sales Quantity and Sales Mix Variance
Enzo and Gabe operate ice cream stores that sell ice cream in cones in Muntinlupa City
and its suburbs. Its budget and operating data for 20X4 follow:

Budgeted Data for 20X4 Actual Operating Results in 20X4


Selling Variable Selling Variable
Price Per Cost Per Price Per Cost Per
Flavor Pieces Piece Piece Pieces Piece Piece
Vanilla 250,000 P120 P50 180,000 P100 P45
Chocolate 300,000 150 60 270,000 135 50
Strawberry 200,000 180 70 330,000 200 75
Mango 50,000 250 100 150,000 300 120

REQUIRED:
1. Compute for the individual flavors and total sold:
a. Sales volume variances
b. Sales mix variances
c. Sales quantity variances
2. Assess the operation of 20X4 based on your analyses
Answer:
Sales Mix
Budget Actual
Flavor Quantity Mix Quantity Mix
Vanilla 250,000 0.31250 180,000 0.18750
Chocolate 300,000 0.37500 270,000 0.28125
Strawberry 200,000 0.25000 330,000 0.34375
Mango 50,000 0.06250 150,000 0.18750
Total 800,000 1.0000 930,000 1.0000

Requirement 1.a. Sales Volume Variance


Budgeted Sales
Sales Quantity Contribution Volume
Flavor Actual Budget Difference Margin/Unit Variance
Vanilla 180,000 250,000 70,000 U x ₱ 7.00 = ₱ (490,000) U
Chocolate 270,000 300,000 30,000 U x ₱ 9.00 = (270,000) U
Strawberry 330,000 200,000 130,000 F x ₱ 11.00 = 1,430,000 F
Mango 150,000 50,000 100,000 F x ₱ 15.00 = 1,500,000 F
Total 930,000 800,000 ₱ 2,620,000 F

Requirement 1.b. Sales Mix Variance


Total Budgeted Sales
Sales Quantity Actual Contribution Volume
Flavor Actual Budget Difference Quantity Margin/Unit Variance
Vanilla 0.18750 0.31250 0.12500 x 960,000 x ₱ 7.00 = ₱ (840,000) U
Chocolate 0.28125 0.37500 0.09375 x 960,000 x ₱ 9.00 = (810,000) U
Strawberry 0.34375 0.25000 0.09375 x 960,000 x ₱ 11.00 = 990,000 F
Mango 0.18750 0.06250 0.12500 x 960,000 x ₱ 15.00 = 1,800,000 F
Total 1.0000 1.0000 ₱ 1,140,000 F

Requirement 1.c. Sales Quantity Variance


Budgeted Budgeted Sales
Total Sales Quantity Sales Contribution Volume
Flavor Actual Budget Difference Mix Margin/Unit Variance
Vanilla 960,000 800,000 160,000 x 0.31250 x ₱ 7.00 = 350,000 F
Chocolate 960,000 800,000 160,000 x 0.37500 x ₱ 9.00 = 540,000 F
Strawberry 960,000 800,000 160,000 x 0.25000 x ₱ 11.00 = 440,000 F
Mango 960,000 800,000 160,000 x 0.06250 x ₱ 15.00 = 150,000 F
Total 1.0000 ₱ 1,480,000 F

Recap
Flavor Sales Mix Variance Sales Quantity Variance Sales Volume Variance
Vanilla ₱ (840,000) U + ₱ 350,000 F = ₱ (490,000) U
Chocolate (810,000) U + 540,000 F = (270,000) U
Strawberry 990,000 F + 440,000 F = 1,430,000 F
Mango 1,800,000 F + 150,000 F = 1,500,000 F
Total ₱ 1,140,000 F + ₱ 1,480,000 F = ₱ 2,620,000 F

Requirement 2

Overall, the firm has enjoyed a good year. The total sales substantially
exceeds the budgeted amount (20%). The increase in sales could have been a
result of the increase of the entire market size lot gelatin and other competing
merchandises. In any event, the firm still had an excellent operation by selling
more units of the flavors with high contribution margins. The favorable sales mix
variances in two of the flavor suggest that the two flavors with high contribution
margins account for all the increases in sales

Illustrative Problem 16.21: Marker Share, Market Size and Sales Volume Variance

MBR Company produces and sells gold-plated souvenir mugs. It expects


to sell 1,600 units in 20X4 for 743 each to cam a P23 contribution margin per
unit. Tne president expects the total market to be 32,000 units for the year.

In 20X4, the University of the West won the national basketball


championship. MBR sold 3,000 at P75 per unit with P40 in variable costs per
unit. Total market was 100,000 units.

REQUIRED: For MBR

(1) What is the market share variance?


(2) What is the market size variance?
(3) What is the sales volume variance?

Answer:
(1) Market share:
Actual: 3,000 / 100,000 = 3%
Market: 1,600 / 32,000 = 5%
Market share variance = (3% - 5%) (100,000) (P25)
= P (50,000) unfavorable
(2) Market size variance = (100,000 – 32,000) (0.05) (P25)
= P85,000 favorable
(3) Sales Volume variance = (3,000 -1,600) (P25)
= P35,000 favorable
Reconcilation:
Market share variance P50,000 Unfavorable
Market size variance 85,000 Favorable
Sales volume variance P35,000 Favorable
REVIEW QUESTION AND PROBLEMS

Questions
1. Define productivity.

2. Why is it important for a firm that follows a strategy of being the cost leader to
improve productivity?

3. What are the criteria often used to assess productivity? Give their advantages
and disadvantages.

4. Distinguish between operational productivity and financial productivity

5. Distinguish between partial productivity and total productivity.

6. Why is operational productivity measures preferred over financial productivity


measure by manufacturing personnel?

7. Give some important measurements for assessing marketing effectiveness.

8. Distinguish between a sales price variance and a sales volume variance.

9. Distinguish between a market size variance and a market share variance.

10. Do you agree with the following statement? Why? Why not?

a. As long as a firm sells more units than the number of units specified in the
master budget, the firm will not have an unfavorable sales volume variance.

b. A favorable sales quantity variance indicates that the marketing manager has
done a good job.
c. An improvement in earnings growth can be achieved at the expense of
market shares (indicated by and unfavorable market share variance).

11. What are the relationships between a market size variance, a market share
variance, a sales quantity variance, and a sales volume variance?

Problems

Problem 1 (Operational and Financial Partial Productivity)

In the fourth quarter of 20X3 star company embarked on a major drive to


improve productivity. The drive included redesigned products, reengineered
manufacturing processes, and productivity improvement courses. The drive was
completed in the last quarter of 20X4. The controller”s office has gathered the
following year-end data for the assessment of the drive.
20X3 20X4
Units manufactured and sold 15,000 18,000
Selling price of the product P 40 P 40
Materials used (pounds) 12,000 12,600
Cost per pound of materials P 8 P 10
Labor-hours 6,000 5,000
Hourly wage rate P 20 P 25
Power (kwh) 1,000 2,000
Cost power per kwh P 2 P 2

Required:
1. Prepare a summary contribution-approach income statement for each year and
calculate the total exchange in profits.
2. Compute the operational partial productivity ratios for each of the production
factors for 20X3 and 20X4.
3. Compute the financial partial productivity ratios for each of the production factors
for 20X3 and 20X4.
4. On the basis of the operational and financial partial productivity you computed,
what conclusions can you draw on the productivity of the firm in 20X4 relative to
20X3.
5. Separate the changes in the financial partial productivity ratios from 20X3 to
20X4 into productivity changes, input price changes, and output changes.
6. Discuss additional insights on the relative productivity between 20X3 and 20X4
into the detailed information provided by the separations of the change in the
financial partial productivity ratios.

Problem 2 (Direct Labor Rate and Efficiency, Variances, Productivity Measure, and
Standard Cost)

Texas manufacturing Inc. assembles industrial testing instruments. The


firm has two departments: assembly and testing. Operating data for 20X3 and
20X4 are

20X3 20X4
Assembly department:
Actual direct-labor-hours per instrument 25 20
Actual wage rate per hour P30 P36
Standard direct-labor-hours per instrument 24 21
Standard wage rare per hour P28 P35
Testing department:
Actual direct-labor-hours per instrument 12 10
Actual wage rate per hour P20 P24
Standard direct-labor-hours per instrument 14
11
Standard wage rate per hour P21
P25

The firm assembled and tested 20,000 instruments in both 20X3b and 20X4

Required.
1. Calculate the direct labor rate and efficiency variances for both years at both
departments.
2. Compute the direct labor operational partial productivities in both years for both
departments.
3. Determine the financial partial productivity for both department for both years.
4. Compare your answer for requirements 2 and 3. Comment on your results.
5. How do the productivity measures differ from the variance analysis? Do they offer
different perspectives for strategic decisions of the firm?
Problem 3 (Sales Variance)

Arrow Brewery has two main products – premium and regular beer. Its operating
results and masted budget for 20X3 (000 omitted) are

Operating Results of 20X3 Master Budget for 20X3


Premium Regular Total Premium Regular Total
Barrels 180 540 720 240 360 600
Sales ₱ 28,800 ₱ 62,100 ₱ 90,900 ₱ 36,000 ₱ 43,200 ₱ 79,200
Variable expenses 16,200 40,500 56,700 21,600 27,000 48,600

Contribution margin ₱ 12,600 ₱ 21,600 ₱ 34,200 ₱ 14,400 ₱ 16,200 ₱ 30,600


Fixed expenses 10,000 5,000 15,000 10,000 5,000 15,000
Operating income ₱ 2,600 ₱ 16,600 ₱ 19,200 ₱ 4,400 ₱ 11,200 ₱ 15,600

Sam Kortes CEO, estimated at the she prepared the masted budget that total
industry sales would be 1,500,000 barrels during the period. After the years was
over Mar Gopez, the controller, reported the total industry sales were 1,600,000
barrels.
Required: Calculate the

1. Selling price variances for the period for each of the products and for the firm.
2. Sales volume variances for the period for each of the products and for the firm.
3. Sales quantity variances for the firm and for each of the products and for the firm.
4. Sales mix variance for the period for each of the products and for the firm.
5. The sum of the sales quantity variance and sales mix variance and verify that this
total equals the sales volume variances.
6. Market size variances.
7. Market share variances.
8. The sum of market size variance and market share variance and verify that this
total equal the sales quantity variance.

Problem 3 (Productivity and Ethics)

Francis Interiors installs custom interiors for luxury mobile homes. In its
most recent negotiation with the union, the firm proposed to share equally
productivity gain in direct labor. In return the union agreed not to demand wage
increases. Most unions members, however, are very skeptical about the honesty
of the management in calculating the productivity measures. Nevertheless, union
members voted to give the program a try. Nic Tan, the management accountant
responsible for determining productivity measures, collected these data at the
end of 20X3.
20X3 20X4
Number of installations 560 500
Direct labor-hours 112,000 99,000

Kel Juan, the CEO, is very anxious to demonstrate the firm’s good
intentions by showing the labor union a positive result. He suggests to Tan that
some of the direct labor-hours are actually indirect. He believes that the hours
spent on details are indirect because these hours cannot be allocated to specific
types of work. Following his suggestion, Tan clarifies 12,000 hours as indirect
labor.

Required:

1. Evaluate whether Juan's suggestion to reclassify some of the direct labor-hours


as indirect labor is ethical.
2. Would it be ethical for Tan to modify his calculations?

Problem 5 (Small Business Market Size and Share Variances)

Empress Designs is a small business


Master Budget
Welcome Signs Doghouse Signs Total
Units 50 25 75
Sales ₱ 1,000 ₱ 250 ₱ 1,250
Variable costs 900 120 1,020
Contribution margin ₱ 100 ₱ 130 ₱ 230
Fixed costs 75 75 150
Operating income ₱ 25 ₱ 55 ₱ 80
run out of
the house Actual Results of its owner. For the past six
Welcome Signs Doghouse Signs Total
Units 45 35 80
Sales ₱ 675 ₱ 420 ₱ 1,095
Variable costs 580 270 850
Contribution margin ₱ 95 ₱ 150 ₱ 245
Fixed costs 75 75 150
Operating income ₱ 20 ₱ 75 ₱ 95
months, the company has been selling two products, welcome signs and
doghouse signs. The owner has become concerned about the market
effectiveness of the company. The master budget and actual results for the
month of March 20X3 are

The industry totals for welcome signs are 3,000 budgeted and 2,550 actual.

Required:

1. Show a comparison of Empress Designs’ market shares.


2. What is the market share variance?
3. What is the market size variance?
4. Explain possible reasons for these variances.
5. How might Empress Designs improve the future?

Multiple Choice

Questions 1 through 6 are based on the following data:

Rosales Automotive Company manufactures fuel-injection systems. It


manufactured and sold 60,000 units in 20X3 and 64,000 units in 20X4 at P25 per
unit. In 20X3 the firm used 75,000 pounds of alloy X-45 at P7.20 per pound and
spent 10,000 direct labor-hours at an hourly wage rate of P30. In 20x4 the firm
used 89,600 pounds of alloy X-45 at P6.80 per pound and spent 10,847 direct
labor-hours at an hourly wage rate of P32. The total amount for all other
expenses remains the same at P450.000 each year, Paolo Rosales, CEO, was
disappointed that while the total sales increased, the P195,616 operating income
earned in 20X4 is only 93 percent of the amount earned in 20X3, which was
P210,0000.

1. The operational partial productivity of direct materials for 20X3 and 20X4 are
20X3 20X4
a. 0.8 0.7143
b. 0.1111 0.105
c. 6.0 5.9002
d. 0.2 0.1844
2. The operational partial productivity of direct labor for 20X3 and 20X4 are
20X3 20X4
a. 0.8 0.7143
b. 0.1111 0.105
c. 6.0 5.9002
d. 0.2 0.1844

3. The financial partial productivity of direct material for 20X3 and 20X4 are
20X3 20X4
a. 0.8 0.7143
b. 0.1111 0.105
c. 6.0 5.9002
d. 0.2 0.1844

4. The financial partial productivity of direct labor for 20X3 and 20X4 are
20X3 20X4
a. 0.8 0.7143
b. 0.1111 0.105
c. 6.0 5.9002
d. 0.2 0.1844

5. The total productivity for 20X3 as measured in both units and sales pesos are
Units n
a. 0.071429 P1.7857
b. 0.066919 P1.6730
c. 0.004510 P0.1127
d. 0.071249 P1.7587

6. The decrease in productivity in 20X4 as measured in both units and sales pesos
are
Units n
a. 0.071429 P1.7857
b. 0.066919 P1.6730
c. 0.004510 P0.1127
d. 0.071249 P1.7587

7. Patrick, Inc. provided the following information for a production factor:

Budgeted production 10,000 units


Actual production 9,500 units
Budgeted input 9,750 gallons
Actual input 8,950 gallons

What is the operational partial productivity ratio of the production factor?

a. 0.97 units per gallon


b. 1.02 units per gallon
c. 1.06 units per gallon
d. 1.12 units per gallon

Questions 8 and 9 are based on the following data:

ETC Corporation makes small parts from steel alloy sheets ETC’s
management has some ability to substitute direct materials for direct
manufacturing labor. If workers cut the steel carefully, ETC can manufacture
more parts out of a metal sleet, but this requires more directs manufacturing
labor-hours. Alternatively, ETC can use fewer direct manufacturing labor-hours if
it is willing to tolerate more waste of direct materials. ETC provides this
information for the years 20X3 and 20X4:

20X3 20X4
Output units 400,000 486,000
Direct manufacturing labor-hours used 10,000 13,500
Wages per hour ₱ 26 ₱ 25
Direct materials used 160 tons 180 tons
Direct materials cost per ton ₱ 3,375 ₱ 3,125

8. The financial partial productivity for direct materials for 20X4 is


a. 0.7407. c. 1.5385.
b. 0.864. d. 1.44.

9. The total productivity in units for 20X3 is


a. 0.864. c. 0.5.
b. 0.7407. d. 0.54.

Questions 10 and 12 are based on the following data:

Ching Company manufactures one product. Its budgeted and operating results for 20X3
are:
Budgeted Actual
Units sold 90,000 100,000
Unit contribution margin ₱ 8.00 ₱10.00
Unit selling price ₱ 20.00 ₱21.00

Industry volume was estimated to be 1,500,000 units at the time the budget was
prepared. Actual industry volume for the period was 2,000,000 units.

10. The market size variance is


a. P160,000 U. c. P240,000 U.
b. P 80,000 U. d. P240,000 F.

11. The market share variance is


a. P160,000 U. c. P240,000 U.
b. P 80,000 U. d. P240,000 F.

12. The sales quantity variance is


a. P160,000 U. c. P240,000 U.
b. P 80,000 U. d. P240,000 F.

Questions 13 and 21 are based on the following data:

Cai & Ltd., manufactures two products that sell to the same market. Its budget and
operating results for 20X3 are:
Budgeted Actual
Unit sales
Product A 30,000 35,000
Product B 60,000 65,000
Unit contribution margin
Product A ₱ 4.00 ₱ 3.00
Product B ₱10.00 ₱12.00
Unit selling price
Product A ₱10.00 ₱12.00
Product B ₱25.00 ₱ 24.00

Industry volume was estimated to be 1,500,000 units at the time the budget was
prepared. Actual industry volume for the period was 2,000,000 units.

13. The budgeted average unit contribution margin is


a. P 8.00. c. P 10.00.
b. P 4.00. d. P 7.50.

14. The sales volume contribution margin variance for each product is
a. P70,000 U. c. P50,000 F.
b. P70,000 F. d. P20,000 U.
15. The sales mix contribution variances for each product is
Product A Product B
a. P 6,667 F P 16,667 F
b. P 6,667 U P 16,667 U
c. P 6,667 F P 16,667 U
d. P 6,667 U P 16,667 U

16. The sales quantity contribution margin variance for each product is
Product A Product B
a. P 13,333 U P 66,667 U
b. P 13,333 F P 66,667 U
c. P 13,333 U P 66,667 F
d. P 13,333 F P 66,667 F

17. The market size contribution margin is


a. P240,000 U.
b. P240,000 F.
c. P160,000 U.
d. P160,000 F.
18. The market share contribution margin variance is
a. P240,000 U.
b. P240,000 F.
c. P160,000 U.
d. P160,000 F.

19. The total flexible budget contribution margin variance is


a. P950,000 F.
b. P950,000 U.
c. P130,000 F.
d. P130,000 U.

20. The total variable cost price variance if the total contribution margin price
variance is P50,000 favorable is
a. P50,000 F.
b. P5,000 F.
c. P5,000 U.
d. P45,000 F.

21. The total variable cost efficiency variance if the total contribution margin price
variance is P50,000 favorable is
a. P45,000 F.
b. P45,000 F.
c. P50,000 F.
d. P45,000 U.

Questions 22 and 24 are based on the following data:

Chips Galore sells two RISC chips to small machine tool manufacturers: R66 and R100.
Pertinent data for 20X3:
Budgeted Actual
R66 R100 R66 R100
Selling price per chip ₱ 50 ₱ 160 ₱ 55 ₱ 155
Variable cost per unit 40 90 43 95
Contribution margin ₱ 10 ₱ 70 ₱ 12 ₱ 60
Fixed cost per chip 6 30 5 25
Operating income ₱ 4 ₱ 40 ₱ 7 ₱ 35
Sales in units 1,200 400 1,000 1,000
22. What is the r66 sales quantity variance?
a. P400 F
b. P1,000 F
c. P1,200 F
d. P3,000 F

23. What is the R100 sales mix variance?


a. P20,000 F
b. P30,000 F
c. P35,000 F
d. P40,000 F

24. What is the total sales volume variance?


a. P10,000 F
b. P12,400 F
c. P22,000 F
d. P40,000 F
CHAPTER
EXECUTIVE PERFORMANCE

1 MEASURES AND COMPENSATION

7
__________________________________________________________

EXPECTED LEARNING OUTCOMES


After studying this chapter, you should be able to…

7. Enumerate the objectives of management compensation

8. Understand cash and noncash compensation

9. Explain the three aspects of a bonus plan: the base for determining
compensation, the compensation pool from which the bonus is funded, and
the bonus payment options
CHAPTER 17
EXECUTIVE PERFORMANCE MEASURES
AND COMPENSATION

Recruiting, motivating, rewarding and retaining effective managers are critical to the success of
all firms. An important and integral part of the determination of a strategic competitive
advantage of a firm is an effective management compensation plan.

Objectives of Management Compensation

The firm's key objective is to develop management compensation plans that support its strategic
objectives, as set forth by management and the owners. The objectives of management
compensation are therefore consistent with the three objectives of management control:

1. To motivate managers to exert a high level of effort to achieve the goals set by top
management.
2. To provide the incentive for managers, acting autonomously, to make decisions consistent
with the goals set by top management.
3. To determine fairly the rewards earned by managers for their effort and skill and the
effectiveness of their decision making.

Executive Performance Measures and Compensation

Studies show that division manager's compensation arrangements include a mix of salary,
bonuses and long-term compensation tied to earnings and stock price of the company such as
stock options and noncash compensation. The goal of such compensation arrangements is to
balance division and company wide incentives, as well as short-term and long-term incentives.

One survey of companies reported the average annual incentive component of compensation as
follows:
(1) Average annual cash and stock compensation based on long-run performance equal to
57% of current salary, and
(2) Bonuses based on short-run performance equal to 40% of current salary.

These percentages vary widely over the sample; some firms use stronger performance incentives
than others.
Cash Compensation
Cash compensation includes salaries and bonuses. A company may reward good managerial
performance by granting periodic raises. Salary raises, once affected, are however usually
permanent while bonuses give a company more flexibility. Many companies use a combination
of salary and bonus to reward performance by keeping salaries fairly level and allowing bonuses
to fluctuate with reported income. Of course, income-based compensation can encourage
dysfunctional behavior such as the manager may engage in unethical practices like
(1) Postponing needed maintenance, or
(2) Postponing revenue recognition at the end of the year in which maximum bonus has
already been achieved to the next year.

Noncash Compensation
Noncash compensation is also important. Some managers may trade off increases in salary for
improvement in title, office location and trappings, use of expense accounts or use of corporate
country club facilities and so forth. Autonomy in the conduct of their daily business can also
make the manager efficient and an important perquisite (a type of fringe benefit over and above
salary).

Stock options which give executives the right to buy company stock at a specified price (usually
lower than market price) within a specified period, are often used to motivate executives to
improve the company's long-run performance to increase the stock price.

Designers of executive or manager's compensation plans emphasize three factors:

(1) achievement of organization goals,


(2) ease of administering the plans, and
(3) ensuring that affected executives perceive the plan.

Bonus Plans

As stated earlier, bonus compensation is the fastest growing element of total compensation and
often the largest part. A wide variety of bonus pay plans can be categorized according to the three
key aspects:
 The base of the compensation, that is, how the bonus pay is determined. The three most
common bases are (1) stock price, (2) cost, revenue, profit or investment unit-based
performance, and (3) the balanced scorecard.
 Compensation pools, that is, the source from which the bonus pay is funded. The two
most common compensation pools are earnings in the manager's own unit and firmwide
pool based on the firm's total earnings.
Payment options, that is, how the bonus is to be awarded. The two common options are cash and
stock (typically ordinary shares). The cash or stock can either be awarded currently or deferred to
future years. Stock can either be awarded directly or granted in the form of stock options.

Bases for Bonus Compensation

The choice of a base comes from a consideration of the compensation objectives as outlined in
Figure 17-1.

Figure 17-1: Advantages and Disadvantages of Different Bonus


Compensation Bases Relative to Compensation Objectives

Motivation Right Decision Fairness


Stock price (+/-) Depends on (+) Consistent with (-) Lack of
whether stock and shareholders' interests. controllability.
stock options are
included in base pay
and bonus
(+) aligns management
compensation with
shareholder interests
Strategic performance (+) Strongly motivating (+) Generally a good (+) Intuitive, clear, and
Measures (cost, if noncontrollable measure of economic easily understood
revenue, profit, and factors are excluded performance (-) Measurement
investment units) (-) Typically has only a issues: differences in
short-term focus accounting
(-) If bonus is very high, conventions, cost
creates an incentive for allocation methods,
inaccurate reporting financing methods, and
so on.
Balanced scorecard (+) Strongly motivating (+) Consistent with (+) If carefully defined
(critical success if noncontrollable management’s strategy and measured, critical
factors) factors are excluded (-) Can be subject to success factors are
(+) aligns management inaccurate reporting of likely to be perceived
compensation with nonfinancial factors as fair
shareholder interests (-) Potential
measurement issues,
as above
Key: (+) means the base has a positive effect on the objective.
(-) means the base has a negative effect on the objective.
Bonus Compensation Pools

A unit-based pool is a basis for determining a bonus according to the performance of a


manager’s unit.

A firmwide pool is a basis for determining the bonus available to all managers through an
amount set aside for this purpose.

Figure 17-2 summarizes the advantage and disadvantage of each approach to bonus pools.

Figure 17-2: Advantages and Disadvantages of Different


Bonus Pools Relative to Compensation Objectives

Motivation Right Decision Fairness


Unit based (+) Strong motivation for an (-) Provides the (-) Does not separate
effective manager - the incentive for individual the performance of the
upside potential managers not to unit from the manager's
(-) Unmotivating for cooperate with and performance.
manager for economically support other units
weaker units when reeded for the
good of me firm

Firmwid (+) Helps to attract (+) Effort for the good of (+) Separates the
e and retain good the overall firm is rewarded performance of the manager
managers - motivates teamwork and from that of the unit
throughout the firm, sharing of assets among
even in units
economically
weaker units

(-) Not as strongly (+) Can appear to be fairer to


motivating as the shareholders and others who
unit-based pool are concerned that executive
pay is too high

Key: (+) means the pool has a positive effect on the objective.
(-) means the pool has a negative effect on the objective.
Bonus Payment Options

The four most common payment options are:

Current bonus (cash and/or stock) based on current (usually annual) performance, the most
common bonus form.

Deferred bonus (cash and/or stock) earned currently but not paid for two or more years.
Deferred plans are used to avoid or delay taxes or to affect the manager's future total income
stream in some desired way. This type of plan can also be used to retain key managers because
the deferred compensation is paid only if the manager stays with the firm.

Stock options confer the right to purchase stock at some future date at a predetermined price.
They are used to motivate managers to increase stock price for the benefit of the shareholders.
When exercised, stock options also have the positive effect of increasing the executive's
ownership in the firm, thereby further increasing the executive's alignment with shareholder
interests. For this reason, many firms require executives to own a significant amount of stock in
the company.

Performance shares grant stock for achieving certain performance goals over two
years or more.

The advantages and disadvantages of the four plans are shown in Figure 17-3.

Figure 17-3: Advantages and Disadvantages of Bonus Payment Options Relative to


Compensation Objectives

Motivation Right Decision Fairness

Current (+) Strong motivation (-) Short-term focus (-) (+/-) Depends on the
bonus for current performance; Risk-averse manager clarity of the bonus
stronger motivation than avoids risky but arrangement and the
for deferred plans potentially beneficial consistency with which it
projects is applied

Deferred (+) Strong motivation Same as for current Same as for current bonus
bonus for current bonus
performance, but not as
strong as for the current
bonus plan since the
reward is delayed

Stock options (+) Unlimited upside (+) Incentive to (-) Uncontrollable factors
potential is highly consider longer-term affect stock price
motivating issues Also, same as for current
(-) Delay and (+) Provides better risk bonus
uncertainty in reward incentives than for
reduces motivation current or deferred
bonus plans
(+) Consistent with
shareholder interests

Performance Same as for stock (+) Incentive to (+/-) Depends on the


shares options consider long-term clarity of the bonus
factors that affect arrangements and the
stock price consistency with which it
(+) Consistent with the is applied
firm's strategy, when
critical success factors
are used
(+) Consistent with
shareholder interests
when earnings per
share is used

Key: (+) means the payment option has a positive effect.


(-) means the payment option has a negative effect.

Performance Measures at the Individual Activity Level

When evaluating performance at the individual activity level two issues are involved:
First: Designing performance measures of activities that require multiple tasks,
and
Second: Designing performance measures for activities done in teams.

Performing Tasks

It is a common business practice that employers want their employees to allocate their time and
effort intelligently among the various tasks or aspects of their jobs.
For example, marketing representatives sell products, provide customer support and gather
market information. Production works are responsible for both the quantity and quality of their
output.
The performance measurement should measure the different aspects of an employee's job and to
balance incentives so that all aspects are properly emphasized.

Team-based Compensation Arrangements

Pooling of talents of employees with multiple skills, knowledge, experiences and judgments can
resolve many businesses problems, be they manufacturing, marketing and design-related. A team
accomplishes better securities than individual employees acting alone. Business establishments
reward individuals or a team on the basis of team performance such as achieving regional sales
target by the regional team. Such team-based incentives encourage individuals to help one
another as they strive toward a common goal. To encourage development of team skills, some
companies use a checklist of team skills, such as communication and willingness to help. The
desirability of team-based compensation depends, to a great extent, on the culture and
management style of a particular organization.
Some criticisms on team-based compensation are
(1) Incentives for individual employees to excel are diminished, harming overall
performance, and
(2) Some team members who are not productive contributors to the team's success
nevertheless share in the team's rewards thereby dampening the interest and morale of the
gond performers.

Team-based incentive compensation encourages employees to work together to achieve common


goals. Individual-based incentive compensation rewards employees for their own performance,
consistent with responsibility accounting.

A mix of both types of incentives encourages employees to maximize their own performance
while working together in the best interest of the company as a whole.

Environmental and Ethical Responsibilities

As companies try to achieve the performance goals of their organizations, managers should be
aware constantly of their environmental and ethical responsibilities.
Illegal practices (such as bribery and corruption) and
Environmental pollution (such as water and air pollution) carry heavy fines and are a prison
offense under the laws of many countries. Business ethics present difficulties in a single-country
context, but they pose more problems in a global context.

Ethical behavior on the part of managers is paramount. They should not be tainted by "creative
accounting" resulting in overstatement of assets, understatement of liabilities, fictitious revenues
and understatement of costs. Additionally, management should promptly and severely reprimand
unethical conduct irrespective of the benefits that might accrue to the company from such action.

A strong underlying system is important for enforcing contracts and provides the basis for
confidence in ethical dealings. Other ethical problems with bribes and differing business laws
exist. US companies that contract with overseas firms may find themselves the target of
unfavorable publicity on use of child labor. The stories of bribery of Middle Eastern officials are
legendary. In some countries, these bribes are a necessary part of doing business. Insider trading
is not against the law in Europe and it is definitely illegal in the US.

Socially responsible companies set very strict environmental goals and measure and report their
performance against them. For example, a company makes environmental performance a line
item on every employee's salary appraisal report. Another company appraises employees on their
part in reducing solid waste, outing emissions and discharges and implementing environmental
problems,

REVIEW QUESTIONS AND PROBLEMS


Questions

1 What is incentive compensation? What type of organization is best suited to incentive


compensation plans?
2. What are the four guidelines for effective incentive compensation systems? Briefly
discuss each.
3. There are four broad approaches to distributing the proceeds of a bonus pool in a
profit-sharing plan:
1. Each person's share is based on her salary.
2. Each person receives an equal share.
3. Each person's share is based on his position in the organization (larger payments
to people at higher levels).
4. Each person's share is based on individual performance.

Required:
a. For each of these alternatives, give 2 reasons to support that alternative.
b. For each of these alternatives, give 2 reasons against that alternative.

4. Describe each of the following:


a. cash bonus
6. profit sharing
c. gain sharing
d. stock option plan
When should an organization use of them?
Problems

Problem 1

PK Corporation has a profit-sharing plan that is worded as follows:

The company will make available a profit-sharing pool that will be the maximum of the
following two items:

1. 20 percent of its profit in excess of the largest profit level which is 18 percent
of assets, or,
2. P2,000,000.

The individual employee will receive a share of the profit-sharing pool that is equal to the ratio
of that employee’s salary. To the total salary paid to all employees. The company earned
P20,000,000 in 20X4 and had net assets of P60,000,000. Total salaries for 20X4 were
P12,000,000.

Required
a. What would be the amount available for distribution from the profit-sharing pool?
b. What would Jo Marcelo’s profit share be assuming she earned P50,000 during 20x4?

Problem 2

FAC corporation has a profit-sharing plan that is worded as follows:


The company will make available a profit-sharing pool that will be the maximum of the
following two
items:

1. 25 percent of profits in excess of the largest profit level which is 18 percent of


assets, or,
2. P3,200,000.

The individual employee will receive a share of the profit-sharing pool that is equal to the ratio
of that employee’s salary to the total salary paid to all employees. The company earned
P30,000,000 in 20x4 and had net assets of P72,000,000. Total salaries for 20X4 were
P10,000,000.
Required:
a. What would be the amount available for distribution from the profit-sharing pool?
b. What would Francis Argante’s profit share be assuming he earned P40,000 during 20X4?

Multiple Choice

1. All of the following statements are true except


a. Cash bonuses, meals, and trips are examples of extrinsic rewards.
b. Pay for performance systems base rewards on achieving or exceeding some measured
performance.
c. An intrinsic reward is based on performance and is any reward that one person provides
to another person to recognize a job well done.
d. In an effective compensation system, each employee should be paid a basic wage that
reflects the market assessment for his skills and experience.

2. Which of the following statement is false?


a. Incentive compensation systems work best in organizations in which employees have no
skill or have not been empowered.
b. Profit sharing is a group incentive compensation plan focused on rewarding short-term
performance.
c. A stock option is the right to purchase a unit of the organization’s stock at a specified
price for a set time limit.
d. An important element of control is motivating employees to pursue the organization’s
interests as they undertake their daily jobs.

3. Which of the following is NOT a true statement?


a. An important element of control is motivating employees to pursue the organization’s
interests as they undertake their daily jobs.
b. An important element of motivation is compensation.
c. Hygiene factors relate to the job context and define the environment of the individual’s
work.
d. Compensation is not useful in motivating employees.

4. _____________ is based on performance and is any reward that one person provides to
another in recognition of a job well done.
a. Valence
b. Intrinsic rewards
c. Extrinsic rewards
d. Hygiene factors

5. Which of the following is an intrinsic reward?


a. Cash bonuses
b. Job satisfaction
c. Trips
d. Meals
6. Which of the following is an extrinsic reward?
a. Stock bonuses
b. Recognition in the organization’s newsletter
c. Recognition on a plaque
d. All of the above are extrinsic rewards.

7. Which of the following is NOT an attribute of effective performance measurement


systems?
a. The person must understand the job.
b. The job’s performance measures should reflect the organization’s key success factors.
c. The performance measurement system should set clear standards or targets for
performance.
d. The pay for performance systems base rewards on only net income.

8. _____________ systems base rewards on achieving or exceeding some measured


performance.
a. Pay for performance
b. Base salary agreement
c. Intrinsic reward
d. Marketing

9. Which of the following is NOT an attribute of effective performance measurement


systems?
a. The performance measurement system should be accurate.
b. The reward system should focus on individual or group rewards depending on the nature
of the job.
c. The performance measurement system should set clear standards or targets for
performance.
d. Incentive wage for hours worked.

10. Under the independent wage policy guideline for effective incentive compensation
systems, wage and incentive compensation policy for senior management should be
developed by:
a. Senior management
b. A board of directors’ compensation committee.
c. Employees.
d. Middle management.

11. Which of the following is true about the independent wage policy for effective incentive
systems?
a. Senior management should have its own wage and incentive compensation
b. The compensation committee should not operate independently of senior management’s
direction.
c. A board of director’s compensation committee should design the incentive compensation
plan for senior management.
d. All of the above are true statements.
12. Which of the following is true about the participation guideline for effective incentive
compensation systems?
a. Many experts believe that only the senior management should participate in an incentive
compensation plan.
b. Many experts believe that all employees should participate in an incentive compensation
plan.
c. Incentive plans do not need to be documented clearly.
d. Many experts feel that the incentive compensation should be about 200% of the
employee’s basic wage for senior levels of the organization.

13. Which of the following guidelines is being described by the statement below?

A board of directors’ compensation committee should design the incentive


compensation plan for senior management without direct influence from the senior
management.
a. Fairness
b. Participation
c. Basic wage level
d. Independent wage policy

14. _____________ is a group incentive compensation plan focused on rewarding short-term


performance.
a. A cash bonus
b. Profit sharing
c. Gain sharing
d. A stock options

15. _____________ is (are) also called lump-sum rewards, pay for performance, and merit
pay.
a. Cash bonuses
b. Profit sharing
c. Gain sharing
d. Stock options

16. _____________ is the right to purchase a unit of the organization’s stock at a specified
price.
a. A cash bonus
b. Profit sharing
c. Gain sharing
d. A stock option

17. Which of the following would not be an advantage for distributing the proceeds of a
bonus pool in a profit-sharing plan based on each person’s salary?
a. Easy to administer
b. Likely to be considered fair
c. Always reflects contributions made
d. Easy to calculate

18. Which of the following would not be an advantage for distributing the proceeds of a
bonus pool in a profit-sharing plan based on an equal share?
a. Easy to administer
b. May have little motivational effect
c. Likely to be considered fair
d. Reflects how people often divide rewards

19. Single performance measures can often


a. Increase an employee’s overall performance by focusing his or her attention on only one
aspect of their work.
b. Create employee myopia by focusing their attention on only one aspect of their work
c. Lead to greater job satisfaction for employees
d. Increase the level of teamwork in an organization

20. Reward system designers consider all of the following when designing an incentive
system except
a. The level of uncertainly about goal achievement
b. The personalities of employees
c. The risk attitudes of employees
d. The work ethic of employees

21. Participation in decision-making involves


a. Filling out budget requests that are passed on to a superior.
b. Telling a superior where you would like the budget set.
c. A joint decision-making process in which all parties agree to the levels at which the
budget should be set.
d. Electing a spokesperson to tell a superior where you would like the budget set.

22. Empowering employees mean


a. They are free to strike at any time.
b. They can hold secret meetings on company time.
c. They get a greater share of the raise pool than they did before.
d. They are given the ability to suggest and make changes to their work environment.

23. Participation in decision-making may lead to the following benefits except


a. Increased job satisfaction
b. Increased tensions between coworkers
c. Improved morale
d. Greater commitment to the decision

24. Which of the following is NOT an attribute of effective performance measurement


systems?
a. The person must understand the job
b. The reward system should focus on individual or group rewards depending on the nature
of the job
c. The performance measurement system should be accurate
d. The performance measurement system should set clear standards or targets for
performance
Glossary of Terms

Absorption costing (Full-costing) - A product-costing method that assigns all manufacturing costs to a
product; direct materials, direct labor, variable overhead, and fixed overhead.
Accounting rate of return - The rate of return obtained by dividing the average accounting net income
by the original investment (or by average investment).
Activity - A basic unit or work performed within an organization. It also can be defined as an aggregation
of actions within an organization useful to managers for purposes of planning, controlling, and decision
making.
Activity-bases costing (ABC) – A cost assignment approach that first direct and driver tracing to assign
costs to activities and then uses drivers to assign costs to cost objects.
Administrative costs – All costs associated with the general administration of the organization that
cannot be reasonably assigned to either marketing or production.
Annuity – A series of future cash flows.
Benchmarking – An approach that uses best practices as the standard for evaluating activity
performance.
Binding constraints – Constraints whose resources are fully utilized.
Budget committee – A committee responsible for setting budgetary policies and goals, reviewing and
approving the budget and resolving any differences that may arise in the budgetary process.
Budgetary slack – The process of padding the budget by over-estimating costs and underestimating
revenues.
Budget – Plan of action expressed in financial terms.
Capital Budgeting – The process of making capital investment decision.
Capital investment decisions – The process of planning, setting goals and priorities, arranging financing
and identifying criteria for making long-term investments.
Carrying costs – The costs of holding inventory.
Cash budget – A detailed plan that outlines all sources and uses of cash
Centralized decision making – A system in which decisions are made at the top level of an organization
and local managers are given the charge to implement them.
Certified Internal Auditor (CIA) – A person who has passed a comprehensive examination designed to
ensure technical competence and has two years’ experience.
Certified Management Accountant (CMA) – A person who has passed a rigorous qualifying
examination, has met an experience requirement, and participates in continuing education.
Certified Public Accountant (CPA) – A person who is permitted (by law) to serve as an external auditor
and who must pass a national examination and be licensed by the state in which he or she practices.
Coefficient of correlation – The square root of the coefficient of determination, which is used to express
not only the degree of correlation between two variables but also the direction of the relationship.
Compounding of interest – Paying interest on interest.
Constraint set – The collection of all constraints that pertain to a particular optimization problem.
Constraints – Mathematical expressions that express resource limitations.
Continuous improvement – The process of searching for ways of increasing the overall efficiency and
productivity of activities by reducing waste, increasing quality, and reducing costs.
Control – The process of setting standards, receiving feedback on actual performance, and talking
corrective actions whenever actual performance deviates significantly from planned performance.
Control costs – Cost incurred from performing control activities.
Control limits – The maximum allowable deviation from a standard.
Controllable costs – Costs that managers have the power to influence.
Controller – The chief accounting officer; supervises all accounting departments.
Controlling – The managerial activity of monitoring a plan’s implementation and taking corrective action
as needed.
Conversion cost – The sum of direct labor cost and overhead cost.
Cost – The cash or cash equivalent value sacrificed for goods and services that are expected to bring a
current of future benefit to the organization.
Cost assignment – The process of associating that costs, once measured, with the units produced.
Cost center – A division of a company that is evaluated on the basis of cost.
Cost measurement – The act of determining the peso amounts of direct materials, direct labor, and
overhead used in production.
Cost of capital – The cost of investment funds, usually viewed as a weighted average of the cost of funds
from all sources
Cost of goods manufactured – The total cost of goods completed during the current period.
Costs of goods sold – The cost of direct materials, direct labor, and overhead attached to the units sold.
Costs of quality – Costs incurred because poor quality may exist or because poor quality does exist.
Currency appreciation – When one country’s currency becomes stronger and can purchase more units of
another country’s currency.
Currency deprecation – When one country’s currency becomes weaker and can purchase fewer units of
another country’s currency.
Currency risk management – A company’s management of its transaction, economic, and translation
exposure due to exchange rate fluctuations.
Cycle time – The length of time required to produce one unit of a product.
Decentralization – The granting of decision-making freedom to lover operating levels.
Decentralization decision-making – A system in which decisions are made and implemented by lower-
level managers.
Decision making – The process of choosing among competing alternatives.
Dependent variable – A variable whose value depends on the value of another variable. For example, Y
in the cost formula Y = F + VX depends on the value of X.
Direct costs – Costs that can be easily and accurately traced to a cost object.
Direct Fixed expenses – Fixed costs that are directly traceable to a given segment and, consequently,
disappear if the segment is eliminated.
Direct materials budget – A budget that outlines the expected usage of materials production and
purchases of the direct materials required.
Discount Factor – The factor used to convert a future cash flow to its present value.
Discount rate – The rate of return used to compute the present value of future cash flows.
Discounted cash flows – Future cash flows expressed in present-value terms.
Double-loop feedback – Information about both the effectiveness of strategy implementation and the
validity of assumptions underlying the strategy.
Drum-Buffer-Rope (DBR) System – The TOC inventory management system that relies on the drum
beat of the major constrained resources, time buffers and ropes to determines inventory levels.
Dysfunctional behaviors – Individual behaviors that conflicts with the goals of the organization.
Economic order quantity (EOQ) – The amount that should be ordered (or produced) to minimize the
total ordering (or setup) and carrying costs.
Economic risk – The possibility that a firm’s present value of future cash flows can be affected by
exchange fluctuations.
Economic value added (EVA) – A performance measure that is calculated by taking the after-tax
operating profit minus the total cost of capital.
Environmental detection costs – Costs incurred to detect poor environmental performance.
Environmental external failure costs – Costs incurred after contaminants are introduced into the
environment.
Environmental internal failure costs – Costs incurred after contaminants are produced but before they
are introduced into the environment.
Environmental prevention costs – Costs incurred to prevent damage to the environment.
Ethical behavior – Choosing actions that are “right”, “proper” and “just”. Our behavior can be right or
wrong, it can be proper or improper, and the decisions we make can be fair or unfair.
Exchange rates – The rates at which foreign currency can be exchanged for domestic currency.
Expenses – Expired costs.
External failure costs – Costs incurred because products fail to conform to requirements after being sold
to outside parties.
External linkages – The relationship of a firm’s activities within its segment of the value chain which
those activities of its suppliers and customers.
Failure activities – Activities performed by an organization or its customers in response to poor quality
(poor quality does exist).
Failure costs – The costs incurred by an organization because failure activities are performed.
Financial accounting information system – An accounting information subsystem that is primarily
concerned with producing outputs for external users and uses well-specified economic events as inputs
and processes that meet certain rules and conventions.
Financial budget – The portions of the master budget that include the cash budget, the budgeted balance
sheet, the budgeted statement of cash flows, and the capital budget.
Financial perspective – A balanced scorecard viewpoint that describes the financial consequences of
actions taken in the other three perspective.
Financial productivity measure – A productivity measure in which inputs and outputs are expressed in
peso.
Fixed cost – Cost that, in total, are constant within the relevant range as the activity output varies.
Flexible budget – A budget that can specify costs for a range of activity.
Forward contract – An agreement that requires the buyer to exchange a specified amount of a currency
at a specified rate (the forward rate) on a specified future date.
Full environmental costing – The assignment of all environmental costs, both private and societal, to
products.
Functional-based costing (FBC) - An approach for assigning costs of shared resources to products and
other cost objects using only production or unit-level drivers.
Functional-based management (FBM) – A managerial approach that attempts to control costs by
focusing on the efficiency of organizational subunits.
Functional-based responsibility accounting system – A control system defined by centering
responsibility on organizational units and individuals with traditional budgets and standard costing used
to evaluate and monitor performance.
Future value – The value that will accumulate by the end of an investment’s life if the investment earns a
specified compounded return.
Gainsharing – Providing cash incentives for a company’s entire workforce that are keyed to quality and
productivity gains.
Goal congruence – The alignment of a manager’s personal goals with those of the organization.
Hedging – A way of insuring against gains and losses on foreign currency exchange.
Hidden quality costs – Opportunity costs resulting from poor quality.
Homogeneous cost pool – A collection of overhead costs associated with activities that have the same
process and the same level and can use the same activity driver to assign costs to products.
Ideal standards – Standards that reflect perfect operating conditions.
Improvement analysis – A life-cycle assessment step where efforts are made to reduce the environmental
impacts revealed by the inventory and impact steps.
Incentives – The positive or negative measures taken by an organization to induce a manager to exert
effort toward achieving the organization’s goals.
Independent projects – Projects that, if accepted or rejected, will not affect the cash flows of another
project.
Independent variable – A variable whose value does not depend on the value of another variable. For
example, in the cost formula Y = F + VX, the variable X is an independent variable.
Industrial value chain – The Linked set of value-creating activities from basic raw materials to end-use
customers.
Innovation process – A process that anticipates the emerging and potential needs of customers and
creates new products and services to satisfy those needs.
Internal business process perspective – A balanced scorecard viewpoint that describes the internal
processes needed to provide value of customers and owners.
Internal constraints – Limiting factors found within the firm (such as machine time availability).
Internal failure costs – Costs incurred because products and services fail to conform to requirements
where lack of conformity is discovered prior to external sale.
Internal measures – Measures that relate to the processes and capabilities that create value for customers
and shareholders.
Internal rate of return – The rate of return that equates the present value of a project’s cash inflows with
the present value of a project’s cash inflows with the present value of its cash outflows (i.e., it sets the
NPV equal zero). Also, the rate of return being earned on funds that remain internally invested in a
project.
Internal value chain – The set of activities required to design, develop, produce, market, distribute, and
service a product (the product can be service).
Investment center – A division of company that is evaluated on the basis of return on investment.
JIT manufacturing – A demand-pull system whose objective is to eliminate waste by producing a
product only when it is needed and only in the quantities demanded by customers.
JIT purchasing – A purchasing method that requires suppliers to deliver parts and materials just in time
to be used in production.
Kaizen costing – Efforts to reduce the costs of existing products and processes.
Kaizen standard – An interim standard that reflects the planned improvement for a coming period.
Keep-or-drop decisions – Relevant costing analysis that focus on keeping or dropping a segment of a
business.
Lead measures (performance drivers) – Factors that drive future performance.
Lead time – For purchasing the time to receive an order after it is placed. For manufacturing, the time to
produce a product from start to finish.
Learning and growth (infrastructure) perspective – A balanced scorecard viewpoint that defines the
capabilities that an organization need to create long-term growth and improvement.
Life cycle costs – All costs that are associated with the product for its entire life cycle.
Life-cycle assessment – An approach that identifies the environmental consequences of a product
through its entire life cycle and then searches for opportunities to obtain environmental improvements.
Life-cycle cost assessment – A method that assigns costs and benefits to environmental consequences
and improvements.
Life-cycle cost management – The management of value-chain activities so that a long-term competitive
advantage is created.
Linear programming – A method that searches among possible solutions until it finds the optimal
solution.
Line positions – Positions that have direct responsibility for the basic objectives of an organization.
Long run – A period of time in which all costs are variable.
Loose constraints – Constraints whose limited resources are not fully used by a product mix.
Make-or-buy decisions – Relevant costing analysis that focus on whether a component should be made
internally or purchased externally.
Management accounting information system – An information system that produces outputs using
inputs and processes needed to satisfy specific management objectives.
Margin – The ratio of net operating income to sales.
Margin of safety – The units sold or expected to be sold or sales revenue earned or expected to be earned
above the break-even volume.
Marketing (selling) costs – The costs necessary to market and distribute a product or service.
Markup – The percentage applied to base cost; it includes desired profit and any costs not included in the
base cost.
Master budget – The collection of all area and activity budgets representing a firm’s comprehensive plan
of action.
Maximum transfer price – The transfer price that will make the buying divisions no worse off if an
output is sold internally.
Method of least squares – A statistical method to find a line that best fits a set of data. It is used to break
out the fixed and variable components of a mixed cost.
Minimum transfer price – The transfer price that will make the selling division no worse off it an output
is sold internally.
Mixed cost – Costs that have both fixed and a variable component.
Multinational corporation (MNC) - A corporation for which a significant amount of business is done in
more than one country.
Mutually exclusive projects – Projects that, if accepted, preclude the acceptance of competing projects.

Net income- Operating income less income taxes.


Net present value -the difference between the present value of a project's cash inflows and the
present value of its cash outflows.
Nondiscounting models - Capital investment models that identify criteria for accepting or
rejecting projects without considering the time value of money.
Nonfinancial measures - Measures expressed in nonmonetary units.
Noninventoriable (period) costs - Costs that are expensed in the period in which they are
incurred.
Nonvalue-added activities - All activities other than those that are absolutely essential to remain
in business.
Nonvalue-added costs - Costs that are caused either by non-value-added activities or by the
inefficient performance of value-added activities.
Normal activity capacity - The average activity output for a given period.
Normal costing - An approach that assigns the actual costs of direct materiala and direct labor to
products but uses a predetermined rate to aasign overhead costs.
Objective function. The function to be optimized, usually a profit function; thus, optimization
usually means maximizing profits.

Objective measures. Measures that can be readily qualified and verified.

Observable quality costs. Quality costs that are available from an organization's accounting
records.

Operating budgets. Budgets associated with the income-producing activities of an organization.

Operating expenses. The money an organization spends in turning inventories into throughput.
Operating income. Revenues minus expenses from the firm's normal operations. Income taxes
are excuded.

Operation costing. A hybrid costing method that assign materials cost to a product using a job-
order approach and assigns conversion costs using a process approach.
Opportunity cost. The benefit sacrificed or foregone when one alternative is chosen over
another.

Optimal solution. The feasible solution that produces the best value for the objective function
(the largest value if seeking to maximize the objective function; the minimum otherwise).

Ordering costs. The costs of placing and receiving an order.

Outsourcing. The payment by a company for a business friction that was formerly done in
house.

Overhead. All production costs other than direct materials and direct labor.

Parallel processing. A processing pattern in which two or more sequential processes are
required to produce a finished good.

Partial productivity measurement. A ratio that measures productive efficiency for one input.

Participative budgeting. An approach to budgeting that allows managers who will be held
accountable for budgetary performance to participate in the budget's development.

Payback period. The time required for a project to return its investment.

Performance. The measure of how consistent and well a product functions.

Physical flow schedule. A schedule that reconciles units to account for with units accounted for.
The physical units are not adjusted for percent of completion.

Planning. Setting objectives and identifying methods to achieve those objectives.

Predatory pricing. The practice of setting prices below cost for the purpose of injuring
competitors and eliminating competition.

Present value. The current value of a future cash flow. It represents the amount that must be
invested now if the future cash flow is to be received assuming compounding at given rate of
interest.

Prevention costs. Costs incurred to prevent defects in products or services being produced.

Price discrimination. The charging of different prices to different customers for essentially the
same product.
Price gouging. A subjective term referring to the practice of setting an "excessively" high price.

Price-recovery component. The difference between the total profit change and the profit-linked
productivity change.
Price standards. The price that should be paid per unit of input.

Prime cost. The sum of direct materials cost and direct labor cost.

Process improvement. Incremental and constant increases in the efficiency of an existing


process.

Process innovation (business reengineering). The performance of a process in a radically new


way with the objective of achieving dramatic improvements in response time, quality, and
efficiency.

Process value analysis. An approach that focuses on processes and activities and emphasizes
systemwide performance instead of individual performance.
Process value chain. The innovation, operations, and post-sales service processes.

Process-costing system. A costing system that accumulates production costs by process or by


department for a given period of time.

Product cost. A cost assignment method that satisfies a well-specified managerial objective.

Product life cycle. The time a product-exists - from conception to abandonment.

Product budget. A budget that shows how many units must be produced to meet sales needs and
satisfy ending inventory requirements.

Production drivers. Drivers that are highly correlated with production output (volume).

Productivity. The efficient production of output, using the least quantity of inputs possible.

Productivity measurement. The assessment of productivity changes.

Profit center. A division of a company that is evaluated on the basis of operating income or
profit.

Pseudoparticipation. A budgetary system in which top management solicits inputs from lower-
level managers and then ignores those inputs. Thus, in reality, budgets are dictated from above.

Quality of conformance. Conforming to the design requirements of the product.

Quality product or service. A product that meets or exceeds customer expectations.


Quantity standards. The quantity of input allowed per unit of output.

Realized external failure costs. Environmental costs caused by environmental degradation and
paid for by the responsible organization.

Reciprocal method. A method that simultaneously allocates service costs to all user
departments. It gives full consideration to interactions among support departments.

Reorder point. The point in time at which a new order (or setup) should be initiated.

Required rate of return. The minimum rate of return that a project must earn in order to be
acceptable. Usually corresponds to the cost of capital.

Resource drivers. Factors that measure the consumption of resources by activities.

Responsibility accounting. A system that measures the results of each responsibility center
according to the information managers need to operate their center.

Responsibility center. A segment of the business whose manager is accountable for specified
sets of activities.

Return on investment (ROI). The ratio of operating income to average operating assets.

Revenue center. A segment of the business that is evaluated on the basis of sales.

Safety stock. Extra inventory carried to serve as insurance against fluctuations in demand.

Sales budget. A budget that describes expected sales in units and dollars for the coming period.

Segment. A submit of a company of sufficient importance to warrant the production of


performance reports.

Segment margin. The contribution a segment makes to cover common fixed costs and provide
for profit after direct fixed costs and variable costs are deducted from the segment's sales
revenue.

Segmented reporting. The preparation of financial performance reports for each segment of
importance within a firm.

Sell or process further. Relevant costing analysis that focuses on whether a product should be
processed beyond the split-off point.

Selling and administrative expenses budget. A budget that outlines planned expenditures for
nonmanufacturing activities.
Sensitivity analysis. The "what if" process of altering certain key variables to assess the effect
on the original outcome.

Sequential processing. A processing pattern in which units pass from one process to another in
a set order.

Setup costs. The costs of preparing equipment and facilities so that they can be used for
production.

Simplex method. An algorithm that identifies the optimal solution for a linear programming
problem.

Single-loop feedback. Information about the effectiveness of strategy implementation.

Special-order decisions. Relevant costing analyses that focus on whether a specially priced
order should be accepted or rejected.

Static budget. A budget for a particular level of activity.

Step cost. A cost function in which cost is defined for ranges of activity usage rather than point
values. The function has the property of displaying constant cost over a range of activity usage
and then changing to a different cost level as a new range of activity usage is undertaken.

Stockout cost. The costs of insufficient inventory.

Strategic-based responsibility accounting system (Balanced Scorecard). A responsibility


accounting system objectives and measures for four different perspectives: the financial
perspective, the customer perspective, the process perspective, and the learning and growth
(infrastructure) perspective.

Strategic cost management. The use of cost data to develop and identify superior strategies that
will produce a competitive advantage.

Strategic decision making. Choosing among alternative strategies with the goal of selecting a
strategy or strategies that provide a company with a reasonable assurance for long-term growth
and survival.

Strategic plan. The long-term plan for future activities and operations, usually involving at least
five years.

Strategy. The process of choosing a business's market and customer segments, identifying its
critical internal business process, and selecting the individual and organizational capabilities
needed to meet internal, customer and financial objectives.

Sunk cost. A cost for which the outlay has already been made and that cannot be affected by a
future decision.
Sustainable development. Development that meets the needs of the present without
compromising the ability of future generations to meet their own needs.

Tactical decision making. Choosing among alternatives with an immediate or limited end in
view.
Target cost. The difference between the sales price needed to achieve a projected market share
and the desired per unit profit.

Target costing. A method of determining the cost of a product or service based on the price
(target price) that customers are willing to pay.

Technical efficiency. The point at which, for any mix of inputs that will produce a given output,
no more of any one input is used than is absolutely necessary.

Testable strategy. A set of linked objectives aimed at an overall goal that can be restated into a
sequence of cause-and-effect hypotheses.

Throughput. The rate at which an organization generates money through sales.

Time buffer. The inventory needed to keep the constrained resource busy for a specified time
interval.

Total preventive maintenance. A program of preventive maintenance that has zero machine
failures as its standard.

Total productive efficiency. The point at which technical and price efficiency are achieved.

Total quality management. An approach to quality in which manufacturers strive to create an


environment that will enable workers to manufacture perfect (zero-defect) products.

Transaction risk. The possibility that future cash transactions will be affected by changing
exchange rates.

Transfer price. The price charged for goods transferred from one division to another.

Transfer pricing problem. The problem of finding a transfer pricing system that simultaneously
satisfies the three objectives of accurate performance evaluation, goal congruence, and
autonomy.

Treasurer. The person responsible for the finance function. Specifically, the treasurer raises
capital and manages cash and investments.

Unit-level activities. Activities that are performed each time a unit is produced.
Unit-level activity drivers. Factors that measure the consumption of unit-level activities by
products and other cost objects.

Value-added activities. Activities that are necessary for a business to achieve corporate
objectives and remain in business.

Value-added costs. Costs caused by value-added activities.


Value activity rate. Total variable activity cost divided by the amount of activity driver used.

Variable costing. A product costing method that assigns only variable manufacturing costs to
production: direct materials, direct labor, and variable overhead. Fixed overhead is treated as a
period cost.

Weighted average costing method. A process-costing method that combines beginning


inventory costs with current-period costs to compute unit costs. Costs and output from the
current period and the previous period are averaged to compute unit costs.

Whole-life cost. The life cycle cost of a product plus costs that consumers incur, including
operation, support, maintenance, and disposal.

Zero defects.
A quality performance standard that requires all products and services to be produced and
delivered according to specifications.

Zero-based budgeting. An alternative approach to budgeting in which the prior year's budgeted
level is not taken for granted. Instead, the existing operations are analyzed, and continuance of
the activity or operation must be justified on the basis of its need or usefulness to the
organization.
APPENDIX

Time Value of Money Concepts


Basic Concepts
1. A peso today is worth more than a peso to be received in the future. The difference between
the present value of cash flows and their future value represents the time value of money.
Interest is the rent paid for the use of money over time.
2. The future value of a single amount is the amount of money that a peso will grow to at
some point in the future. It is computed by multiplying the single amount by (1 + i)n, where i
is the interest rate and n the number of compounding periods. The Future Value of P1 table
allows for the calculation of future value for any single amount by providing the factors for
various combinations of i and n. (Table 1)
3. The present value of a single amount is the amount of money today that is equivalent to a
given amount to be received or paid in the future. It is computed by dividing the future
amount by (1 + i)n. The Present Value of P1 table simplifies the calculation of the present
value of any future amount (Table 2)
4. An annuity is a series of equal-sized cash flows occurring over equal intervals of time. An
ordinary annuity exists when the cash flows occur at the end of each period. An annuity due
exists when the cash flows occur at the beginning of each period.
5. The future value of an ordinary annuity (FVA) is the future value of a series of equal-sized
cash flows with the first payment taking place at the end of the first compounding period.
The last payment will not earn any interest since it is made at the end of the annuity period.
(Table 3)
6. The present value of an ordinary annuity (PVA) is the present value of a series of equal-
sized cash flows with the first payment taking place at the end of the first compounding
period. (Table 4)
7. The future value of an annuity due (FVAD) is the future value of a series of equal-sized
cash flows with the first payment taking place at the beginning of the annuity period (the
beginning of the first compounding period). (Table 5)
8. The present value of an annuity due (PVAD) is the present value of a series of equal-sized
cash flows with the first payment taking place at the beginning of the annuity period. The
present value of a deferred annuity is the present
616 Appendix

value of a series of equal-sized cash flows with the first payment taking place more than
one time period after the date of the agreement. (Table 6)
9. The tables (Tables 1 to 6) can be used to solve PV and FV calculations. When tables are not
available or if more precise results are required, a computer or calculator programmed to
solve present and future values may be used.
Summary of Time Value of Money Concepts
Concepts Summary Formula Table
Future value The amount of money that a
(FV) of P1 peso will grow to at some FV = P1 (P1 + i)n 1
point in the future.

Present value The amount of money today


(PV) of P1 that is equivalent to a given P1
PV = n 2
amount to be received or paid (1+l)
in the future.

Future value The future value of a series of


of an ordinary equal-sized cash flows with n
(1+l) −1
annuity (PVA) the first payment taking place FVA = 3
of P1 at the end of the first l
compounding period.

Present value The present value of a series


of an ordinary of equal-sized cash flows with 1
1− n
annuity (PVA) the first payment taking place PVA = (1+l) 4
of P1 at the end of the first l
compounding period.

Future value The future value of a series of

[ ]
of an annuity equal-sized cash flows with n
(1+l) −1
due (FVAD) of the first payment taking place FVAD = x (1+l) 5
P1 at the beginning of the annuity l
period.

Present value The present value of a series

[ ]
of an annuity of equal-sized cash flows with 1
1−
due (PVAD) of the first payment taking place PVAD = (1+l)n 6
P1 at the beginning of the annuity x (1+l)
l
period.

10. Most accounting applications of the time value of money involve the present value of
annuities. Certain long-term leases require the lessee to compute the present value of future
lease payments to value the leased asset and corresponding lease obligation. And pension
plans require the payment of deferred annuities to retirees.
Appendix 617

11. TABLE 1 Future Value of P1


Number of periods
n 2% 212% 3% 4% 5% 6% 7% 8% 9% 10%
1 1.02000 1.02500 1.03000 1.04000 1.05000 1.06000 1.07000 1.08000 1.09000 1.10000
2 1.04040 1.05063 1.06090 1.08160 1.10250 1.12360 1.14490 1.16640 1.18810 1.21000
3 1.06121 1.07689 1.09273 1.12486 1.15763 1.19102 1.22504 1.25971 1.29503 1.33100
4 1.08243 1.10381 1.12551 1.16986 1.21551 1.26248 1.31080 1.36059 1.41158 1.46410
5 1.10408 1.13141 1.15927 1.21665 1.27628 1.33823 1.40255 1.46933 1.53862 1.61051
6 1.12616 1.15969 1.19405 1.26532 1.34010 1.41852 1.50073 1.58687 1.67710 1.77156
7 1.14869 1.18869 1.22987 1.31593 1.40710 1.50360 1.60578 1.71382 1.82804 1.94872
8 1.17166 1.21840 1.26677 1.36857 1.47746 1.59385 1.71819 1.85093 1.99256 2.14359
9 1.19509 1.24886 1.30477 1.42331 1.55133 1.68948 1.83846 1.99900 2.17189 2.35795
10 1.21899 1.28008 1.34392 1.48024 1.62889 1.79085 1.96715 2.15892 2.36736 2.59374
11 1.24337 1.31209 1.38423 1.53945 1.71034 1.89830 2.10485 2.33164 2.58043 2.85312
12 1.26824 1.34489 1.42576 1.60103 1.79586 2.01220 2.25219 2.51817 2.81266 3.13843
13 1.29361 1.37851 1.46853 1.66507 1.88565 2.13293 2.40985 2.71962 3.06580 3.45227
14 1.31948 1.41297 1.51259 1.73168 1.97993 2.26090 2.57953 2.93719 3.34173 3.79750
15 1.34587 1.44830 1.55797 1.80094 2.07893 2.39656 2.75903 3.17217 3.64248 4.17725
16 1.37279 1.48451 1.60471 1.97298 2.18287 2.54035 2.95216 3.42594 3.97031 4.59497
17 1.40024 1.52162 1.65285 1.94790 2.29202 2.69277 3.15882 3.70002 4.32763 5.05447
18 1.42825 1.55966 1.70243 2.02582 2.40662 2.85434 3.37993 3.99602 4.71712 5.55992
19 1.45681 1.59865 1.75351 2.10685 2.52695 3.02560 3.61653 4.31570 5.14666 6.11591
20 1.48595 1.63862 1.80611 2.19112 2.65330 3.20714 3.86968 4.66096 5.60441 6.72750
21 1.51567 1.67958 1.86029 2.27877 2.78596 3.39956 4.14056 5.03383 6.10881 7.40025
22 1.54598 1.72157 1.91610 2.36992 2.92526 3.60354 4.43040 5.43654 6.65860 8.14027
23 1.57690 1.76461 1.97359 2.48472 3.07152 3.81975 4.74053 5.87146 7.25787 8.95430
24 1.60844 1.80873 2.03279 2.56330 3.22510 4.04893 5.07237 6.34118 7.91108 9.83973
25 1.64061 1.85394 2.09378 2.66584 3.38365 4.29187 5.42743 6.84848 8.62308 10.83471
618 Appendix

TABLE 1 Future Value of P1 (continued)


Number of periods
n 11% 12% 14% 15% 16% 18% 20% 22% 24% 25%
1 1.11000 1.12000 1.14000 1.15000 1.16000 1.18000 1.20000 1.22000 1.24000 1.25000
2 1.23210 1.25440 1.29960 1.32250 1.34560 1.39240 1.44000 1.48840 1.53760 1.56250
3 1.36763 1.40493 1.48154 1.52088 1.56090 1.64303 1.72800 1.81585 1.90662 1.95313
4 1.51807 1.57353 1.68896 1.74091 1.81064 1.93878 2.07360 2.21533 2.36421 2.44141
5 1.68506 1.76234 1.92541 2.01136 2.10034 2.28776 2.48832 2.70271 2.93163 3.05176
6 1.87041 1.97382 2.19497 2.31306 2.43640 2.69955 2.98598 3.29730 3.63522 3.81470
7 2.07616 2.21068 2.50227 2.66002 2.82622 3.18547 3.58318 4.02271 4.50767 4.76837
8 2.30454 2.47596 2.85259 3.05902 3.27841 3.75885 4.29982 4.90771 5.58951 5.96046
9 2.55804 2.77308 3.25195 3.51788 3.80296 4.43535 5.15978 5.98740 6.93099 7.45058
10 2.83942 3.10585 3.70722 4.04556 4.41144 5.23384 6.19174 7.30463 8.59443 9.31323
11 3.15176 3.47855 4.22623 4.65239 5.11726 6.17593 7.43008 8.91165 10.65709 11.64153
12 3.49845 3.89598 4.81790 5.35025 5.93603 7.28759 8.91610 10.87221 13.21479 14.55192
13 3.88328 4.36349 5.49241 6.15279 6.88579 8.59936 10.69932 13.26410 16.38634 18.18989
14 4.31044 4.88711 6.26135 7.07571 7.98752 10.14724 12.83918 16.18220 20.31906 22.73737
15 4.78459 5.47357 7.13794 8.13706 9.26552 11.97375 15.40702 19.74229 25.19563 28.42171
16 5.31089 6.13039 8.13725 9.35762 10.74800 14.12902 18.48843 24.08559 31.24259 35.51714
17 5.80509 6.86604 9.27646 10.76126 12.46768 16.67225 22.18611 29.38442 38.74081 44.48092
18 6.54355 7.68997 10.57517 12.37545 14.46451 19.67325 26.62333 35.84899 48.03860 55.51115
19 7.26334 8.61276 12.05569 14.23177 16.77652 23.21444 31.94800 43.73577 59.56786 69.38894
20 8.06231 9.64629 13.74349 16.36654 19.46076 27.39303 38.33760 53.35764 73.84615 86.73617
21 8.94917 10.80385 15.66758 18.82152 22.57448 32.32378 46.00512 65.09632 91.59155 108.42022
22 9.93357 12.10031 17.86104 21.64475 26.18640 38.14206 55.20614 79.41751 113.57352 135.52527
23 11.02627 13.55235 20.36158 24.89146 30.37622 45.00763 66.24737 96.88936 140.83116 169.40659
24 12.10031 15.17863 23.21221 28.62518 35.23642 53.10901 79.49685 118.20502 174.63064 211.75824
25 13.58546 17.00006 26.46192 32.91895 40.87424 62.66863 95.39622 144.21013 216.54199 264.69780
Appendix 621

TABLE 2 Present Value of P1


Number of periods
n 2% 212% 3% 4% 5% 6% 7% 8% 9% 10%
1 0.98039 0.97561 0.97087 0.96154 0.95238 0.94340 0.93458 0.91593 0.91743 0.90909
2 0.96117 0.95181 0.84260 0.92456 0.90703 0.89000 0.87344 0.85734 0.84168 0.82645
3 0.94232 0.92860 0.41514 0.88900 0.86384 0.83962 0.81630 0.79383 0.77218 0.75131
4 0.93385 0.90595 0.88849 0.85480 0.82270 0.79209 0.76290 0.73503 0.70843 0.68301
5 0.90573 0.88385 0.86261 0.82193 0.78353 0.74729 0.71299 0.68058 0.64993 0.62092
6 0.88797 0.86230 0.83748 0.79031 0.74622 0.70296 0.66634 0.63017 0.59627 0.56447
7 0.87056 0.84127 0.81309 0.75992 0.71068 0.66506 0.62275 0.58349 0.54703 0.51316
8 0.85349 0.82075 0.78941 0.73069 0.67684 0.62741 0.58201 0.54027 0.50187 0.46651
9 0.83676 0.80073 0.76642 0.70259 0.64461 0.59190 0.54393 0.50025 0.46043 0.42410
10 0.82035 0.78120 0.74409 0.67556 0.61391 0.55839 0.50835 0.46319 0.42241 0.38554
11 0.80426 0.76214 0.72242 0.64958 0.58468 0.52679 0.47509 0.42888 0.38753 0.35049
12 0.78849 0.74056 0.70138 0.62460 0.55684 0.49697 0.44401 0.39711 0.35553 0.31863
13 0.77303 0.72542 0.68095 0.60057 0.53032 0.46884 0.41496 0.36770 0.32618 0.28966
14 0.75788 0.70773 0.66112 0.57748 0.50507 0.44230 0.38782 0.34046 0.29925 0.26333
15 0.74301 0.69047 0.64186 0.55526 0.48102 0.41727 0.36245 0.31524 0.27454 0.23939
16 0.72845 0.67362 0.62317 0.53391 0.45811 0.39365 0.33873 0.29189 0.25187 0.21763
17 0.71416 0.65720 0.60502 0.51337 0.43630 0.37136 0.31657 0.27027 0.23107 0.19784
18 0.70016 0.64117 0.58730 0.49363 0.41552 0.35034 0.29586 0.25025 0.21199 0.17986
19 0.68643 0.62553 0.57029 0.47464 0.39573 0.33051 0.27651 0.23171 0.19449 0.16351
20 0.67297 0.61027 0.55368 0.45639 0.37689 0.31180 0.25832 0.21455 0.17843 0.14864
21 0.65978 0.59539 0.53755 0.43883 0.35894 0.29416 0.24151 0.19866 0.16370 0.13513
22 0.64684 0.58086 0.52189 0.42196 0.34185 0.27751 0.22571 0.18394 0.15018 0.12285
23 0.63146 0.56670 0.50669 0.40573 0.32557 0.26180 0.21095 0.17032 0.13778 0.11168
24 0.62172 0.55288 0.49190 0.39012 0.31007 0.24698 0.19715 0.15770 0.12640 0.10153
Appendix 621

25 0.60952 0.53939 0.47761 0.37512 0.29530 0.23300 0.18425 0.14602 0.11597 0.09230
620 Appendix

TABLE 2 Present Value of P1 (continued)


Number of periods
n 11% 12% 14% 15% 16% 18% 20% 22% 24% 25%
1 0.90090 0.89286 0.87719 0.86957 0.86207 0.84746 0.83333 0.81967 0.80645 0.80000
2 0.81162 0.79719 0.76947 0.75614 0.74316 0.71818 0.69444 0.67186 0.65036 0.64000
3 0.73119 0.71178 0.67497 0.65751 0.64066 0.60863 0.57870 0.55071 0.52449 0.51200
4 0.65873 0.63552 0.59208 0.57175 0.55229 0.51579 0.48225 0.45140 0.42297 0.40960
5 0.59345 0.56743 0.51937 0.49718 0.47611 0.43711 0.40188 0.37000 0.34111 0.32768
6 0.53464 0.50663 0.45559 0.43233 0.41044 0.37043 0.33490 0.30328 0.27509 0.26214
7 0.48166 0.45235 0.39964 0.37594 0.35383 0.31393 0.27908 0.24859 0.22184 0.20972
8 0.43393 0.40388 0.35056 0.32690 0.30503 0.26604 0.23257 0.20376 0.17891 0.16777
9 0.39092 0.36061 0.30751 0.28426 0.26295 0.22546 0.19381 0.16702 0.14428 0.13422
10 0.35218 0.32197 0.26974 0.24718 0.22668 0.19106 0.16151 0.13690 0.11635 0.10737
11 0.37128 0.28748 0.23662 0.21494 0.19542 0.16192 0.13459 0.11221 0.09383 0.08590
12 0.28584 0.25668 0.20756 0.18691 0.16846 0.13722 0.11216 0.09198 0.07567 0.06872
13 0.25751 0.22917 0.18207 0.16253 0.14523 0.11629 0.09346 0.07539 0.06103 0.05498
14 0.23199 0.20462 0.15971 0.14133 0.12520 0.09855 0.07789 0.06180 0.04921 0.04398
15 0.20900 0.18270 0.14010 0.12289 0.10793 0.08352 0.06491 0.0506 0.03969 0.03518
16 0.18829 0.16312 0.12289 0.10686 0.09304 0.07078 0.05409 0.04152 0.03201 0.02815
17 0.16963 0.14564 0.10780 0.09293 0.08021 0.05998 0.04507 0.03403 0.02581 0.02252
18 0.15282 0.13004 0.09456 0.08081 0.06914 0.05083 0.03756 0.02789 0.02082 0.01801
19 0.13768 0.11611 0.08295 0.07027 0.05961 0.04308 0.03130 0.02286 0.01679 0.01441
20 0.12403 0.10367 0.07276 0.06110 0.05139 0.03651 0.02608 0.01874 0.01354 0.01153

21 0.11174 0.09256 0.06383 0.05313 0.04430 0.03094 0.02174 0.01536 0.01092 0.00922
22 0.10067 0.08264 0.05599 0.04620 0.03819 0.02622 0.01811 0.01259 0.00880 0.00738

23 0.09069 0.07379 0.05911 0.04017 0.03292 0.02222 0.01509 0.01032 0.00710 0.00590
24 0.08170 0.06588 0.04308 0.03493 0.02838 0.01883 0.01258 0.00846 0.00573 0.00472
25 0.07361 0.05882 0.03779 0.03038 0.02447 0.01596 0.01048. 0.00693 0.00462 0.00378
TABLE 3 Future Value of an Ordinary Annuity of P1
Number of periods
n 2% 212% 3% 4% 5% 6% 7% 8% 9% 10%
1 1.00000 1.00000 1.00000 1.00000. 1.00000 1.00000. 1.00000. 1.00000 1.00000 1.00000.
2 2.02000 2.02500 2.03000 2.04000 2.05000 2.06000 2.07000 2.08000 2.09000 2.09000
3 3.06040 3.07563 3.09090 3.12160 3.15250 3.18360 3.21490 3.24640 3.27810 3.31000
4 4.12161 4.15252 4.18363 4.24646 4.31013 4.37462 4.43994 4.50611 4.57313 4.64100
5 5.20404 5.25633 5.30914 5.41632 5.52563 5.63709 5.75074 5.86660 5.98471 6.10510
6 6.30812 6.38774 6.46841 6.63298 6.80191 6.97532 7.15329 7.33593 7.52333 7.71561
7 7.43428 7.54753 7.66246 7.89829 8.14201 8.39384 8.65402 8.92280 9.20043 9.48717
8 8.58297 8.73612 8.89234 9.21423 9.54911 9.89747 10.25980 10.63663 11.02847 11.43589
9 9.75463 9.95452 10.15911 10.58280 11.02656 11.49132 11.97799 12.48756 13.02104 13.57948
10 10.94972 11.20338 11.46388 12.00611 12.57789 13.18079 13.81645 14.48656 15.19293 15.93742
11 12.16872 12.48347 12.80780 13.48635 14.20679 14.97164 15.78360 16.64549 17.56029 18.53117
12 13.41209 13.79555 14.19203 15.02581 15.91713 16.86994 17.88845 18.97713 20.14072 21.38428
13 14.68033 15.14044 15.61779 16.62684 17.71298 18.88214 20.14064 21.49530 22.95338 24.52271
14 15.97394 16.51895 17.08632 18.29191 19.59863 21.01507 22.55049 24.21492 26.01919 27.97498
15 17.29342 17.93193 18.59891 20.02359 21.57856 23.27597 25.12902 27.15211 29.36092 31.77248
16 18.63929 19.38022 20.15688 21.82453 23.65749 25.67253 27.88805 30.32428 33.00340 35.94973
17 20.01207 20.86473 21.76159 23.69751 25.84037 28.21288 30.84022 33.75023 36.97370 40.54470
18 21.41231 22.38635 23.41444 25.64541 28.13238 30.90565 33.99903 37.45024 41.30134 45.59917
19 22.84056 23.94601 25.11687 27.67123 30.53900 33.75999 37.37896 41.44626 46.01846 51.15909
20 24.29737 25.54466 26.87037 29.77808 33.06595 36.78559 40.99549 45.76196 51.16012 57.27500
21 25.78332 27.18327 28.67649 31.96920 35.71925 39.99273 44.86518 50.42292 56.76453 64.00250
22 27.29898 28.86286 30.53678 34.24797 38.50521 43.39229 49.00574 55.45676 62.87334 71.40275
23 28.84496 30.58443 32.45288 36.61789 41.43048 46.99583 53.43614 60.89330 69.53194 79.54302
24 30.42186 32.34904 34.42647 39.08260 44.50200 50.81558 58.17667 66.76476 76.78981 88.49733
25 32.03030 34.15776 36.45926 41.64591 47.72710 54.86451 63.24904 73.10594 84.70090 98.34706
TABLE 3 Future Value of an Ordinary Annuity of P1 (continued)

Number of periods
n 11% 12% 14% 15% 16% 18% 20% 22% 24% 25%
1 1.00000 1.00000 1.00000 1.00000 1.00000 1.00000 1.00000 1.00000 1.00000 1.00000
2 211000 2.12000 2.14000 2.15000 2.16000 2.18000 2.20000 2.22000 2.24000 2.25000
3 3.34210 3.37440 3.43960 3.47250 3.50560 3.57240 3.64000 3.70840 3.77760 3.81250
4 4.70973 4.77933 4.92114 4.99338 5.06650 5.21543 5.36800 5.52425 5.68422 5.76563
5 6.22780 6.35285 6.61010 6.74238 6.87714 7.15421 7.44160 7.73958 8.04844 8.20703
6 7.91286 8.11519 8.53552 8.75374 8.97748 9.44197 9.92992 10.44229 10.98006 11.25879
7 9.78327 10.08901 10.73049 11.06680 11.41387 12.14152 12.91590 13.73959 14.61528 15.07349
8 11.85943 12.29969 13.23276 13.72682 14.24009 15.32700 16.49908 17.76231 19.12294 19.84186
9 14.16397 14.77566 16.08535 16.78584 17.51851 19.08585 20.79890 22.67001 24.71245 25.80232
10 16.72201 17.54874 19.33730 20.30372 21.32147 23.52131 25.95868 28.65742 31.64344 33.25290
11 19.56143 20.65458 23.04452 24.34928 25.73290 28.75514 32.15042 35.96205 40.23787 42.56613
12 22.71319 24.13313 27.27075 29.00167 30.85017 34.93107 39.58050 44.87370 50.89495 54.20766
13 26.21164 28.02911 32.08865 34.35192 36.78620 42.21866 48.49660 55.74591 64.10974 68.75958
14 30.09492 32.39260 37.58107 40.50471 43.67199 50.81802 59.19592 69.01001 80.49608 86.94947
15 34.40536 37.27971 43.84241 47.58041 51.65951 60.96527 72.03511 85.19221 100.81514 109.68684
16 39.18995 42.75328 50.98035 55.71747 60.92503 72.93901 87.44213 104.93450 126.01077 138.10855
17 44.50084 48.88367 59.11760 65.07509 71.67303 87.06804 105.93056 129.02009 157.41499 173.63568
18 50.39594 55.74971 68.39407 75.83636 84.14072 103.74028 128.11667 158.40451 195.99416 218.04460
19 56.93949 63.43968 78.96923 88.21181 98.60323 123.41353 154.74000 194.25350 244.03276 273.55576
20 64.20283 72.05244 91.02493 102.44358 115.37975 146.62797 186.68800 237.98927 303.60062 342.94470
21 72.26514 81.69874 104.76842 118.81012 134.84051 174.02100 225.02560 291.34691 377.46477 429.68087
22 81.21431 92.50258 120.43600 137.63164 157.25336 206.34479 271.03072 356.44323 469.05632 538.10109
23 91.14788 104.60289 138.29704 159.27638 183.60138 244.48685 326.23686 435.86075 582.62984 673.62636
24 102.17415 118.15524 158.65862 184.16784 213.97761 289.49448 392.48424 532.75011 723.46100 843.03295
25 114.41331 133.33387 181.87083 212.79302 249.21402 342.60349 471.98108 650.95513 898.09164 1054.79118
List of References
Cabrera, Ma. Elenita B., Gilbert Anthony B., Management Accounting, Concepts and Applications, 2017
Edition, GIC Enterprises, Manila 2017
Cabrera, Ma. Elenita B., CPA Examination Reviewer and Quizzer in Management Advisory Services, 2017
Edition, GIC Enterprises, Manila 2017
Edward Blocher J., et. al., Cost Management, A Strategic Emphasis, 4th Edition, McGraw-Hill/Irwin,
USA 2015.
Garrison, Ray H., Noreen, Eric W., Managerial Accounting, 17th Edition, McGraw-Hill/Irwin, USA
2020.
Gleim, Irvin N., Flescher, Dale L., CMA/ CFM Review, 14th Edition, 2018.
Horngren, Charles T., Foster, George and Datar, Sukant M., Cost Accounting. A Managerial Emphasis,
14" Edition, Prentice Hall, Inc., 2019.
Moscove, Stephen A., Wright, Arnold, Cost Accounting with Managerial Applications, 10th Edition,
Houghton Mifflin Co, USA 2019.
Newton, Grant W., Decision Analysis and Information Systems, 6 Edition, Malibu Publishing Co., 2019.
Rickets, Don., Gray, Jack., Managerial Accounting, 7th Edition, Houghton Mifflin Co, USA 2020.
Wilson, James D., Colford, James P., Controllership, The Work of the Managerial Accountant, 7th
Edition, John Wiley & Sons, Inc., USA, 2019.

You might also like